Sunteți pe pagina 1din 196

Republic of the Philippines That as being OIC Governor of the Province of Rizal and in the performance of my

SUPREME COURT duties thereof, I among others, have signed as I did sign the unnumbered
Manila memorandum ordering the replacement of all the barangay officials of all the
barangay(s) in the Municipality of Taytay, Rizal;
EN BANC
That the above cited memorandum dated December 1, 1986 was signed by me
personally on February 8,1987;
G.R. No. 78059 August 31, 1987

That said memorandum was further deciminated (sic) to all concerned the
ALFREDO M. DE LEON, ANGEL S. SALAMAT, MARIO C. STA. ANA, JOSE C. TOLENTINO, ROGELIO J. DE LA
following day, February 9. 1987.
ROSA and JOSE M. RESURRECCION, petitioners,
vs.
HON. BENJAMIN B. ESGUERRA, in his capacity as OIC Governor of the Province of Rizal, HON. ROMEO FURTHER AFFIANT SAYETH NONE.
C. DE LEON, in his capacity as OIC Mayor of the Municipality of Taytay, Rizal, FLORENTINO G. MAGNO,
REMIGIO M. TIGAS, RICARDO Z. LACANIENTA, TEODORO V. MEDINA, ROSENDO S. PAZ, and TERESITA L.
Pasig, Metro Manila, March 23, 1987.
TOLENTINO, respondents.

Before us now, petitioners pray that the subject Memoranda of February 8, 1987 be declared null and
void and that respondents be prohibited from taking over their positions of Barangay Captain and
Barangay Councilmen, respectively. Petitioners maintain that pursuant to Section 3 of the Barangay
MELENCIO-HERRERA, J.: Election Act of 1982 (BP Blg. 222), their terms of office "shall be six (6) years which shall commence on
June 7, 1982 and shall continue until their successors shall have elected and shall have qualified," or up
to June 7, 1988. It is also their position that with the ratification of the 1987 Constitution, respondent
An original action for Prohibition instituted by petitioners seeking to enjoin respondents from replacing
OIC Governor no longer has the authority to replace them and to designate their successors.
them from their respective positions as Barangay Captain and Barangay Councilmen of Barangay
Dolores, Municipality of Taytay, Province of Rizal.
On the other hand, respondents rely on Section 2, Article III of the Provisional Constitution, promulgated
on March 25, 1986, which provided:
As required by the Court, respondents submitted their Comment on the Petition, and petitioner's their
Reply to respondents' Comment.
SECTION 2. All elective and appointive officials and employees under the 1973
Constitution shall continue in office until otherwise provided by proclamation or
In the Barangay elections held on May 17, 1982, petitioner Alfredo M. De Leon was elected Barangay
executive order or upon the designation or appointment and qualification of their
Captain and the other petitioners Angel S. Salamat, Mario C. Sta. Ana, Jose C. Tolentino, Rogelio J. de la
successors, if such appointment is made within a period of one year from February
Rosa and Jose M. Resurreccion, as Barangay Councilmen of Barangay Dolores, Taytay, Rizal under Batas
25,1986.
Pambansa Blg. 222, otherwise known as the Barangay Election Act of 1982.

By reason of the foregoing provision, respondents contend that the terms of office of elective and
On February 9, 1987, petitioner Alfredo M, de Leon received a Memorandum antedated December 1,
appointive officials were abolished and that petitioners continued in office by virtue of the aforequoted
1986 but signed by respondent OIC Governor Benjamin Esguerra on February 8, 1987 designating
provision and not because their term of six years had not yet expired; and that the provision in the
respondent Florentino G. Magno as Barangay Captain of Barangay Dolores, Taytay, Rizal. The designation
Barangay Election Act fixing the term of office of Barangay officials to six (6) years must be deemed to
made by the OIC Governor was "by authority of the Minister of Local Government."
have been repealed for being inconsistent with the aforequoted provision of the Provisional
Constitution.
Also on February 8, 1987, respondent OIC Governor signed a Memorandum, antedated December 1,
1986 designating respondents Remigio M. Tigas, Ricardo Z. Lacanienta Teodoro V. Medina, Roberto S.
Examining the said provision, there should be no question that petitioners, as elective officials under the
Paz and Teresita L. Tolentino as members of the Barangay Council of the same Barangay and
1973 Constitution, may continue in office but should vacate their positions upon the occurrence of any
Municipality.
of the events mentioned. 1

That the Memoranda had been antedated is evidenced by the Affidavit of respondent OIC Governor, the
Since the promulgation of the Provisional Constitution, there has been no proclamation or executive
pertinent portions of which read:
order terminating the term of elective Barangay officials. Thus, the issue for resolution is whether or not
the designation of respondents to replace petitioners was validly made during the one-year period which
xxx xxx xxx ended on February 25, 1987.

That I am the OIC Governor of Rizal having been appointed as such on March 20,
1986;
Considering the candid Affidavit of respondent OIC Governor, we hold that February 8, 1977, should be
considered as the effective date of replacement and not December 1,1986 to which it was ante dated, in
keeping with the dictates of justice.

But while February 8, 1987 is ostensibly still within the one-year deadline, the aforequoted provision in
the Provisional Constitution must be deemed to have been overtaken by Section 27, Article XVIII of the
1987 Constitution reading.

SECTION 27. This Constitution shall take effect immediately upon its ratification by
a majority of the votes cast in a plebiscite held for the purpose and shall supersede
all previous Constitutions.

The 1987 Constitution was ratified in a plebiscite on February 2, 1987. By that date, therefore, the
Provisional Constitution must be deemed to have been superseded. Having become inoperative,
respondent OIC Governor could no longer rely on Section 2, Article III, thereof to designate respondents
to the elective positions occupied by petitioners.

Petitioners must now be held to have acquired security of tenure specially considering that the Barangay
Election Act of 1982 declares it "a policy of the State to guarantee and promote the autonomy of the
barangays to ensure their fullest development as self-reliant communities. 2 Similarly, the 1987
Constitution ensures the autonomy of local governments and of political subdivisions of which the
barangays form a part, 3 and limits the President's power to "general supervision" over local
governments. 4 Relevantly, Section 8, Article X of the same 1987 Constitution further provides in part:

Sec. 8. The term of office of elective local officials, except barangay officials, which
shall be determined by law, shall be three years ...

Until the term of office of barangay officials has been determined by law, therefore, the term of office of
six (6) years provided for in the Barangay Election Act of 1982 5 should still govern.

Contrary to the stand of respondents, we find nothing inconsistent between the term of six (6) years for
elective Barangay officials and the 1987 Constitution, and the same should, therefore, be considered as
still operative, pursuant to Section 3, Article XVIII of the 1987 Constitution, reading:

Sec. 3. All existing laws, decrees, executive orders, proclamations letters of


instructions, and other executive issuances not inconsistent, with this Constitution
shall remain operative until amended, repealed or revoked.

WHEREFORE, (1) The Memoranda issued by respondent OIC Governor on February 8, 1987 designating
respondents as the Barangay Captain and Barangay Councilmen, respectively, of Barangay Dolores,
Taytay, Rizal, are both declared to be of no legal force and effect; and (2) the Writ of Prohibition is
granted enjoining respondents perpetually from proceeding with the ouster/take-over of petitioners'
positions subject of this Petition. Without costs.

SO ORDERED.
Republic of the Philippines district who shall have the same qualifications as those of Congressmen, to be elected on the second
SUPREME COURT Tuesday of November, 1970 in accordance with the Revised Election Code.
Manila
After the adoption of said Res. No. 2 in 1967 but before the November elections of that year, Congress,
EN BANC acting as a legislative body, enacted Republic Act No. 4914 implementing the aforesaid Resolution No. 2
and practically restating in toto the provisions of said Resolution No. 2.

On June 17, 1969, Congress, also acting as a Constituent Assembly, passed Resolution No. 4 amending
the aforesaid Resolution No. 2 of March 16, 1967 by providing that the convention "shall be composed of
G.R. No. L-32432 September 11, 1970
320 delegates apportioned among the existing representative districts according to the number of their
respective inhabitants: Provided, that a representative district shall be entitled to at least two delegates,
MANUEL B. IMBONG, petitioner, who shall have the same qualifications as those required of members of the House of
vs. Representatives," 1 "and that any other details relating to the specific apportionment of delegates,
JAIME FERRER, as Chairman of the Comelec, LINO M. PATAJO and CESAR MILAFLOR, as members election of delegates to, and the holding of, the Constitutional Convention shall be embodied in an
thereof, respondents. implementing legislation: Provided, that it shall not be inconsistent with the provisions of this
Resolution." 2
G.R. No. L-32443 September 11, 1970
On August 24, 1970, Congress, acting as a legislative body, enacted Republic Act No. 6132, implementing
IN THE MATTER OF A PETITION FOR DECLARATORY JUDGMENT REGARDING THE VALIDITY OF R.A. No. Resolutions Nos. 2 and 4, and expressly repealing R.A. No.
6132, OTHERWISE KNOWN AS THE CONSTITUTIONAL CONVENTION ACT OF 1970. RAUL M. 4914. 3
GONZALES,petitioner,
vs. Petitioner Raul M. Gonzales assails the validity of the entire law as well as the particular provisions
COMELEC, respondent. embodied in Sections 2, 4, 5, and par. 1 of 8(a). Petitioner Manuel B. Imbong impugns the
constitutionality of only par. I of Sec. 8(a) of said R.A. No. 6132 practically on the same grounds advanced
Manuel B. Imbong in his own behalf. by petitioner Gonzales.

Raul M. Gonzales in his own behalf. I

Office of the Solicitor General Felix Q. Antonio, Acting Assistant Solicitor General Ricardo L. Pronove, Jr., The validity of Sec. 4 of R.A. No. 6132, which considers, all public officers and employees, whether
and Solicitors Raul I. Goco, Bernardo P. Pardo, Rosalio A. de Leon, Vicente A. Torres and Guillermo C. elective or appointive, including members of the Armed Forces of the Philippines, as well as officers and
Nakar for respondents. employees of corporations or enterprises of the government, as resigned from the date of the filing of
their certificates of candidacy, was recently sustained by this Court, on the grounds, inter alia, that the
same is merely an application of and in consonance with the prohibition in Sec. 2 of Art. XII of the
Lorenzo Taada, Arturo Tolentino, Jovito Salonga and Emmanuel Pelaez as amici curiae. Constitution and that it does not constitute a denial of due process or of the equal protection of the law.
Likewise, the constitutionality of paragraph 2 of Sec. 8(a) of R.A. No. 6132 was upheld. 4

II
MAKASIAR, J.:
Without first considering the validity of its specific provisions, we sustain the constitutionality of the
These two separate but related petitions for declaratory relief were filed pursuant to Sec. 19 of R.A. No. enactment of R.A. No. 6132 by Congress acting as a legislative body in the exercise of its broad law-
6132 by petitioners Manuel B. Imbong and Raul M. Gonzales, both members of the Bar, taxpayers and making authority, and not as a Constituent Assembly, because
interested in running as candidates for delegates to the Constitutional Convention. Both impugn the
constitutionality of R.A. No. 6132, claiming during the oral argument that it prejudices their rights as 1. Congress, when acting as a Constituent Assembly pursuant to Art. XV of the
such candidates. After the Solicitor General had filed answers in behalf the respondents, hearings were Constitution, has full and plenary authority to propose Constitutional amendments
held at which the petitioners and the amici curiae, namely Senator Lorenzo Taada, Senator Arturo or to call a convention for the purpose, by a three-fourths vote of each House in
Tolentino, Senator Jovito Salonga, and Senator Emmanuel Pelaez argued orally. joint session assembled but voting separately. Resolutions Nos. 2 and 4 calling for
a constitutional convention were passed by the required three-fourths vote.
It will be recalled that on March 16, 1967, Congress, acting as a Constituent Assembly pursuant to Art. XV
of the Constitution, passed Resolution No. 2 which among others called for a Constitutional Convention 2. The grant to Congress as a Constituent Assembly of such plenary authority to
to propose constitutional amendments to be composed of two delegates from each representative call a constitutional convention includes, by virtue of the doctrine of necessary
implication, all other powers essential to the effective exercise of the principal to Congress, which, accordingly employed a formula for the necessary computation to effect the desired
power granted, such as the power to fix the qualifications, number, proportional representation.
apportionment, and compensation of the delegates as well as appropriation of
funds to meet the expenses for the election of delegates and for the operation of
The records of the proceedings on Senate Bill No. 77 sponsored by Senator Pelaez which is now R.A. No.
the Constitutional Convention itself, as well as all other implementing details
6132, submitted to this Tribunal by the amici curiae, show that it based its apportionment of the
indispensable to a fruitful convention. Resolutions Nos. 2 and 4 already embody
delegates on the 1970 official preliminary population census taken by the Bureau of Census and Statistics
the above-mentioned details, except the appropriation of funds.
from May 6 to June 30, 1976; and that Congress adopted the formula to effect a reasonable
apportionment of delegates. The Director of the Bureau of Census and Statistics himself, in a letter to
3. While the authority to call a constitutional convention is vested by the present Senator Pelaez dated July 30, 1970, stated that "on the basis of the preliminary count of the population,
Constitution solely and exclusively in Congress acting as a Constituent Assembly, we have computed the distribution of delegates to the Constitutional Convention based on Senate Bill 77
the power to enact the implementing details, which are now contained in (p. 2 lines 5 to 32 and p. 3 line 12) which is a fair and an equitable method of distributing the delegates
Resolutions Nos. 2 and 4 as well as in R.A. No. 6132, does not exclusively pertain to pursuant to the provisions of the joint Resolution of both Houses No. 2, as amended. Upon your request
Congress acting as a Constituent Assembly. Such implementing details are matters at the session of the Senate-House Conference Committee meeting last night, we are submitting
within the competence of Congress in the exercise of its comprehensive legislative herewith the results of the computation on the basis of the above-stated method."
power, which power encompasses all matters not expressly or by necessary
implication withdrawn or removed by the Constitution from the ambit of
Even if such latest census were a preliminary census, the same could still be a valid basis for such
legislative action. And as lone as such statutory details do not clash with any
apportionment.6 The fact that the lone and small congressional district of Batanes, may be over-
specific provision of the constitution, they are valid.
represented, because it is allotted two delegates by R.A. No. 6132 despite the fact that it has a
population very much less than several other congressional districts, each of which is also allotted only
4. Consequently, when Congress, acting as a Constituent Assembly, omits to two delegates, and therefore under-represented, vis-a-vis Batanes alone, does not vitiate the
provide for such implementing details after calling a constitutional convention, apportionment as not effecting proportional representation. Absolute proportional apportionment is not
Congress, acting as a legislative body, can enact the necessary implementing required and is not possible when based on the number of inhabitants, for the population census cannot
legislation to fill in the gaps, which authority is expressly recognized in Sec. 8 of be accurate nor complete, dependent as it is on the diligence of the census takers, aggravated by the
Res No. 2 as amended by Res. No. 4. constant movement of population, as well as daily death and birth. It is enough that the basis employed
is reasonable and the resulting apportionment is substantially proportional. Resolution No. 4 fixed a
minimum of two delegates for a congressional district.
5. The fact that a bill providing for such implementing details may be vetoed by the
President is no argument against conceding such power in Congress as a legislative
body nor present any difficulty; for it is not irremediable as Congress can override While there may be other formulas for a reasonable apportionment considering the evidence submitted
the Presidential veto or Congress can reconvene as a Constituent Assembly and to Congress by the Bureau of Census and Statistics, we are not prepared to rule that the computation
adopt a resolution prescribing the required implementing details. formula adopted by, Congress for proportional representation as, directed in Res. No. 4 is unreasonable
and that the apportionment provided in R.A. No. 6132 does not constitute a substantially proportional
representation.
III

In the Macias case, relied on by petitioner Gonzales, the apportionment law, which was nullified as
Petitioner Raul M. Gonzales asserts that Sec. 2 on the apportionment of delegates is not in accordance
unconstitutional, granted more representatives to a province with less population than the provinces
with proportional representation and therefore violates the Constitution and the intent of the law itself,
with more inhabitants. Such is not the case here, where under Sec. 2 of R.A. No. 6132 Batanes is allotted
without pinpointing any specific provision of the Constitution with which it collides.
only two delegates, which number is equal to the number of delegates accorded other provinces with
more population. The present petitions therefore do not present facts which fit the mould of the
Unlike in the apportionment of representative districts, the Constitution does not expressly or impliedly doctrine in the case of Macias et al. vs. Comelec, supra.
require such apportionment of delegates to the convention on the basis of population in each
congressional district. Congress, sitting as a Constituent Assembly, may constitutionally allocate one
The impossibility of absolute proportional representation is recognized by the Constitution itself when it
delegate for, each congressional district or for each province, for reasons of economy and to avoid
directs that the apportionment of congressional districts among the various provinces shall be "as nearly
having an unwieldy convention. If the framers of the present Constitution wanted the apportionment of
as may be according to their respective inhabitants, but each province shall have at least one member"
delegates to the convention to be based on the number of inhabitants in each representative district,
(Sec. 5, Art. VI, Phil. Const., emphasis supplied). The employment of the phrase "as nearly as may be
they would have done so in so many words as they did in relation to the apportionment of the
according to their respective inhabitants" emphasizes the fact that the human mind can only
representative districts. 5
approximate a reasonable apportionment but cannot effect an absolutely proportional representation
with mathematical precision or exactitude.
The apportionment provided for in Sec. 2 of R.A. No. 6132 cannot possibly conflict with its own intent
expressed therein; for it merely obeyed and implemented the intent of Congress acting as a Constituent
IV
Assembly expressed in Sec. 1 of Res. No. 4, which provides that the 320 delegates should be apportioned
among the existing representative districts according to the number of their respective inhabitants, but
fixing a minimum of at least two delegates for a representative district. The presumption is that the Sec. 5 of R.A. 6132 is attacked on the ground that it is an undue deprivation of liberty without due
factual predicate, the latest available official population census, for such apportionment was presented process of law and denies the equal protection of the laws. Said Sec. 5 disqualifies any elected delegate
from running "for any public office in any election" or from assuming "any appointive office or position in As heretofore intimated, the inhibition is relevant to the object of the law, which is to insure that the
any branch of the government government until after the final adjournment of the Constitutional proposed amendments are meaningful to the masses of our people and not designed for the
Convention." enhancement of selfishness, greed, corruption, or injustice.

That the citizen does not have any inherent nor natural right to a public office, is axiomatic under our Lastly, the disqualification applies to all the delegates to the convention who will be elected on the
constitutional system. The State through its Constitution or legislative body, can create an office and second Tuesday of November, 1970.
define the qualifications and disqualifications therefor as well as impose inhibitions on a public officer.
Consequently, only those with qualifications and who do not fall under any constitutional or statutory
V
inhibition can be validly elected or appointed to a public office. The obvious reason for the questioned
inhibition, is to immunize the delegates from the perverting influence of self-interest, party interest or
vested interest and to insure that he dedicates all his time to performing solely in the interest of the Paragraph 1, Sec. 8(a) of R.A. No. 6132 is impugned by both petitioners as violative of the constitutional
nation his high and well nigh sacred function of formulating the supreme law of the land, which may guarantees of due process, equal protection of the laws, freedom of expressions, freedom of assembly
endure for generations and which cannot easily be changed like an ordinary statute. With the and freedom of association.
disqualification embodied in Sec. 5, the delegate will not utilize his position as a bargaining leverage for
concessions in the form of an elective or appointive office as long as the convention has not finally This Court ruled last year that the guarantees of due process, equal protection of the laws, peaceful
adjourned. The appointing authority may, by his appointing power, entice votes for his own proposals. assembly, free expression, and the right of association are neither absolute nor illimitable rights; they are
Not love for self, but love for country must always motivate his actuations as delegate; otherwise the always subject to the pervasive and dormant police power of the State and may be lawfully abridged to
several provisions of the new Constitution may only satisfy individual or special interests, subversive of serve appropriate and important public interests. 8
the welfare of the general citizenry. It should be stressed that the disqualification is not permanent but
only temporary only to continue until the final adjournment of the convention which may not extend
beyond one year. The convention that framed the present Constitution finished its task in approximately In said Gonzalez vs. Comelec case the Court applied the clear and present danger test to determine
seven months from July 30, 1934 to February 8, 1935. whether a statute which trenches upon the aforesaid Constitutional guarantees, is a legitimate exercise
of police power. 9

As admitted by petitioner Gonzales, this inhibition finds analogy in the constitutional provision
prohibiting a member of Congress, during the time for which he was elected, from being appointed to Paragraph 1 of Sec. 8(a), R.A. No. 6132 prohibits:
any civil office which may have been created or the emolument whereof shall have been increased while
he was a member of the Congress. (Sec. 16, Art. VI, Phil. Constitution.) 1. any candidate for delegate to the convention

As observed by the Solicitor General in his Answer, the overriding objective of the challenged (a) from representing, or
disqualification, temporary in nature, is to compel the elected delegates to serve in full their term as
such and to devote all their time to the convention, pursuant to their representation and commitment to
the people; otherwise, his seat in the convention will be vacant and his constituents will be deprived of a (b) allowing himself to be represented as being a candidate
voice in the convention. The inhibition is likewise "designed to prevent popular political figures from of any political party or any other organization; and
controlling elections or positions. Also it is a brake on the appointing power, to curtail the latter's desire
to 'raid' the convention of "talents" or attempt to control the convention." (p. 10, Answer in L-32443.) 2. any political party, political group, political committee, civic, religious,
professional or other organizations or organized group of whatever nature from
Thus the challenged disqualification prescribed in Sec. 5 of R.A. No. 6132 is a valid limitation on the right
to public office pursuant to state police power as it is reasonable and not arbitrary. (a) intervening in the nomination of any such candidate or in
the filing of his certificate, or
The discrimination under Sec. 5 against delegates to the Constitutional Convention is likewise
constitutional; for it is based on a substantial distinction which makes for real differences, is germane to (b) from giving aid or support directly or indirectly, material
the purposes of the law, and applies to all members of the same class. 7 The function of a delegate is or otherwise, favorable to or against his campaign for
more far-reaching and its effect more enduring than that of any ordinary legislator or any other public election.
officer. A delegate shapes the fundamental law of the land which delineates the essential nature of the
government, its basic organization and powers, defines the liberties of the people, and controls all other
The ban against all political parties or organized groups of whatever nature contained in par. 1 of Sec.
laws. Unlike ordinary statutes, constitutional amendments cannot be changed in one or two years. No
8(a), is confined to party or organization support or assistance, whether material, moral, emotional or
other public officer possesses such a power, not even the members of Congress unless they themselves,
otherwise. The very Sec. 8(a) in its provisos permits the candidate to utilize in his campaign the help of
propose constitutional amendments when acting as a Constituent Assembly pursuant to Art. XV of the
the members of his family within the fourth civil degree of consanguinity or affinity, and a campaign staff
Constitution. The classification, therefore, is neither whimsical nor repugnant to the sense of justice of
composed of not more than one for every ten precincts in his district. It allows the full exercise of his
the community.
freedom of expression and his right to peaceful assembly, because he cannot be denied any permit to
hold a public meeting on the pretext that the provision of said section may or will be violated. The right
of a member of any political party or association to support him or oppose his opponent is preserved as
long as such member acts individually. The very party or organization to which he may belong or which The debasement of the electoral process as a substantive evil exists today and is one of the major
may be in sympathy with his cause or program of reforms, is guaranteed the right to disseminate compelling interests that moved Congress into prescribing the total ban contained in par. 1 of Sec. 8(a)
information about, or to arouse public interest in, or to advocate for constitutional reforms, programs, of R.A. No. 6132, to justify such ban. In the said Gonzales vs. Comelec case, this Court gave "due
policies or constitutional proposals for amendments. recognition to the legislative concern to cleanse, and if possible, render spotless, the electoral
process," 14 impressed as it was by the explanation made by the author of R.A. No. 4880, Sen. Lorenzo
Taada, who appeared as amicus curiae, "that such provisions were deemed by the legislative body to be
It is therefore patent that the restriction contained in Sec. 8(a) is so narrow that the basic constitutional
part and parcel of the necessary and appropriate response not merely to a clear and present danger but
rights themselves remain substantially intact and inviolate. And it is therefore a valid infringement of the
to the actual existence of a grave and substantive evil of excessive partisanship, dishonesty and
aforesaid constitutional guarantees invoked by petitioners.
corruption as well as violence that of late has marred election campaigns and partisan political activities
in this country. He did invite our attention likewise to the well-settled doctrine that in the choice of
In the aforesaid case of Gonzales vs. Comelec, supra, this Court unanimously sustained the validity of the remedies for an admitted malady requiring governmental action, on the legislature primarily rests the
limitation on the period for nomination of candidates in Sec. 50-A of R.A. No. 4880, thus: responsibility. Nor should the cure prescribed by it, unless clearly repugnant to fundamental rights, be
ignored or disregarded." 15
The prohibition of too early nomination of candidates presents a question that is
not too formidable in character. According to the act: "It shall be unlawful for any But aside from the clear and imminent danger of the debasement of the electoral process, as conceded
political party, political committee, or political group to nominate candidates for by Senator Pelaez, the basic motivation, according to Senate Majority Floor Leader Senator Arturo
any elective public office voted for at large earlier than one hundred and fifty days Tolentino, the sponsor of the Puyat-Tolentino amendment embodied in par. 1 of Sec. 8(a) of R.A. No.
immediately preceding an election, and for any other elective public office earlier 6132, is to assure the candidates equal protection of the laws by according them equality of
than ninety days immediately preceding an election. chances. 16 The primary purpose of the prohibition then is also to avert the clear and present danger of
another substantive evil, the denial of the equal protection of the laws. The candidates must depend on
The right of association is affected. Political parties have less freedom as to the their individual merits and not on the support of political parties or organizations. Senator Tolentino and
time during which they may nominate candidates; the curtailment is not such, Senator Salonga emphasized that under this provision, the poor candidate has an even chance as against
however, as to render meaningless such a basic right. Their scope of legitimate the rich candidate. We are not prepared to disagree with them, because such a conclusion, predicated as
activities, save this one, is not unduly narrowed. Neither is there infringement of it is on empirical logic, finds support in our recent political history and experience. Both Senators
their freedom to assemble. They can do so, but not for such a purpose. We sustain stressed that the independent candidate who wins in the election against a candidate of the major
its validity. We do so unanimously. 10 political parties, is a rare phenomenon in this country and the victory of an independent candidate
mainly rests on his ability to match the resources, financial and otherwise, of the political parties or
organizations supporting his opponent. This position is further strengthened by the principle that the
In said Gonzales vs. Comelec case, this Court likewise held that the period for the conduct of an election guarantee of social justice under Sec. V, Art. II of the Constitution, includes the guarantee of equal
campaign or partisan political activity may be limited without offending the aforementioned opportunity, equality of political rights, and equality before the law enunciated by Mr. Justice Tuazon in
constitutional guarantees as the same is designed also to prevent a "clear and present danger of a the case Guido vs. Rural Progress Administration. 17
substantive evil, the debasement of the electoral process." 11

While it may be true that a party's support of a candidate is not wrong per se it is equally true that
Even if the partisan activity consists of (a) forming organizations, associations, clubs, committees or Congress in the exercise of its broad law-making authority can declare certain acts as mala prohibita
other group of persons for the purpose of soliciting votes and/or undertaking any campaign or when justified by the exigencies of the times. One such act is the party or organization support
propaganda for or against a party or candidate; (b) holding political conventions, caucuses, conferences, proscribed in Sec. 8(a),which ban is a valid limitation on the freedom of association as well as expression,
meetings, rallies, parades or other similar assemblies for the purpose of soliciting votes and/or for the reasons aforestated.
undertaking any campaign or propaganda for or against any candidate or party; and (c) giving, soliciting,
or receiving contributions for election campaign either directly or indirectly, (Sec. 50-B, pars. (a), (b), and
(c), R.A. 4880), the abridgment was still affirmed as constitutional by six members of this Court, which Senator Tolentino emphasized that "equality of chances may be better attained by banning all
could not "ignore ... the legislative declaration that its enactment was in response to a serious organization support." 18
substantive evil affecting the electoral process, not merely in danger of happening, but actually in
existence, and likely to continue unless curbed or remedied. To assert otherwise would be to close one's The questioned par. 1 of Sec. 8 (a) likewise can easily pass the balancing-of-interest test. 19
eyes to the reality of the situation." 12;
In the apt words of the Solicitor General:
Likewise, because four members dissented, this Court in said case of Gonzales vs. Comelec, supra, failed
to muster the required eight votes to declare as unconstitutional the limitation on the period for (a)
It is to be noted that right now the nation is on the threshold of rewriting its
making speeches, announcements or commentaries or holding interviews for or against the election of
Constitution in a hopeful endeavor to find a solution to the grave economic, social
any party or candidate for public office; (b) publishing or distributing campaign literature or materials;
and political problems besetting the country. Instead of directly proposing the
and (e) directly or indirectly soliciting votes and/or undertaking any campaign or propaganda for or
amendments Congress has chosen to call a Constitutional Convention which shall
against any candidate or party specified in Sec. 50-B, pars. (c), (d) & (e) of R.A. 4880. 13
have the task of fashioning a document that shall embody the aspirations and
ideals of the people. Because what is to be amended is the fundamental law of the
land, it is indispensable that the Constitutional Convention be composed of
delegates truly representative of the people's will. Public welfare demands that Constitution and by laws of such civic, religious, or professional associations usually prohibit the
the delegates should speak for the entire nation, and their voices be not those of a association from engaging in partisan political activity or supporting any candidate for an elective office.
particular segment of the citizenry, or of a particular class or group of people, be Hence, they must likewise respect the ban.
they religious, political, civic or professional in character. Senator Pelaez, Chairman
of the Senate Committee on Codes and Constitutional Amendments, eloquently
The freedom of association also implies the liberty not to associate or join with others or join any existing
stated that "the function of a constitution is not to represent anyone in interest or
organization. A person may run independently on his own merits without need of catering to a political
set of interests, not to favor one group at the expense or disadvantage of the
party or any other association for support. And he, as much as the candidate whose candidacy does not
candidates but to encompass all the interests that exist within our society and
evoke sympathy from any political party or organized group, must be afforded equal chances. As
to blend them into one harmonious and balanced whole. For the constitutional
emphasized by Senators Tolentino and Salonga, this ban is to assure equal chances to a candidate with
system means, not the predominance of interests, but the harmonious balancing
talent and imbued with patriotism as well as nobility of purpose, so that the country can utilize their
thereof."
services if elected.

So that the purpose for calling the Constitutional Convention will not be deflated
Impressed as We are by the eloquent and masterly exposition of Senator Taada for the invalidation of
or frustrated, it is necessary that the delegatee thereto be independent, beholden
par. 1 of Sec. 8(a) of R.A. No. 6132, demonstrating once again his deep concern for the preservation of
to no one but to God, country and conscience.
our civil liberties enshrined in the Bill of Rights, We are not persuaded to entertain the belief that the
challenged ban transcends the limits of constitutional invasion of such cherished immunities.
xxx xxx xxx
WHEREFORE, the prayers in both petitions are hereby denied and R.A. No. 6132 including Secs. 2, 4, 5,
The evil therefore, which the law seeks to prevent lies in the election of delegates and 8(a), paragraph 1, thereof, cannot be declared unconstitutional. Without costs.
who, because they have been chosen with the aid and resources of organizations,
cannot be expected to be sufficiently representative of the people. Such delegates
Reyes, J.B.L., Dizon and Castro, JJ., concur.
could very well be the spokesmen of narrow political, religious or economic
interest and not of the great majority of the people. 20
Makalintal, J., concurs in the result.
We likewise concur with the Solicitor General that the equal protection of the laws is not unduly
subverted in par. I of Sec. 8(a); because it does not create any hostile discrimination against any party or
group nor does it confer undue favor or privilege on an individual as heretofore stated. The
discrimination applies to all organizations, whether political parties or social, civic, religious, or
professional associations. The ban is germane to the objectives of the law, which are to avert the
debasement of the electoral process, and to attain real equality of chances among individual candidates
and thereby make real the guarantee of equal protection of the laws.

The political parties and the other organized groups have built-in advantages because of their machinery
and other facilities, which, the individual candidate who is without any organization support, does not
have. The fact that the other civic of religious organizations cannot have a campaign machinery as
efficient as that of a political party, does not vary the situation; because it still has that much built-in
advantage as against the individual candidate without similar support. Moreover, these civic religious
and professional organization may band together to support common candidates, who advocates the
reforms that these organizations champion and believe are imperative. This is admitted by petitioner
Gonzales thru the letter of Senator Ganzon dated August 17, 1970 attached to his petition as Annex "D",
wherein the Senator stated that his own "Timawa" group had agreed with the Liberal Party in Iloilo to
support petitioner Gonzales and two others as their candidates for the convention, which organized
support is nullified by the questioned ban, Senator Ganzon stressed that "without the group moving and
working in joint collective effort" they cannot "exercise effective control and supervision over our
leaders the Women's League, the area commanders, etc."; but with their joining with the LP's they
"could have presented a solid front with very bright chances of capturing all seats."

The civic associations other than political parties cannot with reason insist that they should be exempted
from the ban; because then by such exemption they would be free to utilize the facilities of the
campaign machineries which they are denying to the political parties. Whenever all organization engages
in a political activity, as in this campaign for election of delegates to the Constitutional Convention, to
that extent it partakes of the nature of a political organization. This, despite the fact that the
Republic of the Philippines interests would be adequately protected already, the Court had to limit the number of intervenors from
SUPREME COURT the ranks of the delegates to the Convention who, more or less, have legal interest in the success of the
Manila respondents, and so, only Delegates Raul S. Manglapus, Jesus G. Barrera, Pablo S. Trillana III, Victor de la
Serna, Marcelo B. Fernan, Jose Y. Feria, Leonardo Siguion Reyna, Victor Ortega and Juan B. Borra, all
distinguished lawyers in their own right, have been allowed to intervene jointly. The Court feels that with
EN BANC
such an array of brilliant and dedicated counsel, all interests involved should be duly and amply
represented and protected. At any rate, notwithstanding that their corresponding motions for leave to
G.R. No. L-34150 October 16, 1971 intervene or to appear as amicus curiae 1 have been denied, the pleadings filed by the other delegates
and some private parties, the latter in representation of their minor children allegedly to be affected by
ARTURO M. TOLENTINO, petitioner, the result of this case with the records and the Court acknowledges that they have not been without
vs. value as materials in the extensive study that has been undertaken in this case.
COMMISSION ON ELECTIONS, and THE CHIEF ACCOUNTANT, THE AUDITOR, and THE DISBURSING
OFFICER OF THE 1971 CONSTITUTIONAL CONVENTION, respondents, RAUL S. MANGLAPUS, JESUS G. The background facts are beyond dispute. The Constitutional Convention of 1971 came into being by
BARRERA, PABLO S. TRILLANA III, VICTOR DE LA SERNA, MARCELO B. FERNAN, JOSE Y. FERIA, virtue of two resolutions of the Congress of the Philippines approved in its capacity as a constituent
LEONARDO SIGUION REYNA, VICTOR F. ORTEGA, and JUAN V. BORRA, Intervenors. assembly convened for the purpose of calling a convention to propose amendments to the Constitution
namely, Resolutions 2 and 4 of the joint sessions of Congress held on March 16, 1967 and June 17, 1969
Arturo M. Tolentino in his own behalf. respectively. The delegates to the said Convention were all elected under and by virtue of said
resolutions and the implementing legislation thereof, Republic Act 6132. The pertinent portions of
Resolution No 2 read as follows:
Ramon A. Gonzales for respondents Chief Accountant and Auditor of the 1971 Constitutional Convention.

SECTION 1. There is hereby called a convention to propose amendments to the


Emmanuel Pelaez, Jorge M. Juco and Tomas L. Echivarre for respondent Disbursing Officer of the 1971 Constitution of the Philippines, to be composed of two elective Delegates from
Constitutional Convention. each representative district who shall have the same qualifications as those
required of Members of the House of Representatives.
Intervenors in their own behalf.
xxx xxx xxx

SECTION 7. The amendments proposed by the Convention shall be valid and


BARREDO, J.: considered part of the Constitution when approved by a majority of the votes cast
in an election at which they are submitted to the people for their ratification
pursuant to Article XV of the Constitution.
Petition for prohibition principally to restrain the respondent Commission on Elections "from
undertaking to hold a plebiscite on November 8, 1971," at which the proposed constitutional
amendment "reducing the voting age" in Section 1 of Article V of the Constitution of the Philippines to Resolution No. 4 merely modified the number of delegates to represent the different cities and
eighteen years "shall be, submitted" for ratification by the people pursuant to Organic Resolution No. 1 provinces fixed originally in Resolution No 2.
of the Constitutional Convention of 1971, and the subsequent implementing resolutions, by declaring
said resolutions to be without the force and effect of law in so far as they direct the holding of such After the election of the delegates held on November 10, 1970, the Convention held its inaugural session
plebiscite and by also declaring the acts of the respondent Commission (COMELEC) performed and to be on June 1, 1971. Its preliminary labors of election of officers, organization of committees and other
done by it in obedience to the aforesaid Convention resolutions to be null and void, for being violative of preparatory works over, as its first formal proposal to amend the Constitution, its session which began
the Constitution of the Philippines. on September 27, 1971, or more accurately, at about 3:30 in the morning of September 28, 1971, the
Convention approved Organic Resolution No. 1 reading thus: .
As a preliminary step, since the petition named as respondent only the COMELEC, the Count required
that copies thereof be served on the Solicitor General and the Constitutional Convention, through its CC ORGANIC RESOLUTION NO. 1
President, for such action as they may deem proper to take. In due time, respondent COMELEC filed its
answer joining issues with petitioner. To further put things in proper order, and considering that the
fiscal officers of the Convention are indispensable parties in a proceeding of this nature, since the acts A RESOLUTION AMENDING SECTION ONE OF ARTICLE V OF THE CONSTITUTION OF
sought to be enjoined involve the expenditure of funds appropriated by law for the Convention, the THE PHILIPPINES SO AS TO LOWER THE VOTING AGE TO 18
Court also ordered that the Disbursing Officer, Chief Accountant and Auditor of the Convention be made
respondents. After the petition was so amended, the first appeared thru Senator Emmanuel Pelaez and BE IT RESOLVED as it is hereby resolved by the 1971 Constitutional Convention:
the last two thru Delegate Ramon Gonzales. All said respondents, thru counsel, resist petitioner's action.
Section 1. Section One of Article V of the Constitution of the Philippines is
For reasons of orderliness and to avoid unnecessary duplication of arguments and even possible amended to as follows:
confusion, and considering that with the principal parties being duly represented by able counsel, their
Section 1. Suffrage may be exercised by (male) citizens of On September 30, 1971, COMELEC "RESOLVED to inform the Constitutional Convention that it will hold
the Philippines not otherwise disqualified by law, who are the plebiscite on condition that:
(twenty-one) EIGHTEEN years or over and are able to read
and write, and who shall have resided in the Philippines for
(a) The Constitutional Convention will undertake the printing of separate official
one year and in the municipality wherein they propose to
ballots, election returns and tally sheets for the use of said plebiscite at its
vote for at least six months preceding the election.
expense;

Section 2. This amendment shall be valid as part of the Constitution of the


(b) The Constitutional Convention will adopt its own security measures for the
Philippines when approved by a majority of the votes cast in a plebiscite to
printing and shipment of said ballots and election forms; and
coincide with the local elections in November 1971.

(c) Said official ballots and election forms will be delivered to the Commission in
Section 3. This partial amendment, which refers only to the age qualification for
time so that they could be distributed at the same time that the Commission will
the exercise of suffrage shall be without prejudice to other amendments that will
distribute its official and sample ballots to be used in the elections on November 8,
be proposed in the future by the 1971 Constitutional Convention on other portions
1971.
of the amended Section or on other portions of the entire Constitution.

What happened afterwards may best be stated by quoting from intervenors' Governors' statement of
Section 4. The Convention hereby authorizes the use of the sum of P75,000.00
the genesis of the above proposal:
from its savings or from its unexpended funds for the expense of the advanced
plebiscite; provided, however that should there be no savings or unexpended
sums, the Delegates waive P250.00 each or the equivalent of 2-1/2 days per diem. The President of the Convention also issued an order forming an Ad Hoc
Committee to implement the Resolution.
By a letter dated September 28, 1971, President Diosdado Macapagal, called upon respondent Comelec
"to help the Convention implement (the above) resolution." The said letter reads: This Committee issued implementing guidelines which were approved by the
President who then transmitted them to the Commission on Elections.
September 28, 1971
The Committee on Plebiscite and Ratification filed a report on the progress of the
implementation of the plebiscite in the afternoon of October 7,1971, enclosing
The Commission on Elections Manila
copies of the order, resolution and letters of transmittal above referred to (Copy of
the report is hereto attached as Annex 8-Memorandum).
Thru the Chairman
RECESS RESOLUTION
Gentlemen:
In its plenary session in the evening of October 7, 1971, the Convention approved
Last night the Constitutional Convention passed Resolution No. 1 quoted as a resolution authored by Delegate Antonio Olmedo of Davao Oriental, calling for a
follows: recess of the Convention from November 1, 1971 to November 9, 1971 to permit
the delegates to campaign for the ratification of Organic Resolution No. 1. (Copies
of the resolution and the transcript of debate thereon are hereto attached as
xxx xxx xxx
Annexes 9 and 9-A Memorandum, respectively).

(see above)
RESOLUTION CONFIRMING IMPLEMENTATION

Pursuant to the provision of Section 14, Republic Act No. 6132 otherwise known as
On October 12, 1971, the Convention passed Resolution No. 24 submitted by
the Constitutional Convention Act of 1971, may we call upon you to help the
Delegate Jose Ozamiz confirming the authority of the President of the Convention
Convention implement this resolution:
to implement Organic Resolution No. 1, including the creation of the Ad Hoc
Committee ratifying all acts performed in connection with said implementation.
Sincerely,
Upon these facts, the main thrust of the petition is that Organic Resolution No. 1 and the other
. implementing resolutions thereof subsequently approved by the Convention have no force and effect as
laws in so far as they provide for the holding of a plebiscite co-incident with the elections of eight
senators and all city, provincial and municipal officials to be held on November 8, 1971, hence all of
Comelec's acts in obedience thereof and tending to carry out the holding of the plebiscite directed by
said resolutions are null and void, on the ground that the calling and holding of such a plebiscite is, by largest number of votes in said chamber, purporting to act, on behalf of the party
the Constitution, a power lodged exclusively in Congress, as a legislative body, and may not be exercised having the second largest number of votes therein of two (2) Senators belonging
by the Convention, and that, under Section 1, Article XV of the Constitution, the proposed amendment in to the first party, as members, for the second party, of the Senate Electoral
question cannot be presented to the people for ratification separately from each and all of the other Tribunal; and in the fourth, we declared unconstitutional an act of Congress
amendments to be drafted and proposed by the Convention. On the other hand, respondents and purporting to apportion the representatives districts for the House of
intervenors posit that the power to provide for, fix the date and lay down the details of the plebiscite for Representatives, upon the ground that the apportionment had not been made as
the ratification of any amendment the Convention may deem proper to propose is within the authority may be possible according to the number of inhabitants of each province. Thus we
of the Convention as a necessary consequence and part of its power to propose amendments and that rejected the theory, advanced in these four (4) cases that the issues therein raised
this power includes that of submitting such amendments either individually or jointly at such time and were political questions the determination of which is beyond judicial review.
manner as the Convention may direct in discretion. The Court's delicate task now is to decide which of
these two poses is really in accord with the letter and spirit of the Constitution.
Indeed, the power to amend the Constitution or to propose amendments thereto
is not included in the general grant of legislative powers to Congress (Section 1,
As a preliminary and prejudicial matter, the intervenors raise the question of jurisdiction. They contend Art. VI, Constitution of the Philippines). It is part of the inherent powers of the
that the issue before Us is a political question and that the Convention being legislative body of the people as the repository sovereignty in a republican state, such as ours (Section
highest order is sovereign, and as such, its acts impugned by petitioner are beyond the control of the 1, Art. 11, Constitution of the Philippines) to make, and, hence, to amend their
Congress and the courts. In this connection, it is to be noted that none of the respondent has joined own Fundamental Law. Congress may propose amendments to the Constitution
intervenors in this posture. In fact, respondents Chief Accountant and Auditor of the convention merely because the same explicitly grants such power. (Section 1, Art. XV,
expressly concede the jurisdiction of this Court in their answer acknowledging that the issue herein is a Constitution of the Philippines) Hence, when exercising the same, it is said that
justifiable one. Senators and members of the House of Representatives act, not as members of
Congress, but as component elements of a constituent assembly. When acting as
such, the members of Congress derive their authority from the Constitution, unlike
Strangely, intervenors cite in support of this contention portions of the decision of this Court in the case
the people, when performing the same function, (Of amending the Constitution)
of Gonzales v. Comelec, 21 SCRA 774, wherein the members of the Court, despite their being divided in
for their authority does not emanate from the Constitution they are the very
their opinions as to the other matters therein involved, were precisely unanimous in upholding its
source of all powers of government including the Constitution itself.
jurisdiction. Obviously, distinguished counsel have either failed to grasp the full impact of the portions of
Our decision they have quoted or would misapply them by taking them out of context.
Since, when proposing, as a constituent assembly, amendments to the
Constitution, the members of Congress derive their authority from the
There should be no more doubt as to the position of this Court regarding its jurisdiction vis-a-vis the
Fundamental Law, it follows, necessarily, that they do not have the final say on
constitutionality of the acts of the Congress, acting as a constituent assembly, and, for that matter, those
whether or not their acts are within or beyond constitutional limits. Otherwise,
of a constitutional convention called for the purpose of proposing amendments to the Constitution,
they could brush aside and set the same at naught, contrary to the basic tenet that
which concededly is at par with the former. A simple reading of Our ruling in that very case
ours is a government of laws, not of men, and to the rigid nature of our
of Gonzales relied upon by intervenors should dispel any lingering misgivings as regards that point.
Constitution. Such rigidity is stressed by the fact that the Constitution expressly
Succinctly but comprehensively, Chief Justice Concepcion held for the Court thus: .
confers upon the Supreme Court, (And, inferentially, to lower courts.) the power
to declare a treaty unconstitutional. (Sec. 2(1), Art. VIII of the Constitution),
As early as Angara vs. Electoral Commission (63 Phil. 139, 157), this Court despite the eminently political character of treaty-making power.
speaking through one of the leading members of the Constitutional Convention
and a respected professor of Constitutional Law, Dr. Jose P. Laurel declared that
In short, the issue whether or not a Resolution of Congress acting as a
"the judicial department is the only constitutional organ which can be called upon
constituent assembly violates the Constitution is essentially justiciable not
to determine the proper allocation of powers between the several departments
political, and, hence, subject to judicial review, and, to the extent that this view
and among the integral or constituent units thereof."
may be inconsistent with the stand taken in Mabanag v. Lopez Vito, (supra) the
latter should be deemed modified accordingly. The Members of the Court are
It is true that in Mabanag v. Lopez Vito (supra), this Court characterizing the issue unanimous on this point.
submitted thereto as a political one declined to pass upon the question whether or
not a given number of votes cast in Congress in favor of a proposed amendment to
No one can rightly claim that within the domain of its legitimate authority, the Convention is not
the Constitution which was being submitted to the people for ratification
supreme. Nowhere in his petition and in his oral argument and memoranda does petitioner point
satisfied the three-fourths vote requirement of the fundamental law. The force of
otherwise. Actually, what respondents and intervenors are seemingly reluctant to admit is that the
this precedent has been weakened, however, by Suanes v. Chief Accountant of the
Constitutional Convention of 1971, as any other convention of the same nature, owes its existence and
Senate (81 Phil. 818), Avelino v. Cuenco, (L-2851, March 4 & 14, 1949), Taada v.
derives all its authority and power from the existing Constitution of the Philippines. This Convention has
Cuenco, (L-10520, Feb. 28, 1957) and Macias v. Commission on Elections, (L-18684,
not been called by the people directly as in the case of a revolutionary convention which drafts the first
Sept. 14, 1961). In the first we held that the officers and employees of the Senate
Constitution of an entirely new government born of either a war of liberation from a mother country or
Electoral Tribunal are under its supervision and control, not of that of the Senate
of a revolution against an existing government or of a bloodless seizure of power a la coup d'etat. As to
President, as claimed by the latter; in the second, this Court proceeded to
such kind of conventions, it is absolutely true that the convention is completely without restrain and
determine the number of Senators necessary for quorum in the Senate; in the
omnipotent all wise, and it is as to such conventions that the remarks of Delegate Manuel Roxas of the
third, we nullified the election, by Senators belonging to the party having the
Constitutional Convention of 1934 quoted by Senator Pelaez refer. No amount of rationalization can say where the one leaves off and the other begins. In times of social disquietude or
belie the fact that the current convention came into being only because it was called by a resolution of a political excitement, the great landmark of the Constitution are apt to be forgotten
joint session of Congress acting as a constituent assembly by authority of Section 1, Article XV of the or marred, if not entirely obliterated. In cases of conflict, the judicial department is
present Constitution which provides: the only constitutional organ which can be called upon to determine the proper
allocation of powers between the several departments and among the integral or
constituent units thereof.
ARTICLE XV AMENDMENTS

As any human production our Constitution is of course lacking perfection and


SECTION 1. The Congress in joint session assembled, by a vote of three-fourths of
perfectibility, but as much as it was within the power of our people, acting through
all the Members of the Senate and of the House of Representatives voting
their delegates to so provide, that instrument which is the expression of their
separately, may propose amendments to this Constitution or call a convention for
sovereignty however limited, has established a republican government intended to
the purpose. Such amendments shall be valid as part of this Constitution when
operate and function as a harmonious whole, under a system of check and
approved by a majority of the votes cast at an election at which the amendments
balances and subject to specific limitations and restrictions provided in the said
are submitted to the people for their ratification.
instrument. The Constitution sets forth in no uncertain language the restrictions
and limitations upon governmental powers and agencies. If these restrictions and
True it is that once convened, this Convention became endowed with extra ordinary powers generally limitations are transcended it would be inconceivable if the Constitution had not
beyond the control of any department of the existing government, but the compass of such powers can provided for a mechanism by which to direct the course of government along
be co-extensive only with the purpose for which the convention was called and as it may propose cannot constitutional channels, for then the distribution of powers would be mere
have any effect as part of the Constitution until the same are duly ratified by the people, it necessarily verbiage, the bill of rights mere expressions of sentiment and the principles of
follows that the acts of convention, its officers and members are not immune from attack on good government mere political apothegms. Certainly the limitations and
constitutional grounds. The present Constitution is in full force and effect in its entirety and in everyone restrictions embodied in our Constitution are real as they should be in any living
of its parts the existence of the Convention notwithstanding, and operates even within the walls of that Constitution. In the United States where no express constitutional grant is found in
assembly. While it is indubitable that in its internal operation and the performance of its task to propose their constitution, the possession of this moderating power of the courts, not to
amendments to the Constitution it is not subject to any degree of restraint or control by any other speak of its historical origin and development there, has been set at rest by
authority than itself, it is equally beyond cavil that neither the Convention nor any of its officers or popular acquiescence for a period of more than one and half centuries. In our
members can rightfully deprive any person of life, liberty or property without due process of law, deny to case, this moderating power is granted, if not expressly, by clear implication from
anyone in this country the equal protection of the laws or the freedom of speech and of the press in section 2 of Article VIII of our Constitution.
disregard of the Bill of Rights of the existing Constitution. Nor, for that matter, can such Convention
validly pass any resolution providing for the taking of private property without just compensation or for
The Constitution is a definition of the powers or government. Who is to determine
the imposition or exacting of any tax, impost or assessment, or declare war or call the Congress to a
the nature, scope and extent of such powers? The Constitution itself has provided
special session, suspend the privilege of the writ of habeas corpus, pardon a convict or render judgment
for the instrumentality of the judiciary as the rational way. And when the judiciary
in a controversy between private individuals or between such individuals and the state, in violation of
mediates to allocate constitutional boundaries, it does not assert any superiority
the distribution of powers in the Constitution.
over the other departments; it does not in reality nullify or invalidate an act of the
legislature, but only asserts the solemn and sacred obligation assigned to it by the
It being manifest that there are powers which the Convention may not and cannot validly assert, much Constitution to determine conflicting claims of authority under the Constitution
less exercise, in the light of the existing Constitution, the simple question arises, should an act of the and to establish for the parties in an actual controversy the rights which that
Convention be assailed by a citizen as being among those not granted to or inherent in it, according to instrument secures and guarantees to them. This is in truth all that is involved in
the existing Constitution, who can decide whether such a contention is correct or not? It is of the very what is termed "judicial supremacy" which properly is the power of judicial review
essence of the rule of law that somehow somewhere the Power and duty to resolve such a grave under the Constitution. Even then, this power of judicial review is limited to actual
constitutional question must be lodged on some authority, or we would have to confess that the cases and controversies to be exercised after full opportunity of argument by the
integrated system of government established by our founding fathers contains a wide vacuum no parties, and limited further to the constitutional question raised or the very lis
intelligent man could ignore, which is naturally unworthy of their learning, experience and craftsmanship mota presented. Any attempt at abstraction could only lead to dialectics and
in constitution-making. barren legal questions and to strike conclusions unrelated to actualities. Narrowed
as its functions is in this manner the judiciary does not pass upon questions of
We need not go far in search for the answer to the query We have posed. The very decision of Chief wisdom, justice or expediency of legislation. More than that, courts accord the
Justice Concepcion in Gonzales, so much invoked by intervenors, reiterates and reinforces the irrefutable presumption of constitutionality to legislative enactments, not only because the
logic and wealth of principle in the opinion written for a unanimous Court by Justice Laurel in Angara vs. legislature is presumed to abide by the Constitution but also because the judiciary
Electoral Commission, 63 Phil., 134, reading: in the determination of actual cases and controversies must reflect the wisdom
and justice of the people as expressed through their representatives in the
executive and legislative departments of the government.
... (I)n the main, the Constitution has blocked out with deft strokes and in bold
lines, allotment of power to the executive, the legislative and the judicial
departments of the government. The overlapping and interlacing of functions and But much as we might postulate on the internal checks of power provided in our
duties between the several departments, however, sometimes makes it hard to Constitution, it ought not the less to be remembered that, in the language of
James Madison, the system itself is not "the chief palladium of constitutional February 29, 1920) and Spain (arts. 121-123, Title IX, Constitution of the Republic
liberty ... the people who are authors of this blessing must also be its guardians ... of 1931) especial constitutional courts are established to pass upon the validity of
their eyes must be ever ready to mark, their voices to pronounce ... aggression on ordinary laws. In our case, the nature of the present controversy shows the
the authority of their Constitution." In the last and ultimate analysis then, must the necessity of a final constitutional arbiter to determine the conflict of authority
success of our government in the unfolding years to come be tested in the crucible between two agencies created by the Constitution. Were we to decline to take
of Filipino minds and hearts than in consultation rooms and court chambers. cognizance of the controversy, who will determine the conflict? And if the conflict
were left undecided and undetermined, would not a void be thus created in our
constitutional system which may in the long run prove destructive of the entire
In the case at bar, the National Assembly has by resolution (No. 8) of December 3,
framework? To ask these questions is to answer them. Natura vacuum abhorret,
1935, confirmed the election of the herein petitioner to the said body. On the
so must we avoid exhaustion in our constitutional system. Upon principle, reason,
other hand, the Electoral Commission has by resolution adopted on December 9,
and authority, we are clearly of the opinion that upon the admitted facts of the
1935, fixed said date as the last day for the filing of protests against the election,
present case, this court has jurisdiction over the Electoral Commission and the
returns and qualifications of members of the National Assembly; notwithstanding
subject matter of the present controversy for the purpose of determining the
the previous confirmations made by the National Assembly as aforesaid. If, as
character, scope and extent of the constitutional grant to the Electoral
contended by the petitioner, the resolution of the National Assembly has the
Commission as "the sole judge of all contests relating to the election, returns and
effect of cutting off the power of the Electoral Commission to entertain protests
qualifications of the members of the National Assembly." .
against the election, returns and qualifications of members of the National
Assembly, submitted after December 3, 1935 then the resolution of the Electoral
Commission of December 9, 1935, is mere surplusage and had no effect. But, if, as As the Chief Justice has made it clear in Gonzales, like Justice Laurel did in Angara, these postulates just
contended by the respondents, the Electoral Commission has the sole power of quoted do not apply only to conflicts of authority between the three existing regular departments of the
regulating its proceedings to the exclusion of the National Assembly, then the government but to all such conflicts between and among these departments, or, between any of them,
resolution of December 9, 1935, by which the Electoral Commission fixed said date on the one hand, and any other constitutionally created independent body, like the electoral tribunals in
as the last day for filing protests against the election, returns and qualifications of Congress, the Comelec and the Constituent assemblies constituted by the House of Congress, on the
members of the National Assembly, should be upheld. other. We see no reason of logic or principle whatsoever, and none has been convincingly shown to Us
by any of the respondents and intervenors, why the same ruling should not apply to the present
Convention, even if it is an assembly of delegate elected directly by the people, since at best, as already
Here is then presented an actual controversy involving as it does a conflict of a
demonstrated, it has been convened by authority of and under the terms of the present Constitution..
grave constitutional nature between the National Assembly on the one hand and
the Electoral Commission on the other. From the very nature of the republican
government established in our country in the light of American experience and of Accordingly, We are left with no alternative but to uphold the jurisdiction of the Court over the present
our own, upon the judicial department is thrown the solemn and inescapable case. It goes without saying that We do this not because the Court is superior to the Convention or that
obligation of interpreting the Constitution and defining constitutional boundaries. the Convention is subject to the control of the Court, but simply because both the Convention and the
The Electoral Commission as we shall have occasion to refer hereafter, is a Court are subject to the Constitution and the rule of law, and "upon principle, reason and authority," per
constitutional organ, created for a specific purpose, namely, to determine all Justice Laurel, supra, it is within the power as it is the solemn duty of the Court, under the existing
contests relating to the election, returns and qualifications of the members of the Constitution to resolve the issues in which petitioner, respondents and intervenors have joined in this
National Assembly. Although the Electoral Commission may not be interfered with, case.
when and while acting within the limits of its authority, it does not follow that it is
beyond the reach of the constitutional mechanism adopted by the people and that
II
it is not subject to constitutional restriction. The Electoral Commission is not a
separate department of the government, and even if it were, conflicting claims of
authority under the fundamental law between departmental powers and agencies The issue of jurisdiction thus resolved, We come to the crux of the petition. Is it within the powers of the
of the government are necessarily determined by the judiciary in justiciable and Constitutional Convention of 1971 to order, on its own fiat, the holding of a plebiscite for the ratification
appropriate cases. Discarding the English type and other European types of of the proposed amendment reducing to eighteen years the age for the exercise of suffrage under
constitutional government, the framers of our Constitution adopted the American Section 1 of Article V of the Constitution proposed in the Convention's Organic Resolution No. 1 in the
type where the written constitution is interpreted and given effect by the judicial manner and form provided for in said resolution and the subsequent implementing acts and resolution
department. In some countries which have declined to follow the American of the Convention?
example, provisions have been inserted in their constitutions prohibiting the
courts from exercising the power to interpret the fundamental law. This is taken as At the threshold, the environmental circumstances of this case demand the most accurate and
a recognition of what otherwise would be the rule that in the absence of direct unequivocal statement of the real issue which the Court is called upon to resolve. Petitioner has very
prohibition, courts are bound to assume what is logically their function. For clearly stated that he is not against the constitutional extension of the right of suffrage to the eighteen-
instance, the Constitution of Poland of 1921 expressly provides that courts shall year-olds, as a matter of fact, he has advocated or sponsored in Congress such a proposal, and that, in
have no power to examine the validity of statutes (art. 81, Chap. IV). The former truth, the herein petition is not intended by him to prevent that the proposed amendment here involved
Austrian Constitution contained a similar declaration. In countries whose be submitted to the people for ratification, his only purpose in filing the petition being to comply with his
constitution are silent in this respect, courts have assumed this power. This is true sworn duty to prevent, Whenever he can, any violation of the Constitution of the Philippines even if it is
in Norway, Greece, Australia and South Africa. Whereas, in Czechoslovakia (arts. 2 committed in the course of or in connection with the most laudable undertaking. Indeed, as the Court
and 3, Preliminary Law to Constitutional Charter of the Czechoslavak, Republic, sees it, the specific question raised in this case is limited solely and only to the point of whether or not it
is within the power of the Convention to call for a plebiscite for the ratification by the people of the obligation to express Our views thereon. The Court considers it to be of the utmost importance that the
constitutional amendment proposed in the abovequoted Organic Resolution No. 1, in the manner and Convention should be untrammelled and unrestrained in the performance of its constitutionally as
form provided in said resolution as well as in the subject question implementing actions and resolution signed mission in the manner and form it may conceive best, and so the Court may step in to clear up
of the Convention and its officers, at this juncture of its proceedings, when as it is a matter of common doubts as to the boundaries set down by the Constitution only when and to the specific extent only that
knowledge and judicial notice, it is not set to adjourn sine die, and is, in fact, still in the preliminary it would be necessary to do so to avoid a constitutional crisis or a clearly demonstrable violation of the
stages of considering other reforms or amendments affecting other parts of the existing Constitution; existing Charter. Withal, it is a very familiar principle of constitutional law that constitutional questions
and, indeed, Organic Resolution No. 1 itself expressly provides, that the amendment therein proposed are to be resolved by the Supreme Court only when there is no alternative but to do it, and this rule is
"shall be without prejudice to other amendments that will be proposed in the future by the 1971 founded precisely on the principle of respect that the Court must accord to the acts of the other
Constitutional Convention on other portions of the amended section or on other portions of the entire coordinate departments of the government, and certainly, the Constitutional Convention stands almost
Constitution." In other words, nothing that the Court may say or do, in this case should be understood as in a unique footing in that regard.
reflecting, in any degree or means the individual or collective stand of the members of the Court on the
fundamental issue of whether or not the eighteen-year-olds should be allowed to vote, simply because
In our discussion of the issue of jurisdiction, We have already made it clear that the Convention came
that issue is not before Us now. There should be no doubt in the mind of anyone that, once the Court
into being by a call of a joint session of Congress pursuant to Section I of Article XV of the Constitution,
finds it constitutionally permissible, it will not hesitate to do its part so that the said proposed
already quoted earlier in this opinion. We reiterate also that as to matters not related to its internal
amendment may be presented to the people for their approval or rejection.
operation and the performance of its assigned mission to propose amendments to the Constitution, the
Convention and its officers and members are all subject to all the provisions of the existing Constitution.
Withal, the Court rests securely in the conviction that the fire and enthusiasm of the youth have not Now We hold that even as to its latter task of proposing amendments to the Constitution, it is subject to
blinded them to the absolute necessity, under the fundamental principles of democracy to which the the provisions of Section I of Article XV. This must be so, because it is plain to Us that the framers of the
Filipino people is committed, of adhering always to the rule of law. Surely, their idealism, sincerity and Constitution took care that the process of amending the same should not be undertaken with the same
purity of purpose cannot permit any other line of conduct or approach in respect of the problem before ease and facility in changing an ordinary legislation. Constitution making is the most valued power,
Us. The Constitutional Convention of 1971 itself was born, in a great measure, because of the pressure second to none, of the people in a constitutional democracy such as the one our founding fathers have
brought to bear upon the Congress of the Philippines by various elements of the people, the youth in chosen for this nation, and which we of the succeeding generations generally cherish. And because the
particular, in their incessant search for a peaceful and orderly means of bringing about meaningful Constitution affects the lives, fortunes, future and every other conceivable aspect of the lives of all the
changes in the structure and bases of the existing social and governmental institutions, including the people within the country and those subject to its sovereignty, every degree of care is taken in preparing
provisions of the fundamental law related to the well-being and economic security of the and drafting it. A constitution worthy of the people for which it is intended must not be prepared in
underprivileged classes of our people as well as those concerning the preservation and protection of our haste without adequate deliberation and study. It is obvious that correspondingly, any amendment of
natural resources and the national patrimony, as an alternative to violent and chaotic ways of achieving the Constitution is of no less importance than the whole Constitution itself, and perforce must be
such lofty ideals. In brief, leaving aside the excesses of enthusiasm which at times have justifiably or conceived and prepared with as much care and deliberation. From the very nature of things, the drafters
unjustifiably marred the demonstrations in the streets, plazas and campuses, the youth of the of an original constitution, as already observed earlier, operate without any limitations, restraints or
Philippines, in general, like the rest of the people, do not want confusion and disorder, anarchy and inhibitions save those that they may impose upon themselves. This is not necessarily true of subsequent
violence; what they really want are law and order, peace and orderliness, even in the pursuit of what conventions called to amend the original constitution. Generally, the framers of the latter see to it that
they strongly and urgently feel must be done to change the present order of things in this Republic of their handiwork is not lightly treated and as easily mutilated or changed, not only for reasons purely
ours. It would be tragic and contrary to the plain compulsion of these perspectives, if the Court were to personal but more importantly, because written constitutions are supposed to be designed so as to last
allow itself in deciding this case to be carried astray by considerations other than the imperatives of the for some time, if not for ages, or for, at least, as long as they can be adopted to the needs and exigencies
rule of law and of the applicable provisions of the Constitution. Needless to say, in a larger measure than of the people, hence, they must be insulated against precipitate and hasty actions motivated by more or
when it binds other departments of the government or any other official or entity, the Constitution less passing political moods or fancies. Thus, as a rule, the original constitutions carry with them
imposes upon the Court the sacred duty to give meaning and vigor to the Constitution, by interpreting limitations and conditions, more or less stringent, made so by the people themselves, in regard to the
and construing its provisions in appropriate cases with the proper parties, and by striking down any act process of their amendment. And when such limitations or conditions are so incorporated in the original
violative thereof. Here, as in all other cases, We are resolved to discharge that duty. constitution, it does not lie in the delegates of any subsequent convention to claim that they may ignore
and disregard such conditions because they are as powerful and omnipotent as their original
counterparts.
During these twice when most anyone feels very strongly the urgent need for constitutional reforms, to
the point of being convinced that meaningful change is the only alternative to a violent revolution, this
Court would be the last to put any obstruction or impediment to the work of the Constitutional Nothing of what is here said is to be understood as curtailing in any degree the number and nature and
Convention. If there are respectable sectors opining that it has not been called to supplant the existing the scope and extent of the amendments the Convention may deem proper to propose. Nor does the
Constitution in its entirety, since its enabling provision, Article XV, from which the Convention itself Court propose to pass on the issue extensively and brilliantly discussed by the parties as to whether or
draws life expressly speaks only of amendments which shall form part of it, which opinion is not without not the power or duty to call a plebiscite for the ratification of the amendments to be proposed by the
persuasive force both in principle and in logic, the seemingly prevailing view is that only the collective Convention is exclusively legislative and as such may be exercised only by the Congress or whether the
judgment of its members as to what is warranted by the present condition of things, as they see it, can said power can be exercised concurrently by the Convention with the Congress. In the view the Court
limit the extent of the constitutional innovations the Convention may propose, hence the complete takes of present case, it does not perceive absolute necessity to resolve that question, grave and
substitution of the existing constitution is not beyond the ambit of the Convention's authority. Desirable important as it may be. Truth to tell, the lack of unanimity or even of a consensus among the members
as it may be to resolve, this grave divergence of views, the Court does not consider this case to be of the Court in respect to this issue creates the need for more study and deliberation, and as time is of
properly the one in which it should discharge its constitutional duty in such premises. The issues raised the essence in this case, for obvious reasons, November 8, 1971, the date set by the Convention for the
by petitioner, even those among them in which respondents and intervenors have joined in an apparent plebiscite it is calling, being nigh, We will refrain from making any pronouncement or expressing Our
wish to have them squarely passed upon by the Court do not necessarily impose upon Us the imperative
views on this question until a more appropriate case comes to Us. After all, the basis of this decision is as ever be or not, because Congress has reserved those for future action, what kind of judgment can he
important and decisive as any can be. render on the proposal?

The ultimate question, therefore boils down to this: Is there any limitation or condition in Section 1 of But the situation actually before Us is even worse. No one knows what changes in the fundamental
Article XV of the Constitution which is violated by the act of the Convention of calling for a plebiscite on principles of the constitution the Convention will be minded to approve. To be more specific, we do not
the sole amendment contained in Organic Resolution No. 1? The Court holds that there is, and it is the have any means of foreseeing whether the right to vote would be of any significant value at all. Who can
condition and limitation that all the amendments to be proposed by the same Convention must be say whether or not later on the Convention may decide to provide for varying types of voters for each
submitted to the people in a single "election" or plebiscite. It being indisputable that the amendment level of the political units it may divide the country into. The root of the difficulty in other words, lies in
now proposed to be submitted to a plebiscite is only the first amendment the Convention propose We that the Convention is precisely on the verge of introducing substantial changes, if not radical ones, in
hold that the plebiscite being called for the purpose of submitting the same for ratification of the people almost every part and aspect of the existing social and political order enshrined in the present
on November 8, 1971 is not authorized by Section 1 of Article XV of the Constitution, hence all acts of Constitution. How can a voter in the proposed plebiscite intelligently determine the effect of the
the Convention and the respondent Comelec in that direction are null and void. reduction of the voting age upon the different institutions which the Convention may establish and of
which presently he is not given any idea?
We have arrived at this conclusion for the following reasons:
We are certain no one can deny that in order that a plebiscite for the ratification of an amendment to
the Constitution may be validly held, it must provide the voter not only sufficient time but ample basis
1. The language of the constitutional provision aforequoted is sufficiently clear. lt says distinctly that
for an intelligent appraisal of the nature of the amendment per se as well as its relation to the other
either Congress sitting as a constituent assembly or a convention called for the purpose "may propose
parts of the Constitution with which it has to form a harmonious whole. In the context of the present
amendments to this Constitution," thus placing no limit as to the number of amendments that Congress
state of things, where the Convention has hardly started considering the merits of hundreds, if not
or the Convention may propose. The same provision also as definitely provides that
thousands, of proposals to amend the existing Constitution, to present to the people any single proposal
"such amendments shall be valid as part of this Constitution when approved by a majority of the votes
or a few of them cannot comply with this requirement. We are of the opinion that the present
cast at an election at which the amendments are submitted to the people for their ratification," thus
Constitution does not contemplate in Section 1 of Article XV a plebiscite or "election" wherein the
leaving no room for doubt as to how many "elections" or plebiscites may be held to ratify any
people are in the dark as to frame of reference they can base their judgment on. We reject the
amendment or amendments proposed by the same constituent assembly of Congress or convention, and
rationalization that the present Constitution is a possible frame of reference, for the simple reason that
the provision unequivocably says "an election" which means only one.
intervenors themselves are stating that the sole purpose of the proposed amendment is to enable the
eighteen year olds to take part in the election for the ratification of the Constitution to be drafted by the
(2) Very little reflection is needed for anyone to realize the wisdom and appropriateness of this Convention. In brief, under the proposed plebiscite, there can be, in the language of Justice Sanchez,
provision. As already stated, amending the Constitution is as serious and important an undertaking as speaking for the six members of the Court in Gonzales, supra, "no proper submission".
constitution making itself. Indeed, any amendment of the Constitution is as important as the whole of it
if only because the Constitution has to be an integrated and harmonious instrument, if it is to be viable
III
as the framework of the government it establishes, on the one hand, and adequately formidable and
reliable as the succinct but comprehensive articulation of the rights, liberties, ideology, social ideals, and
national and nationalistic policies and aspirations of the people, on the other. lt is inconceivable how a The Court has no desire at all to hamper and hamstring the noble work of the Constitutional Convention.
constitution worthy of any country or people can have any part which is out of tune with its other parts.. Much less does the Court want to pass judgment on the merits of the proposal to allow these eighteen
years old to vote. But like the Convention, the Court has its own duties to the people under the
Constitution which is to decide in appropriate cases with appropriate parties Whether or not the
A constitution is the work of the people thru its drafters assembled by them for the purpose. Once the
mandates of the fundamental law are being complied with. In the best light God has given Us, we are of
original constitution is approved, the part that the people play in its amendment becomes harder, for
the conviction that in providing for the questioned plebiscite before it has finished, and separately from,
when a whole constitution is submitted to them, more or less they can assumed its harmony as an
the whole draft of the constitution it has been called to formulate, the Convention's Organic Resolution
integrated whole, and they can either accept or reject it in its entirety. At the very least, they can
No. 1 and all subsequent acts of the Convention implementing the same violate the condition in Section
examine it before casting their vote and determine for themselves from a study of the whole document
1, Article XV that there should only be one "election" or plebiscite for the ratification of all the
the merits and demerits of all or any of its parts and of the document as a whole. And so also, when an
amendments the Convention may propose. We are not denying any right of the people to vote on the
amendment is submitted to them that is to form part of the existing constitution, in like fashion they can
proposed amendment; We are only holding that under Section 1, Article XV of the Constitution, the
study with deliberation the proposed amendment in relation to the whole existing constitution and or
same should be submitted to them not separately from but together with all the other amendments to
any of its parts and thereby arrive at an intelligent judgment as to its acceptability.
be proposed by this present Convention.

This cannot happen in the case of the amendment in question. Prescinding already from the fact that
IN VIEW OF ALL THE FOREGOING, the petition herein is granted. Organic Resolution No. 1 of the
under Section 3 of the questioned resolution, it is evident that no fixed frame of reference is provided
Constitutional Convention of 1971 and the implementing acts and resolutions of the Convention, insofar
the voter, as to what finally will be concomitant qualifications that will be required by the final draft of
as they provide for the holding of a plebiscite on November 8, 1971, as well as the resolution of the
the constitution to be formulated by the Convention of a voter to be able to enjoy the right of suffrage,
respondent Comelec complying therewith (RR Resolution No. 695) are hereby declared null and void.
there are other considerations which make it impossible to vote intelligently on the proposed
The respondents Comelec, Disbursing Officer, Chief Accountant and Auditor of the Constitutional
amendment, although it may already be observed that under Section 3, if a voter would favor the
Convention are hereby enjoined from taking any action in compliance with the said organic resolution. In
reduction of the voting age to eighteen under conditions he feels are needed under the circumstances,
view of the peculiar circumstances of this case, the Court declares this decision immediately executory.
and he does not see those conditions in the ballot nor is there any possible indication whether they will
No costs.
Republic of the Philippines Delfin alleged in his petition that he is a founding member of the Movement for People's Initiative, 6 a
SUPREME COURT group of citizens desirous to avail of the system intended to institutionalize people power; that he and
Manila the members of the Movement and other volunteers intend to exercise the power to directly propose
amendments to the Constitution granted under Section 2, Article XVII of the Constitution; that the
exercise of that power shall be conducted in proceedings under the control and supervision of the
EN BANC
COMELEC; that, as required in COMELEC Resolution No. 2300, signature stations shall be established all
over the country, with the assistance of municipal election registrars, who shall verify the signatures
affixed by individual signatories; that before the Movement and other volunteers can gather signatures,
it is necessary that the time and dates to be designated for the purpose be first fixed in an order to be
G.R. No. 127325 March 19, 1997 issued by the COMELEC; and that to adequately inform the people of the electoral process involved, it is
likewise necessary that the said order, as well as the Petition on which the signatures shall be affixed, be
published in newspapers of general and local circulation, under the control and supervision of the
MIRIAM DEFENSOR SANTIAGO, ALEXANDER PADILLA, and MARIA ISABEL ONGPIN, petitioners, COMELEC.
vs.
COMMISSION ON ELECTIONS, JESUS DELFIN, ALBERTO PEDROSA & CARMEN PEDROSA, in their
capacities as founding members of the People's Initiative for Reforms, Modernization and Action The Delfin Petition further alleged that the provisions sought to be amended are Sections 4 and 7 of
(PIRMA), respondents. Article VI, 7Section 4 of Article VII, 8 and Section 8 of Article X 9 of the Constitution. Attached to the
petition is a copy of a "Petition for Initiative on the 1987 Constitution" 10 embodying the proposed
amendments which consist in the deletion from the aforecited sections of the provisions concerning
SENATOR RAUL S. ROCO, DEMOKRASYA-IPAGTANGGOL ANG KONSTITUSYON (DIK), MOVEMENT OF term limits, and with the following proposition:
ATTORNEYS FOR BROTHERHOOD INTEGRITY AND NATIONALISM, INC. (MABINI), INTEGRATED BAR OF
THE PHILIPPINES (IBP), and LABAN NG DEMOKRATIKONG PILIPINO (LABAN), petitioners-intervenors.
DO YOU APPROVE OF LIFTING THE TERM LIMITS OF ALL ELECTIVE GOVERNMENT
OFFICIALS, AMENDING FOR THE PURPOSE SECTIONS 4 AND 7 OF ARTICLE VI,
SECTION 4 OF ARTICLE VII, AND SECTION 8 OF ARTICLE X OF THE 1987 PHILIPPINE
CONSTITUTION?
DAVIDE, JR., J.:
According to Delfin, the said Petition for Initiative will first be submitted to the people, and after it is
The heart of this controversy brought to us by way of a petition for prohibition under Rule 65 of the signed by at least twelve per cent of the total number of registered voters in the country it will be
Rules of Court is the right of the people to directly propose amendments to the Constitution through the formally filed with the COMELEC.
system of initiative under Section 2 of Article XVII of the 1987 Constitution. Undoubtedly, this demands
special attention, as this system of initiative was unknown to the people of this country, except perhaps Upon the filing of the Delfin Petition, which was forthwith given the number UND 96-037 (INITIATIVE),
to a few scholars, before the drafting of the 1987 Constitution. The 1986 Constitutional Commission the COMELEC, through its Chairman, issued an Order 11 (a) directing Delfin "to cause the publication of
itself, through the original proponent 1 and the main sponsor 2 of the proposed Article on Amendments the petition, together with the attached Petition for Initiative on the 1987 Constitution (including the
or Revision of the Constitution, characterized this system as "innovative". 3 Indeed it is, for both under proposal, proposed constitutional amendment, and the signature form), and the notice of hearing in
the 1935 and 1973 Constitutions, only two methods of proposing amendments to, or revision of, the three (3) daily newspapers of general circulation at his own expense" not later than 9 December 1996;
Constitution were recognized, viz., (1) by Congress upon a vote of three-fourths of all its members and and (b) setting the case for hearing on 12 December 1996 at 10:00 a.m.
(2) by a constitutional convention. 4 For this and the other reasons hereafter discussed, we resolved to
give due course to this petition.
At the hearing of the Delfin Petition on 12 December 1996, the following appeared: Delfin and Atty. Pete
Q. Quadra; representatives of the People's Initiative for Reforms, Modernization and Action (PIRMA);
On 6 December 1996, private respondent Atty. Jesus S. Delfin filed with public respondent Commission intervenor-oppositor Senator Raul S. Roco, together with his two other lawyers, and representatives of,
on Elections (hereafter, COMELEC) a "Petition to Amend the Constitution, to Lift Term Limits of Elective or counsel for, the Integrated Bar of the Philippines (IBP), Demokrasya-Ipagtanggol ang Konstitusyon
Officials, by People's Initiative" (hereafter, Delfin Petition) 5 wherein Delfin asked the COMELEC for an (DIK), Public Interest Law Center, and Laban ng Demokratikong Pilipino (LABAN). 12 Senator Roco, on that
order same day, filed a Motion to Dismiss the Delfin Petition on the ground that it is not the initiatory petition
properly cognizable by the COMELEC.
1. Fixing the time and dates for signature gathering all over the country;
After hearing their arguments, the COMELEC directed Delfin and the oppositors to file their "memoranda
2. Causing the necessary publications of said Order and the attached "Petition for and/or oppositions/memoranda" within five days. 13
Initiative on the 1987 Constitution, in newspapers of general and local circulation;
On 18 December 1996, the petitioners herein Senator Miriam Defensor Santiago, Alexander Padilla,
3. Instructing Municipal Election Registrars in all Regions of the Philippines, to and Maria Isabel Ongpin filed this special civil action for prohibition raising the following arguments:
assist Petitioners and volunteers, in establishing signing stations at the time and on
the dates designated for the purpose.
(1) The constitutional provision on people's initiative to amend the Constitution On 2 January 1997, private respondents, through Atty Quadra, filed their Comment 15 on the petition.
can only be implemented by law to be passed by Congress. No such law has been They argue therein that:
passed; in fact, Senate Bill No. 1290 entitled An Act Prescribing and Regulating
Constitution Amendments by People's Initiative, which petitioner Senator Santiago
1. IT IS NOT TRUE THAT "IT WOULD ENTAIL EXPENSES TO THE NATIONAL
filed on 24 November 1995, is still pending before the Senate Committee on
TREASURY FOR GENERAL REGISTRATION OF VOTERS AMOUNTING TO AT LEAST
Constitutional Amendments.
PESOS: ONE HUNDRED EIGHTY MILLION (P180,000,000.00)" IF THE "COMELEC
GRANTS THE PETITION FILED BY RESPONDENT DELFIN BEFORE THE COMELEC.
(2) It is true that R.A. No. 6735 provides for three systems of initiative, namely,
initiative on the Constitution, on statutes, and on local legislation. However, it
2. NOT A SINGLE CENTAVO WOULD BE SPENT BY THE NATIONAL GOVERNMENT IF
failed to provide any subtitle on initiative on the Constitution, unlike in the other
THE COMELEC GRANTS THE PETITION OF RESPONDENT DELFIN. ALL EXPENSES IN
modes of initiative, which are specifically provided for in Subtitle II and Subtitle III.
THE SIGNATURE GATHERING ARE ALL FOR THE ACCOUNT OF RESPONDENT DELFIN
This deliberate omission indicates that the matter of people's initiative to amend
AND HIS VOLUNTEERS PER THEIR PROGRAM OF ACTIVITIES AND EXPENDITURES
the Constitution was left to some future law. Former Senator Arturo Tolentino
SUBMITTED TO THE COMELEC. THE ESTIMATED COST OF THE DAILY PER DIEM OF
stressed this deficiency in the law in his privilege speech delivered before the
THE SUPERVISING SCHOOL TEACHERS IN THE SIGNATURE GATHERING TO BE
Senate in 1994: "There is not a single word in that law which can be considered as
DEPOSITED and TO BE PAID BY DELFIN AND HIS VOLUNTEERS IS P2,571,200.00;
implementing [the provision on constitutional initiative]. Such implementing
provisions have been obviously left to a separate law.
3. THE PENDING PETITION BEFORE THE COMELEC IS ONLY ON THE SIGNATURE
GATHERING WHICH BY LAW COMELEC IS DUTY BOUND "TO SUPERVISE CLOSELY"
(3) Republic Act No. 6735 provides for the effectivity of the law after publication in
PURSUANT TO ITS "INITIATORY JURISDICTION" UPHELD BY THE HONORABLE
print media. This indicates that the Act covers only laws and not constitutional
COURT IN ITS RECENT SEPTEMBER 26, 1996 DECISION IN THE CASE OF SUBIC BAY
amendments because the latter take effect only upon ratification and not after
METROPOLITAN AUTHORITY VS. COMELEC, ET AL. G.R. NO. 125416;
publication.

4. REP. ACT NO. 6735 APPROVED ON AUGUST 4, 1989 IS THE ENABLING LAW
(4) COMELEC Resolution No. 2300, adopted on 16 January 1991 to govern "the
IMPLEMENTING THE POWER OF PEOPLE INITIATIVE TO PROPOSE AMENDMENTS
conduct of initiative on the Constitution and initiative and referendum on national
TO THE CONSTITUTION. SENATOR DEFENSOR-SANTIAGO'S SENATE BILL NO. 1290
and local laws, is ultra vires insofar asinitiative on amendments to the Constitution
IS A DUPLICATION OF WHAT ARE ALREADY PROVIDED FOR IN REP. ACT NO. 6735;
is concerned, since the COMELEC has no power to provide rules and regulations
for the exercise of the right of initiative to amend the Constitution. Only Congress
is authorized by the Constitution to pass the implementing law. 5. COMELEC RESOLUTION NO. 2300 PROMULGATED ON JANUARY 16, 1991
PURSUANT TO REP. ACT 6735 WAS UPHELD BY THE HONORABLE COURT IN THE
RECENT SEPTEMBER 26, 1996 DECISION IN THE CASE OF SUBIC BAY
(5) The people's initiative is limited to amendments to the Constitution, not
METROPOLITAN AUTHORITY VS. COMELEC, ET AL. G.R. NO. 125416 WHERE THE
to revision thereof. Extending or lifting of term limits constitutes a revision and is,
HONORABLE COURT SAID: "THE COMMISSION ON ELECTIONS CAN DO NO LESS BY
therefore, outside the power of the people's initiative.
SEASONABLY AND JUDICIOUSLY PROMULGATING GUIDELINES AND RULES FOR
BOTH NATIONAL AND LOCAL USE, IN IMPLEMENTING OF THESE LAWS."
(6) Finally, Congress has not yet appropriated funds for people's initiative; neither
the COMELEC nor any other government department, agency, or office has
6. EVEN SENATOR DEFENSOR-SANTIAGO'S SENATE BILL NO. 1290 CONTAINS A
realigned funds for the purpose.
PROVISION DELEGATING TO THE COMELEC THE POWER TO "PROMULGATE SUCH
RULES AND REGULATIONS AS MAY BE NECESSARY TO CARRY OUT THE PURPOSES
To justify their recourse to us via the special civil action for prohibition, the petitioners allege that in the OF THIS ACT." (SEC. 12, S.B. NO. 1290, ENCLOSED AS ANNEX E, PETITION);
event the COMELEC grants the Delfin Petition, the people's initiative spearheaded by PIRMA would entail
expenses to the national treasury for general re-registration of voters amounting to at least P180 million,
7. THE LIFTING OF THE LIMITATION ON THE TERM OF OFFICE OF ELECTIVE
not to mention the millions of additional pesos in expenses which would be incurred in the conduct of
OFFICIALS PROVIDED UNDER THE 1987 CONSTITUTION IS NOT A "REVISION" OF
the initiative itself. Hence, the transcendental importance to the public and the nation of the issues
THE CONSTITUTION. IT IS ONLY AN AMENDMENT. "AMENDMENT ENVISAGES AN
raised demands that this petition for prohibition be settled promptly and definitely, brushing aside
ALTERATION OF ONE OR A FEW SPECIFIC PROVISIONS OF THE CONSTITUTION.
technicalities of procedure and calling for the admission of a taxpayer's and legislator's suit. 14 Besides,
REVISION CONTEMPLATES A RE-EXAMINATION OF THE ENTIRE DOCUMENT TO
there is no other plain, speedy, and adequate remedy in the ordinary course of law.
DETERMINE HOW AND TO WHAT EXTENT IT SHOULD BE ALTERED." (PP. 412-413,
2ND. ED. 1992, 1097 PHIL. CONSTITUTION, BY JOAQUIN G. BERNAS, S.J.).
On 19 December 1996, this Court (a) required the respondents to comment on the petition within a non-
extendible period of ten days from notice; and (b) issued a temporary restraining order, effective
Also on 2 January 1997, private respondent Delfin filed in his own behalf a Comment 16 which starts off
immediately and continuing until further orders, enjoining public respondent COMELEC from proceeding
with an assertion that the instant petition is a "knee-jerk reaction to a draft 'Petition for Initiative on the
with the Delfin Petition, and private respondents Alberto and Carmen Pedrosa from conducting a
1987 Constitution'. . . which is not formally filed yet." What he filed on 6 December 1996 was an
signature drive for people's initiative to amend the Constitution.
"Initiatory Pleading" or "Initiatory Petition," which was legally necessary to start the signature campaign (4) Extension of term limits of elected officials constitutes a mere amendment to
to amend the Constitution or to put the movement to gather signatures under COMELEC power and the Constitution, not a revision thereof.
function. On the substantive allegations of the petitioners, Delfin maintains as follows:
(5) COMELEC Resolution No. 2300 was validly issued under Section 20 of R.A. No.
(1) Contrary to the claim of the petitioners, there is a law, R.A. No. 6735, which 6735 and under the Omnibus Election Code. The rule-making power of the
governs the conduct of initiative to amend the Constitution. The absence therein COMELEC to implement the provisions of R.A. No. 6735 was in fact upheld by this
of a subtitle for such initiative is not fatal, since subtitles are not requirements for Court in Subic Bay Metropolitan Authority vs. COMELEC.
the validity or sufficiency of laws.
On 14 January 1997, this Court (a) confirmed nunc pro tunc the temporary restraining order; (b) noted
(2) Section 9(b) of R.A. No. 6735 specifically provides that the proposition in the aforementioned Comments and the Motion to Lift Temporary Restraining Order filed by private
an initiative to amend the Constitution approved by the majority of the votes cast respondents through Atty. Quadra, as well as the latter's Manifestation stating that he is the counsel for
in the plebiscite shall become effective as of the day of the plebiscite. private respondents Alberto and Carmen Pedrosa only and the Comment he filed was for the Pedrosas;
and (c) granted the Motion for Intervention filed on 6 January 1997 by Senator Raul Roco and allowed
him to file his Petition in Intervention not later than 20 January 1997; and (d) set the case for hearing on
(3) The claim that COMELEC Resolution No. 2300 is ultra vires is contradicted by (a)
23 January 1997 at 9:30 a.m.
Section 2, Article IX-C of the Constitution, which grants the COMELEC the power to
enforce and administer all laws and regulations relative to the conduct of an
election, plebiscite, initiative, referendum, and recall; and (b) Section 20 of R.A. On 17 January 1997, the Demokrasya-Ipagtanggol ang Konstitusyon (DIK) and the Movement of
6735, which empowers the COMELEC to promulgate such rules and regulations as Attorneys for Brotherhood Integrity and Nationalism, Inc. (MABINI), filed a Motion for Intervention.
may be necessary to carry out the purposes of the Act. Attached to the motion was their Petition in Intervention, which was later replaced by an Amended
Petition in Intervention wherein they contend that:
(4) The proposed initiative does not involve a revision of, but mere amendment to,
the Constitution because it seeks to alter only a few specific provisions of the (1) The Delfin proposal does not involve a mere amendment to, but a revision of,
Constitution, or more specifically, only those which lay term limits. It does not seek the Constitution because, in the words of Fr. Joaquin Bernas, S.J., 18 it would
to reexamine or overhaul the entire document. involve a change from a political philosophy that rejects unlimited tenure to one
that accepts unlimited tenure; and although the change might appear to be an
isolated one, it can affect other provisions, such as, on synchronization of elections
As to the public expenditures for registration of voters, Delfin considers petitioners' estimate of P180
and on the State policy of guaranteeing equal access to opportunities for public
million as unreliable, for only the COMELEC can give the exact figure. Besides, if there will be a plebiscite
service and prohibiting political dynasties. 19 Arevision cannot be done
it will be simultaneous with the 1997 Barangay Elections. In any event, fund requirements
by initiative which, by express provision of Section 2 of Article XVII of the
for initiative will be a priority government expense because it will be for the exercise of the sovereign
Constitution, is limited to amendments.
power of the people.

(2) The prohibition against reelection of the President and the limits provided for
In the Comment 17 for the public respondent COMELEC, filed also on 2 January 1997, the Office of the
all other national and local elective officials are based on the philosophy of
Solicitor General contends that:
governance, "to open up the political arena to as many as there are Filipinos
qualified to handle the demands of leadership, to break the concentration of
(1) R.A. No. 6735 deals with, inter alia, people's initiative to amend the political and economic powers in the hands of a few, and to promote effective
Constitution. Its Section 2 on Statement of Policy explicitly affirms, recognizes, and proper empowerment for participation in policy and decision-making for the
guarantees that power; and its Section 3, which enumerates the three systems common good"; hence, to remove the term limits is to negate and nullify the noble
of initiative, includes initiative on the Constitution and defines the same as the vision of the 1987 Constitution.
power to propose amendments to the Constitution. Likewise, its Section 5
repeatedly mentionsinitiative on the Constitution.
(3) The Delfin proposal runs counter to the purpose of initiative, particularly in a
conflict-of-interest situation. Initiative is intended as a fallback position that may
(2) A separate subtitle on initiative on the Constitution is not necessary in R.A. No. be availed of by the people only if they are dissatisfied with the performance of
6735 because, being national in scope, that system of initiative is deemed included their elective officials, but not as a premium for good performance. 20
in the subtitle on National Initiative and Referendum; and Senator Tolentino
simply overlooked pertinent provisions of the law when he claimed that nothing
(4) R.A. No. 6735 is deficient and inadequate in itself to be called the enabling law
therein was provided for initiative on the Constitution.
that implements the people's initiative on amendments to the Constitution. It fails
to state (a) the proper parties who may file the petition, (b) the appropriate
(3) Senate Bill No. 1290 is neither a competent nor a material proof that R.A. No. agency before whom the petition is to be filed, (c) the contents of the petition, (d)
6735 does not deal with initiative on the Constitution. the publication of the same, (e) the ways and means of gathering the signatures of
the voters nationwide and 3% per legislative district, (f) the proper parties who
may oppose or question the veracity of the signatures, (g) the role of the COMELEC
in the verification of the signatures and the sufficiency of the petition, (h) the respondents to comment thereon within a nonextendible period of five days from receipt of the said
appeal from any decision of the COMELEC, (I) the holding of a plebiscite, and (g) Petition in Intervention.
the appropriation of funds for such people's initiative. Accordingly, there being no
enabling law, the COMELEC has no jurisdiction to hear Delfin's petition.
At the hearing of the case on 23 January 1997, the parties argued on the following pivotal issues, which
the Court formulated in light of the allegations and arguments raised in the pleadings so far filed:
(5) The deficiency of R.A. No. 6735 cannot be rectified or remedied by COMELEC
Resolution No. 2300, since the COMELEC is without authority to legislate the
1. Whether R.A. No. 6735, entitled An Act Providing for a System of Initiative and
procedure for a people's initiative under Section 2 of Article XVII of the
Referendum and Appropriating Funds Therefor, was intended to include or
Constitution. That function exclusively pertains to Congress. Section 20 of R.A. No.
cover initiative on amendments to the Constitution; and if so, whether the Act, as
6735 does not constitute a legal basis for the Resolution, as the former does not
worded, adequately covers such initiative.
set a sufficient standard for a valid delegation of power.

2. Whether that portion of COMELEC Resolution No. 2300 (In re: Rules and
On 20 January 1997, Senator Raul Roco filed his Petition in
Regulations Governing the Conduct of Initiative on the Constitution, and Initiative
Intervention. 21 He avers that R.A. No. 6735 is the enabling law that implements the people's right to
and Referendum on National and Local Laws) regarding the conduct of initiative on
initiate constitutional amendments. This law is a consolidation of Senate Bill No. 17 and House Bill No.
amendments to the Constitution is valid, considering the absence in the law of
21505; he co-authored the House Bill and even delivered a sponsorship speech thereon. He likewise
specific provisions on the conduct of such initiative.
submits that the COMELEC was empowered under Section 20 of that law to promulgate COMELEC
Resolution No. 2300. Nevertheless, he contends that the respondent Commission is without jurisdiction
to take cognizance of the Delfin Petition and to order its publication because the said petition is not the 3. Whether the lifting of term limits of elective national and local officials, as
initiatory pleading contemplated under the Constitution, Republic Act No. 6735, and COMELEC proposed in the draft "Petition for Initiative on the 1987 Constitution," would
Resolution No. 2300. What vests jurisdiction upon the COMELEC in an initiative on the Constitution is the constitute a revision of, or an amendment to, the Constitution.
filing of a petition for initiative which is signedby the required number of registered voters. He also
submits that the proponents of a constitutional amendment cannot avail of the authority and resources 4. Whether the COMELEC can take cognizance of, or has jurisdiction over, a
of the COMELEC to assist them is securing the required number of signatures, as the COMELEC's role in petition solely intended to obtain an order (a) fixing the time and dates for
an initiative on the Constitution is limited to the determination of the sufficiency of the initiative petition signature gathering; (b) instructing municipal election officers to assist Delfin's
and the call and supervision of a plebiscite, if warranted. movement and volunteers in establishing signature stations; and (c) directing or
causing the publication of, inter alia, the unsigned proposed Petition for Initiative
On 20 January 1997, LABAN filed a Motion for Leave to Intervene. on the 1987 Constitution.

The following day, the IBP filed a Motion for Intervention to which it attached a Petition in Intervention 5. Whether it is proper for the Supreme Court to take cognizance of the petition
raising the following arguments: when there is a pending case before the COMELEC.

(1) Congress has failed to enact an enabling law mandated under Section 2, Article After hearing them on the issues, we required the parties to submit simultaneously their respective
XVII of the 1987 Constitution. memoranda within twenty days and requested intervenor Senator Roco to submit copies of the
deliberations on House Bill No. 21505.
(2) COMELEC Resolution No. 2300 cannot substitute for the required implementing
law on the initiative to amend the Constitution. On 27 January 1997, LABAN filed its Petition in Intervention wherein it adopts the allegations and
arguments in the main Petition. It further submits that the COMELEC should have dismissed the Delfin
Petition for failure to state a sufficient cause of action and that the Commission's failure or refusal to do
(3) The Petition for Initiative suffers from a fatal defect in that it does not have the
so constituted grave abuse of discretion amounting to lack of jurisdiction.
required number of signatures.

On 28 January 1997, Senator Roco submitted copies of portions of both the Journal and the Record of
(4) The petition seeks, in effect a revision of the Constitution, which can be
the House of Representatives relating to the deliberations of House Bill No. 21505, as well as the
proposed only by Congress or a constitutional convention. 22
transcripts of stenographic notes on the proceedings of the Bicameral Conference Committee,
Committee on Suffrage and Electoral Reforms, of 6 June 1989 on House Bill No. 21505 and Senate Bill
On 21 January 1997, we promulgated a Resolution (a) granting the Motions for Intervention filed by the No. 17.
DIK and MABINI and by the IBP, as well as the Motion for Leave to Intervene filed by LABAN; (b)
admitting the Amended Petition in Intervention of DIK and MABINI, and the Petitions in Intervention of
Private respondents Alberto and Carmen Pedrosa filed their Consolidated Comments on the Petitions in
Senator Roco and of the IBP; (c) requiring the respondents to file within a nonextendible period of five
Intervention of Senator Roco, DIK and MABINI, and IBP. 23 The parties thereafter filed, in due time, their
days their Consolidated Comments on the aforesaid Petitions in Intervention; and (d) requiring LABAN to
separate memoranda. 24
file its Petition in Intervention within a nonextendible period of three days from notice, and the
As we stated in the beginning, we resolved to give due course to this special civil action. It must also be noted that intervenor Roco claims that the COMELEC has no jurisdiction over the Delfin
Petition because the said petition is not supported by the required minimum number of signatures of
registered voters. LABAN also asserts that the COMELEC gravely abused its discretion in refusing to
For a more logical discussion of the formulated issues, we shall first take up the fifth issue which appears
dismiss the Delfin Petition, which does not contain the required number of signatures. In light of these
to pose a prejudicial procedural question.
claims, the instant case may likewise be treated as a special civil action for certiorari under Section I of
Rule 65 of the Rules of Court.
I
In any event, as correctly pointed out by intervenor Roco in his Memorandum, this Court may brush
THE INSTANT PETITION IS VIABLE DESPITE THE PENDENCY IN THE COMELEC OF THE DELFIN aside technicalities of procedure in
PETITION. cases of transcendental importance. As we stated in Kilosbayan, Inc. v. Guingona, Jr. 28

Except for the petitioners and intervenor Roco, the parties paid no serious attention to the fifth A party's standing before this Court is a procedural technicality which it may, in the
issue, i.e., whether it is proper for this Court to take cognizance of this special civil action when there is a exercise of its discretion, set aside in view of the importance of issues raised. In
pending case before the COMELEC. The petitioners provide an affirmative answer. Thus: the landmark Emergency Powers Cases, this Court brushed aside this technicality
because the transcendental importance to the public of these cases demands that
28. The Comelec has no jurisdiction to take cognizance of the petition filed by they be settled promptly and definitely, brushing aside, if we must, technicalities
private respondent Delfin. This being so, it becomes imperative to stop the of procedure.
Comelec from proceeding any further, and under the Rules of Court, Rule 65,
Section 2, a petition for prohibition is the proper remedy. II

29. The writ of prohibition is an extraordinary judicial writ issuing out of a court of R.A. NO. 6735 INTENDED TO INCLUDE THE SYSTEM OF INITIATIVE ON AMENDMENTS TO THE
superior jurisdiction and directed to an inferior court, for the purpose of CONSTITUTION, BUT IS, UNFORTUNATELY, INADEQUATE TO COVER THAT SYSTEM.
preventing the inferior tribunal from usurping a jurisdiction with which it is not
legally vested. (People v. Vera, supra., p. 84). In this case the writ is an urgent
Section 2 of Article XVII of the Constitution provides:
necessity, in view of the highly divisive and adverse environmental consequences
on the body politic of the questioned Comelec order. The consequent climate of
legal confusion and political instability begs for judicial statesmanship. Sec. 2. Amendments to this Constitution may likewise be directly proposed by the
people through initiative upon a petition of at least twelve per centum of the total
number of registered voters, of which every legislative district must be
30. In the final analysis, when the system of constitutional law is threatened by the
represented by at least three per centum of the registered voters therein. No
political ambitions of man, only the Supreme Court
amendment under this section shall be authorized within five years following the
can save a nation in peril and uphold the paramount majesty of the Constitution. 25
ratification of this Constitution nor oftener than once every five years thereafter.

It must be recalled that intervenor Roco filed with the COMELEC a motion to dismiss the Delfin Petition
The Congress shall provide for the implementation of the exercise of this right.
on the ground that the COMELEC has no jurisdiction or authority to entertain the petition. 26 The
COMELEC made no ruling thereon evidently because after having heard the arguments of Delfin and the
oppositors at the hearing on 12 December 1996, it required them to submit within five days their This provision is not self-executory. In his book, 29 Joaquin Bernas, a member of the 1986 Constitutional
memoranda or oppositions/memoranda. 27 Earlier, or specifically on 6 December 1996, it practically gave Commission, stated:
due course to the Delfin Petition by ordering Delfin to cause the publication of the petition, together
with the attached Petition for Initiative, the signature form, and the notice of hearing; and by setting the
Without implementing legislation Section 2 cannot operate. Thus, although this
case for hearing. The COMELEC's failure to act on Roco's motion to dismiss and its insistence to hold on
mode of amending the Constitution is a mode of amendment which bypasses
to the petition rendered ripe and viable the instant petition under Section 2 of Rule 65 of the Rules of
congressional action, in the last analysis it still is dependent on congressional
Court, which provides:
action.

Sec. 2. Petition for prohibition. Where the proceedings of any tribunal,


Bluntly stated, the right of the people to directly propose amendments to the Constitution
corporation, board, or person, whether exercising functions judicial or ministerial,
through the system of initiative would remain entombed in the cold niche of the Constitution
are without or in excess of its or his jurisdiction, or with grave abuse of discretion,
until Congress provides for its implementation. Stated otherwise, while the Constitution has
and there is no appeal or any other plain, speedy and adequate remedy in the
recognized or granted that right, the people cannot exercise it if Congress, for whatever
ordinary course of law, a person aggrieved thereby may file a verified petition in
reason, does not provide for its implementation.
the proper court alleging the facts with certainty and praying that judgment be
rendered commanding the defendant to desist from further proceedings in the
action or matter specified therein. This system of initiative was originally included in Section 1 of the draft Article on Amendment or
Revision proposed by the Committee on Amendments and Transitory Provisions of the 1986
Constitutional Commission in its Committee Report No. 7 (Proposed Resolution No. 332). 30 That section appropriations which would have to be legislated so that the
reads as follows: plebiscite could be called. We deemed it best that this
matter be left to the legislature. The Gentleman is right. In
any event, as envisioned, no amendment through the power
Sec. 1. Any amendment to, or revision of, this Constitution may be proposed:
of initiative can be called until after five years from the date
of the ratification of this Constitution. Therefore, the first
(a) by the National Assembly upon a vote of three-fourths of all its members; or amendment that could be proposed through the exercise of
this initiative power would be after five years. It is
(b) by a constitutional convention; or reasonably expected that within that five-year period, the
National Assembly can come up with the appropriate rules
governing the exercise of this power.
(c) directly by the people themselves thru initiative as provided for in Article___
Section ___of the Constitution. 31
FR. BERNAS. Since the matter is left to the legislature the
details on how this is to be carried out is it possible that,
After several interpellations, but before the period of amendments, the Committee submitted in effect, what will be presented to the people for
a new formulation of the concept of initiative which it denominated as Section 2; thus: ratification is the work of the legislature rather than of the
people? Does this provision exclude that possibility?
MR. SUAREZ. Thank you, Madam President. May we
respectfully call attention of the Members of the MR. SUAREZ. No, it does not exclude that possibility because
Commission that pursuant to the mandate given to us last even the legislature itself as a body could propose that
night, we submitted this afternoon a complete Committee amendment, maybe individually or collectively, if it fails to
Report No. 7 which embodies the proposed provision muster the three-fourths vote in order to constitute itself as
governing the matter of initiative. This is now covered by a constituent assembly and submit that proposal to the
Section 2 of the complete committee report. With the people for ratification through the process of an initiative.
permission of the Members, may I quote Section 2:

xxx xxx xxx


The people may, after five years from the date of the last plebiscite held, directly
propose amendments to this Constitution thru initiative upon petition of at least
ten percent of the registered voters. MS. AQUINO. Do I understand from the sponsor that the
intention in the proposal is to vest constituent power in the
people to amend the Constitution?
This completes the blanks appearing in the original Committee Report No. 7. 32

MR. SUAREZ. That is absolutely correct, Madam President.


The interpellations on Section 2 showed that the details for carrying out Section 2 are left to the
legislature. Thus:
MS. AQUINO. I fully concur with the underlying precept of
the proposal in terms of institutionalizing popular
FR. BERNAS. Madam President, just two simple, clarificatory participation in the drafting of the Constitution or in the
questions. amendment thereof, but I would have a lot of difficulties in
terms of accepting the draft of Section 2, as written. Would
First, on Section 1 on the matter of initiative upon petition the sponsor agree with me that in the hierarchy of legal
of at least 10 percent, there are no details in the provision on mandate, constituent power has primacy over all other legal
how to carry this out. Do we understand, therefore, that we mandates?
are leaving this matter to the legislature?
MR. SUAREZ. The Commissioner is right, Madam President.
MR. SUAREZ. That is right, Madam President.
MS. AQUINO. And would the sponsor agree with me that in
FR. BERNAS. And do we also understand, therefore, that for the hierarchy of legal values, the Constitution is source of all
as long as the legislature does not pass the necessary legal mandates and that therefore we require a great deal of
implementing law on this, this will not operate? circumspection in the drafting and in the amendments of
the Constitution?
MR. SUAREZ. That matter was also taken up during the
committee hearing, especially with respect to the budget MR. SUAREZ. That proposition is nondebatable.
MS. AQUINO. Such that in order to underscore the primacy in Section 1 to include the process of revision; whereas
of constituent power we have a separate article in the the process of initiation to amend, which is given to the
constitution that would specifically cover the process and public, would only apply to amendments?
the modes of amending the Constitution?
MR. SUAREZ. That is right. Those were the terms envisioned
MR. SUAREZ. That is right, Madam President. in the Committee. 35

MS. AQUINO. Therefore, is the sponsor inclined, as the Amendments to the proposed Section 2 were thereafter introduced by then Commissioner Hilario G.
provisions are drafted now, to again concede to the Davide, Jr., which the Committee accepted. Thus:
legislature the process or the requirement of determining the
mechanics of amending the Constitution by people's
MR. DAVIDE. Thank you Madam President. I propose to
initiative?
substitute the entire Section 2 with the following:

MR. SUAREZ. The matter of implementing this could very


MR. DAVIDE. Madam President, I have modified the
well be placed in the hands of the National Assembly, not
proposed amendment after taking into account the
unless we can incorporate into this provision the mechanics
modifications submitted by the sponsor himself and the
that would adequately cover all the conceivable
honorable Commissioners Guingona, Monsod, Rama, Ople,
situations. 33
de los Reyes and Romulo. The modified amendment in
substitution of the proposed Section 2 will now read as
It was made clear during the interpellations that the aforementioned Section 2 is limited to proposals to follows: "SECTION 2. AMENDMENTS TO THIS
AMEND not to REVISE the Constitution; thus: CONSTITUTION MAY LIKEWISE BE DIRECTLY PROPOSED BY
THE PEOPLE THROUGH INITIATIVE UPON A PETITION OF AT
LEAST TWELVE PERCENT OF THE TOTAL NUMBER Of
MR. SUAREZ. . . . This proposal was suggested on the theory
REGISTERED VOTERS, OF WHICH EVERY LEGISLATIVE
that this matter of initiative, which came about because of
DISTRICT MUST BE REPRESENTED BY AT LEAST THREE
the extraordinary developments this year, has to be
PERCENT OF THE REGISTERED VOTERS THEREOF. NO
separated from the traditional modes of amending the
AMENDMENT UNDER THIS SECTION SHALL BE AUTHORIZED
Constitution as embodied in Section 1. The committee
WITHIN FIVE YEARS FOLLOWING THE RATIFICATION OF THIS
members felt that this system of initiative should not extend
CONSTITUTION NOR OFTENER THAN ONCE EVERY FIVE
to the revision of the entire Constitution, so we removed it
YEARS THEREAFTER.
from the operation of Section 1 of the proposed Article on
Amendment or Revision. 34
THE NATIONAL ASSEMBLY SHALL BY LAW PROVIDE FOR THE
IMPLEMENTATION OF THE EXERCISE OF THIS RIGHT.
xxx xxx xxx

MR. SUAREZ. Madam President, considering that the


MS. AQUINO. In which case, I am seriously bothered by
proposed amendment is reflective of the sense contained in
providing this process of initiative as a separate section in
Section 2 of our completed Committee Report No. 7, we
the Article on Amendment. Would the sponsor be amenable
accept the proposed amendment. 36
to accepting an amendment in terms of realigning Section 2
as another subparagraph (c) of Section 1, instead of setting
it up as another separate section as if it were a self- The interpellations which ensued on the proposed modified amendment to Section 2 clearly
executing provision? showed that it was a legislative act which must implement the exercise of the right. Thus:

MR. SUAREZ. We would be amenable except that, as we MR. ROMULO. Under Commissioner Davide's amendment, is
clarified a while ago, this process of initiative is limited to the it possible for the legislature to set forth certain procedures
matter of amendment and should not expand into a to carry out the initiative. . .?
revision which contemplates a total overhaul of the
Constitution. That was the sense that was conveyed by the
MR. DAVIDE. It can.
Committee.

xxx xxx xxx


MS. AQUINO. In other words, the Committee was
attempting to distinguish the coverage of modes (a) and (b)
MR. ROMULO. But the Commissioner's amendment does process of amendment must be made more rigorous and
not prevent the legislature from asking another body to set difficult than probably initiating an ordinary legislation or
the proposition in proper form. putting an end to a law proposed by the National Assembly
by way of a referendum. I cannot agree to reducing the
requirement approved by the Committee on the Legislative
MR. DAVIDE. The Commissioner is correct. In other words,
because it would require another voting by the Committee,
the implementation of this particular right would be subject
and the voting as precisely based on a requirement of 10
to legislation, provided the legislature cannot determine
percent. Perhaps, I might present such a proposal, by way of
anymore the percentage of the requirement.
an amendment, when the Commission shall take up the
Article on the Legislative or on the National Assembly on
MR. ROMULO. But the procedures, including the plenary sessions. 39
determination of the proper form for submission to the
people, may be subject to legislation.
The Davide modified amendments to Section 2 were subjected to amendments, and the final version,
which the Commission approved by a vote of 31 in favor and 3 against, reads as follows:
MR. DAVIDE. As long as it will not destroy the substantive
right to initiate. In other words, none of the procedures to
MR. DAVIDE. Thank you Madam President. Section 2, as
be proposed by the legislative body must diminish or impair
amended, reads as follows: "AMENDMENT TO THIS
the right conceded here.
CONSTITUTION MAY LIKEWISE BE DIRECTLY PROPOSED BY
THE PEOPLE THROUGH INITIATIVE UPON A PETITION OF AT
MR. ROMULO. In that provision of the Constitution can the LEAST TWELVE PERCENT OF THE TOTAL NUMBER OF
procedures which I have discussed be legislated? REGISTERED VOTERS, OF WHICH EVERY LEGISLATIVE
DISTRICT MUST BE REPRESENTED BY AT LEAST THREE
MR. DAVIDE. Yes. 37 PERCENT OF THE REGISTERED VOTERS THEREOF. NO
AMENDMENT UNDER THIS SECTION SHALL BE AUTHORIZED
WITHIN FIVE YEARS FOLLOWING THE RATIFICATION OF THIS
Commissioner Davide also reaffirmed that his modified amendment strictly confines initiative to CONSTITUTION NOR OFTENER THAN ONCE EVERY FIVE
AMENDMENTS to NOT REVISION of the Constitution. Thus: YEARS THEREAFTER.

MR. DAVIDE. With pleasure, Madam President. THE NATIONAL ASSEMBLY SHALL BY LAW PROVIDE
FOR THE IMPLEMENTATION OF THE EXERCISE OF THIS
MR. MAAMBONG. My first question: Commissioner Davide's RIGHT. 40
proposed amendment on line 1 refers to "amendment."
Does it not cover the word "revision" as defined by The entire proposed Article on Amendments or Revisions was approved on second reading on
Commissioner Padilla when he made the distinction 9 July 1986.41 Thereafter, upon his motion for reconsideration, Commissioner Gascon was
between the words "amendments" and "revision"? allowed to introduce an amendment to Section 2 which, nevertheless, was withdrawn. In
view thereof, the Article was again approved on Second and Third Readings on 1 August
MR. DAVIDE. No, it does not, because "amendments" and 1986. 42
"revision" should be covered by Section 1. So insofar as
initiative is concerned, it can only relate to "amendments" However, the Committee on Style recommended that the approved Section 2 be amended by changing
not "revision." 38 "percent" to "per centum" and "thereof" to "therein" and deleting the phrase "by law" in the second
paragraph so that said paragraph reads: The Congress 43 shall provide for the implementation of the
Commissioner Davide further emphasized that the process of proposing amendments exercise of this right. 44 This amendment was approved and is the text of the present second paragraph
through initiative must be more rigorous and difficult than the initiative on legislation. Thus: of Section 2.

MR. DAVIDE. A distinction has to be made that under this The conclusion then is inevitable that, indeed, the system of initiative on the Constitution under Section
proposal, what is involved is an amendment to the 2 of Article XVII of the Constitution is not self-executory.
Constitution. To amend a Constitution would ordinarily
require a proposal by the National Assembly by a vote of Has Congress "provided" for the implementation of the exercise of this right? Those who answer the
three-fourths; and to call a constitutional convention would question in the affirmative, like the private respondents and intervenor Senator Roco, point to us R.A.
require a higher number. Moreover, just to submit the issue No. 6735.
of calling a constitutional convention, a majority of the
National Assembly is required, the import being that the
There is, of course, no other better way for Congress to implement the exercise of the right than through The foregoing conclusion is further buttressed by the fact that this section was lifted from Section 1 of
the passage of a statute or legislative act. This is the essence or rationale of the last minute amendment Senate Bill No. 17, which solely referred to a statement of policy on local initiative and referendum and
by the Constitutional Commission to substitute the last paragraph of Section 2 of Article XVII then appropriately used the phrases "propose and enact," "approve or reject" and "in whole or in part." 52
reading:
Second. It is true that Section 3 (Definition of Terms) of the Act defines initiative on amendments to the
The Congress 45 shall by law provide for the implementation of the exercise of this Constitution and mentions it as one of the three systems of initiative, and that Section 5 (Requirements)
right. restates the constitutional requirements as to the percentage of the registered voters who must submit
the proposal. But unlike in the case of the other systems of initiative, the Act does not provide for the
contents of a petition forinitiative on the Constitution. Section 5, paragraph (c) requires, among other
with
things, statement of the proposed law sought to be enacted, approved or rejected, amended or repealed,
as the case may be. It does not include, as among the contents of the petition, the provisions of the
The Congress shall provide for the implementation of the exercise of this right. Constitution sought to be amended, in the case of initiative on the Constitution. Said paragraph (c) reads
in full as follows:
This substitute amendment was an investiture on Congress of a power to provide for the rules
implementing the exercise of the right. The "rules" means "the details on how [the right] is to (c) The petition shall state the following:
be carried out." 46
c.1 contents or text of the proposed law sought to be enacted, approved or
We agree that R.A. No. 6735 was, as its history reveals, intended to cover initiative to propose rejected, amended or repealed, as the case may be;
amendments to the Constitution. The Act is a consolidation of House Bill No. 21505 and Senate Bill No.
17. The former was prepared by the Committee on Suffrage and Electoral Reforms of the House of
c.2 the proposition;
Representatives on the basis of two House Bills referred to it, viz., (a) House Bill No. 497, 47 which dealt
with the initiative and referendum mentioned
in Sections 1 and 32 of Article VI of the Constitution; and (b) House Bill No. 988, 48 which dealt with the c.3 the reason or reasons therefor;
subject matter of House Bill No. 497, as well as with initiative and referendum under Section 3 of Article
X (Local Government) and initiative provided for in Section 2 of Article XVII of the Constitution. Senate
c.4 that it is not one of the exceptions provided therein;
Bill No. 17 49 solely dealt with initiative and referendum concerning ordinances or resolutions of local
government units. The Bicameral Conference Committee consolidated Senate Bill No. 17 and House Bill
No. 21505 into a draft bill, which was subsequently approved on 8 June 1989 by the Senate 50 and by the c.5 signatures of the petitioners or registered voters; and
House of Representatives. 51 This approved bill is now R.A. No. 6735.
c.6 an abstract or summary proposition is not more than one hundred (100) words
But is R.A. No. 6735 a full compliance with the power and duty of Congress to "provide for the which shall be legibly written or printed at the top of every page of the petition.
implementation of the exercise of the right?" (Emphasis supplied).

A careful scrutiny of the Act yields a negative answer. The use of the clause "proposed laws sought to be enacted, approved or rejected, amended
or repealed" only strengthens the conclusion that Section 2, quoted earlier, excludes initiative
on amendments to the Constitution.
First. Contrary to the assertion of public respondent COMELEC, Section 2 of the Act does not suggest an
initiative on amendments to the Constitution. The said section reads:
Third. While the Act provides subtitles for National Initiative and Referendum (Subtitle II) and for Local
Initiative and Referendum (Subtitle III), no subtitle is provided for initiative on the Constitution. This
Sec. 2. Statement and Policy. The power of the people under a system of
conspicuous silence as to the latter simply means that the main thrust of the Act is initiative and
initiative and referendum to directly propose, enact, approve or reject, in whole or
referendum on national and local laws. If Congress intended R.A. No. 6735 to fully provide for the
in part, the Constitution, laws, ordinances, or resolutions passed by any legislative
implementation of the initiative on amendments to the Constitution, it could have provided for a subtitle
body upon compliance with the requirements of this Act is hereby affirmed,
therefor, considering that in the order of things, the primacy of interest, or hierarchy of values, the right
recognized and guaranteed. (Emphasis supplied).
of the people to directly propose amendments to the Constitution is far more important than the
initiative on national and local laws.
The inclusion of the word "Constitution" therein was a delayed afterthought. That word is
neither germane nor relevant to said section, which exclusively relates to initiative and
We cannot accept the argument that the initiative on amendments to the Constitution is subsumed
referendum on national laws and local laws, ordinances, and resolutions. That section is silent
under the subtitle on National Initiative and Referendum because it is national in scope. Our reading of
as to amendments on the Constitution. As pointed out earlier, initiative on the Constitution is
Subtitle II (National Initiative and Referendum) and Subtitle III (Local Initiative and Referendum) leaves
confined only to proposals to AMEND. The people are not accorded the power to "directly
no room for doubt that the classification is not based on the scope of the initiative involved, but on
propose, enact, approve, or reject, in whole or in part, the Constitution" through the system
its nature and character. It is "national initiative," if what is proposed to be adopted or enacted is
of initiative. They can only do so with respect to "laws, ordinances, or resolutions."
a national law, or a law which only Congress can pass. It is "local initiative" if what is proposed to be
adopted or enacted is a law, ordinance, or resolution which only the legislative bodies of the Upon the other hand, Section 18 on "Authority of Courts" under subtitle III on Local Initiative and
governments of the autonomous regions, provinces, cities, municipalities, and barangays can pass. This Referendum is misplaced, 54 since the provision therein applies to both national and local initiative and
classification of initiative into national and local is actually based on Section 3 of the Act, which we quote referendum. It reads:
for emphasis and clearer understanding:
Sec. 18. Authority of Courts. Nothing in this Act shall prevent or preclude the
Sec. 3. Definition of terms proper courts from declaring null and void any proposition approved pursuant to
this Act for violation of the Constitution or want of capacity of the local legislative
body to enact the said measure.
xxx xxx xxx

Curiously, too, while R.A. No. 6735 exerted utmost diligence and care in providing for the details in the
There are three (3) systems of initiative, namely:
implementation of initiative and referendum on national and local legislation thereby giving them special
attention, it failed, rather intentionally, to do so on the system of initiative on amendments to the
a.1 Initiative on the Constitution which refers to a petition proposing amendments Constitution. Anent the initiative on national legislation, the Act provides for the following:
to the Constitution;
(a) The required percentage of registered voters to sign the petition and the contents of the petition;
a.2 Initiative on Statutes which refers to a petition proposing to enact a national
legislation; and
(b) The conduct and date of the initiative;

a.3 Initiative on local legislation which refers to a petition proposing to enact a


(c) The submission to the electorate of the proposition and the required number of votes for its
regional, provincial, city, municipal, or barangay law, resolution or ordinance.
approval;
(Emphasis supplied).

(d) The certification by the COMELEC of the approval of the proposition;


Hence, to complete the classification under subtitles there should have been a subtitle on initiative on
amendments to the Constitution. 53
(e) The publication of the approved proposition in the Official Gazette or in a newspaper of general
circulation in the Philippines; and
A further examination of the Act even reveals that the subtitling is not accurate. Provisions not germane
to the subtitle on National Initiative and Referendum are placed therein, like (1) paragraphs (b) and (c) of
Section 9, which reads: (f) The effects of the approval or rejection of the proposition. 55

(b) The proposition in an initiative on the Constitution approved by the majority of As regards local initiative, the Act provides for the following:
the votes cast in the plebiscite shall become effective as to the day of the
plebiscite.
(a) The preliminary requirement as to the number of signatures of registered voters for the petition;

(c) A national or local initiative proposition approved by majority of the votes cast
(b) The submission of the petition to the local legislative body concerned;
in an election called for the purpose shall become effective fifteen (15) days after
certification and proclamation of the Commission. (Emphasis supplied).
(c) The effect of the legislative body's failure to favorably act thereon, and the invocation of the power of
initiative as a consequence thereof;
(2) that portion of Section 11 (Indirect Initiative) referring to indirect initiative with the legislative bodies
of local governments; thus:
(d) The formulation of the proposition;
Sec. 11. Indirect Initiative. Any duly accredited people's organization, as defined
by law, may file a petition for indirect initiative with the House of Representatives, (e) The period within which to gather the signatures;
and other legislative bodies. . . .
(f) The persons before whom the petition shall be signed;
and (3) Section 12 on Appeal, since it applies to decisions of the COMELEC on the findings of
sufficiency or insufficiency of the petition for initiative or referendum, which could be (g) The issuance of a certification by the COMELEC through its official in the local government unit
petitions for both national and localinitiative and referendum. concerned as to whether the required number of signatures have been obtained;
(h) The setting of a date by the COMELEC for the submission of the proposition to the registered voters specifies the public agency to apply it. It indicates the circumstances under which the legislative
for their approval, which must be within the period specified therein; command is to be effected. 62

(i) The issuance of a certification of the result; Insofar as initiative to propose amendments to the Constitution is concerned, R.A. No. 6735 miserably
failed to satisfy both requirements in subordinate legislation. The delegation of the power to the
COMELEC is then invalid.
(j) The date of effectivity of the approved proposition;

III
(k) The limitations on local initiative; and

COMELEC RESOLUTION NO. 2300, INSOFAR AS IT PRESCRIBES RULES AND REGULATIONS ON


(l) The limitations upon local legislative bodies. 56
THE CONDUCT OF INITIATIVE ON AMENDMENTS TO THE CONSTITUTION, IS VOID.

Upon the other hand, as to initiative on amendments to the Constitution, R.A. No. 6735, in all of its
It logically follows that the COMELEC cannot validly promulgate rules and regulations to implement the
twenty-three sections, merely (a) mentions, the word "Constitution" in Section 2; (b) defines "initiative
exercise of the right of the people to directly propose amendments to the Constitution through the
on the Constitution" and includes it in the enumeration of the three systems of initiative in Section 3; (c)
system of initiative. It does not have that power under R.A. No. 6735. Reliance on the COMELEC's power
speaks of "plebiscite" as the process by which the proposition in an initiative on the Constitution may be
under Section 2(1) of Article IX-C of the Constitution is misplaced, for the laws and regulations referred
approved or rejected by the people; (d) reiterates the constitutional requirements as to the number of
to therein are those promulgated by the COMELEC under (a) Section 3 of Article IX-C of the Constitution,
voters who should sign the petition; and (e) provides for the date of effectivity of the approved
or (b) a law where subordinate legislation is authorized and which satisfies the "completeness" and the
proposition.
"sufficient standard" tests.

There was, therefore, an obvious downgrading of the more important or the paramount system of
IV
initiative. RA. No. 6735 thus delivered a humiliating blow to the system of initiative on amendments to
the Constitution by merely paying it a reluctant lip service. 57
COMELEC ACTED WITHOUT JURISDICTION OR WITH GRAVE ABUSE OF DISCRETION IN
ENTERTAINING THE DELFIN PETITION.
The foregoing brings us to the conclusion that R.A. No. 6735 is incomplete, inadequate, or wanting in
essential terms and conditions insofar as initiative on amendments to the Constitution is concerned. Its
lacunae on this substantive matter are fatal and cannot be cured by "empowering" the COMELEC "to Even if it be conceded ex gratia that R.A. No. 6735 is a full compliance with the power of Congress to
promulgate such rules and regulations as may be necessary to carry out the purposes of [the] Act. 58 implement the right to initiate constitutional amendments, or that it has validly vested upon the
COMELEC the power of subordinate legislation and that COMELEC Resolution No. 2300 is valid, the
COMELEC acted without jurisdiction or with grave abuse of discretion in entertaining the Delfin Petition.
The rule is that what has been delegated, cannot be delegated or as expressed in a Latin maxim: potestas
delegata non delegari potest. 59 The recognized exceptions to the rule are as follows:
Under Section 2 of Article XVII of the Constitution and Section 5(b) of R.A. No. 6735, a petition for
initiative on the Constitution must be signed by at least 12% of the total number of registered voters of
(1) Delegation of tariff powers to the President under Section 28(2) of Article VI of the Constitution;
which every legislative district is represented by at least 3% of the registered voters therein. The Delfin
Petition does not contain signatures of the required number of voters. Delfin himself admits that he has
(2) Delegation of emergency powers to the President under Section 23(2) of Article VI of the not yet gathered signatures and that the purpose of his petition is primarily to obtain assistance in his
Constitution; drive to gather signatures. Without the required signatures, the petition cannot be deemed validly
initiated.
(3) Delegation to the people at large;
The COMELEC acquires jurisdiction over a petition for initiative only after its filing. The petition then is
(4) Delegation to local governments; and the initiatory pleading. Nothing before its filing is cognizable by the COMELEC, sitting en banc. The only
participation of the COMELEC or its personnel before the filing of such petition are (1) to prescribe the
form of the petition; 63 (2) to issue through its Election Records and Statistics Office a certificate on the
(5) Delegation to administrative bodies. 60 total number of registered voters in each legislative district; 64 (3) to assist, through its election registrars,
in the establishment of signature stations; 65 and (4) to verify, through its election registrars, the
Empowering the COMELEC, an administrative body exercising quasi-judicial functions, to promulgate signatures on the basis of the registry list of voters, voters' affidavits, and voters' identification cards
rules and regulations is a form of delegation of legislative authority under no. 5 above. However, in every used in the immediately preceding election. 66
case of permissible delegation, there must be a showing that the delegation itself is valid. It is valid only
if the law (a) is complete in itself, setting forth therein the policy to be executed, carried out, or Since the Delfin Petition is not the initiatory petition under R.A. No. 6735 and COMELEC Resolution No.
implemented by the delegate; and (b) fixes a standard the limits of which are sufficiently determinate 2300, it cannot be entertained or given cognizance of by the COMELEC. The respondent Commission
and determinable to which the delegate must conform in the performance of his functions. 61 A must have known that the petition does not fall under any of the actions or proceedings under the
sufficient standard is one which defines legislative policy, marks its limits, maps out its boundaries and
COMELEC Rules of Procedure or under Resolution No. 2300, for which reason it did not assign to the
petition a docket number. Hence, the said petition was merely entered as UND, meaning, undocketed.
That petition was nothing more than a mere scrap of paper, which should not have been dignified by the
Order of 6 December 1996, the hearing on 12 December 1996, and the order directing Delfin and the
oppositors to file their memoranda or oppositions. In so dignifying it, the COMELEC acted without
jurisdiction or with grave abuse of discretion and merely wasted its time, energy, and resources.

The foregoing considered, further discussion on the issue of whether the proposal to lift the term limits
of elective national and local officials is an amendment to, and not a revision of, the Constitution is
rendered unnecessary, if not academic.

CONCLUSION

This petition must then be granted, and the COMELEC should be permanently enjoined from
entertaining or taking cognizance of any petition for initiative on amendments to the Constitution until a
sufficient law shall have been validly enacted to provide for the implementation of the system.

We feel, however, that the system of initiative to propose amendments to the Constitution should no
longer be kept in the cold; it should be given flesh and blood, energy and strength. Congress should not
tarry any longer in complying with the constitutional mandate to provide for the implementation of the
right of the people under that system.

WHEREFORE, judgment is hereby rendered

a) GRANTING the instant petition;

b) DECLARING R.A. No. 6735 inadequate to cover the system of initiative on amendments to the
Constitution, and to have failed to provide sufficient standard for subordinate legislation;

c) DECLARING void those parts of Resolution No. 2300 of the Commission on Elections prescribing rules
and regulations on the conduct of initiative or amendments to the Constitution; and

d) ORDERING the Commission on Elections to forthwith DISMISS the DELFIN petition (UND-96-037).

The Temporary Restraining Order issued on 18 December 1996 is made permanent as against the
Commission on Elections, but is LIFTED as against private respondents.

Resolution on the matter of contempt is hereby reserved.

SO ORDERED
Republic of the Philippines The questions ask, to wit:
SUPREME COURT
Manila
(1) Do you want martial law to be continued?

EN BANC
(2) Whether or not you want martial law to be continued, do you approve the following amendments to
the Constitution? For the purpose of the second question, the referendum shall have the effect of a
G.R. No. L-44640 October 12, 1976 plebiscite within the contemplation of Section 2 of Article XVI of the Constitution.

PABLO C. SANIDAD and PABLITO V. SANIDAD, petitioner, PROPOSED AMENDMENTS:


vs.
HONORABLE COMMISSION ON ELECTIONS and HONORABLE NATIONAL TREASURER, respondents.
1. There shall be, in lieu of the interim National Assembly, an interim Batasang Pambansa. Members of
the interim Batasang Pambansa which shall not be more than 120, unless otherwise provided by law,
G.R. No. L-44684. October 12,1976 shall include the incumbent President of the Philippines, representatives elected from the different
regions of the nation, those who shall not be less than eighteen years of age elected by their respective
sectors, and those chosen by the incumbent President from the members of the Cabinet. Regional
VICENTE M. GUZMAN, petitioner,
representatives shall be apportioned among the regions in accordance with the number of their
vs.
respective inhabitants and on the basis of a uniform and progressive ratio while the sectors shall be
COMMISSION ELECTIONS, respondent.
determined by law. The number of representatives from each region or sector and the, manner of their
election shall be prescribed and regulated by law.
G.R. No. L-44714. October 12,1976
2. The interim Batasang Pambansa shall have the same powers and its members shall have the same
RAUL M. GONZALES, RAUL T. GONZALES, JR., and ALFREDO SALAPANTAN, petitioners, functions, responsibilities, rights, privileges, and disqualifications as the interim National Assembly and
vs. the regular National Assembly and the members thereof. However, it shall not exercise the power
HONORABLE COMMISSION ON SELECTIONS and HONORABLE NATIONAL TREASURER, respondents. provided in Article VIII, Section 14(l) of the Constitution.

MARTIN, J,: 3. The incumbent President of the Philippines shall, within 30 days from the election and selection of the
members, convene the interim Batasang Pambansa and preside over its sessions until the Speaker shall
The capital question raised in these prohibition suits with preliminary injunction relates to the power of have been elected. The incumbent President of the Philippines shall be the Prime Minister and he shall
the incumbent President of the Philippines to propose amendments to the present Constitution in the continue to exercise all his powers even after the interim Batasang Pambansa is organized and ready to
absence of the interim National Assembly which has not been convened. discharge its functions and likewise he shall continue to exercise his powers and prerogatives under the
nineteen hundred and thirty five. Constitution and the powers vested in the President and the Prime
Minister under this Constitution.
On September 2, 1976, President Ferdinand E. Marcos issued Presidential Decree No. 991 calling for a
national referendum on October 16, 1976 for the Citizens Assemblies ("barangays") to resolve, among
other things, the issues of martial law, the I . assembly, its replacement, the powers of such replacement, 4. The President (Prime Minister) and his Cabinet shall exercise all the powers and functions, and
the period of its existence, the length of the period for tile exercise by the President of his present discharge the responsibilities of the regular President (Prime Minister) and his Cabinet, and shall be
powers.1 subject only to such disqualifications as the President (Prime Minister) may prescribe. The President
(Prime Minister) if he so desires may appoint a Deputy Prime Minister or as many Deputy Prime
Ministers as he may deem necessary.
Twenty days after or on September 22, 1976, the President issued another related decree, Presidential
Decree No. 1031, amending the previous Presidential Decree No. 991, by declaring the provisions of
presidential Decree No. 229 providing for the manner of voting and canvass of votes in "barangays" 5. The incumbent President shall continue to exercise legislative powers until martial law shall have been
(Citizens Assemblies) applicable to the national referendum-plebiscite of October 16, 1976. Quite lifted.
relevantly, Presidential Decree No. 1031 repealed Section 4, of Presidential Decree No. 991, the full text
of which (Section 4) is quoted in the footnote below. 2 6. Whenever in the judgment of the President (Prime Minister), there exists a grave emergency or a
threat or imminence thereof, or whenever the interim Batasang Pambansa or the regular National
On the same date of September 22, 1976, the President issued Presidential Decree No. 1033, stating the Assembly fails or is unable to act adequately on any matter for any reason that in his judgment requires
questions to be submitted to the people in the referendum-plebiscite on October 16, 1976. The Decree immediate action, he may, in order to meet the exigency, issue the necessary decrees, orders or letters
recites in its "whereas" clauses that the people's continued opposition to the convening of the National of instructions, which shall form part of the law of the land.
Assembly evinces their desire to have such body abolished and replaced thru a constitutional
amendment, providing for a legislative body, which will be submitted directly to the people in the 7. The barangays and sanggunians shall continue as presently constituted but their functions, powers,
referendum-plebiscite of October 16. and composition may be altered by law.
Referenda conducted thru the barangays and under the Supervision of the Commission on Elections may I
be called at any time the government deems it necessary to ascertain the will of the people regarding
any important matter whether of national or local interest.
Justiciability of question raised.

8. All provisions of this Constitution not inconsistent with any of these amendments shall continue in full
1. As a preliminary resolution, We rule that the petitioners in L-44640 (Pablo C. Sanidad and Pablito V.
force and effect.
Sanidad) possess locus standi to challenge the constitutional premise of Presidential Decree Nos. 991,
1031, and 1033. It is now an ancient rule that the valid source of a stature Presidential Decrees are of
9. These amendments shall take effect after the incumbent President shall have proclaimed that they such nature-may be contested by one who will sustain a direct injuries as a in result of its enforcement.
have been ratified by I majority of the votes cast in the referendum-plebiscite." At the instance of taxpayers, laws providing for the disbursement of public funds may be enjoined, upon
the theory that the expenditure of public funds by an officer of the State for the purpose of executing an
unconstitutional act constitutes a misapplication of such funds. 4 The breadth of Presidential Decree No.
The Commission on Elections was vested with the exclusive supervision and control of the October 1976
991 carries all appropriation of Five Million Pesos for the effective implementation of its purposes. 5
National Referendum-Plebiscite.
Presidential Decree No. 1031 appropriates the sum of Eight Million Pesos to carry out its provisions. 6
The interest of the aforenamed petitioners as taxpayers in the lawful expenditure of these amounts of
On September 27, 1976, PABLO C. SANIDAD and PABLITO V. SANIDAD, father and son, commenced L- public money sufficiently clothes them with that personality to litigate the validity of the Decrees
44640 for Prohibition with Preliminary Injunction seeking to enjoin the Commission on Elections from appropriating said funds. Moreover, as regards taxpayer's suits, this Court enjoys that open discretion to
holding and conducting the Referendum Plebiscite on October 16; to declare without force and effect entertain the same or not. 7 For the present case, We deem it sound to exercise that discretion
Presidential Decree Nos. 991 and 1033, insofar as they propose amendments to the Constitution, as well affirmatively so that the authority upon which the disputed Decrees are predicated may be inquired into.
as Presidential Decree No. 1031, insofar as it directs the Commission on Elections to supervise, control,
hold, and conduct the Referendum-Plebiscite scheduled on October 16, 1976.
2. The Solicitor General would consider the question at bar as a pure political one, lying outside the
domain of judicial review. We disagree. The amending process both as to proposal and ratification, raises
Petitioners contend that under the 1935 and 1973 Constitutions there is no grant to the incumbent a judicial question. 8 This is especially true in cases where the power of the Presidency to initiate the of
President to exercise the constituent power to propose amendments to the new Constitution. As a normally exercised by the legislature, is seriously doubted. Under the terms of the 1973 Constitution, the
consequence, the Referendum-Plebiscite on October 16 has no constitutional or legal basis. power to propose amendments o the constitution resides in the interim National Assembly in the period
of transition (See. 15, Transitory provisions). After that period, and the regular National Assembly in its
On October 5, 1976, the Solicitor General filed the comment for respondent Commission on Elections, active session, the power to propose amendments becomes ipso facto the prerogative of the regular
The Solicitor General principally maintains that petitioners have no standing to sue; the issue raised is National Assembly (Sec. 1, pars. 1 and 2 of Art. XVI, 1973 constitution). The normal course has not been
political in nature, beyond judicial cognizance of this Court; at this state of the transition period, only the followed. Rather than calling the National Assembly to constitute itself into a constituent assembly the
incumbent President has the authority to exercise constituent power; the referendum-plebiscite is a step incumbent President undertook the proposal of amendments and submitted the proposed amendments
towards normalization. thru Presidential Decree 1033 to the people in a Referendum-Plebiscite on October 16. Unavoidably, the
regularity regularity of the procedure for amendments, written in lambent words in the very Constitution
sought to be amended, raises a contestable issue. The implementing Presidential Decree Nos. 991, 1031,
On September 30, 1976, another action for Prohibition with Preliminary Injunction, docketed as L-44684, and 1033, which commonly purport to have the force and effect of legislation are assailed as invalid, thus
was instituted by VICENTE M. GUZMAN, a delegate to the 1971 Constitutional Convention, asserting that the issue of the validity of said Decrees is plainly a justiciable one, within the competence of this Court to
the power to propose amendments to, or revision of the Constitution during the transition period is pass upon. Section 2 (2), Article X of the new Constitution provides: "All cases involving the
expressly conferred on the interim National Assembly under Section 16, Article XVII of the Constitution.3 constitutionality of a treaty, executive agreement, or law may shall be heard and decided by the
Supreme Court en banc and no treaty, executive agreement, or law may be declared unconstitutional
Still another petition for Prohibition with Preliminary Injunction was filed on October 5, 1976 by RAUL M. without the concurrence of at least ten Members. ..." The Supreme Court has the last word in the
GONZALES, his son RAUL, JR., and ALFREDO SALAPANTAN, docketed as L- 44714, to restrain the construction not only of treaties and statutes, but also of the Constitution itself The amending, like all
implementation of Presidential Decrees relative to the forthcoming Referendum-Plebiscite of October other powers organized in the Constitution, is in form a delegated and hence a limited power, so that the
16. Supreme Court is vested with that authorities to determine whether that power has been discharged
within its limits.
These last petitioners argue that even granting him legislative powers under Martial Law, the incumbent
President cannot act as a constituent assembly to propose amendments to the Constitution; a Political questions are neatly associated with the wisdom, of the legality of a particular act. Where the
referendum-plebiscite is untenable under the Constitutions of 1935 and 1973; the submission of the vortex of the controversy refers to the legality or validity of the contested act, that matter is definitely
proposed amendments in such a short period of time for deliberation renders the plebiscite a nullity; to justiciable or non-political. What is in the heels of the Court is not the wisdom of the act of the
lift Martial Law, the President need not consult the people via referendum; and allowing 15-.year olds to incumbent President in proposing amendments to the Constitution, but his constitutional authority to
vote would amount to an amendment of the Constitution, which confines the right of suffrage to those perform such act or to assume the power of a constituent assembly. Whether the amending process
citizens of the Philippines 18 years of age and above. confers on the President that power to propose amendments is therefore a downright justiciable
question. Should the contrary be found, the actuation of the President would merely be a brutum
fulmen. If the Constitution provides how it may be amended, the judiciary as the interpreter of that
We find the petitions in the three entitled cases to be devoid of merit. Constitution, can declare whether the procedure followed or the authority assumed was valid or not. 10
We cannot accept the view of the Solicitor General, in pursuing his theory of non-justiciability, that the vote of two-thirds of all its Members, call a constitutional convention or, by a
question of the President's authority to propose amendments and the regularity of the procedure majority vote of all its Members, submit the question of calling such a convention
adopted for submission of the proposal to the people ultimately lie in the judgment of the A clear to the electorate in an election.
Descartes fallacy of vicious circle. Is it not that the people themselves, by their sovereign act, provided
for the authority and procedure for the amending process when they ratified the present Constitution in
SECTION 2. Any amendment to, or revision of, this Constitution shall be valid when
1973? Whether, therefore, the constitutional provision has been followed or not is the proper subject of
ratified by a majority of the votes cast in a plebiscite which shall be held not later
inquiry, not by the people themselves of course who exercise no power of judicial but by the Supreme
than three months after the approval of such amendment or revision.
Court in whom the people themselves vested that power, a power which includes the competence to
determine whether the constitutional norms for amendments have been observed or not. And, this
inquiry must be done a prior not a posterior i.e., before the submission to and ratification by the people. In the present period of transition, the interim National Assembly instituted in the Transitory Provisions
is conferred with that amending power. Section 15 of the Transitory Provisions reads:
Indeed, the precedents evolved by the Court or, prior constitutional cases underline the preference of
the Court's majority to treat such issue of Presidential role in the amending process as one of non- SECTION 15. The interim National Assembly, upon special call by the interim Prime
political impression. In the Plebiscite Cases, 11 the contention of the Solicitor General that the issue on Minister, may, by a majority vote of all its Members, propose amendments to this
the legality of Presidential Decree No. 73 "submitting to the Pilipino people (on January 15, 1973) for Constitution. Such amendments shall take effect when ratified in accordance with
ratification or rejection the Constitution of the Republic of the Philippines proposed by the 1971 Article Sixteen hereof.
Constitutional Convention and appropriating fund s therefore "is a political one, was rejected and the
Court unanimously considered the issue as justiciable in nature. Subsequently in the Ratification There are, therefore, two periods contemplated in the constitutional life of the nation, i.e., period of
Cases 12involving the issue of whether or not the validity of Presidential Proclamation No. 1102. normalcy and period of transition. In times of normally, the amending process may be initiated by the
announcing the Ratification by the Filipino people of the constitution proposed by the 1971 proposals of the (1) regular National Assembly upon a vote of three-fourths of all its members; or (2) by
Constitutional Convention," partakes of the nature of a political question, the affirmative stand of' the a Constitutional Convention called by a vote of two-thirds of all the Members of the National Assembly.
Solicitor General was dismissed, the Court ruled that the question raised is justiciable. Chief Justice However the calling of a Constitutional Convention may be submitted to the electorate in an election
Concepcion, expressing the majority view, said, Thus, in the aforementioned plebiscite cases, We voted upon by a majority vote of all the members of the National Assembly. In times of transition,
rejected the theory of the respondents therein that the question whether Presidential Decree No. 73 amendments may be proposed by a majority vote of all the Members of the National Assembly upon
calling a plebiscite to be held on January 15, 1973, for the ratification or rejection of the proposed new special call by the interim Prime Minister,.
Constitution, was valid or not, was not a proper subject of judicial inquiry because, they claimed, it
partook of a political nature, and We unanimously declared that the issue was a justiciable one. With
Identical unanimity. We overruled the respondent's contention in the 1971 habeas corpus cases, 2. This Court in Aquino v. COMELEC," had already settled that the incumbent President is vested with
questioning Our authority to determine the constitutional sufficiency of the factual bases of the that prerogative of discretion as to when he shall initially convene the interim National Assembly.
Presidential proclamation suspending the privilege of the writ of habeas corpus on August 21, 1971, Speaking for the majority opinion in that case, Justice Makasiar said: "The Constitutional Convention
despite the opposite view taken by this Court in Barcelon vs. Baker and Montenegro vs. Castaneda, intended to leave to the President the determination of the time when he shall initially convene the
insofar as it adhered to the former case, which view We, accordingly, abandoned and refused to apply. interim National Assembly, consistent with the prevailing conditions of peace and order in the country."
For the same reason, We did not apply and expressly modified, in Gonzales vs. Commission on Elections, Concurring, Justice Fernandez, himself a member of that Constitutional Convention, revealed: "(W)hen
the political-question theory adopted in Mabanag vs. Lopez Vito." 13 The return to Barcelon vs. Baker and the Delegates to the Constitutional Convention voted on the Transitory Provisions, they were aware of
Mabanag vs. Lopez Vito, urged by the Solicitor General, was decisively refused by the Court. Chief Justice the fact that under the same, the incumbent President was given the discretion as to when he could
Concepcion continued: "The reasons adduced in support thereof are, however, substantially the same as convene the interim National Assembly; it was so stated plainly by the sponsor, Delegate Yaneza; as a
those given in support on the political question theory advanced in said habeas corpus and plebiscite matter of fact, the proposal that it be convened 'immediately', made by Delegate Pimentel (V) was
cases, which were carefully considered by this Court and found by it to be legally unsound and rejected. The President's decision to defer the convening of the interim National Assembly soon found
constitutionally untenable. As a consequence. Our decisions in the aforementioned habeas corpus cases support from the people themselves. In the plebiscite of January 10-15, 1973, at which the ratification of
partakes of the nature and effect of a stare decisis which gained added weight by its virtual reiteration." the 1973 Constitution was submitted, the people voted against the convening of the interim National
Assembly. In the referendum of July 24, 1973, the Citizens Assemblies ("bagangays") reiterated their
sovereign will to withhold the convening of the interim National Assembly. Again, in the referendum of
II February 27, 1975, the proposed question of whether the interim National Assembly shall be initially
convened was eliminated, because some of the members of Congress and delegates of the
The amending process as laid out Constitutional Convention, who were deemed automatically members of the I interim National
Assembly, were against its inclusion since in that referendum of January, 1973, the people had already
resolved against it.
in the new Constitution.

3. In sensu strictiore, when the legislative arm of the state undertakes the proposals of amendment to a
1. Article XVI of the 1973 Constitution on Amendments ordains:
Constitution, that body is not in the usual function of lawmaking. lt is not legislating when engaged in the
amending process.16 Rather, it is exercising a peculiar power bestowed upon it by the fundamental
SECTION 1. (1) Any amendment to, or revision of, this Constitution may be charter itself. In the Philippines, that power is provided for in Article XVI of the 1973 Constitution (for the
proposed by the National Assembly upon a vote of three-fourths of all its regular National Assembly) or in Section 15 of the Transitory Provisions (for the National Assembly).
Members, or by a constitutional convention. (2) The National Assembly may, by a While ordinarily it is the business of the legislating body to legislate for the nation by virtue of
constitutional conferment amending of the Constitution is not legislative in character. In political science Minister, who shall then exercise their respective powers vested by this
a distinction is made between constitutional content of an organic character and that of a legislative Constitution.
character'. The distinction, however, is one of policy, not of law. 17 Such being the case, approval of the
President of any proposed amendment is a misnomer 18 The prerogative of the President to approve or
All proclamations, orders, decrees, instructions, and acts promulgated, issued, or
disapprove applies only to the ordinary cases of legislation. The President has nothing to do with
done by the incumbent President shall be part of the law of the land, and shall
proposition or adoption of amendments to the Constitution. 19
remain valid, binding, and effective even after lifting of martial law or the
ratification of this Constitution, unless modified, revoked, or superseded by
III subsequent proclamations, orders, decrees, instructions, or other acts of the
incumbent President, or unless expressly and explicitly modified or repealed by
the regular National Assembly.
Concentration of Powers

"It is unthinkable," said Justice Fernandez, a 1971 Constitutional Convention delegate, "that the
in the President during
Constitutional Convention, while giving to the President the discretion when to call the interim National
Assembly to session, and knowing that it may not be convened soon, would create a vacuum in the
crisis government. exercise of legislative powers. Otherwise, with no one to exercise the lawmaking powers, there would be
paralyzation of the entire governmental machinery."24 Paraphrasing Rossiter, this is an extremely
1. In general, the governmental powers in crisis government the Philippines is a crisis government today important factor in any constitutional dictatorship which extends over a period of time. The separation
are more or less concentrated in the President. 20 According to Rossiter, "(t)he concentration of of executive and legislature ordained in the Constitution presents a distinct obstruction to efficient crisis
government power in a democracy faced by an emergency is a corrective to the crisis inefficiencies government. The steady increase in executive power is not too much a cause for as the steady increase
inherent in the doctrine of the separation of powers. In most free states it has generally been regarded in the magnitude and complexity of the problems the President has been called upon by the Filipino
as imperative that the total power of the government be parceled out among three mutually people to solve in their behalf, which involve rebellion, subversion, secession, recession, inflation, and
independent branches executive, legislature, and judiciary. It is believed to be destructive of economic crisis-a crisis greater than war. In short, while conventional constitutional law just confines the
constitutionalism if any one branch should exercise any two or more types of power, and certainly a total President's power as Commander-in-Chief to the direction of the operation of the national forces, yet
disregard of the separation of powers is, as Madison wrote in the Federalist, No. 47, 'the very definition the facts of our political, social, and economic disturbances had convincingly shown that in meeting the
of tyranny.' In normal times the separation of powers forms a distinct obstruction to arbitrary same, indefinite power should be attributed to tile President to take emergency measures 25
governmental action. By this same token, in abnormal times it may form an insurmountable barrier to a
decisive emergency action in behalf of the state and its independent existence. There are moments in IV
the life of any government when all powers must work together in unanimity of purpose and action,
even if this means the temporary union of executive, legislative, and judicial power in the hands of one
Authority of the incumbent
man. The more complete the separation of powers in a constitutional system, the more difficult and yet
the more necessary will be their fusion in time of crisis. This is evident in a comparison of the crisis
potentialities of the cabinet and presidential systems of government. In the former the all-important President t to propose
harmony of legislature and executive is taken for granted; in the latter it is neither guaranteed nor to be
to confidently expected. As a result, cabinet is more easily established and more trustworthy than
amendments to the Constitution.
presidential dictatorship. The power of the state in crisis must not only be concentrated and expanded; it
must also be freed from the normal system of constitutional and legal limitations. 21 John Locke, on the
other hand, claims for the executive in its own right a broad discretion capable even of setting aside the 1. As earlier pointed out, the power to legislate is constitutionally consigned to the interim National
ordinary laws in the meeting of special exigencies for which the legislative power had not Assembly during the transition period. However, the initial convening of that Assembly is a matter fully
provided. 22 The rationale behind such broad emergency powers of the Executive is the release of the addressed to the judgment of the incumbent President. And, in the exercise of that judgment, the
government from "the paralysis of constitutional restrains" so that the crisis may be ended and normal President opted to defer convening of that body in utter recognition of the people's preference.
times restored. Likewise, in the period of transition, the power to propose amendments to the Constitution lies in the
interim National Assembly upon special call by the President (See. 15 of the Transitory Provisions). Again,
harking to the dictates of the sovereign will, the President decided not to call the interim National
2. The presidential exercise of legislative powers in time of martial law is now a conceded valid at. That
Assembly. Would it then be within the bounds of the Constitution and of law for the President to assume
sun clear authority of the President is saddled on Section 3 (pars. 1 and 2) of the Transitory Provisions,
that constituent power of the interim Assembly vis-a-vis his assumption of that body's legislative
thus: 23
functions? The answer is yes. If the President has been legitimately discharging the legislative functions
of the interim Assembly, there is no reason why he cannot validly discharge the function of that
The incumbent President of the Philippines shall initially convene the interim Assembly to propose amendments to the Constitution, which is but adjunct, although peculiar, to its
National Assembly and shall preside over its sessions until the interim Speaker gross legislative power. This, of course, is not to say that the President has converted his office into a
shall have been elected. He shall continue to exercise his powers and prerogatives constituent assembly of that nature normally constituted by the legislature. Rather, with the interim
under the nineteen hundred and thirty-five Constitution and the powers vested in National Assembly not convened and only the Presidency and the Supreme Court in operation, the urges
the President and the Prime Minister under this Constitution until the calls upon of absolute necessity render it imperative upon the President to act as agent for and in behalf of the
the interim National Assembly to elect the interim President and the interim Prime people to propose amendments to the Constitution. Parenthetically, by its very constitution, the
Supreme Court possesses no capacity to propose amendments without constitutional infractions. For the
President to shy away from that actuality and decline to undertake the amending process would leave VI
the governmental machineries at a stalemate or create in the powers of the State a destructive vacuum,
thereby impeding the objective of a crisis government "to end the crisis and restore normal times." In
Referendum-Plebiscite not
these parlous times, that Presidential initiative to reduce into concrete forms the constant voices of the
people reigns supreme. After all, constituent assemblies or constitutional conventions, like the President
now, are mere agents of the people .26 rendered nugatory by the

2. The President's action is not a unilateral move. As early as the referendums of January 1973 and participation of the 15-year olds.
February 1975, the people had already rejected the calling of the interim National Assembly. The Lupong
Tagapagpaganap of the Katipunan ng mga Sanggunian, the Pambansang Katipunan ng mga Barangay, 1. October 16 is in parts a referendum and a plebiscite. The question - (1) Do you want martial law to be
and the Pambansang Katipunan ng mga Barangay, representing 42,000 barangays, about the same continued? - is a referendum question, wherein the 15-year olds may participate. This was prompted by
number of Kabataang Barangay organizations, Sanggunians in 1,458 municipalities, 72 provinces, 3 sub- the desire of the Government to reach the larger mas of the people so that their true pulse may be felt
provinces, and 60 cities had informed the President that the prevailing sentiment of the people is for the to guide the President in pursuing his program for a New Order. For the succeeding question on the
abolition of the interim National Assembly. Other issues concerned the lifting of martial law and proposed amendments, only those of voting age of 18 years may participate. This is the plebiscite aspect,
amendments to the Constitution .27 The national organizations of Sangguniang Bayan presently as contemplated in Section 2, Article XVI of the new Constitution. 36 On this second question, it would
proposed to settle the issues of martial law, the interim Assembly, its replacement, the period of its only be the votes of those 18 years old and above which will have valid bearing on the results. The fact
existence, the length of the period for the exercise by the President of its present powers in a that the voting populace are simultaneously asked to answer the referendum question and the plebiscite
referendum to be held on October 16 . 28 The Batasang Bayan (legislative council) created under question does not infirm the referendum-plebiscite. There is nothing objectionable in consulting the
Presidential Decree 995 of September 10, 1976, composed of 19 cabinet members, 9 officials with people on a given issue, which is of current one and submitting to them for ratification of proposed
cabinet rank, 91 members of the Lupong Tagapagpaganap (executive committee) of the Katipunan ng constitutional amendments. The fear of commingled votes (15-year olds and 18-year olds above) is
mga Sangguniang Bayan voted in session to submit directly to the people in a plebiscite on October 16, readily dispelled by the provision of two ballot boxes for every barangay center, one containing the
the previously quoted proposed amendments to the Constitution, including the issue of martial ballots of voters fifteen years of age and under eighteen, and another containing the ballots of voters
law .29 Similarly, the "barangays" and the "sanggunians" endorsed to the President the submission of eighteen years of age and above. 37 The ballots in the ballot box for voters fifteen years of age and under
the proposed amendments to the people on October 16. All the foregoing led the President to initiate eighteen shall be counted ahead of the ballots of voters eighteen years and above contained in another
the proposal of amendments to the Constitution and the subsequent issuance of Presidential Decree No, ballot box. And, the results of the referendum-plebiscite shall be separately prepared for the age
1033 on September 22, 1976 submitting the questions (proposed amendments) to the people in the groupings, i.e., ballots contained in each of the two boxes. 38
National Referendum-Plebiscite on October 16.

2. It is apt to distinguish here between a "referendum" and a "plebiscite." A "referendum" is merely


V consultative in character. It is simply a means of assessing public reaction to the given issues submitted
to the people foe their consideration, the calling of which is derived from or within the totality of the
The People is Sovereign executive power of the President. 39 It is participated in by all citizens from the age of fifteen, regardless
of whether or not they are illiterates, feeble-minded, or ex- convicts . 40 A "plebiscite," on the other hand,
involves the constituent act of those "citizens of the Philippines not otherwise disqualified by law, who
1. Unlike in a federal state, the location of sovereignty in a unitary state is easily seen. In the Philippines,
are eighteen years of age or over, and who shall have resided in the Philippines for at least one year and
a republican and unitary state, sovereignty "resides in the people and all government authority
in the place wherein they propose to vote for at least six months preceding the election Literacy,
emanates from them.30 In its fourth meaning, Savigny would treat people as "that particular organized
property or any other substantive requirement is not imposed. It is generally associated with the
assembly of individuals in which, according to the Constitution, the highest power exists." 31 This is the
amending process of the Constitution, more particularly, the ratification aspect.
concept of popular sovereignty. It means that the constitutional legislator, namely the people, is
sovereign 32 In consequence, the people may thus write into the Constitution their convictions on any
subject they choose in the absence of express constitutional prohibition. 33 This is because, as Holmes VII
said, the Constitution "is an experiment, as all life is all experiment." 34 "The necessities of orderly
government," wrote Rottschaefer, "do not require that one generation should be permitted to 1. There appeals to be no valid basis for the claim that the regime of martial law stultifies in main the
permanently fetter all future generations." A constitution is based, therefore, upon a self-limiting freedom to dissent. That speaks of a bygone fear. The martial law regime which, in the observation of
decision of the people when they adopt it. 35 Justice Fernando, 41 is impressed with a mild character recorded no State imposition for a muffled voice.
To be sure, there are restraints of the individual liberty, but on certain grounds no total suppression of
2. The October 16 referendum-plebiscite is a resounding call to the people to exercise their sovereign that liberty is aimed at. The for the referendum-plebiscite on October 16 recognizes all the embracing
power as constitutional legislator. The proposed amendments, as earlier discussed, proceed not from freedoms of expression and assembly The President himself had announced that he would not
the thinking of a single man. Rather, they are the collated thoughts of the sovereign will reduced only countenance any suppression of dissenting views on the issues, as he is not interested in winning a "yes"
into enabling forms by the authority who can presently exercise the powers of the government. In equal or "no" vote, but on the genuine sentiment of the people on the issues at hand. 42 Thus, the dissenters
vein, the submission of those proposed amendments and the question of martial law in a referendum- soon found their way to the public forums, voicing out loud and clear their adverse views on the
plebiscite expresses but the option of the people themselves implemented only by the authority of the proposed amendments and even (in the valid ratification of the 1973 Constitution, which is already a
President. Indeed, it may well be said that the amending process is a sovereign act, although the settled matter. 43 Even government employees have been held by the Civil Service Commission free to
authority to initiate the same and the procedure to be followed reside somehow in a particular body. participate in public discussion and even campaign for their stand on the referendum-plebiscite issues. 44
VIII 3. Is the submission to the people of the proposed amendments within the time frame allowed therefor
a sufficient and proper submission?
Time for deliberation
Upon the first issue, Chief Justice Fred Ruiz Castro and Associate Justices Enrique M. Fernando, Claudio
Teehankee, Antonio P. Barredo, Cecilia Munoz Palma, Hermogenes Concepcion Jr. and Ruperto G. Martin
is not short.
are of the view that the question posed is justiciable, while Associate Justices Felix V. Makasiar, Felix Q.
Antonio and Ramon C. Aquino hold the view that the question is political.
1. The period from September 21 to October 16 or a period of 3 weeks is not too short for free debates
or discussions on the referendum-plebiscite issues. The questions are not new. They are the issues of the
Upon the second issue, Chief Justice Castro and Associate Justices Barredo, Makasiar, Antonio, Aquino,
day. The people have been living with them since the proclamation of martial law four years ago. The
Concepcion Jr. and Martin voted in the affirmative, while Associate Justices Teehankee and Munoz
referendums of 1973 and 1975 carried the same issue of martial law. That notwithstanding, the
Palma voted in the negative. Associate Justice Fernando, conformably to his concurring and dissenting
contested brief period for discussion is not without counterparts in previous plebiscites for constitutional
opinion in Aquino vs. Enrile (59 SCRA 183), specifically dissents from the proposition that there is
amendments. Justice Makasiar, in the Referendum Case, recalls: "Under the old Society, 15 days were
concentration of powers in the Executive during periods of crisis, thus raising serious doubts as to the
allotted for the publication in three consecutive issues of the Official Gazette of the women's suffrage
power of the President to propose amendments.
amendment to the Constitution before the scheduled plebiscite on April 30, 1937 (Com. Act No. 34). The
constitutional amendment to append as ordinance the complicated Tydings-Kocialskowski was published
in only three consecutive issues of the Official Gazette for 10 days prior to the scheduled plebiscite (Com. Upon the third issue, Chief Justice Castro and Associate Justices Barredo, Makasiar, Aquino, Concepcion
Act 492). For the 1940 Constitutional amendments providing for the bicameral Congress, the reelection Jr. and Martin are of the view that there is a sufficient and proper submission of the proposed
of the President and Vice President, and the creation of the Commission on Elections, 20 days of amendments for ratification by the people. Associate Justices Barredo and Makasiar expressed the hope,
publication in three consecutive issues of the Official Gazette was fixed (Com Act No. 517). And the however that the period of time may be extended. Associate Justices Fernando, Makasiar and Antonio
Parity Amendment, an involved constitutional amendment affecting the economy as well as the are of the view that the question is political and therefore beyond the competence and cognizance of
independence of the Republic was publicized in three consecutive issues of the Official Gazette for 20 this Court, Associate Justice Fernando adheres to his concurrence in the opinion of Chief Justice
days prior to the plebiscite (Rep. Act No. 73)." 45 Concepcion in Gonzales vs. COMELEC (21 SCRA 774).Associate Justices Teehankee and MUNOZ Palma
hold that prescinding from the President's lack of authority to exercise the constituent power to propose
the amendments, etc., as above stated, there is no fair and proper submission with sufficient
2. It is worthy to note that Article XVI of the Constitution makes no provision as to the specific date when
information and time to assure intelligent consent or rejection under the standards set by this Court in
the plebiscite shall be held, but simply states that it "shall be held not later than three months after the
the controlling cases of Gonzales, supra, and Tolentino vs. COMELEC (41 SCRA 702).
approval of such amendment or revision." In Coleman v. Miller, 46 the United States Supreme court held
that this matter of submission involves "an appraisal of a great variety of relevant conditions, political,
social and economic," which "are essentially political and not justiciable." The constituent body or in the Chief Justice Castro and Associate Justices Barredo, Makasiar, Antonio, Aquino, Concepcion Jr. and
instant cases, the President, may fix the time within which the people may act. This is because proposal Martin voted to dismiss the three petitions at bar. For reasons as expressed in his separate opinion,
and ratification are not treated as unrelated acts, but as succeeding steps in a single endeavor, the Associate Justice Fernando concurs in the result. Associate Justices Teehankee and Munoz Palma voted
natural inference being that they are not to be widely separated in time; second, it is only when there is to grant the petitions.
deemed to be a necessity therefor that amendments are to be proposed, the reasonable implication
being that when proposed, they are to be considered and disposed of presently, and third, ratification is
ACCORDINGLY, the vote being 8 to 2 to dismiss, the said petitions are hereby dismissed. This decision is
but the expression of the approbation of the people, hence, it must be done contemporaneously. 47 In
immediately executory.
the words of Jameson, "(a)n alteration of the Constitution proposed today has relation to the sentiment
and the felt needs of today, and that, if not ratified early while that sentiment may fairly be supposed to
exist. it ought to be regarded as waived, and not again to be voted upon, unless a second time proposed SO ORDERED.
by proper body

IN RESUME

The three issues are

1. Is the question of the constitutionality of Presidential Decrees Nos. 991, 1031 and 1033 political or
justiciable?

2. During the present stage of the transition period, and under, the environmental circumstances now
obtaining, does the President possess power to propose amendments to the Constitution as well as set
up the required machinery and prescribe the procedure for the ratification of his proposals by the
people?
Republic of the Philippines Marketing/Reservation System Contract or other type of
SUPREME COURT contract specified by the Highest Bidder in its strategic plan
Manila for the Manila Hotel. . . .

EN BANC b. The Highest Bidder must execute the Stock Purchase and
Sale Agreement with GSIS . . . .

K. DECLARATION OF THE WINNING BIDDER/STRATEGIC


PARTNER
G.R. No. 122156 February 3, 1997

The Highest Bidder will be declared the Winning Bidder/Strategic Partner after the
MANILA PRINCE HOTEL petitioner,
following conditions are met:
vs.
GOVERNMENT SERVICE INSURANCE SYSTEM, MANILA HOTEL CORPORATION, COMMITTEE ON
PRIVATIZATION and OFFICE OF THE GOVERNMENT CORPORATE COUNSEL, respondents. a. Execution of the necessary contracts with GSIS/MHC not
later than October 23, 1995 (reset to November 3, 1995);
and

b. Requisite approvals from the GSIS/MHC and COP


BELLOSILLO, J.:
(Committee on Privatization)/OGCC (Office of the
Government Corporate Counsel) are obtained. 3
The FiIipino First Policy enshrined in the 1987 Constitution, i.e., in the grant of rights, privileges, and
concessions covering the national economy and patrimony, the State shall give preference to qualified
Pending the declaration of Renong Berhad as the winning bidder/strategic partner and the execution of
Filipinos, 1 is in oked by petitioner in its bid to acquire 51% of the shares of the Manila Hotel Corporation
the necessary contracts, petitioner in a letter to respondent GSIS dated 28 September 1995 matched the
(MHC) which owns the historic Manila Hotel. Opposing, respondents maintain that the provision is not
bid price of P44.00 per share tendered by Renong Berhad. 4 In a subsequent letter dated 10 October
self-executing but requires an implementing legislation for its enforcement. Corollarily, they ask whether
1995 petitioner sent a manager's check issued by Philtrust Bank for Thirty-three Million Pesos
the 51% shares form part of the national economy and patrimony covered by the protective mantle of
(P33.000.000.00) as Bid Security to match the bid of the Malaysian Group, Messrs. Renong Berhad . .
the Constitution.
. 5 which respondent GSIS refused to accept.

The controversy arose when respondent Government Service Insurance System (GSIS), pursuant to the
On 17 October 1995, perhaps apprehensive that respondent GSIS has disregarded the tender of the
privatization program of the Philippine Government under Proclamation No. 50 dated 8 December 1986,
matching bid and that the sale of 51% of the MHC may be hastened by respondent GSIS and
decided to sell through public bidding 30% to 51% of the issued and outstanding shares of respondent
consummated with Renong Berhad, petitioner came to this Court on prohibition and mandamus. On 18
MHC. The winning bidder, or the eventual "strategic partner," is to provide management expertise
October 1995 the Court issued a temporary restraining order enjoining respondents from perfecting and
and/or an international marketing/reservation system, and financial support to strengthen the
consummating the sale to the Malaysian firm.
profitability and performance of the Manila Hotel. 2 In a close bidding held on 18 September 1995 only
two (2) bidders participated: petitioner Manila Prince Hotel Corporation, a Filipino corporation, which
offered to buy 51% of the MHC or 15,300,000 shares at P41.58 per share, and Renong Berhad, a On 10 September 1996 the instant case was accepted by the Court En Banc after it was referred to it by
Malaysian firm, with ITT-Sheraton as its hotel operator, which bid for the same number of shares at the First Division. The case was then set for oral arguments with former Chief Justice Enrique M.
P44.00 per share, or P2.42 more than the bid of petitioner. Fernando and Fr. Joaquin G. Bernas, S.J., as amici curiae.

Pertinent provisions of the bidding rules prepared by respondent GSIS state In the main, petitioner invokes Sec. 10, second par., Art. XII, of the 1987 Constitution and submits that
the Manila Hotel has been identified with the Filipino nation and has practically become a historical
monument which reflects the vibrancy of Philippine heritage and culture. It is a proud legacy of an earlier
I. EXECUTION OF THE NECESSARY CONTRACTS WITH
generation of Filipinos who believed in the nobility and sacredness of independence and its power and
GSIS/MHC
capacity to release the full potential of the Filipino people. To all intents and purposes, it has become a
part of the national patrimony. 6 Petitioner also argues that since 51% of the shares of the MHC carries
1. The Highest Bidder must comply with the conditions set forth below by October with it the ownership of the business of the hotel which is owned by respondent GSIS, a government-
23, 1995 (reset to November 3, 1995) or the Highest Bidder will lose the right to owned and controlled corporation, the hotel business of respondent GSIS being a part of the tourism
purchase the Block of Shares and GSIS will instead offer the Block of Shares to the industry is unquestionably a part of the national economy. Thus, any transaction involving 51% of the
other Qualified Bidders: shares of stock of the MHC is clearly covered by the term national economy, to which Sec. 10, second
par., Art. XII, 1987 Constitution, applies. 7
a. The Highest Bidder must negotiate and execute with the
GSIS/MHC the Management Contract, International
It is also the thesis of petitioner that since Manila Hotel is part of the national patrimony and its business administered.11 Under the doctrine of constitutional supremacy, if a law or contract violates any norm of
also unquestionably part of the national economy petitioner should be preferred after it has matched the constitution that law or contract whether promulgated by the legislative or by the executive branch
the bid offer of the Malaysian firm. For the bidding rules mandate that if for any reason, the Highest or entered into by private persons for private purposes is null and void and without any force and effect.
Bidder cannot be awarded the Block of Shares, GSIS may offer this to the other Qualified Bidders that Thus, since the Constitution is the fundamental, paramount and supreme law of the nation, it is deemed
have validly submitted bids provided that these Qualified Bidders are willing to match the highest bid in written in every statute and contract.
terms of price per share. 8
Admittedly, some constitutions are merely declarations of policies and principles. Their provisions
Respondents except. They maintain that: First, Sec. 10, second par., Art. XII, of the 1987 Constitution is command the legislature to enact laws and carry out the purposes of the framers who merely establish
merely a statement of principle and policy since it is not a self-executing provision and requires an outline of government providing for the different departments of the governmental machinery and
implementing legislation(s) . . . Thus, for the said provision to Operate, there must be existing laws "to lay securing certain fundamental and inalienable rights of citizens. 12 A provision which lays down a general
down conditions under which business may be done." 9 principle, such as those found in Art. II of the 1987 Constitution, is usually not self-executing. But a
provision which is complete in itself and becomes operative without the aid of supplementary or
enabling legislation, or that which supplies sufficient rule by means of which the right it grants may be
Second, granting that this provision is self-executing, Manila Hotel does not fall under the term national
enjoyed or protected, is self-executing. Thus a constitutional provision is self-executing if the nature and
patrimony which only refers to lands of the public domain, waters, minerals, coal, petroleum and other
extent of the right conferred and the liability imposed are fixed by the constitution itself, so that they can
mineral oils, all forces of potential energy, fisheries, forests or timber, wildlife, flora and fauna and all
be determined by an examination and construction of its terms, and there is no language indicating that
marine wealth in its territorial sea, and exclusive marine zone as cited in the first and second paragraphs
the subject is referred to the legislature for action. 13
of Sec. 2, Art. XII, 1987 Constitution. According to respondents, while petitioner speaks of the guests who
have slept in the hotel and the events that have transpired therein which make the hotel historic, these
alone do not make the hotel fall under the patrimonyof the nation. What is more, the mandate of the As against constitutions of the past, modern constitutions have been generally drafted upon a different
Constitution is addressed to the State, not to respondent GSIS which possesses a personality of its own principle and have often become in effect extensive codes of laws intended to operate directly upon the
separate and distinct from the Philippines as a State. people in a manner similar to that of statutory enactments, and the function of constitutional
conventions has evolved into one more like that of a legislative body. Hence, unless it is expressly
provided that a legislative act is necessary to enforce a constitutional mandate, the presumption now is
Third, granting that the Manila Hotel forms part of the national patrimony, the constitutional provision
that all provisions of the constitution are self-executing If the constitutional provisions are treated as
invoked is still inapplicable since what is being sold is only 51% of the outstanding shares of the
requiring legislation instead of self-executing, the legislature would have the power to ignore and
corporation, not the hotel building nor the land upon which the building stands. Certainly, 51% of the
practically nullify the mandate of the fundamental law. 14 This can be cataclysmic. That is why the
equity of the MHC cannot be considered part of the national patrimony. Moreover, if the disposition of
prevailing view is, as it has always been, that
the shares of the MHC is really contrary to the Constitution, petitioner should have questioned it right
from the beginning and not after it had lost in the bidding.
. . . in case of doubt, the Constitution should be considered self-executing rather
than non-self-executing . . . . Unless the contrary is clearly intended, the provisions
Fourth, the reliance by petitioner on par. V., subpar. J. 1., of the bidding rules which provides that if for
of the Constitution should be considered self-executing, as a contrary rule would
any reason, the Highest Bidder cannot be awarded the Block of Shares, GSIS may offer this to the other
give the legislature discretion to determine when, or whether, they shall be
Qualified Bidders that have validly submitted bids provided that these Qualified Bidders are willing to
effective. These provisions would be subordinated to the will of the lawmaking
match the highest bid in terms of price per share, is misplaced. Respondents postulate that the privilege
body, which could make them entirely meaningless by simply refusing to pass the
of submitting a matching bid has not yet arisen since it only takes place if for any reason, the Highest
needed implementing statute. 15
Bidder cannot be awarded the Block of Shares. Thus the submission by petitioner of a matching bid is
premature since Renong Berhad could still very well be awarded the block of shares and the condition
giving rise to the exercise of the privilege to submit a matching bid had not yet taken place. Respondents argue that Sec. 10, second par., Art. XII, of the 1987 Constitution is clearly not self-
executing, as they quote from discussions on the floor of the 1986 Constitutional Commission
Finally, the prayer for prohibition grounded on grave abuse of discretion should fail since respondent
GSIS did not exercise its discretion in a capricious, whimsical manner, and if ever it did abuse its MR. RODRIGO. Madam President, I am asking this question
discretion it was not so patent and gross as to amount to an evasion of a positive duty or a virtual refusal as the Chairman of the Committee on Style. If the wording of
to perform a duty enjoined by law. Similarly, the petition for mandamus should fail as petitioner has no "PREFERENCE" is given to QUALIFIED FILIPINOS," can it be
clear legal right to what it demands and respondents do not have an imperative duty to perform the act understood as a preference to qualified Filipinos vis-a-
required of them by petitioner. vis Filipinos who are not qualified. So, why do we not make
it clear? To qualified Filipinos as against aliens?
We now resolve. A constitution is a system of fundamental laws for the governance and administration
of a nation. It is supreme, imperious, absolute and unalterable except by the authority from which it THE PRESIDENT. What is the question of Commissioner
emanates. It has been defined as the fundamental and paramount law of the nation. 10 It prescribes the Rodrigo? Is it to remove the word "QUALIFIED?".
permanent framework of a system of government, assigns to the different departments their respective
powers and duties, and establishes certain fixed principles on which government is founded. The
MR. RODRIGO. No, no, but say definitely "TO QUALIFIED
fundamental conception in other words is that it is a supreme law to which all other laws must conform
FILIPINOS" as against whom? As against aliens or over
and in accordance with which all private rights must be determined and all public authority
aliens?
MR. NOLLEDO. Madam President, I think that is understood. enforceable constitutional rights but merely guidelines for legislation. The very terms of the provisions
We use the word "QUALIFIED" because the existing laws or manifest that they are only principles upon which the legislations must be based. Res ipsa loquitur.
prospective laws will always lay down conditions under
which business may be done. For example, qualifications on
On the other hand, Sec. 10, second par., Art. XII of the of the 1987 Constitution is a mandatory, positive
the setting up of other financial structures, et
command which is complete in itself and which needs no further guidelines or implementing laws or
cetera (emphasis supplied by respondents)
rules for its enforcement. From its very words the provision does not require any legislation to put it in
operation. It is per se judicially enforceable When our Constitution mandates that [i]n the grant of rights,
MR. RODRIGO. It is just a matter of style. privileges, and concessions covering national economy and patrimony, the State shall give preference to
qualified Filipinos, it means just that qualified Filipinos shall be preferred. And when our Constitution
declares that a right exists in certain specified circumstances an action may be maintained to enforce
MR. NOLLEDO Yes, 16
such right notwithstanding the absence of any legislation on the subject; consequently, if there is no
statute especially enacted to enforce such constitutional right, such right enforces itself by its own
Quite apparently, Sec. 10, second par., of Art XII is couched in such a way as not to make it appear that it inherent potency and puissance, and from which all legislations must take their bearings. Where there is
is non-self-executing but simply for purposes of style. But, certainly, the legislature is not precluded from a right there is a remedy. Ubi jus ibi remedium.
enacting other further laws to enforce the constitutional provision so long as the contemplated statute
squares with the Constitution. Minor details may be left to the legislature without impairing the self-
As regards our national patrimony, a member of the 1986 Constitutional Commission 34 explains
executing nature of constitutional provisions.

The patrimony of the Nation that should be conserved and developed refers not
In self-executing constitutional provisions, the legislature may still enact legislation to facilitate the
only to out rich natural resources but also to the cultural heritage of out race. It
exercise of powers directly granted by the constitution, further the operation of such a provision,
also refers to our intelligence in arts, sciences and letters. Therefore, we should
prescribe a practice to be used for its enforcement, provide a convenient remedy for the protection of
develop not only our lands, forests, mines and other natural resources but also the
the rights secured or the determination thereof, or place reasonable safeguards around the exercise of
mental ability or faculty of our people.
the right. The mere fact that legislation may supplement and add to or prescribe a penalty for the
violation of a self-executing constitutional provision does not render such a provision ineffective in the
absence of such legislation. The omission from a constitution of any express provision for a remedy for We agree. In its plain and ordinary meaning, the term patrimony pertains to heritage. 35 When the
enforcing a right or liability is not necessarily an indication that it was not intended to be self-executing. Constitution speaks of national patrimony, it refers not only to the natural resources of the Philippines,
The rule is that a self-executing provision of the constitution does not necessarily exhaust legislative as the Constitution could have very well used the term natural resources, but also to the cultural
power on the subject, but any legislation must be in harmony with the constitution, further the exercise heritage of the Filipinos.
of constitutional right and make it more available. 17 Subsequent legislation however does not
necessarily mean that the subject constitutional provision is not, by itself, fully enforceable.
Manila Hotel has become a landmark a living testimonial of Philippine heritage. While it was
restrictively an American hotel when it first opened in 1912, it immediately evolved to be truly Filipino,
Respondents also argue that the non-self-executing nature of Sec. 10, second par., of Art. XII is implied Formerly a concourse for the elite, it has since then become the venue of various significant events
from the tenor of the first and third paragraphs of the same section which undoubtedly are not self- which have shaped Philippine history. It was called the Cultural Center of the 1930's. It was the site of the
executing. 18 The argument is flawed. If the first and third paragraphs are not self-executing because festivities during the inauguration of the Philippine Commonwealth. Dubbed as the Official Guest House
Congress is still to enact measures to encourage the formation and operation of enterprises fully owned of the Philippine Government. it plays host to dignitaries and official visitors who are accorded the
by Filipinos, as in the first paragraph, and the State still needs legislation to regulate and exercise traditional Philippine hospitality. 36
authority over foreign investments within its national jurisdiction, as in the third paragraph, then
a fortiori, by the same logic, the second paragraph can only be self-executing as it does not by its
The history of the hotel has been chronicled in the book The Manila Hotel: The Heart and Memory of a
language require any legislation in order to give preference to qualified Filipinos in the grant of rights,
City. 37During World War II the hotel was converted by the Japanese Military Administration into a
privileges and concessions covering the national economy and patrimony. A constitutional provision may
military headquarters. When the American forces returned to recapture Manila the hotel was selected
be self-executing in one part and non-self-executing in another. 19
by the Japanese together with Intramuros as the two (2) places fro their final stand. Thereafter, in the
1950's and 1960's, the hotel became the center of political activities, playing host to almost every
Even the cases cited by respondents holding that certain constitutional provisions are merely statements political convention. In 1970 the hotel reopened after a renovation and reaped numerous international
of principles and policies, which are basically not self-executing and only placed in the Constitution as recognitions, an acknowledgment of the Filipino talent and ingenuity. In 1986 the hotel was the site of a
moral incentives to legislation, not as judicially enforceable rights are simply not in point. Basco failed coup d' etatwhere an aspirant for vice-president was "proclaimed" President of the Philippine
v. Philippine Amusements and Gaming Corporation 20 speaks of constitutional provisions on personal Republic.
dignity, 21 the sanctity of family life, 22 the vital role of the youth in nation-building 23 the promotion of
social justice, 24 and the values of education. 25Tolentino v. Secretary of Finance 26 refers to the
For more than eight (8) decades Manila Hotel has bore mute witness to the triumphs and failures, loves
constitutional provisions on social justice and human rights 27 and on education. 28 Lastly, Kilosbayan,
and frustrations of the Filipinos; its existence is impressed with public interest; its own historicity
Inc. v. Morato 29 cites provisions on the promotion of general welfare, 30 the sanctity of family life, 31 the
associated with our struggle for sovereignty, independence and nationhood. Verily, Manila Hotel has
vital role of the youth in nation-building 32 and the promotion of total human liberation and
become part of our national economy and patrimony. For sure, 51% of the equity of the MHC comes
development. 33A reading of these provisions indeed clearly shows that they are not judicially
within the purview of the constitutional shelter for it comprises the majority and controlling stock, so
that anyone who acquires or owns the 51% will have actual control and management of the hotel. In this
instance, 51% of the MHC cannot be disassociated from the hotel and the land on which the hotel edifice MR. NOLLEDO. The amendment will read: "IN THE GRANT
stands. Consequently, we cannot sustain respondents' claim that theFilipino First Policy provision is not OF RIGHTS, PRIVILEGES AND CONCESSIONS COVERING THE
applicable since what is being sold is only 51% of the outstanding shares of the corporation, not the Hotel NATIONAL ECONOMY AND PATRIMONY, THE STATE SHALL
building nor the land upon which the building stands. 38 GIVE PREFERENCE TO QUALIFIED FILIPINOS." And the word
"Filipinos" here, as intended by the proponents, will include
not only individual Filipinos but also Filipino-controlled
The argument is pure sophistry. The term qualified Filipinos as used in Our Constitution also includes
entities or entities fully-controlled by Filipinos. 40
corporations at least 60% of which is owned by Filipinos. This is very clear from the proceedings of the
1986 Constitutional Commission
The phrase preference to qualified Filipinos was explained thus
THE PRESIDENT. Commissioner Davide is recognized.
MR. FOZ. Madam President, I would like to request
Commissioner Nolledo to please restate his amendment so
MR. DAVIDE. I would like to introduce an amendment to the
that I can ask a question.
Nolledo amendment. And the amendment would consist in
substituting the words "QUALIFIED FILIPINOS" with the
following: "CITIZENS OF THE PHILIPPINES OR MR. NOLLEDO. "IN THE GRANT OF RIGHTS, PRIVILEGES AND
CORPORATIONS OR ASSOCIATIONS WHOSE CAPITAL OR CONCESSIONS COVERING THE NATIONAL ECONOMY AND
CONTROLLING STOCK IS WHOLLY OWNED BY SUCH PATRIMONY, THE STATE SHALL GIVE PREFERENCE TO
CITIZENS. QUALIFIED FILIPINOS."

xxx xxx xxx MR FOZ. In connection with that amendment, if a foreign


enterprise is qualified and a Filipino enterprise is also
qualified, will the Filipino enterprise still be given a
MR. MONSOD. Madam President, apparently the proponent
preference?
is agreeable, but we have to raise a question. Suppose it is a
corporation that is 80-percent Filipino, do we not give it
preference? MR. NOLLEDO. Obviously.

MR. DAVIDE. The Nolledo amendment would refer to an MR. FOZ. If the foreigner is more qualified in some aspects
individual Filipino. What about a corporation wholly owned than the Filipino enterprise, will the Filipino still be
by Filipino citizens? preferred?

MR. MONSOD. At least 60 percent, Madam President. MR. NOLLEDO. The answer is "yes."

MR. DAVIDE. Is that the intention? MR. FOZ. Thank you, 41

MR. MONSOD. Yes, because, in fact, we would be limiting it Expounding further on the Filipino First Policy provision Commissioner Nolledo continues
if we say that the preference should only be 100-percent
Filipino.
MR. NOLLEDO. Yes, Madam President. Instead of "MUST," it will be "SHALL THE
STATE SHALL GlVE PREFERENCE TO QUALIFIED FILIPINOS. This embodies the so-
MR: DAVIDE. I want to get that meaning clear because called "Filipino First" policy. That means that Filipinos should be given preference
"QUALIFIED FILIPINOS" may refer only to individuals and not in the grant of concessions, privileges and rights covering the national
to juridical personalities or entities. patrimony. 42

MR. MONSOD. We agree, Madam President. 39 The exchange of views in the sessions of the Constitutional Commission regarding the subject provision
was still further clarified by Commissioner Nolledo 43
xxx xxx xxx
Paragraph 2 of Section 10 explicitly mandates the "Pro-Filipino" bias in all
economic concerns. It is better known as the FILIPINO FIRST Policy . . . This
MR. RODRIGO. Before we vote, may I request that the
provision was never found in previous Constitutions . . . .
amendment be read again.
The term "qualified Filipinos" simply means that preference shall be given to those When the Constitution addresses the State it refers not only to the people but also to the government as
citizens who can make a viable contribution to the common good, because of elements of the State. After all, government is composed of three (3) divisions of power legislative,
credible competence and efficiency. It certainly does NOT mandate the pampering executive and judicial. Accordingly, a constitutional mandate directed to the State is correspondingly
and preferential treatment to Filipino citizens or organizations that are directed to the three(3) branches of government. It is undeniable that in this case the subject
incompetent or inefficient, since such an indiscriminate preference would be constitutional injunction is addressed among others to the Executive Department and respondent GSIS, a
counter productive and inimical to the common good. government instrumentality deriving its authority from the State.

In the granting of economic rights, privileges, and concessions, when a choice has It should be stressed that while the Malaysian firm offered the higher bid it is not yet the winning bidder.
to be made between a "qualified foreigner" end a "qualified Filipino," the latter The bidding rules expressly provide that the highest bidder shall only be declared the winning bidder
shall be chosen over the former." after it has negotiated and executed the necessary contracts, and secured the requisite approvals. Since
the "Filipino First Policy provision of the Constitution bestows preference on qualified Filipinos the mere
tending of the highest bid is not an assurance that the highest bidder will be declared the winning
Lastly, the word qualified is also determinable. Petitioner was so considered by respondent GSIS and
bidder. Resultantly, respondents are not bound to make the award yet, nor are they under obligation to
selected as one of the qualified bidders. It was pre-qualified by respondent GSIS in accordance with its
enter into one with the highest bidder. For in choosing the awardee respondents are mandated to abide
own guidelines so that the sole inference here is that petitioner has been found to be possessed of
by the dictates of the 1987 Constitution the provisions of which are presumed to be known to all the
proven management expertise in the hotel industry, or it has significant equity ownership in another
bidders and other interested parties.
hotel company, or it has an overall management and marketing proficiency to successfully operate the
Manila Hotel. 44
Adhering to the doctrine of constitutional supremacy, the subject constitutional provision is, as it should
be, impliedly written in the bidding rules issued by respondent GSIS, lest the bidding rules be nullified for
The penchant to try to whittle away the mandate of the Constitution by arguing that the subject
being violative of the Constitution. It is a basic principle in constitutional law that all laws and contracts
provision is not self-executory and requires implementing legislation is quite disturbing. The attempt to
must conform with the fundamental law of the land. Those which violate the Constitution lose their
violate a clear constitutional provision by the government itself is only too distressing. To adopt
reason for being.
such a line of reasoning is to renounce the duty to ensure faithfulness to the Constitution. For, even
some of the provisions of the Constitution which evidently need implementing legislation have juridical
life of their own and can be the source of a judicial remedy. We cannot simply afford the government a Paragraph V. J. 1 of the bidding rules provides that [if] for any reason the Highest Bidder cannot be
defense that arises out of the failure to enact further enabling, implementing or guiding legislation. In awarded the Block of Shares, GSIS may offer this to other Qualified Bidders that have validly submitted
fine, the discourse of Fr. Joaquin G. Bernas, S.J., on constitutional government is apt bids provided that these Qualified Bidders are willing to match the highest bid in terms of price per
share. 47 Certainly, the constitutional mandate itself is reason enough not to award the block of shares
immediately to the foreign bidder notwithstanding its submission of a higher, or even the highest, bid. In
The executive department has a constitutional duty to implement laws, including
fact, we cannot conceive of a stronger reason than the constitutional injunction itself.
the Constitution, even before Congress acts provided that there are
discoverable legal standards for executive action. When the executive acts, it must
be guided by its own understanding of the constitutional command and of In the instant case, where a foreign firm submits the highest bid in a public bidding concerning the grant
applicable laws. The responsibility for reading and understanding the Constitution of rights, privileges and concessions covering the national economy and patrimony, thereby exceeding
and the laws is not the sole prerogative of Congress. If it were, the executive the bid of a Filipino, there is no question that the Filipino will have to be allowed to match the bid of the
would have to ask Congress, or perhaps the Court, for an interpretation every time foreign entity. And if the Filipino matches the bid of a foreign firm the award should go to the Filipino. It
the executive is confronted by a constitutional command. That is not how must be so if we are to give life and meaning to the Filipino First Policy provision of the 1987
constitutional government operates. 45 Constitution. For, while this may neither be expressly stated nor contemplated in the bidding rules, the
constitutional fiat is, omnipresent to be simply disregarded. To ignore it would be to sanction a perilous
skirting of the basic law.
Respondents further argue that the constitutional provision is addressed to the State, not to respondent
GSIS which by itself possesses a separate and distinct personality. This argument again is at best
specious. It is undisputed that the sale of 51% of the MHC could only be carried out with the prior This Court does not discount the apprehension that this policy may discourage foreign investors. But the
approval of the State acting through respondent Committee on Privatization. As correctly pointed out by Constitution and laws of the Philippines are understood to be always open to public scrutiny. These are
Fr. Joaquin G. Bernas, S.J., this fact alone makes the sale of the assets of respondents GSIS and MHC a given factors which investors must consider when venturing into business in a foreign jurisdiction. Any
"state action." In constitutional jurisprudence, the acts of persons distinct from the government are person therefore desiring to do business in the Philippines or with any of its agencies or instrumentalities
considered "state action" covered by the Constitution (1) when the activity it engages in is a "public is presumed to know his rights and obligations under the Constitution and the laws of the forum.
function;" (2) when the government is so significantly involved with the private actor as to make the
government responsible for his action; and, (3) when the government has approved or authorized the
The argument of respondents that petitioner is now estopped from questioning the sale to Renong
action. It is evident that the act of respondent GSIS in selling 51% of its share in respondent MHC comes
Berhad since petitioner was well aware from the beginning that a foreigner could participate in the
under the second and third categories of "state action." Without doubt therefore the transaction.
bidding is meritless. Undoubtedly, Filipinos and foreigners alike were invited to the bidding. But
although entered into by respondent GSIS, is in fact a transaction of the State and therefore subject to
foreigners may be awarded the sale only if no Filipino qualifies, or if the qualified Filipino fails to match
the constitutional command. 46
the highest bid tendered by the foreign entity. In the case before us, while petitioner was already
preferred at the inception of the bidding because of the constitutional mandate, petitioner had not yet
matched the bid offered by Renong Berhad. Thus it did not have the right or personality then to compel
respondent GSIS to accept its earlier bid. Rightly, only after it had matched the bid of the foreign firm always defer to the Constitution in the proper governance of a free society; after all, there is nothing so
and the apparent disregard by respondent GSIS of petitioner's matching bid did the latter have a cause of sacrosanct in any economic policy as to draw itself beyond judicial review when the Constitution is
action. involved. 49

Besides, there is no time frame for invoking the constitutional safeguard unless perhaps the award has Nationalism is inherent, in the very concept of the Philippines being a democratic and republican state,
been finally made. To insist on selling the Manila Hotel to foreigners when there is a Filipino group with sovereignty residing in the Filipino people and from whom all government authority emanates. In
willing to match the bid of the foreign group is to insist that government be treated as any other nationalism, the happiness and welfare of the people must be the goal. The nation-state can have no
ordinary market player, and bound by its mistakes or gross errors of judgment, regardless of the higher purpose. Any interpretation of any constitutional provision must adhere to such basic concept.
consequences to the Filipino people. The miscomprehension of the Constitution is regrettable. Thus we Protection of foreign investments, while laudible, is merely a policy. It cannot override the demands of
would rather remedy the indiscretion while there is still an opportunity to do so than let the government nationalism. 50
develop the habit of forgetting that the Constitution lays down the basic conditions and parameters for
its actions.
The Manila Hotel or, for that matter, 51% of the MHC, is not just any commodity to be sold to the
highest bidder solely for the sake of privatization. We are not talking about an ordinary piece of property
Since petitioner has already matched the bid price tendered by Renong Berhad pursuant to the bidding in a commercial district. We are talking about a historic relic that has hosted many of the most important
rules, respondent GSIS is left with no alternative but to award to petitioner the block of shares of MHC events in the short history of the Philippines as a nation. We are talking about a hotel where heads of
and to execute the necessary agreements and documents to effect the sale in accordance not only with states would prefer to be housed as a strong manifestation of their desire to cloak the dignity of the
the bidding guidelines and procedures but with the Constitution as well. The refusal of respondent GSIS highest state function to their official visits to the Philippines. Thus the Manila Hotel has played and
to execute the corresponding documents with petitioner as provided in the bidding rules after the latter continues to play a significant role as an authentic repository of twentieth century Philippine history and
has matched the bid of the Malaysian firm clearly constitutes grave abuse of discretion. culture. In this sense, it has become truly a reflection of the Filipino soul a place with a history of
grandeur; a most historical setting that has played a part in the shaping of a country. 51
The Filipino First Policy is a product of Philippine nationalism. It is embodied in the 1987 Constitution not
merely to be used as a guideline for future legislation but primarily to be enforced; so must it be This Court cannot extract rhyme nor reason from the determined efforts of respondents to sell the
enforced. This Court as the ultimate guardian of the Constitution will never shun, under any reasonable historical landmark this Grand Old Dame of hotels in Asia to a total stranger. For, indeed, the
circumstance, the duty of upholding the majesty of the Constitution which it is tasked to defend. It is conveyance of this epic exponent of the Filipino psyche to alien hands cannot be less than
worth emphasizing that it is not the intention of this Court to impede and diminish, much less mephistophelian for it is, in whatever manner viewed, a veritable alienation of a nation's soul for some
undermine, the influx of foreign investments. Far from it, the Court encourages and welcomes more pieces of foreign silver. And so we ask: What advantage, which cannot be equally drawn from a qualified
business opportunities but avowedly sanctions the preference for Filipinos whenever such preference is Filipino, can be gained by the Filipinos Manila Hotel and all that it stands for is sold to a non-
ordained by the Constitution. The position of the Court on this matter could have not been more Filipino? How much of national pride will vanish if the nation's cultural heritage is entrusted to a foreign
appropriately articulated by Chief Justice Narvasa entity? On the other hand, how much dignity will be preserved and realized if the national patrimony is
safekept in the hands of a qualified, zealous and well-meaning Filipino? This is the plain and simple
meaning of the Filipino First Policy provision of the Philippine Constitution. And this Court, heeding the
As scrupulously as it has tried to observe that it is not its function to substitute its
clarion call of the Constitution and accepting the duty of being the elderly watchman of the nation, will
judgment for that of the legislature or the executive about the wisdom and
continue to respect and protect the sanctity of the Constitution.
feasibility of legislation economic in nature, the Supreme Court has not been
spared criticism for decisions perceived as obstacles to economic progress and
development . . . in connection with a temporary injunction issued by the Court's WHEREFORE, respondents GOVERNMENT SERVICE INSURANCE SYSTEM, MANILA HOTEL CORPORATION,
First Division against the sale of the Manila Hotel to a Malaysian Firm and its COMMITTEE ON PRIVATIZATION and OFFICE OF THE GOVERNMENT CORPORATE COUNSEL are directed
partner, certain statements were published in a major daily to the effect that to CEASE and DESIST from selling 51% of the shares of the Manila Hotel Corporation to RENONG
injunction "again demonstrates that the Philippine legal system can be a major BERHAD, and to ACCEPT the matching bid of petitioner MANILA PRINCE HOTEL CORPORATION to
obstacle to doing business here. purchase the subject 51% of the shares of the Manila Hotel Corporation at P44.00 per share and
thereafter to execute the necessary clearances and to do such other acts and deeds as may be necessary
for purpose.
Let it be stated for the record once again that while it is no business of the Court to
intervene in contracts of the kind referred to or set itself up as the judge of
whether they are viable or attainable, it is its bounden duty to make sure that they SO ORDERED.
do not violate the Constitution or the laws, or are not adopted or implemented
with grave abuse of discretion amounting to lack or excess of jurisdiction. It will
never shirk that duty, no matter how buffeted by winds of unfair and ill-informed
criticism. 48

Privatization of a business asset for purposes of enhancing its business viability and preventing further
losses, regardless of the character of the asset, should not take precedence over non-material values. A
commercial, nay even a budgetary, objective should not be pursued at the expense of national pride and
dignity. For the Constitution enshrines higher and nobler non-material values. Indeed, the Court will
Republic of the Philippines In March 2009, Congress amended RA 3046 by enacting RA 9522, the statute now under scrutiny. The
SUPREME COURT change was prompted by the need to make RA 3046 compliant with the terms of the United Nations
Manila Convention on the Law of the Sea (UNCLOS III),5 which the Philippines ratified on 27 February
1984.6 Among others, UNCLOS III prescribes the water-land ratio, length, and contour of baselines of
archipelagic States like the Philippines7 and sets the deadline for the filing of application for the extended
EN BANC
continental shelf.8 Complying with these requirements, RA 9522 shortened one baseline, optimized the
location of some basepoints around the Philippine archipelago and classified adjacent territories,
G.R No. 187167 August 16, 2011 namely, the Kalayaan Island Group (KIG) and the Scarborough Shoal, as "regimes of islands" whose
islands generate their own applicable maritime zones.
PROF. MERLIN M. MAGALLONA, AKBAYAN PARTY-LIST REP. RISA HONTIVEROS, PROF. HARRY C.
ROQUE, JR., AND UNIVERSITY OF THE PHILIPPINES COLLEGE OF LAW STUDENTS, ALITHEA BARBARA Petitioners, professors of law, law students and a legislator, in their respective capacities as "citizens,
ACAS, VOLTAIRE ALFERES, CZARINA MAY ALTEZ, FRANCIS ALVIN ASILO, SHERYL BALOT, RUBY AMOR taxpayers or x x x legislators,"9 as the case may be, assail the constitutionality of RA 9522 on two
BARRACA, JOSE JAVIER BAUTISTA, ROMINA BERNARDO, VALERIE PAGASA BUENAVENTURA, EDAN principal grounds, namely: (1) RA 9522 reduces Philippine maritime territory, and logically, the reach of
MARRI CAETE, VANN ALLEN DELA CRUZ, RENE DELORINO, PAULYN MAY DUMAN, SHARON ESCOTO, the Philippine states sovereign power, in violation of Article 1 of the 1987 Constitution, 10 embodying the
RODRIGO FAJARDO III, GIRLIE FERRER, RAOULLE OSEN FERRER, CARLA REGINA GREPO, ANNA MARIE terms of the Treaty of Paris11 and ancillary treaties,12 and (2) RA 9522 opens the countrys waters
CECILIA GO, IRISH KAY KALAW, MARY ANN JOY LEE, MARIA LUISA MANALAYSAY, MIGUEL RAFAEL landward of the baselines to maritime passage by all vessels and aircrafts, undermining Philippine
MUSNGI, MICHAEL OCAMPO, JAKLYN HANNA PINEDA, WILLIAM RAGAMAT, MARICAR RAMOS, ENRIK sovereignty and national security, contravening the countrys nuclear-free policy, and damaging marine
FORT REVILLAS, JAMES MARK TERRY RIDON, JOHANN FRANTZ RIVERA IV, CHRISTIAN RIVERO, DIANNE resources, in violation of relevant constitutional provisions.13
MARIE ROA, NICHOLAS SANTIZO, MELISSA CHRISTINA SANTOS, CRISTINE MAE TABING, VANESSA
ANNE TORNO, MARIA ESTER VANGUARDIA, and MARCELINO VELOSO III, Petitioners,
In addition, petitioners contend that RA 9522s treatment of the KIG as "regime of islands" not only
vs.
results in the loss of a large maritime area but also prejudices the livelihood of subsistence
HON. EDUARDO ERMITA, IN HIS CAPACITY AS EXECUTIVE SECRETARY, HON. ALBERTO ROMULO, IN HIS
fishermen.14 To buttress their argument of territorial diminution, petitioners facially attack RA 9522 for
CAPACITY AS SECRETARY OF THE DEPARTMENT OF FOREIGN AFFAIRS, HON. ROLANDO ANDAYA, IN HIS
what it excluded and included its failure to reference either the Treaty of Paris or Sabah and its use of
CAPACITY AS SECRETARY OF THE DEPARTMENT OF BUDGET AND MANAGEMENT, HON. DIONY
UNCLOS IIIs framework of regime of islands to determine the maritime zones of the KIG and the
VENTURA, IN HIS CAPACITY AS ADMINISTRATOR OF THE NATIONAL MAPPING & RESOURCE
Scarborough Shoal.
INFORMATION AUTHORITY, and HON. HILARIO DAVIDE, JR., IN HIS CAPACITY AS REPRESENTATIVE OF
THE PERMANENT MISSION OF THE REPUBLIC OF THE PHILIPPINES TO THE UNITED
NATIONS,Respondents. Commenting on the petition, respondent officials raised threshold issues questioning (1) the petitions
compliance with the case or controversy requirement for judicial review grounded on petitioners
alleged lack of locus standiand (2) the propriety of the writs of certiorari and prohibition to assail the
DECISION
constitutionality of RA 9522. On the merits, respondents defended RA 9522 as the countrys compliance
with the terms of UNCLOS III, preserving Philippine territory over the KIG or Scarborough Shoal.
CARPIO, J.: Respondents add that RA 9522 does not undermine the countrys security, environment and economic
interests or relinquish the Philippines claim over Sabah.
The Case
Respondents also question the normative force, under international law, of petitioners assertion that
This original action for the writs of certiorari and prohibition assails the constitutionality of Republic Act what Spain ceded to the United States under the Treaty of Paris were the islands and all the
No. 95221(RA 9522) adjusting the countrys archipelagic baselines and classifying the baseline regime of waters found within the boundaries of the rectangular area drawn under the Treaty of Paris.
nearby territories.
We left unacted petitioners prayer for an injunctive writ.
The Antecedents
The Issues
In 1961, Congress passed Republic Act No. 3046 (RA 3046)2 demarcating the maritime baselines of the
Philippines as an archipelagic State.3 This law followed the framing of the Convention on the Territorial The petition raises the following issues:
Sea and the Contiguous Zone in 1958 (UNCLOS I),4 codifying, among others, the sovereign right of States
parties over their "territorial sea," the breadth of which, however, was left undetermined. Attempts to
1. Preliminarily
fill this void during the second round of negotiations in Geneva in 1960 (UNCLOS II) proved futile. Thus,
domestically, RA 3046 remained unchanged for nearly five decades, save for legislation passed in 1968
(Republic Act No. 5446 [RA 5446]) correcting typographical errors and reserving the drawing of baselines 1. Whether petitioners possess locus standi to bring this suit; and
around Sabah in North Borneo.
2. Whether the writs of certiorari and prohibition are the proper remedies to assail
the constitutionality of RA 9522.
2. On the merits, whether RA 9522 is unconstitutional. the time of the Treaty of Paris, that Spain supposedly ceded to the United States. Petitioners argue that
from the Treaty of Paris technical description, Philippine sovereignty over territorial waters extends
hundreds of nautical miles around the Philippine archipelago, embracing the rectangular area delineated
The Ruling of the Court
in the Treaty of Paris.22

On the threshold issues, we hold that (1) petitioners possess locus standi to bring this suit as citizens and
Petitioners theory fails to persuade us.
(2) the writs of certiorari and prohibition are proper remedies to test the constitutionality of RA 9522. On
the merits, we find no basis to declare RA 9522 unconstitutional.
UNCLOS III has nothing to do with the acquisition (or loss) of territory. It is a multilateral treaty
regulating, among others, sea-use rights over maritime zones (i.e., the territorial waters [12 nautical
On the Threshold Issues
miles from the baselines], contiguous zone [24 nautical miles from the baselines], exclusive economic
Petitioners Possess Locus
zone [200 nautical miles from the baselines]), and continental shelves that UNCLOS III delimits.23 UNCLOS
Standi as Citizens
III was the culmination of decades-long negotiations among United Nations members to codify norms
regulating the conduct of States in the worlds oceans and submarine areas, recognizing coastal and
Petitioners themselves undermine their assertion of locus standi as legislators and taxpayers because the archipelagic States graduated authority over a limited span of waters and submarine lands along their
petition alleges neither infringement of legislative prerogative15 nor misuse of public funds,16 occasioned coasts.
by the passage and implementation of RA 9522. Nonetheless, we recognize petitioners locus standi as
citizens with constitutionally sufficient interest in the resolution of the merits of the case which
On the other hand, baselines laws such as RA 9522 are enacted by UNCLOS III States parties to mark-out
undoubtedly raises issues of national significance necessitating urgent resolution. Indeed, owing to the
specific basepoints along their coasts from which baselines are drawn, either straight or contoured, to
peculiar nature of RA 9522, it is understandably difficult to find other litigants possessing "a more direct
serve as geographic starting points to measure the breadth of the maritime zones and continental shelf.
and specific interest" to bring the suit, thus satisfying one of the requirements for granting citizenship
Article 48 of UNCLOS III on archipelagic States like ours could not be any clearer:
standing.17

Article 48. Measurement of the breadth of the territorial sea, the contiguous zone, the exclusive economic
The Writs of Certiorari and Prohibition
zone and the continental shelf. The breadth of the territorial sea, the contiguous zone, the exclusive
Are Proper Remedies to Test
economic zone and the continental shelf shall be measured from archipelagic baselines drawn in
the Constitutionality of Statutes
accordance with article 47. (Emphasis supplied)

In praying for the dismissal of the petition on preliminary grounds, respondents seek a strict observance
Thus, baselines laws are nothing but statutory mechanisms for UNCLOS III States parties to delimit with
of the offices of the writs of certiorari and prohibition, noting that the writs cannot issue absent any
precision the extent of their maritime zones and continental shelves. In turn, this gives notice to the rest
showing of grave abuse of discretion in the exercise of judicial, quasi-judicial or ministerial powers on the
of the international community of the scope of the maritime space and submarine areas within which
part of respondents and resulting prejudice on the part of petitioners.18
States parties exercise treaty-based rights, namely, the exercise of sovereignty over territorial waters
(Article 2), the jurisdiction to enforce customs, fiscal, immigration, and sanitation laws in the contiguous
Respondents submission holds true in ordinary civil proceedings. When this Court exercises its zone (Article 33), and the right to exploit the living and non-living resources in the exclusive economic
constitutional power of judicial review, however, we have, by tradition, viewed the writs of certiorari and zone (Article 56) and continental shelf (Article 77).
prohibition as proper remedial vehicles to test the constitutionality of statutes,19 and indeed, of acts of
other branches of government.20Issues of constitutional import are sometimes crafted out of statutes
Even under petitioners theory that the Philippine territory embraces the islands and all the
which, while having no bearing on the personal interests of the petitioners, carry such relevance in the
waters within the rectangular area delimited in the Treaty of Paris, the baselines of the Philippines would
life of this nation that the Court inevitably finds itself constrained to take cognizance of the case and pass
still have to be drawn in accordance with RA 9522 because this is the only way to draw the baselines in
upon the issues raised, non-compliance with the letter of procedural rules notwithstanding. The statute
conformity with UNCLOS III. The baselines cannot be drawn from the boundaries or other portions of the
sought to be reviewed here is one such law.
rectangular area delineated in the Treaty of Paris, but from the "outermost islands and drying reefs of
the archipelago."24
RA 9522 is Not Unconstitutional
RA 9522 is a Statutory Tool
UNCLOS III and its ancillary baselines laws play no role in the acquisition, enlargement or, as petitioners
to Demarcate the Countrys
claim, diminution of territory. Under traditional international law typology, States acquire (or conversely,
Maritime Zones and Continental
lose) territory through occupation, accretion, cession and prescription,25 not by executing multilateral
Shelf Under UNCLOS III, not to
treaties on the regulations of sea-use rights or enacting statutes to comply with the treatys terms to
Delineate Philippine Territory
delimit maritime zones and continental shelves. Territorial claims to land features are outside UNCLOS
III, and are instead governed by the rules on general international law.26
Petitioners submit that RA 9522 "dismembers a large portion of the national territory"21 because it
discards the pre-UNCLOS III demarcation of Philippine territory under the Treaty of Paris and related
RA 9522s Use of the Framework
treaties, successively encoded in the definition of national territory under the 1935, 1973 and 1987
of Regime of Islands to Determine the
Constitutions. Petitioners theorize that this constitutional definition trumps any treaty or statutory
Maritime Zones of the KIG and the
provision denying the Philippines sovereign control over waters, beyond the territorial sea recognized at
Scarborough Shoal, not Inconsistent SEC. 2. The baselines in the following areas over which the Philippines likewise exercises sovereignty
with the Philippines Claim of Sovereignty and jurisdiction shall be determined as "Regime of Islands" under the Republic of the Philippines
Over these Areas consistent with Article 121 of the United Nations Convention on the Law of the Sea (UNCLOS):

Petitioners next submit that RA 9522s use of UNCLOS IIIs regime of islands framework to draw the a) The Kalayaan Island Group as constituted under Presidential Decree No. 1596 and
baselines, and to measure the breadth of the applicable maritime zones of the KIG, "weakens our
territorial claim" over that area.27 Petitioners add that the KIGs (and Scarborough Shoals) exclusion
b) Bajo de Masinloc, also known as Scarborough Shoal. (Emphasis supplied)
from the Philippine archipelagic baselines results in the loss of "about 15,000 square nautical miles of
territorial waters," prejudicing the livelihood of subsistence fishermen.28 A comparison of the
configuration of the baselines drawn under RA 3046 and RA 9522 and the extent of maritime space Had Congress in RA 9522 enclosed the KIG and the Scarborough Shoal as part of the Philippine
encompassed by each law, coupled with a reading of the text of RA 9522 and its congressional archipelago, adverse legal effects would have ensued. The Philippines would have committed a breach of
deliberations, vis--vis the Philippines obligations under UNCLOS III, belie this view.1avvphi1 two provisions of UNCLOS III. First, Article 47 (3) of UNCLOS III requires that "[t]he drawing of such
baselines shall not depart to any appreciable extent from the general configuration of the archipelago."
Second, Article 47 (2) of UNCLOS III requires that "the length of the baselines shall not exceed 100
The configuration of the baselines drawn under RA 3046 and RA 9522 shows that RA 9522 merely
nautical miles," save for three per cent (3%) of the total number of baselines which can reach up to 125
followed the basepoints mapped by RA 3046, save for at least nine basepoints that RA 9522 skipped to
nautical miles.31
optimize the location of basepoints and adjust the length of one baseline (and thus comply with UNCLOS
IIIs limitation on the maximum length of baselines). Under RA 3046, as under RA 9522, the KIG and the
Scarborough Shoal lie outside of the baselines drawn around the Philippine archipelago. This undeniable Although the Philippines has consistently claimed sovereignty over the KIG32 and the Scarborough Shoal
cartographic fact takes the wind out of petitioners argument branding RA 9522 as a statutory for several decades, these outlying areas are located at an appreciable distance from the nearest
renunciation of the Philippines claim over the KIG, assuming that baselines are relevant for this purpose. shoreline of the Philippine archipelago,33 such that any straight baseline loped around them from the
nearest basepoint will inevitably "depart to an appreciable extent from the general configuration of the
archipelago."
Petitioners assertion of loss of "about 15,000 square nautical miles of territorial waters" under RA 9522
is similarly unfounded both in fact and law. On the contrary, RA 9522, by optimizing the location of
basepoints, increased the Philippines total maritime space (covering its internal waters, territorial sea The principal sponsor of RA 9522 in the Senate, Senator Miriam Defensor-Santiago, took pains to
and exclusive economic zone) by 145,216 square nautical miles, as shown in the table below:29 emphasize the foregoing during the Senate deliberations:

What we call the Kalayaan Island Group or what the rest of the world call[] the Spratlys and the
Extent of maritime area using
Extent of maritime area using Scarborough Shoal are outside our archipelagic baseline because if we put them inside our baselines we
RA 3046, as amended, taking
RA 9522, taking into account might be accused of violating the provision of international law which states: "The drawing of such
into account the Treaty of
UNCLOS III (in square nautical baseline shall not depart to any appreciable extent from the general configuration of the archipelago." So
Paris delimitation (in square
miles) sa loob ng ating baseline, dapat magkalapit ang mga islands. Dahil malayo ang Scarborough Shoal, hindi
nautical miles)
natin masasabing malapit sila sa atin although we are still allowed by international law to claim them as
Internal or archipelagic our own.
waters 166,858 171,435
This is called contested islands outside our configuration. We see that our archipelago is defined by the
Territorial Sea 274,136 32,106 orange line which [we] call[] archipelagic baseline. Ngayon, tingnan ninyo ang maliit na circle doon sa
Exclusive Economic Zone 382,669 itaas, that is Scarborough Shoal, itong malaking circle sa ibaba, that is Kalayaan Group or the
Spratlys. Malayo na sila sa ating archipelago kaya kung ilihis pa natin ang dating archipelagic baselines
TOTAL 440,994 586,210 para lamang masama itong dalawang circles, hindi na sila magkalapit at baka hindi na tatanggapin ng
United Nations because of the rule that it should follow the natural configuration of the
archipelago.34 (Emphasis supplied)
Thus, as the map below shows, the reach of the exclusive economic zone drawn under RA 9522 even
extends way beyond the waters covered by the rectangular demarcation under the Treaty of Paris. Of
Similarly, the length of one baseline that RA 3046 drew exceeded UNCLOS IIIs limits.1avvphi1 The need
course, where there are overlapping exclusive economic zones of opposite or adjacent States, there will
to shorten this baseline, and in addition, to optimize the location of basepoints using current maps,
have to be a delineation of maritime boundaries in accordance with UNCLOS III.30
became imperative as discussed by respondents:

[T]he amendment of the baselines law was necessary to enable the Philippines to draw the outer limits
of its maritime zones including the extended continental shelf in the manner provided by Article 47 of
Further, petitioners argument that the KIG now lies outside Philippine territory because the baselines [UNCLOS III]. As defined by R.A. 3046, as amended by R.A. 5446, the baselines suffer from some technical
that RA 9522 draws do not enclose the KIG is negated by RA 9522 itself. Section 2 of the law commits to deficiencies, to wit:
text the Philippines continued claim of sovereignty and jurisdiction over the KIG and the Scarborough
Shoal:
1. The length of the baseline across Moro Gulf (from Middle of 3 Rock Awash to Tongquil Article 49. Legal status of archipelagic waters, of the air space over archipelagic waters and of their bed
Point) is 140.06 nautical miles x x x. This exceeds the maximum length allowed under Article and subsoil.
47(2) of the [UNCLOS III], which states that "The length of such baselines shall not exceed 100
nautical miles, except that up to 3 per cent of the total number of baselines enclosing any
1. The sovereignty of an archipelagic State extends to the waters enclosed by the
archipelago may exceed that length, up to a maximum length of 125 nautical miles."
archipelagic baselines drawn in accordance with article 47, described as archipelagic waters,
regardless of their depth or distance from the coast.
2. The selection of basepoints is not optimal. At least 9 basepoints can be skipped or deleted
from the baselines system. This will enclose an additional 2,195 nautical miles of water.
2. This sovereignty extends to the air space over the archipelagic waters, as well as to their
bed and subsoil, and the resources contained therein.
3. Finally, the basepoints were drawn from maps existing in 1968, and not established by
geodetic survey methods. Accordingly, some of the points, particularly along the west coasts
xxxx
of Luzon down to Palawan were later found to be located either inland or on water, not on
low-water line and drying reefs as prescribed by Article 47.35
4. The regime of archipelagic sea lanes passage established in this Part shall not in other
respects affect the status of the archipelagic waters, including the sea lanes, or the exercise
Hence, far from surrendering the Philippines claim over the KIG and the Scarborough Shoal, Congress
by the archipelagic State of its sovereignty over such waters and their air space, bed and
decision to classify the KIG and the Scarborough Shoal as "Regime[s] of Islands under the Republic of
subsoil, and the resources contained therein. (Emphasis supplied)
the Philippines consistent with Article 121"36 of UNCLOS III manifests the Philippine States responsible
observance of its pacta sunt servanda obligation under UNCLOS III. Under Article 121 of UNCLOS III, any
"naturally formed area of land, surrounded by water, which is above water at high tide," such as portions The fact of sovereignty, however, does not preclude the operation of municipal and international law
of the KIG, qualifies under the category of "regime of islands," whose islands generate their own norms subjecting the territorial sea or archipelagic waters to necessary, if not marginal, burdens in the
applicable maritime zones.37 interest of maintaining unimpeded, expeditious international navigation, consistent with the
international law principle of freedom of navigation. Thus, domestically, the political branches of the
Philippine government, in the competent discharge of their constitutional powers, may pass legislation
Statutory Claim Over Sabah under
designating routes within the archipelagic waters to regulate innocent and sea lanes passage.40 Indeed,
RA 5446 Retained
bills drawing nautical highways for sea lanes passage are now pending in Congress.41

Petitioners argument for the invalidity of RA 9522 for its failure to textualize the Philippines claim over
In the absence of municipal legislation, international law norms, now codified in UNCLOS III, operate to
Sabah in North Borneo is also untenable. Section 2 of RA 5446, which RA 9522 did not repeal, keeps open
grant innocent passage rights over the territorial sea or archipelagic waters, subject to the treatys
the door for drawing the baselines of Sabah:
limitations and conditions for their exercise.42 Significantly, the right of innocent passage is a customary
international law,43 thus automatically incorporated in the corpus of Philippine law.44 No modern State
Section 2. The definition of the baselines of the territorial sea of the Philippine Archipelago as provided can validly invoke its sovereignty to absolutely forbid innocent passage that is exercised in accordance
in this Actis without prejudice to the delineation of the baselines of the territorial sea around the with customary international law without risking retaliatory measures from the international
territory of Sabah, situated in North Borneo, over which the Republic of the Philippines has acquired community.
dominion and sovereignty. (Emphasis supplied)
The fact that for archipelagic States, their archipelagic waters are subject to both the right of innocent
UNCLOS III and RA 9522 not passage and sea lanes passage45 does not place them in lesser footing vis--vis continental coastal States
Incompatible with the Constitutions which are subject, in their territorial sea, to the right of innocent passage and the right of transit passage
Delineation of Internal Waters through international straits. The imposition of these passage rights through archipelagic waters under
UNCLOS III was a concession by archipelagic States, in exchange for their right to claim all the waters
landward of their baselines,regardless of their depth or distance from the coast, as archipelagic waters
As their final argument against the validity of RA 9522, petitioners contend that the law
subject to their territorial sovereignty. More importantly, the recognition of archipelagic States
unconstitutionally "converts" internal waters into archipelagic waters, hence subjecting these waters to
archipelago and the waters enclosed by their baselines as one cohesive entity prevents the treatment of
the right of innocent and sea lanes passage under UNCLOS III, including overflight. Petitioners
their islands as separate islands under UNCLOS III.46 Separate islands generate their own maritime zones,
extrapolate that these passage rights indubitably expose Philippine internal waters to nuclear and
placing the waters between islands separated by more than 24 nautical miles beyond the States
maritime pollution hazards, in violation of the Constitution.38
territorial sovereignty, subjecting these waters to the rights of other States under UNCLOS III.47

Whether referred to as Philippine "internal waters" under Article I of the Constitution39 or as


Petitioners invocation of non-executory constitutional provisions in Article II (Declaration of Principles
"archipelagic waters" under UNCLOS III (Article 49 [1]), the Philippines exercises sovereignty over the
and State Policies)48 must also fail. Our present state of jurisprudence considers the provisions in Article
body of water lying landward of the baselines, including the air space over it and the submarine areas
II as mere legislative guides, which, absent enabling legislation, "do not embody judicially enforceable
underneath. UNCLOS III affirms this:
constitutional rights x x x."49 Article II provisions serve as guides in formulating and interpreting
implementing legislation, as well as in interpreting executory provisions of the Constitution.
Although Oposa v. Factoran50 treated the right to a healthful and balanced ecology under Section 16 of
Article II as an exception, the present petition lacks factual basis to substantiate the claimed
constitutional violation. The other provisions petitioners cite, relating to the protection of marine wealth
(Article XII, Section 2, paragraph 251 ) and subsistence fishermen (Article XIII, Section 752 ), are not
violated by RA 9522.

In fact, the demarcation of the baselines enables the Philippines to delimit its exclusive economic zone,
reserving solely to the Philippines the exploitation of all living and non-living resources within such zone.
Such a maritime delineation binds the international community since the delineation is in strict
observance of UNCLOS III. If the maritime delineation is contrary to UNCLOS III, the international
community will of course reject it and will refuse to be bound by it.

UNCLOS III favors States with a long coastline like the Philippines. UNCLOS III creates a sui
generis maritime space the exclusive economic zone in waters previously part of the high seas.
UNCLOS III grants new rights to coastal States to exclusively exploit the resources found within this zone
up to 200 nautical miles.53 UNCLOS III, however, preserves the traditional freedom of navigation of other
States that attached to this zone beyond the territorial sea before UNCLOS III.

RA 9522 and the Philippines Maritime Zones

Petitioners hold the view that, based on the permissive text of UNCLOS III, Congress was not bound to
pass RA 9522.54 We have looked at the relevant provision of UNCLOS III55 and we find petitioners reading
plausible. Nevertheless, the prerogative of choosing this option belongs to Congress, not to this Court.
Moreover, the luxury of choosing this option comes at a very steep price. Absent an UNCLOS III
compliant baselines law, an archipelagic State like the Philippines will find itself devoid of internationally
acceptable baselines from where the breadth of its maritime zones and continental shelf is measured.
This is recipe for a two-fronted disaster: first, it sends an open invitation to the seafaring powers to
freely enter and exploit the resources in the waters and submarine areas around our archipelago;
and second, it weakens the countrys case in any international dispute over Philippine maritime space.
These are consequences Congress wisely avoided.

The enactment of UNCLOS III compliant baselines law for the Philippine archipelago and adjacent areas,
as embodied in RA 9522, allows an internationally-recognized delimitation of the breadth of the
Philippines maritime zones and continental shelf. RA 9522 is therefore a most vital step on the part of
the Philippines in safeguarding its maritime zones, consistent with the Constitution and our national
interest.

WHEREFORE, we DISMISS the petition.

SO ORDERED.
Republic of the Philippines As sequestrator of the 227 shares of stock in question, PCGG did not pay the corresponding monthly
SUPREME COURT membership due thereon totaling P2,959,471.00. On account thereof, the 227 sequestered shares were
Manila declared delinquent to be disposed of in an auction sale.

SECOND DIVISION Apprised of the above development and evidently to prevent the projected auction sale of the same
shares, PCGG filed a complaint for injunction with the Regional Trial Court (RTC) of Bacolod City, thereat
docketed as Civil Case No. 5348. The complaint, however, was dismissed, paving the way for the auction
G.R. No. 129406 March 6, 2006
sale for the delinquent 227 shares of stock. On August 5, 1989, an auction sale was conducted.

REPUBLIC OF THE PHILIPPINES represented by the PRESIDENTIAL COMMISSION ON GOOD


On November 3, 1990, petitioner Republic and private respondent Benedicto entered into a Compromise
GOVERNMENT (PCGG), Petitioner,
Agreement in Civil Case No. 0034. The agreement contained a general release clause5 whereunder
vs.
petitioner Republic agreed and bound itself to lift the sequestration on the 227 NOGCCI shares, among
SANDIGANBAYAN (SECOND DIVISION) and ROBERTO S. BENEDICTO, Respondents.
other Benedictos properties, petitioner Republic acknowledging that it was within private respondent
Benedictos capacity to acquire the same shares out of his income from business and the exercise of his
DECISION profession.6 Implied in this undertaking is the recognition by petitioner Republic that the subject shares
of stock could not have been ill-gotten.
GARCIA, J.:
In a decision dated October 2, 1992, the Sandiganbayan approved the Compromise Agreement and
Before the Court is this petition for certiorari under Rule 65 of the Rules of Court to nullify and set aside accordingly rendered judgment in accordance with its terms.
the March 28, 19951 and March 13, 19972 Resolutions of the Sandiganbayan, Second Division, in Civil
Case No. 0034, insofar as said resolutions ordered the Presidential Commission on Good Government In the process of implementing the Compromise Agreement, either of the parties would, from time to
(PCGG) to pay private respondent Roberto S. Benedicto or his corporations the value of 227 shares of time, move for a ruling by the Sandiganbayan on the proper manner of implementing or interpreting a
stock of the Negros Occidental Golf and Country Club, Inc. (NOGCCI) at P150,000.00 per share, registered specific provision therein.
in the name of said private respondent or his corporations.
On February 22, 1994, Benedicto filed in Civil Case No. 0034 a "Motion for Release from Sequestration
The facts: and Return of Sequestered Shares/Dividends" praying, inter alia, that his NOGCCI shares of stock be
specifically released from sequestration and returned, delivered or paid to him as part of the parties
Civil Case No. 0034 entitled Republic of the Philippines, plaintiff, v. Roberto S. Benedicto, et al., Compromise Agreement in that case. In a Resolution7 promulgated on December 6, 1994, the
defendants, is a complaint for reconveyance, reversion, accounting, reconstitution and damages. The Sandiganbayan granted Benedictos aforementioned motion but placed the subject shares under the
case is one of several suits involving ill-gotten or unexplained wealth that petitioner Republic, through custody of its Clerk of Court, thus:
the PCGG, filed with the Sandiganbayan against private respondent Roberto S. Benedicto and others
pursuant to Executive Order (EO) No. 14,3 series of 1986. WHEREFORE, in the light of the foregoing, the said "Motion for Release From Sequestration and Return
of Sequestered Shares/Dividends" is hereby GRANTED and it is directed that said shares/dividends be
Pursuant to its mandate under EO No. 1,4 series of 1986, the PCGG issued writs placing under delivered/placed under the custody of the Clerk of Court, Sandiganbayan, Manila subject to this Courts
sequestration all business enterprises, entities and other properties, real and personal, owned or disposition.
registered in the name of private respondent Benedicto, or of corporations in which he appeared to have
controlling or majority interest. Among the properties thus sequestered and taken over by PCGG fiscal On March 28, 1995, the Sandiganbayan came out with the herein first assailed Resolution,8 which
agents were the 227 shares in NOGCCI owned by private respondent Benedicto and registered in his clarified its aforementioned December 6, 1994 Resolution and directed the immediate implementation
name or under the names of corporations he owned or controlled. thereof by requiring PCGG, among other things:

Following the sequestration process, PCGG representatives sat as members of the Board of Directors of (b) To deliver to the Clerk of Court the 227 sequestered shares of [NOGCCI] registered in the name of
NOGCCI, which passed, sometime in October 1986, a resolution effecting a corporate policy change. The nominees of ROBERTO S. BENEDICTO free from all liens and encumbrances, or in default thereof, to pay
change consisted of assessing a monthly membership due of P150.00 for each NOGCCI share. Prior to their value at P150,000.00 per share which can be deducted from [the Republics] cash share in the
this resolution, an investor purchasing more than one NOGCCI share was exempt from paying monthly Compromise Agreement. [Words in bracket added] (Emphasis Supplied).
membership due for the second and subsequent shares that he/she owned.
Owing to PCGGs failure to comply with the above directive, Benedicto filed in Civil Case No. 0034 a
Subsequently, on March 29, 1987, the NOGCCI Board passed another resolution, this time increasing the Motion for Compliance dated July 25, 1995, followed by an Ex-Parte Motion for Early Resolution dated
monthly membership due from P150.00 to P250.00 for each share. February 12, 1996. Acting thereon, the Sandiganbayan promulgated yet another Resolution9 on February
23, 1996, dispositively reading:
WHEREFORE, finding merit in the instant motion for early resolution and considering that, indeed, the shares ought not to have come to pass had those fiscal agents prudently not agreed to the passage of
PCGG has not shown any justifiable ground as to why it has not complied with its obligation as set forth the NOGCCI board resolutions charging membership dues on shares without playing representatives.
in the Order of December 6, 1994 up to this date and which Order was issued pursuant to the
Compromise Agreement and has already become final and executory, accordingly, the Presidential
Given the circumstances leading to the auction sale of the subject NOGCCI shares, PCGGs lament about
Commission on Good Government is hereby given a final extension of fifteen (15) days from receipt
public respondent Sandiganbayan having erred or, worse still, having gravely abused its discretion in its
hereof within which to comply with the Order of December 6, 1994 as stated hereinabove.
determination as to who is at fault for the loss of the shares in question can hardly be given cogency.

On April 1, 1996, PCGG filed a Manifestation with Motion for Reconsideration,10 praying for the setting
For sure, even if the Sandiganbayan were wrong in its findings, which does not seem to be in this case, it
aside of the Resolution of February 23, 1996. On April 11, 1996, private respondent Benedicto filed a
is a well-settled rule of jurisprudence that certiorari will issue only to correct errors of jurisdiction, not
Motion to Enforce Judgment Levy. Resolving these two motions, the Sandiganbayan, in its second
errors of judgment. Corollarily, errors of procedure or mistakes in the courts findings and conclusions
assailed Resolution11 dated March 13, 1997, denied that portion of the PCGGs Manifestation with
are beyond the corrective hand of certiorari.14 The extraordinary writ of certiorari may be availed only
Motion for Reconsideration concerning the subject 227 NOGCCI shares and granted Benedictos Motion
upon a showing, in the minimum, that the respondent tribunal or officer exercising judicial or quasi-
to Enforce Judgment Levy.
judicial functions has acted without or in excess of its or his jurisdiction, or with grave abuse of
discretion.15
Hence, the Republics present recourse on the sole issue of whether or not the public respondent
Sandiganbayan, Second Division, gravely abused its discretion in holding that the PCGG is at fault for not
The term "grave abuse of discretion" connotes capricious and whimsical exercise of judgment as is
paying the membership dues on the 227 sequestered NOGCCI shares of stock, a failing which eventually
equivalent to excess, or a lack of jurisdiction.16 The abuse must be so patent and gross as to amount to
led to the foreclosure sale thereof.
an evasion of a positive duty or a virtual refusal to perform a duty enjoined by law, or to act at all in
contemplation of law as where the power is exercised in an arbitrary and despotic manner by reason of
The petition lacks merit. passion or hostility.17 Sadly, this is completely absent in the present case. For, at bottom, the assailed
resolutions of the Sandiganbayan did no more than to direct PCGG to comply with its part of the bargain
under the compromise agreement it freely entered into with private respondent Benedicto. Simply put,
To begin with, PCGG itself does not dispute its being considered as a receiver insofar as the sequestered
the assailed resolutions of the Sandiganbayan have firm basis in fact and in law.
227 NOGCCI shares of stock are concerned.12 PCGG also acknowledges that as such receiver, one of its
functions is to pay outstanding debts pertaining to the sequestered entity or property,13 in this case the
227 NOGCCI shares in question. It contends, however, that membership dues owing to a golf club cannot Lest it be overlooked, the issue of liability for the shares in question had, as both public and private
be considered as an outstanding debt for which PCGG, as receiver, must pay. It also claims to have respondents asserted, long become final and executory. Petitioners narration of facts in its present
exercised due diligence to prevent the loss through delinquency sale of the subject NOGCCI shares, petition is even misleading as it conveniently fails to make reference to two (2) resolutions issued by the
specifically inviting attention to the injunctive suit, i.e., Civil Case No. 5348, it filed before the RTC of Sandiganbayan. We refer to that courts resolutions of December 6, 199418 and February 23, 199619 as
Bacolod City to enjoin the foreclosure sale of the shares. well as several intervening pleadings which served as basis for the decisions reached therein. As it were,
the present petition questions only and focuses on the March 28, 199520 and March 13,
199721 resolutions, which merely reiterated and clarified the graft courts underlying resolution of
The filing of the injunction complaint adverted to, without more, cannot plausibly tilt the balance in
December 6, 1994. And to place matters in the proper perspective, PCGGs failure to comply with the
favor of PCGG. To the mind of the Court, such filing is a case of acting too little and too late. It cannot be
December 6, 1994 resolution prompted the issuance of the clarificatory and/or reiteratory resolutions
over-emphasized that it behooved the PCGGs fiscal agents to preserve, like a responsible father of the
aforementioned.
family, the value of the shares of stock under their administration. But far from acting as such father,
what the fiscal agents did under the premises was to allow the element of delinquency to set in before
acting by embarking on a tedious process of going to court after the auction sale had been announced In a last-ditch attempt to escape liability, petitioner Republic, through the PCGG, invokes state immunity
and scheduled. from suit.22 As argued, the order for it to pay the value of the delinquent shares would fix monetary
liability on a government agency, thus necessitating the appropriation of public funds to satisfy the
judgment claim.23 But, as private respondent Benedicto correctly countered, the PCGG fails to take stock
The PCGGs posture that to the owner of the sequestered shares rests the burden of paying the
of one of the exceptions to the state immunity principle, i.e., when the government itself is the suitor, as
membership dues is untenable. For one, it lost sight of the reality that such dues are basically obligations
in Civil Case No. 0034. Where, as here, the State itself is no less the plaintiff in the main case, immunity
attached to the shares, which, in the final analysis, shall be made liable, thru delinquency sale in case of
from suit cannot be effectively invoked.24 For, as jurisprudence teaches, when the State, through its duly
default in payment of the dues. For another, the PCGG as sequestrator-receiver of such shares is, as
authorized officers, takes the initiative in a suit against a private party, it thereby descends to the level of
stressed earlier, duty bound to preserve the value of such shares. Needless to state, adopting timely
a private individual and thus opens itself to whatever counterclaims or defenses the latter may have
measures to obviate the loss of those shares forms part of such duty and due diligence.
against it.25 Petitioner Republics act of filing its complaint in Civil Case No. 0034 constitutes a waiver of
its immunity from suit. Being itself the plaintiff in that case, petitioner Republic cannot set up its
The Sandiganbayan, to be sure, cannot plausibly be faulted for finding the PCGG liable for the loss of the immunity against private respondent Benedictos prayers in the same case.
227 NOGCCI shares. There can be no quibbling, as indeed the graft court so declared in its assailed and
related resolutions respecting the NOGCCI shares of stock, that PCGGs fiscal agents, while sitting in the
In fact, by entering into a Compromise Agreement with private respondent Benedicto, petitioner
NOGCCI Board of Directors agreed to the amendment of the rule pertaining to membership dues. Hence,
Republic thereby stripped itself of its immunity from suit and placed itself in the same level of its
it is not amiss to state, as did the Sandiganbayan, that the PCGG-designated fiscal agents, no less, had a
adversary. When the State enters into contract, through its officers or agents, in furtherance of a
direct hand in the loss of the sequestered shares through delinquency and their eventual sale through
legitimate aim and purpose and pursuant to constitutional legislative authority, whereby mutual or
public auction. While perhaps anti-climactic to so mention it at this stage, the unfortunate loss of the
reciprocal benefits accrue and rights and obligations arise therefrom, the State may be sued even
without its express consent, precisely because by entering into a contract the sovereign descends to the
level of the citizen. Its consent to be sued is implied from the very act of entering into such
contract,26 breach of which on its part gives the corresponding right to the other party to the agreement.

Finally, it is apropos to stress that the Compromise Agreement in Civil Case No. 0034 envisaged the
immediate recovery of alleged ill-gotten wealth without further litigation by the government, and buying
peace on the part of the aging Benedicto.27 Sadly, that stated objective has come to naught as not only
had the litigation continued to ensue, but, worse, private respondent Benedicto passed away on May 15,
2000,28 with the trial of Civil Case No. 0034 still in swing, so much so that the late Benedicto had to be
substituted by the administratrix of his estate.29

WHEREFORE, the instant petition is hereby DISMISSED.

SO ORDERED.
Plaintiff Gaudencio A. Begosa, now appellee, sought the aid of the judiciary to obtain the benefits to
which he believed he was entitled under the Veterans Bill of Rights. 1 To such a move, there was an
insistent objection, both vigorous and persistent, on the part of defendants, the chairman and the
members of the Philippine Veterans Administration, now appellants. The lower court, then presided by
the then Judge, now Justice of the Court of Appeals, the Honorable Edilberto Soriano, found for
EN BANC plaintiffs, after a careful and meticulous study of the applicable statutory provisions. Not being satisfied
with such a judgment, defendants appealed, relying once more on the principal grounds raised below
[G.R. No. L-25916. April 30, 1970.] that plaintiff should have exhausted his administrative remedies before coming to court and that he was
in fact suing the State without its consent having been obtained. As neither defense is sufficiently
GAUDENCIO A. BEGOSA, Plaintiff-Appellee, v. CHAIRMAN, PHILIPPINE VETERANS ADMINISTRATION; meritorious, we affirm the lower court decision.
and MEMBERS OF THE BOARD OF ADMINISTRATORS, PHILIPPINE VETERANS
ADMINISTRATION, Defendants-Appellants. As noted in such decision, appellees complaint was predicated on his having been "an enlisted man in
good standing of the Philippine Commonwealth Army, inducted in the service of the USAFFE" and having
Jose V . Rosales for Plaintiff-Appellee. taken "active participation in the battle of Bataan" as well as the "liberation drive against the enemy"
thereafter having become "permanently incapacitated from work due to injuries he sustained in line of
Solicitor General, for Defendants-Appellants. duty . . ." 2 It was likewise asserted in his complaint that after having submitted all the supporting papers
relative to his complaint, there was a disapproval on the part of defendants on the ground of his having
been dishonorably discharged, although such an event did not take place until almost five years after the
SYLLABUS end of the war on November 7, 1950 and while he was in the service of a different organization that such
a penalty was imposed on him. 3

1. REMEDIAL LAW; PARTIES; DOCTRINE OF NON-SUABILITY OF STATE, EXCEPTION. It is well settled that Then came the allegation that there was an approval on his claim on September 2, 1964 but effective
where a litigation may have adverse consequences on the public treasury, whether in the disbursements only as of October 5 of that year, and for amount much less than that to which he was entitled under the
of funds or loss of property, the public official proceeded against not being liable in his personal capacity, law. 4 The relief sought was the payment, as of the date to which he believed his right to pension should
then the doctrine of non-suability may appropriately be invoked. It has no application, however, where have been respected, of the sums, which he felt were legally due and owing to him.chanrobles virtual
the suit against such a functionary had to be instituted because of his failure to comply with the duty lawlibrary
imposed by statute appropriating public funds for the benefit of plaintiff or petitioner
The then Judge Soriano noted that there was an admission of certain allegations to the complaint with
2. ID.; ADMINISTRATIVE LAW; EXHAUSTION OF ADMINISTRATIVE REMEDY UNNECESSARY WHERE others being denied, and that the following affirmative and special defenses were interposed:
QUESTION IS LEGAL; CASE AT BAR. It is well established that the principle requiring the previous "Defendants answer admits certain allegations of said complaint, while denying others; set up the
exhaustion of administrative remedies is not applicable where the question in dispute is purely a legal following affirmative and special defenses: (1) payment of disability pension under Republic Act No. 65,
one, or where the controverted act is patently illegal or was performed without jurisdiction or in excess as amended, by the Philippine Veterans Administration commences from the date the proper application
of jurisdiction, or where the respondent is a department Secretary, whose acts as an alter-ego of the therefor is approved; (2) plaintiff has not exhausted all administrative remedies before resorting to court
President bear the implied or assumed approval of the latter, unless actually disapproved by him, or action, hence the present action is premature; (3) inasmuch as the instant action pertains to money
where there are circumstances indicating the urgency of judicial intervention. Where there is a claim against the Government, it must first be presented before the Auditor General as provided by
stipulation of facts, as in this case, the question before the lower court being solely one of law and on existing law on the matter (C.A. 327); and (4) plaintiffs claim is in reality a suit against the Government
the face of the decision, the actuation of appellants being patently illegal, the doctrine of exhaustion of which cannot be entertained by this Court for lack of jurisdiction because the Government has not given
administrative remedies certainly does not come into play. its consent, . . ." 5 The case was then submitted on an agreed statement of facts and the respective
memoranda of the parties.
3. ID.; ID.; FINDINGS OF REGULATORY AGENCIES ENTITLED TO RESPECT. It has often been announced,
and rightly so, that as much as possible the findings of regulatory agencies which are expected to acquire In the decision now on appeal, the question of when appellee is entitled to his pension as well as how
expertise by their jurisdiction being confined to specific matters, deserve to be accorded respect and much it would amount to were fully discussed by the lower court. Thus, as to the former: "From the facts
finality. There is a limit, however, to such a deference paid to the actuations of such bodies. Clearly, just set out, it will be noted that plaintiff filed his said claim for disability pension as far back as March 4,
where there has been a failure to interpret and apply the statutory provisions in question, judicial power 1955; that it was erroneously disapproved on June 21, 1955, because his dishonorable discharge from
should assert itself. Under the theory of separation of powers, it is to the judiciary, and to the judiciary the Army was not a good or proper ground for the said disapproval, and that on reconsideration asked
alone, that the final say on questions of law in appropriate cases coming before it is vested. for by him on November 1, 1957, which he continued to follow up, the Board of Administrators,
Philippine Veterans Administration, composed of herein defendants, which took over the duties of the
Philippine Veterans Board, finally approved his claim on September 2, 1964, at the rate of P30.00 a
DECISION month." 6 After stating that in fairness and good conscience the said claims could be made effective as of
June 21, 1955, when it was erroneously disapproved by appellants, and not on September 2, 1964 when
it was approved on reconsideration, as appellee should not for obvious reason be made to suffer for the
FERNANDO, J.: error of another, the then Judge Soriano observed further: "Had it not been for the said error, it appears
that there was no good ground to deny the said claim, so the latter was valid and meritorious even as of
the date of its filing on March 4, 1955, hence to make the same effective only as of the date of its however, where the suit against such a functionary had to be instituted because of his failure to comply
approval on September 2, 1964 according to defendants stand would be greatly unfair and with the duty imposed by statute appropriating public funds for the benefit of plaintiff or petitioner.
prejudicial to plaintiff. This is especially true in the light of the well-known intent of the legislature in Such is the present case.chanroblesvirtual|awlibrary
passing these pension laws of war veterans, and the no less well-known spirit in which they should be
construed or interpreted by the courts in favor of their beneficiaries." 7 The doctrine announced by us in Ruiz v. Cabahug 12 finds relevance: "We hold that under the facts and
circumstances alleged in the amended complaint, which should be taken on its face value, the suit is not
On the question of how much plaintiff should receive according to law, the appealed decision contains one against the Government, or a claim against it, but one against the officials to compel them to act in
the following: "The next question for resolution refers to the monthly rate or amount to which plaintiff is accordance with the rights to be established by the contending architects, or to prevent them from
entitled by way of pension. According to plaintiff, he should be given a disability pension of P50.00 a making payment and recognition until the contending architects have established their respective rights
month from June 21, 1955 (the effective date of his claim as above found by this Court) until June 21, and interests in the funds retained and in the credit for the work done." 13 As a matter of fact, in an
1957, and P100.00 a month for life from June 22, 1957 when Section 9 of Republic Act No. 65, as earlier case where we sustained the power of a private citizen claiming title to and right of possession of
amended by Republic Act No. 1362, was further amended by Republic Act No. 1920). This contention is a certain property to sue an officer or agent of the government alleged to be illegally withholding the
well taken because the very letter of the Philippine Veterans Administration to plaintiff (Annex F of the same, we likewise expressed this caveat: "However, and this is important, where the judgment in such a
[Agreed Statements of Facts]) contains the following: Note: Re-rating is not required, permanent case would result not only in the recovery of possession of the property in favor of said citizen but also in
disability. By permanent disability, as this Court understands it, is meant that plaintiff is permanently a charge against or financial liability to the Government, then the suit should be regarded as one against
incapacitated from work. Under Section 9 of Republic Act No. 65, as amended by Republic Act No. 1362, the government itself, and, consequently, it cannot prosper or be validly entertained by the courts
which was the law in force when plaintiffs claim for pension should have been approved on June 21, except with the consent of said Government." 14
1955, he was entitled to a pension of P50.00 a month as such permanently incapacitated person, which
monthly rate or amount was increased to P100.00 a month when the said Section 9 was further 2. Nor is the third assignment of error to the effect that the lower court did not require appellee to
amended by Republic Act No. 1920 on June 22, 1957." 8 Why the action of appellants in the form of exhaust his administrative remedies before coming to court any more persuasive. An excerpt from the
resolution could not prevail as against the law was made clear by the decision in this wise: "For one leading case of Gonzales v. Hechanova, 15 the opinion being penned by the present Chief Justice, clearly
thing, the said resolution may not change or amend the meaning of the term permanent disability as demonstrates why appellants argument in this respect is unavailing: "Respondents assail petitioners
used by Congress itself in enacting the said Section 9 of Republic Act No. 65, as amended. For another, as right to the reliefs prayed for because he has not exhausted all administrative remedies available to him
of June 21, 1955 and as of June 21, 1957, plaintiff was already entitled to the said pension of P50.00 and before coming to court. We have already held, however, that the principle requiring the previous
P100.00 a month respectively, and his said right cannot be adversely affected by a resolution which was exhaustion of administrative remedies is not applicable where the question in dispute is purely a legal
allegedly adopted only in 1963." 9 Necessarily, there was in the decision likewise a recognition of the one, or where the controverted act is patently illegal or was performed without jurisdiction or in excess
monthly allowance for each of appellees unmarried minor children below 18 years of age at the time he of jurisdiction, or where the respondent is a department secretary, whose acts as an alter-ego of the
was entitled to the pension to which under the statute he could validly lay President bear the implied or assumed approval of the latter, unless actually disapproved by him, or
claim.chanroblesvirtuallawlibrary where there are circumstances indicating the urgency of judicial intervention." 16 The Gonzales doctrine,
it is to be noted, summarized the views announced in earlier cases. 17 The list of subsequent cases
After rejecting as untenable the defenses that there was no exhaustion of administrative remedies, that reiterating such a doctrine is quite impressive. 18 To be more specific, where there is a stipulation of
the action is in the nature of money claim which should first be presented before the Auditor General, facts, as in this case, the question before the lower court being solely one of law and on the face of the
and that said action is in reality a suit against the Government without the latters consent, the decision decision, the actuation of appellants being patently illegal, the doctrine of exhaustion of administrative
concludes with the following:" [Wherefore], judgment is hereby rendered in accordance with the prayer remedies certainly does not come into play.
of plaintiffs amended complaint, to wit, that defendants make plaintiffs pension effective June 21, 1955
at the rate of P50.00 a month up to June 21, 1957 at the rate of P100.00 a month, plus P10.00 a month 3. The other errors assigned, namely the alleged failure of the lower court to comply with the law in
each for his four unmarried minor children below 18 years old from June 22, 1957 up to September 1, fixing the amounts to which appellee is entitled instead of following the rules and regulations on
1964; and the difference of P70.00 a month, plus P10.00 for his one unmarried minor child below 18 veterans benefits promulgated by appellants and the alleged interference with the purely discretionary
years old from September 2, 1954, and thereafter, with costs against said dependents." 10 matter of a coordinate administrative agent, the Philippine Veterans Administration, can easily be
disposed of. It is to be admitted that appellants as chairman and members of the Philippine Veterans
Appellants elevated the matter to us. The careful and painstaking way in which the controlling statutory Administration, formerly the Philippine Veterans Board, are officials of an administrative body. 19 Nor
provisions were considered and applied by the then Judge Soriano must have impelled them to place may exception be taken to the general principle that as much as possible the courts should view with the
their faith in the alleged failure to respect the doctrines of non-suability and exhaustion of administrative utmost sympathy the exercise of power of administrative tribunals whether in its rule-making or
remedies to obtain a reversal. The appealed decision, however, as will now be shown is not subject to adjudicatory capacity. It has often been announced, and rightly so, that as much as possible the findings
such a reproach. The appeal then, as noted at the outset, is not to be attended with success. of these regulatory agencies which are expected to acquire expertise by their jurisdiction being confined
to specific matters, deserve to be accorded respect and finality. There is a limit, however, to such a
1. The fourth assignment of error assails what it considers to be the failing of the lower court in not deference paid to the actuations of such bodies. Clearly, where there has been a failure to interpret and
holding that the complaint in this case is in effect a suit against the State which has not given its consent apply the statutory provisions in question, judicial power should assert itself. Under the theory of
thereto. We have recently had occasion to reaffirm the force and primacy of the doctrine of non- separation of powers, it is to the judiciary and to the judiciary alone, that the final say on questions of
suability. 11 It does not admit of doubt, then, that if the suit were in fact against the State, the lower law appropriate cases coming before it is vested.chanrobles.com : virtual law library
court should have dismissed the complaint. Nor is it to be doubted that while ostensibly an action may
be against a public official, the defendant may in reality be the government. As a result, it is equally well- When the then Judge Soriano, therefore, as he was called upon to do, saw to it that there was strict
settled that where a litigation may have adverse consequences on the public treasury, whether in the compliance with the amounts of pension required by the law to be granted plaintiff and disregarded the
disbursements of funds or loss of property, the public official proceeded against not being liable in his regulation promulgated under the rule-making power of appellants, the effect of which would make
personal capacity, then the doctrine of non-suability may appropriately be invoked. It has no application, appellee suffer the consequences of an error committed by them, it cannot be truly said that his decision
may be assailed as being offensive to authoritative doctrines. On the contrary, it can stand the test of the
utmost scrutiny. Precisely because the commands of the law were duly carried out, it cannot be set
aside.

WHEREFORE, the decision of the then Judge Edilberto Soriano of the Court of First Instance of Manila
promulgated on January 22, 1966, is affirmed. Without pronouncement as to costs
Republic of the Philippines The marks revealed that he had one or more fractures of the skull and that the grey matter
SUPREME COURT and brain was had suffered material injury. At ten o'clock of the night in question, which was
Manila the time set for performing the operation, his pulse was so weak and so irregular that, in his
opinion, there was little hope that he would live. His right leg was broken in such a way that
the fracture extended to the outer skin in such manner that it might be regarded as double
EN BANC
and the would be exposed to infection, for which reason it was of the most serious nature.

G.R. No. L-11154 March 21, 1916


At another examination six days before the day of the trial, Dr. Saleeby noticed that the
plaintiff's leg showed a contraction of an inch and a half and a curvature that made his leg
E. MERRITT, plaintiff-appellant, very weak and painful at the point of the fracture. Examination of his head revealed a notable
vs. readjustment of the functions of the brain and nerves. The patient apparently was slightly
GOVERNMENT OF THE PHILIPPINE ISLANDS, defendant-appellant. deaf, had a light weakness in his eyes and in his mental condition. This latter weakness was
always noticed when the plaintiff had to do any difficult mental labor, especially when he
Crossfield and O'Brien for plaintiff. attempted to use his money for mathematical calculations.
Attorney-General Avancea for defendant..
According to the various merchants who testified as witnesses, the plaintiff's mental and
TRENT, J.: physical condition prior to the accident was excellent, and that after having received the
injuries that have been discussed, his physical condition had undergone a noticeable
depreciation, for he had lost the agility, energy, and ability that he had constantly displayed
This is an appeal by both parties from a judgment of the Court of First Instance of the city of Manila in before the accident as one of the best constructors of wooden buildings and he could not
favor of the plaintiff for the sum of P14,741, together with the costs of the cause. now earn even a half of the income that he had secured for his work because he had lost 50
per cent of his efficiency. As a contractor, he could no longer, as he had before done, climb up
Counsel for the plaintiff insist that the trial court erred (1) "in limiting the general damages which the ladders and scaffoldings to reach the highest parts of the building.
plaintiff suffered to P5,000, instead of P25,000 as claimed in the complaint," and (2) "in limiting the time
when plaintiff was entirely disabled to two months and twenty-one days and fixing the damage As a consequence of the loss the plaintiff suffered in the efficiency of his work as a contractor,
accordingly in the sum of P2,666, instead of P6,000 as claimed by plaintiff in his complaint." he had to dissolved the partnership he had formed with the engineer. Wilson, because he was
incapacitated from making mathematical calculations on account of the condition of his leg
The Attorney-General on behalf of the defendant urges that the trial court erred: (a) in finding that the and of his mental faculties, and he had to give up a contract he had for the construction of
collision between the plaintiff's motorcycle and the ambulance of the General Hospital was due to the the Uy Chaco building."
negligence of the chauffeur; (b) in holding that the Government of the Philippine Islands is liable for the
damages sustained by the plaintiff as a result of the collision, even if it be true that the collision was due We may say at the outset that we are in full accord with the trial court to the effect that the collision
to the negligence of the chauffeur; and (c) in rendering judgment against the defendant for the sum of between the plaintiff's motorcycle and the ambulance of the General Hospital was due solely to the
P14,741. negligence of the chauffeur.

The trial court's findings of fact, which are fully supported by the record, are as follows: The two items which constitute a part of the P14,741 and which are drawn in question by the plaintiff
are (a) P5,000, the award awarded for permanent injuries, and (b) the P2,666, the amount allowed for
It is a fact not disputed by counsel for the defendant that when the plaintiff, riding on a the loss of wages during the time the plaintiff was incapacitated from pursuing his occupation. We find
motorcycle, was going toward the western part of Calle Padre Faura, passing along the west nothing in the record which would justify us in increasing the amount of the first. As to the second, the
side thereof at a speed of ten to twelve miles an hour, upon crossing Taft Avenue and when record shows, and the trial court so found, that the plaintiff's services as a contractor were worth P1,000
he was ten feet from the southwestern intersection of said streets, the General Hospital per month. The court, however, limited the time to two months and twenty-one days, which the plaintiff
ambulance, upon reaching said avenue, instead of turning toward the south, after passing the was actually confined in the hospital. In this we think there was error, because it was clearly established
center thereof, so that it would be on the left side of said avenue, as is prescribed by the that the plaintiff was wholly incapacitated for a period of six months. The mere fact that he remained in
ordinance and the Motor Vehicle Act, turned suddenly and unexpectedly and long before the hospital only two months and twenty-one days while the remainder of the six months was spent in
reaching the center of the street, into the right side of Taft Avenue, without having sounded his home, would not prevent recovery for the whole time. We, therefore, find that the amount of
any whistle or horn, by which movement it struck the plaintiff, who was already six feet from damages sustained by the plaintiff, without any fault on his part, is P18,075.
the southwestern point or from the post place there.
As the negligence which caused the collision is a tort committed by an agent or employee of the
By reason of the resulting collision, the plaintiff was so severely injured that, according to Dr. Government, the inquiry at once arises whether the Government is legally-liable for the damages
Saleeby, who examined him on the very same day that he was taken to the General Hospital, resulting therefrom.
he was suffering from a depression in the left parietal region, a would in the same place and
in the back part of his head, while blood issued from his nose and he was entirely Act No. 2457, effective February 3, 1915, reads:
unconscious.
An Act authorizing E. Merritt to bring suit against the Government of the Philippine Islands The Government of the Philippine Islands having been "modeled after the Federal and State
and authorizing the Attorney-General of said Islands to appear in said suit. Governments in the United States," we may look to the decisions of the high courts of that country for
aid in determining the purpose and scope of Act No. 2457.
Whereas a claim has been filed against the Government of the Philippine Islands by Mr. E.
Merritt, of Manila, for damages resulting from a collision between his motorcycle and the In the United States the rule that the state is not liable for the torts committed by its officers or agents
ambulance of the General Hospital on March twenty-fifth, nineteen hundred and thirteen; whom it employs, except when expressly made so by legislative enactment, is well settled. "The
Government," says Justice Story, "does not undertake to guarantee to any person the fidelity of the
officers or agents whom it employs, since that would involve it in all its operations in endless
Whereas it is not known who is responsible for the accident nor is it possible to determine the
embarrassments, difficulties and losses, which would be subversive of the public interest." (Claussen vs.
amount of damages, if any, to which the claimant is entitled; and
City of Luverne, 103 Minn., 491, citing U. S. vs. Kirkpatrick, 9 Wheat, 720; 6 L. Ed., 199; and Beers vs.
States, 20 How., 527; 15 L. Ed., 991.)
Whereas the Director of Public Works and the Attorney-General recommended that an Act be
passed by the Legislature authorizing Mr. E. Merritt to bring suit in the courts against the
In the case of Melvin vs. State (121 Cal., 16), the plaintiff sought to recover damages from the state for
Government, in order that said questions may be decided: Now, therefore,
personal injuries received on account of the negligence of the state officers at the state fair, a state
institution created by the legislature for the purpose of improving agricultural and kindred industries; to
By authority of the United States, be it enacted by the Philippine Legislature, that: disseminate information calculated to educate and benefit the industrial classes; and to advance by such
means the material interests of the state, being objects similar to those sought by the public school
SECTION 1. E. Merritt is hereby authorized to bring suit in the Court of First Instance of the system. In passing upon the question of the state's liability for the negligent acts of its officers or agents,
city of Manila against the Government of the Philippine Islands in order to fix the the court said:
responsibility for the collision between his motorcycle and the ambulance of the General
Hospital, and to determine the amount of the damages, if any, to which Mr. E. Merritt is No claim arises against any government is favor of an individual, by reason of the
entitled on account of said collision, and the Attorney-General of the Philippine Islands is misfeasance, laches, or unauthorized exercise of powers by its officers or agents. (Citing
hereby authorized and directed to appear at the trial on the behalf of the Government of said Gibbons vs. U. S., 8 Wall., 269; Clodfelter vs. State, 86 N. C., 51, 53; 41 Am. Rep., 440;
Islands, to defendant said Government at the same. Chapman vs. State, 104 Cal., 690; 43 Am. St. Rep., 158; Green vs. State, 73 Cal., 29; Bourn vs.
Hart, 93 Cal., 321; 27 Am. St. Rep., 203; Story on Agency, sec. 319.)
SEC. 2. This Act shall take effect on its passage.
As to the scope of legislative enactments permitting individuals to sue the state where the cause of
Enacted, February 3, 1915. action arises out of either fort or contract, the rule is stated in 36 Cyc., 915, thus:

Did the defendant, in enacting the above quoted Act, simply waive its immunity from suit or did it also By consenting to be sued a state simply waives its immunity from suit. It does not thereby
concede its liability to the plaintiff? If only the former, then it cannot be held that the Act created any concede its liability to plaintiff, or create any cause of action in his favor, or extend its liability
new cause of action in favor of the plaintiff or extended the defendant's liability to any case not to any cause not previously recognized. It merely gives a remedy to enforce a preexisting
previously recognized. liability and submits itself to the jurisdiction of the court, subject to its right to interpose any
lawful defense.

All admit that the Insular Government (the defendant) cannot be sued by an individual without its
consent. It is also admitted that the instant case is one against the Government. As the consent of the In Apfelbacher vs. State (152 N. W., 144, advanced sheets), decided April 16, 1915, the Act of 1913,
Government to be sued by the plaintiff was entirely voluntary on its part, it is our duty to look carefully which authorized the bringing of this suit, read:
into the terms of the consent, and render judgment accordingly.
SECTION 1. Authority is hereby given to George Apfelbacher, of the town of Summit,
The plaintiff was authorized to bring this action against the Government "in order to fix the responsibility Waukesha County, Wisconsin, to bring suit in such court or courts and in such form or forms
for the collision between his motorcycle and the ambulance of the General Hospital and to determine as he may be advised for the purpose of settling and determining all controversies which he
the amount of the damages, if any, to which Mr. E. Merritt is entitled on account of said collision, . . . ." may now have with the State of Wisconsin, or its duly authorized officers and agents, relative
These were the two questions submitted to the court for determination. The Act was passed "in order to the mill property of said George Apfelbacher, the fish hatchery of the State of Wisconsin on
that said questions may be decided." We have "decided" that the accident was due solely to the the Bark River, and the mill property of Evan Humphrey at the lower end of Nagawicka Lake,
negligence of the chauffeur, who was at the time an employee of the defendant, and we have also fixed and relative to the use of the waters of said Bark River and Nagawicka Lake, all in the county
the amount of damages sustained by the plaintiff as a result of the collision. Does the Act authorize us to of Waukesha, Wisconsin.
hold that the Government is legally liable for that amount? If not, we must look elsewhere for such
authority, if it exists. In determining the scope of this act, the court said:

Plaintiff claims that by the enactment of this law the legislature admitted liability on the part
of the state for the acts of its officers, and that the suit now stands just as it would stand
between private parties. It is difficult to see how the act does, or was intended to do, more The state is liable in this sense when it acts through a special agent, but not when the damage
than remove the state's immunity from suit. It simply gives authority to commence suit for should have been caused by the official to whom properly it pertained to do the act
the purpose of settling plaintiff's controversies with the estate. Nowhere in the act is there a performed, in which case the provisions of the preceding article shall be applicable.
whisper or suggestion that the court or courts in the disposition of the suit shall depart from
well established principles of law, or that the amount of damages is the only question to be
The supreme court of Spain in defining the scope of this paragraph said:
settled. The act opened the door of the court to the plaintiff. It did not pass upon the
question of liability, but left the suit just where it would be in the absence of the state's
immunity from suit. If the Legislature had intended to change the rule that obtained in this That the obligation to indemnify for damages which a third person causes to another by his
state so long and to declare liability on the part of the state, it would not have left so fault or negligence is based, as is evidenced by the same Law 3, Title 15, Partida 7, on that the
important a matter to mere inference, but would have done so in express terms. (Murdock person obligated, by his own fault or negligence, takes part in the act or omission of the third
Grate Co. vs. Commonwealth, 152 Mass., 28; 24 N.E., 854; 8 L. R. A., 399.) party who caused the damage. It follows therefrom that the state, by virtue of such provisions
of law, is not responsible for the damages suffered by private individuals in consequence of
acts performed by its employees in the discharge of the functions pertaining to their office,
In Denning vs. State (123 Cal., 316), the provisions of the Act of 1893, relied upon and considered, are as
because neither fault nor even negligence can be presumed on the part of the state in the
follows:
organization of branches of public service and in the appointment of its agents; on the
contrary, we must presuppose all foresight humanly possible on its part in order that each
All persons who have, or shall hereafter have, claims on contract or for negligence against the branch of service serves the general weal an that of private persons interested in its
state not allowed by the state board of examiners, are hereby authorized, on the terms and operation. Between these latter and the state, therefore, no relations of a private nature
conditions herein contained, to bring suit thereon against the state in any of the courts of this governed by the civil law can arise except in a case where the state acts as a judicial person
state of competent jurisdiction, and prosecute the same to final judgment. The rules of capable of acquiring rights and contracting obligations. (Supreme Court of Spain, January 7,
practice in civil cases shall apply to such suits, except as herein otherwise provided. 1898; 83 Jur. Civ., 24.)

And the court said: That the Civil Code in chapter 2, title 16, book 4, regulates the obligations which arise out of
fault or negligence; and whereas in the first article thereof. No. 1902, where the general
principle is laid down that where a person who by an act or omission causes damage to
This statute has been considered by this court in at least two cases, arising under different
another through fault or negligence, shall be obliged to repair the damage so done, reference
facts, and in both it was held that said statute did not create any liability or cause of action
is made to acts or omissions of the persons who directly or indirectly cause the damage, the
against the state where none existed before, but merely gave an additional remedy to
following articles refers to this persons and imposes an identical obligation upon those who
enforce such liability as would have existed if the statute had not been enacted. (Chapman vs.
maintain fixed relations of authority and superiority over the authors of the damage, because
State, 104 Cal., 690; 43 Am. St. Rep., 158; Melvin vs. State, 121 Cal., 16.)
the law presumes that in consequence of such relations the evil caused by their own fault or
negligence is imputable to them. This legal presumption gives way to proof, however,
A statute of Massachusetts enacted in 1887 gave to the superior court "jurisdiction of all claims against because, as held in the last paragraph of article 1903, responsibility for acts of third persons
the commonwealth, whether at law or in equity," with an exception not necessary to be here ceases when the persons mentioned in said article prove that they employed all the diligence
mentioned. In construing this statute the court, in Murdock Grate Co. vs. Commonwealth (152 Mass., of a good father of a family to avoid the damage, and among these persons, called upon to
28), said: answer in a direct and not a subsidiary manner, are found, in addition to the mother or the
father in a proper case, guardians and owners or directors of an establishment or enterprise,
The statute we are discussing disclose no intention to create against the state a new and the state, but not always, except when it acts through the agency of a special agent,
heretofore unrecognized class of liabilities, but only an intention to provide a judicial tribunal doubtless because and only in this case, the fault or negligence, which is the original basis of
where well recognized existing liabilities can be adjudicated. this kind of objections, must be presumed to lie with the state.

In Sipple vs. State (99 N. Y., 284), where the board of the canal claims had, by the terms of the statute of That although in some cases the state might by virtue of the general principle set forth in
New York, jurisdiction of claims for damages for injuries in the management of the canals such as the article 1902 respond for all the damage that is occasioned to private parties by orders or
plaintiff had sustained, Chief Justice Ruger remarks: "It must be conceded that the state can be made resolutions which by fault or negligence are made by branches of the central administration
liable for injuries arising from the negligence of its agents or servants, only by force of some positive acting in the name and representation of the state itself and as an external expression of its
statute assuming such liability." sovereignty in the exercise of its executive powers, yet said article is not applicable in the case
of damages said to have been occasioned to the petitioners by an executive official, acting in
the exercise of his powers, in proceedings to enforce the collections of certain property taxes
It being quite clear that Act No. 2457 does not operate to extend the Government's liability to any cause owing by the owner of the property which they hold in sublease.
not previously recognized, we will now examine the substantive law touching the defendant's liability for
the negligent acts of its officers, agents, and employees. Paragraph 5 of article 1903 of the Civil Code
reads: That the responsibility of the state is limited by article 1903 to the case wherein it
acts through a special agent (and a special agent, in the sense in which these words are
employed, is one who receives a definite and fixed order or commission, foreign to the
exercise of the duties of his office if he is a special official) so that in representation of the
state and being bound to act as an agent thereof, he executes the trust confided to him. This
concept does not apply to any executive agent who is an employee of the acting
administration and who on his own responsibility performs the functions which are inherent
in and naturally pertain to his office and which are regulated by law and the regulations."
(Supreme Court of Spain, May 18, 1904; 98 Jur. Civ., 389, 390.)

That according to paragraph 5 of article 1903 of the Civil Code and the principle laid down in a
decision, among others, of the 18th of May, 1904, in a damage case, the responsibility of the
state is limited to that which it contracts through a special agent, duly empowered by
a definite order or commission to perform some act or charged with some definite purpose
which gives rise to the claim, and not where the claim is based on acts or omissions imputable
to a public official charged with some administrative or technical office who can be held to
the proper responsibility in the manner laid down by the law of civil responsibility.
Consequently, the trial court in not so deciding and in sentencing the said entity to the
payment of damages, caused by an official of the second class referred to, has by erroneous
interpretation infringed the provisions of articles 1902 and 1903 of the Civil Code. (Supreme
Court of Spain, July 30, 1911; 122 Jur. Civ., 146.)

It is, therefore, evidence that the State (the Government of the Philippine Islands) is only liable,
according to the above quoted decisions of the Supreme Court of Spain, for the acts of its agents, officers
and employees when they act as special agents within the meaning of paragraph 5 of article 1903, supra,
and that the chauffeur of the ambulance of the General Hospital was not such an agent.

For the foregoing reasons, the judgment appealed from must be reversed, without costs in this instance.
Whether the Government intends to make itself legally liable for the amount of damages above set
forth, which the plaintiff has sustained by reason of the negligent acts of one of its employees, by
legislative enactment and by appropriating sufficient funds therefor, we are not called upon to
determine. This matter rests solely with the Legislature and not with the courts.
Republic of the Philippines The principle that the state or its government cannot be sued without its consent has its root in the
SUPREME COURT juridical and practical notion that the state can do no wrong. Demandable and enforceable obligations
Manila which may be the subject of judicial action come into being either by law, contract, quasi-contract, acts
or omissions punishable by law, acts which do not constitute or amount to a crime or a misdemeanor
known at common law as torts and in civil law as culpa aquiliana or extra contractual. An obligation or
EN BANC
liability of the state created by statute is enforceable against the officer or agent charged with the duty
to execute the law. If there should be anything demandable which had been paid or delivered to or
G.R. No. L-4699 November 26, 1952 collected by officers or agents of the state without the authority of law, the action would not be against
the state but against the responsible officers or agents who received what was not due the state or
TEODORA SANTOS, assisted by her husband DONATO DE CASTRO, JOSEFINA SANTOS, assisted by her made the unauthorized collection. Punishable acts or omissions committed by officers or agents of the
husband Santiago Rodriguez and EMILIANA SANTOS, plaintiffs-appellants, state are crimes and violations of law perpetuated by such officers or agents and not by the state. The
vs. same postulate may be applied to torts committed by officers or agents of the state. Nevertheless,
LEONCIO SANTOS, THE ADMINISTRATOR OF THE CIVIL AERONAUTICS ADMINISTRATION, and if, where and when the state or its government enters into a contract, through its officers or agents, in
NATIONAL AIRPORTS CORPORATION, defendants-appellees. furtherance of a legitimate aim and purpose and pursuant to constitutional legislative authority,
whereby mutual or reciprocal benefits accrue and rights and obligations arise therefrom, and if the law
granting the authority to enter into such contract does not provide for or name the officer against whom
Ramon Diokno and Jose W. Diokno for appellants. action may be brought in the event of a breach thereof, the state itself may be sued even without its
Office of the Solicitor General Pompeyo Diaz and Solicitor Esmeraldo Umali for appellees. consent, because by entering into a contract the sovereign state has descended to the level of the citizen
and its consent to be used is implied from the very act of entering into such contract. If the dignity of the
PADILLA, J.: state, the sacredness of the institution, the respect for the government are to be preserved and the
dragging of its name in a suit to be prevented, the legislative department should name the officer or
agent against whom the action may be brought in the event of breach of the contract entered into under
Teodora Santos and her nieces Emiliana and Josefina surnamed Santos complain that from 1945 to 1949 its name and authority. And the omission or failure of the legislative department to do so is no obstacle
Leoncio Santos collected from the Army of the United States of America rentals for the use and or impediment for an individual or citizen, who is aggrieved by the breach of the contract, to bring an
occupation of a parcel of land, known as Lot No. 4 of CAA Survey Plan AERO R-1, containing an area of action against the state itself for the reasons already adverted to, to wit; the descent of the sovereign
21,577 square meters, situated in the Municipality of Las Pias, Province of Rizal, more particularly state to the level of the individual or citizen with whom it entered into a contract and its consent to be
described in the complaint, belonging to them and Leoncio Santos in common by inheritance from their sued implied from the act of entering into such contract.
ancestor, the late Paulino de los Santos, father of Teodora Santos and Leoncio Santos and grandfather of
Josefina Santos and Emiliana Santos, who died sometime in 1919, in the proportion of 1/7 undivided
share for Teodora Santos and 1/14 undivided share each for Josefina Santos and Emiliana Santos and 5/7 The action brought in this case is for partition and accounting of rental received by the defendant
undivided share for Leoncio Santos, for the accounting of which and payment of their respective shares Leoncio Santos from 1945 to December 1949 for the use and occupation of a parcel of land allegedly
therein they made a demand upon Leoncio Santos but the latter failed and refused to do so. They also owned in common by the plaintiffs and the defendant Leoncio Santos in the proportion stated in the
complain that they made a demand upon Leoncio Santos to have the lot partitioned among them but the complaint. It is also averred that the National Airports Corporation created by Republic Act No. 224,
later refused to do so, he having sold the lot to the Administrator of the Civil Aeronautics Administration which had acquired the parcel of land from the defendant Leoncio Santos, was abolished by Executive
on or about 13 May 1949, who is now in possession thereof, and that the sale of the lot made by Leoncio Order no. 365, series of 1950, and in its place and stead the Civil Aeronautics Administration was created
Santos to the Administrator of the Civil Aeronautics Administration insofar as their shares in the lot are and took over all the assets and assumed all the liabilities of the abolished corporation. The Civil
concerned is null and void. Upon these allegations they pray that Leoncio Santos be ordered to render an Aeronautics Administration, even if it is not a juridical entity, cannot legally prevent a party or parties
accounting of the rentals and such other fruits, products and benefits as he might have received from from enforcing their propriety rights under the cloak or shield of lack of juridical personality, because it
1945 on and thereafter and to pay and deliver 1/7 thereof to Teodora Santos and 1/14 thereof each to took over all the powers and assumed all the obligations of the defunct corporation which had entered
Josefina and Emiliana surname Santos; that the parcel of land be partitioned among them in the into the contract in question. In National Airports Corporation vs. Teodoro *, G.R. No. L-5122, 30 April
proportion above-stated; that the purported sale by Leoncio Santos to the National Airports Corporation, 1952, we held that the Civil Aeronautics Administration may be sued and that the principle of state
the predecessor to the Civil Aeronautics Administration, insofar as theirs shares are concerned be immunity from suit does not apply to it.
declared null and void; that the Administrator of the Civil Aeronautics Administration be directed to
vacate the portions of the lot belonging to them a reasonable rental until after possession of their shares If the plaintiffs are not entitled to any share in the parcel of land sold by Leoncio Santos and acquired by
in the lot shall have been restored to them and to pay damages and cost. the National Airports Corporation, now in the possession of its successor, the Civil Aeronautics
Administration, the complaint would have to be dismissed. But if the right to such shares as claimed be
The Administrator of the Civil Aeronautics Administration moved to dismiss the complaint for lack of established, the plaintiffs should not and can be deprived of their proprietary rights in the parcel of land
jurisdiction and insufficiency of the complaint against him, invoking the case of Metropolitan sold by their co-owner without their knowledge and consent. Leoncio Santos would be responsible for
Transportation Service METRAN vs. Paredes, 45 Off. Gaz., 2835, where it has been held that the suit was warranty and eviction to the Civil Aeronautics Administration. If the Torrens title does not show such
against the state which could not be brought without its consent. This motion was granted on the shares of the plaintiffs in the parcel of land sold by Leoncio Santos to the National Airports Corporation,
ground that the Civil Aeronautics Administration not being a juridical person has no capacity to sue and then the action would not lie against the National Airports Corporation or its successor, the Civil
be sued and for that reason it cannot come under the jurisdiction of the court. Aeronautics Administration of land and of their natural or civil fruits of which they had been deprived by
the sale and conveyance of the whole parcel of land to the National Airports Corporation by Leoncio
Santos. The accounting of rentals received would not affect the Civil Aeronautics Administration,
because it would be the exclusive liability of Leoncio Santos.
The order appealed from dismissing the complaint as to the Civil Aeronautics Administration is reversed
and the case remanded to the lower court for further proceedings in accordance with law. No cost shall
be taxed.
Republic of the Philippines States, who is one of the petitioners herein. The letter said that the company did not qualify to receive
SUPREME COURT an award for the projects because of its previous unsatisfactory performance rating on a repair contract
Manila for the sea wall at the boat landings of the U.S. Naval Station in Subic Bay. The letter further said that the
projects had been awarded to third parties. In the abovementioned Civil Case No. 779-M, the company
sued the United States of America and Messrs. James E. Galloway, William I. Collins and Robert Gohier all
EN BANC
members of the Engineering Command of the U.S. Navy. The complaint is to order the defendants to
allow the plaintiff to perform the work on the projects and, in the event that specific performance was
G.R. No. L-35645 May 22, 1985 no longer possible, to order the defendants to pay damages. The company also asked for the issuance of
a writ of preliminary injunction to restrain the defendants from entering into contracts with third parties
UNITED STATES OF AMERICA, CAPT. JAMES E. GALLOWAY, WILLIAM I. COLLINS and ROBERT for work on the projects.
GOHIER,petitioners,
vs. The defendants entered their special appearance for the purpose only of questioning the jurisdiction of
HON. V. M. RUIZ, Presiding Judge of Branch XV, Court of First Instance of Rizal and ELIGIO DE GUZMAN this court over the subject matter of the complaint and the persons of defendants, the subject matter of
& CO., INC., respondents. the complaint being acts and omissions of the individual defendants as agents of defendant United
States of America, a foreign sovereign which has not given her consent to this suit or any other suit for
Sycip, Salazar, Luna & Manalo & Feliciano Law for petitioners. the causes of action asserted in the complaint." (Rollo, p. 50.)

Albert, Vergara, Benares, Perias & Dominguez Law Office for respondents. Subsequently the defendants filed a motion to dismiss the complaint which included an opposition to
the issuance of the writ of preliminary injunction. The company opposed the motion. The trial court
denied the motion and issued the writ. The defendants moved twice to reconsider but to no avail. Hence
the instant petition which seeks to restrain perpetually the proceedings in Civil Case No. 779-M for lack
of jurisdiction on the part of the trial court.
ABAD SANTOS, J.:
The petition is highly impressed with merit.
This is a petition to review, set aside certain orders and restrain the respondent judge from trying Civil
Case No. 779M of the defunct Court of First Instance of Rizal. The traditional rule of State immunity exempts a State from being sued in the courts of another State
without its consent or waiver. This rule is a necessary consequence of the principles of independence
The factual background is as follows: and equality of States. However, the rules of International Law are not petrified; they are constantly
developing and evolving. And because the activities of states have multiplied, it has been necessary to
distinguish them-between sovereign and governmental acts (jure imperii) and private, commercial and
At times material to this case, the United States of America had a naval base in Subic, Zambales. The proprietary acts (jure gestionis). The result is that State immunity now extends only to acts jure imperil
base was one of those provided in the Military Bases Agreement between the Philippines and the United The restrictive application of State immunity is now the rule in the United States, the United Kingdom
States. and other states in western Europe. (See Coquia and Defensor Santiago, Public International Law, pp.
207-209 [1984].)
Sometime in May, 1972, the United States invited the submission of bids for the following projects
The respondent judge recognized the restrictive doctrine of State immunity when he said in his Order
1. Repair offender system, Alava Wharf at the U.S. Naval Station Subic Bay, Philippines. denying the defendants' (now petitioners) motion: " A distinction should be made between a strictly
governmental function of the sovereign state from its private, proprietary or non- governmental acts
(Rollo, p. 20.) However, the respondent judge also said: "It is the Court's considered opinion that
2. Repair typhoon damage to NAS Cubi shoreline; repair typhoon damage to shoreline revetment,
entering into a contract for the repair of wharves or shoreline is certainly not a governmental function
NAVBASE Subic; and repair to Leyte Wharf approach, NAVBASE Subic Bay, Philippines.
altho it may partake of a public nature or character. As aptly pointed out by plaintiff's counsel in his reply
citing the ruling in the case of Lyons, Inc., [104 Phil. 594 (1958)], and which this Court quotes with
Eligio de Guzman & Co., Inc. responded to the invitation and submitted bids. Subsequent thereto, the approval, viz.:
company received from the United States two telegrams requesting it to confirm its price proposals and
for the name of its bonding company. The company complied with the requests. [In its complaint, the
It is however contended that when a sovereign state enters into a contract with a
company alleges that the United States had accepted its bids because "A request to confirm a price
private person, the state can be sued upon the theory that it has descended to the
proposal confirms the acceptance of a bid pursuant to defendant United States' bidding practices."
level of an individual from which it can be implied that it has given its consent to
(Rollo, p. 30.) The truth of this allegation has not been tested because the case has not reached the trial
be sued under the contract. ...
stage.]

xxx xxx xxx


In June, 1972, the company received a letter which was signed by Wilham I. Collins, Director, Contracts
Division, Naval Facilities Engineering Command, Southwest Pacific, Department of the Navy of the United
We agree to the above contention, and considering that the United States America; that any judgment for back or Increased rentals or damages will have to
government, through its agency at Subic Bay, entered into a contract with be paid not by defendants Moore and Tillman and their 64 co-defendants but by
appellant for stevedoring and miscellaneous labor services within the Subic Bay the said U.S. Government. On the basis of the ruling in the case of Land vs. Dollar
Area, a U.S. Naval Reservation, it is evident that it can bring an action before our already cited, and on what we have already stated, the present action must be
courts for any contractual liability that that political entity may assume under the considered as one against the U.S. Government. It is clear hat the courts of the
contract. The trial court, therefore, has jurisdiction to entertain this case ... (Rollo, Philippines including the Municipal Court of Manila have no jurisdiction over the
pp. 20-21.) present case for unlawful detainer. The question of lack of jurisdiction was raised
and interposed at the very beginning of the action. The U.S. Government has not ,
given its consent to the filing of this suit which is essentially against her, though
The reliance placed on Lyons by the respondent judge is misplaced for the following reasons:
not in name. Moreover, this is not only a case of a citizen filing a suit against his
own Government without the latter's consent but it is of a citizen filing an action
In Harry Lyons, Inc. vs. The United States of America, supra, plaintiff brought suit in the Court of First against a foreign government without said government's consent, which renders
Instance of Manila to collect several sums of money on account of a contract between plaintiff and more obvious the lack of jurisdiction of the courts of his country. The principles of
defendant. The defendant filed a motion to dismiss on the ground that the court had no jurisdiction over law behind this rule are so elementary and of such general acceptance that we
defendant and over the subject matter of the action. The court granted the motion on the grounds that: deem it unnecessary to cite authorities in support thereof. (At p. 323.)
(a) it had no jurisdiction over the defendant who did not give its consent to the suit; and (b) plaintiff
failed to exhaust the administrative remedies provided in the contract. The order of dismissal was
In Syquia,the United States concluded contracts with private individuals but the contracts
elevated to this Court for review.
notwithstanding the States was not deemed to have given or waived its consent to be sued for the
reason that the contracts were forjure imperii and not for jure gestionis.
In sustaining the action of the lower court, this Court said:
WHEREFORE, the petition is granted; the questioned orders of the respondent judge are set aside and
It appearing in the complaint that appellant has not complied with the procedure Civil Case No. is dismissed. Costs against the private respondent.
laid down in Article XXI of the contract regarding the prosecution of its claim
against the United States Government, or, stated differently, it has failed to first
exhaust its administrative remedies against said Government, the lower court
acted properly in dismissing this case.(At p. 598.)

It can thus be seen that the statement in respect of the waiver of State immunity from suit was purely
gratuitous and, therefore, obiter so that it has no value as an imperative authority.

The restrictive application of State immunity is proper only when the proceedings arise out of
commercial transactions of the foreign sovereign, its commercial activities or economic affairs. Stated
differently, a State may be said to have descended to the level of an individual and can thus be deemed
to have tacitly given its consent to be sued only when it enters into business contracts. It does not apply
where the contract relates to the exercise of its sovereign functions. In this case the projects are an
integral part of the naval base which is devoted to the defense of both the United States and the
Philippines, indisputably a function of the government of the highest order; they are not utilized for nor
dedicated to commercial or business purposes.

That the correct test for the application of State immunity is not the conclusion of a contract by a State
but the legal nature of the act is shown in Syquia vs. Lopez, 84 Phil. 312 (1949). In that case the plaintiffs
leased three apartment buildings to the United States of America for the use of its military officials. The
plaintiffs sued to recover possession of the premises on the ground that the term of the leases had
expired. They also asked for increased rentals until the apartments shall have been vacated.

The defendants who were armed forces officers of the United States moved to dismiss the suit for lack of
jurisdiction in the part of the court. The Municipal Court of Manila granted the motion to dismiss;
sustained by the Court of First Instance, the plaintiffs went to this Court for review on certiorari. In
denying the petition, this Court said:

On the basis of the foregoing considerations we are of the belief and we hold that
the real party defendant in interest is the Government of the United States of
Republic of the Philippines
SUPREME COURT
Manila

EN BANC

G.R. No. 101949 December 1, 1994

THE HOLY SEE, petitioner,


vs.
THE HON. ERIBERTO U. ROSARIO, JR., as Presiding Judge of the Regional Trial Court of Makati, Branch
61 and STARBRIGHT SALES ENTERPRISES, INC., respondents.

Padilla Law Office for petitioner.

Siguion Reyna, Montecillo & Ongsiako for private respondent.

QUIASON, J.:

This is a petition for certiorari under Rule 65 of the Revised Rules of Court to reverse and set aside the
Orders dated June 20, 1991 and September 19, 1991 of the Regional Trial Court, Branch 61, Makati,
Metro Manila in Civil Case No. 90-183.

The Order dated June 20, 1991 denied the motion of petitioner to dismiss the complaint in Civil Case No.
90-183, while the Order dated September 19, 1991 denied the motion for reconsideration of the June
20,1991 Order.

Petitioner is the Holy See who exercises sovereignty over the Vatican City in Rome, Italy, and is
represented in the Philippines by the Papal Nuncio.

Private respondent, Starbright Sales Enterprises, Inc., is a domestic corporation engaged in the real
estate business.

This petition arose from a controversy over a parcel of land consisting of 6,000 square meters (Lot 5-A,
Transfer Certificate of Title No. 390440) located in the Municipality of Paraaque, Metro Manila and
registered in the name of petitioner.

Said Lot 5-A is contiguous to Lots 5-B and 5-D which are covered by Transfer Certificates of Title Nos.
271108 and 265388 respectively and registered in the name of the Philippine Realty Corporation (PRC).

The three lots were sold to Ramon Licup, through Msgr. Domingo A. Cirilos, Jr., acting as agent to the
sellers. Later, Licup assigned his rights to the sale to private respondent.
In view of the refusal of the squatters to vacate the lots sold to private respondent, a dispute arose as to petitioner prayed that a hearing be conducted to allow it to establish certain facts upon which the said
who of the parties has the responsibility of evicting and clearing the land of squatters. Complicating the defense is based. Private respondent opposed this motion as well as the motion for reconsideration.
relations of the parties was the sale by petitioner of Lot 5-A to Tropicana Properties and Development
Corporation (Tropicana).
On October 1, 1991, the trial court issued an order deferring the resolution on the motion for
reconsideration until after trial on the merits and directing petitioner to file its answer (Rollo, p. 22).
I
Petitioner forthwith elevated the matter to us. In its petition, petitioner invokes the privilege of
On January 23, 1990, private respondent filed a complaint with the Regional Trial Court, Branch 61, sovereign immunity only on its own behalf and on behalf of its official representative, the Papal Nuncio.
Makati, Metro Manila for annulment of the sale of the three parcels of land, and specific performance
and damages against petitioner, represented by the Papal Nuncio, and three other defendants: namely,
On December 9, 1991, a Motion for Intervention was filed before us by the Department of Foreign
Msgr. Domingo A. Cirilos, Jr., the PRC and Tropicana (Civil Case No.
Affairs, claiming that it has a legal interest in the outcome of the case as regards the diplomatic immunity
90-183).
of petitioner, and that it "adopts by reference, the allegations contained in the petition of the Holy See
insofar as they refer to arguments relative to its claim of sovereign immunity from suit" (Rollo, p. 87).
The complaint alleged that: (1) on April 17, 1988, Msgr. Cirilos, Jr., on behalf of petitioner and the PRC,
agreed to sell to Ramon Licup Lots 5-A, 5-B and 5-D at the price of P1,240.00 per square meters; (2) the
Private respondent opposed the intervention of the Department of Foreign Affairs. In compliance with
agreement to sell was made on the condition that earnest money of P100,000.00 be paid by Licup to the
the resolution of this Court, both parties and the Department of Foreign Affairs submitted their
sellers, and that the sellers clear the said lots of squatters who were then occupying the same; (3) Licup
respective memoranda.
paid the earnest money to Msgr. Cirilos; (4) in the same month, Licup assigned his rights over the
property to private respondent and informed the sellers of the said assignment; (5) thereafter, private
respondent demanded from Msgr. Cirilos that the sellers fulfill their undertaking and clear the property II
of squatters; however, Msgr. Cirilos informed private respondent of the squatters' refusal to vacate the
lots, proposing instead either that private respondent undertake the eviction or that the earnest money A preliminary matter to be threshed out is the procedural issue of whether the petition
be returned to the latter; (6) private respondent counterproposed that if it would undertake the eviction for certiorari under Rule 65 of the Revised Rules of Court can be availed of to question the order denying
of the squatters, the purchase price of the lots should be reduced from P1,240.00 to P1,150.00 per petitioner's motion to dismiss. The general rule is that an order denying a motion to dismiss is not
square meter; (7) Msgr. Cirilos returned the earnest money of P100,000.00 and wrote private reviewable by the appellate courts, the remedy of the movant being to file his answer and to proceed
respondent giving it seven days from receipt of the letter to pay the original purchase price in cash; (8) with the hearing before the trial court. But the general rule admits of exceptions, and one of these is
private respondent sent the earnest money back to the sellers, but later discovered that on March 30, when it is very clear in the records that the trial court has no alternative but to dismiss the complaint
1989, petitioner and the PRC, without notice to private respondent, sold the lots to Tropicana, as (Philippine National Bank v. Florendo, 206 SCRA 582 [1992]; Zagada v. Civil Service Commission, 216
evidenced by two separate Deeds of Sale, one over Lot 5-A, and another over Lots 5-B and 5-D; and that SCRA 114 [1992]. In such a case, it would be a sheer waste of time and energy to require the parties to
the sellers' transfer certificate of title over the lots were cancelled, transferred and registered in the undergo the rigors of a trial.
name of Tropicana; (9) Tropicana induced petitioner and the PRC to sell the lots to it and thus enriched
itself at the expense of private respondent; (10) private respondent demanded the rescission of the sale
to Tropicana and the reconveyance of the lots, to no avail; and (11) private respondent is willing and able The other procedural question raised by private respondent is the personality or legal interest of the
to comply with the terms of the contract to sell and has actually made plans to develop the lots into a Department of Foreign Affairs to intervene in the case in behalf of the Holy See (Rollo, pp. 186-190).
townhouse project, but in view of the sellers' breach, it lost profits of not less than P30,000.000.00.
In Public International Law, when a state or international agency wishes to plead sovereign or diplomatic
Private respondent thus prayed for: (1) the annulment of the Deeds of Sale between petitioner and the immunity in a foreign court, it requests the Foreign Office of the state where it is sued to convey to the
PRC on the one hand, and Tropicana on the other; (2) the reconveyance of the lots in question; (3) court that said defendant is entitled to immunity.
specific performance of the agreement to sell between it and the owners of the lots; and (4) damages.
In the United States, the procedure followed is the process of "suggestion," where the foreign state or
On June 8, 1990, petitioner and Msgr. Cirilos separately moved to dismiss the complaint petitioner for the international organization sued in an American court requests the Secretary of State to make a
lack of jurisdiction based on sovereign immunity from suit, and Msgr. Cirilos for being an improper party. determination as to whether it is entitled to immunity. If the Secretary of State finds that the defendant
An opposition to the motion was filed by private respondent. is immune from suit, he, in turn, asks the Attorney General to submit to the court a "suggestion" that the
defendant is entitled to immunity. In England, a similar procedure is followed, only the Foreign Office
issues a certification to that effect instead of submitting a "suggestion" (O'Connell, I International Law
On June 20, 1991, the trial court issued an order denying, among others, petitioner's motion to dismiss 130 [1965]; Note: Immunity from Suit of Foreign Sovereign Instrumentalities and Obligations, 50 Yale
after finding that petitioner "shed off [its] sovereign immunity by entering into the business contract in Law Journal 1088 [1941]).
question" (Rollo, pp. 20-21).

In the Philippines, the practice is for the foreign government or the international organization to first
On July 12, 1991, petitioner moved for reconsideration of the order. On August 30, 1991, petitioner filed secure an executive endorsement of its claim of sovereign or diplomatic immunity. But how the
a "Motion for a Hearing for the Sole Purpose of Establishing Factual Allegation for claim of Immunity as a Philippine Foreign Office conveys its endorsement to the courts varies. In International Catholic
Jurisdictional Defense." So as to facilitate the determination of its defense of sovereign immunity, Migration Commission v. Calleja, 190 SCRA 130 (1990), the Secretary of Foreign Affairs just sent a letter
directly to the Secretary of Labor and Employment, informing the latter that the respondent-employer The Vatican City fits into none of the established categories of states, and the attribution to it of
could not be sued because it enjoyed diplomatic immunity. InWorld Health Organization v. Aquino, 48 "sovereignty" must be made in a sense different from that in which it is applied to other states (Fenwick,
SCRA 242 (1972), the Secretary of Foreign Affairs sent the trial court a telegram to that effect. In Baer v. International Law 124-125 [1948]; Cruz, International Law 37 [1991]). In a community of national states,
Tizon, 57 SCRA 1 (1974), the U.S. Embassy asked the Secretary of Foreign Affairs to request the Solicitor the Vatican City represents an entity organized not for political but for ecclesiastical purposes and
General to make, in behalf of the Commander of the United States Naval Base at Olongapo City, international objects. Despite its size and object, the Vatican City has an independent government of its
Zambales, a "suggestion" to respondent Judge. The Solicitor General embodied the "suggestion" in a own, with the Pope, who is also head of the Roman Catholic Church, as the Holy See or Head of State, in
Manifestation and Memorandum as amicus curiae. conformity with its traditions, and the demands of its mission in the world. Indeed, the world-wide
interests and activities of the Vatican City are such as to make it in a sense an "international state"
(Fenwick, supra., 125; Kelsen, Principles of International Law 160 [1956]).
In the case at bench, the Department of Foreign Affairs, through the Office of Legal Affairs moved with
this Court to be allowed to intervene on the side of petitioner. The Court allowed the said Department to
file its memorandum in support of petitioner's claim of sovereign immunity. One authority wrote that the recognition of the Vatican City as a state has significant implication that
it is possible for any entity pursuing objects essentially different from those pursued by states to be
invested with international personality (Kunz, The Status of the Holy See in International Law, 46 The
In some cases, the defense of sovereign immunity was submitted directly to the local courts by the
American Journal of International Law 308 [1952]).
respondents through their private counsels (Raquiza v. Bradford, 75 Phil. 50 [1945]; Miquiabas v.
Philippine-Ryukyus Command, 80 Phil. 262 [1948]; United States of America v. Guinto, 182 SCRA 644
[1990] and companion cases). In cases where the foreign states bypass the Foreign Office, the courts can Inasmuch as the Pope prefers to conduct foreign relations and enter into transactions as the Holy See
inquire into the facts and make their own determination as to the nature of the acts and transactions and not in the name of the Vatican City, one can conclude that in the Pope's own view, it is the Holy See
involved. that is the international person.

III The Republic of the Philippines has accorded the Holy See the status of a foreign sovereign. The Holy
See, through its Ambassador, the Papal Nuncio, has had diplomatic representations with the Philippine
government since 1957 (Rollo, p. 87). This appears to be the universal practice in international relations.
The burden of the petition is that respondent trial court has no jurisdiction over petitioner, being a
foreign state enjoying sovereign immunity. On the other hand, private respondent insists that the
doctrine of non-suability is not anymore absolute and that petitioner has divested itself of such a cloak B. Sovereign Immunity
when, of its own free will, it entered into a commercial transaction for the sale of a parcel of land located
in the Philippines.
As expressed in Section 2 of Article II of the 1987 Constitution, we have adopted the generally accepted
principles of International Law. Even without this affirmation, such principles of International Law are
A. The Holy See deemed incorporated as part of the law of the land as a condition and consequence of our admission in
the society of nations (United States of America v. Guinto, 182 SCRA 644 [1990]).
Before we determine the issue of petitioner's non-suability, a brief look into its status as a sovereign
state is in order. There are two conflicting concepts of sovereign immunity, each widely held and firmly established.
According to the classical or absolute theory, a sovereign cannot, without its consent, be made a
respondent in the courts of another sovereign. According to the newer or restrictive theory, the
Before the annexation of the Papal States by Italy in 1870, the Pope was the monarch and he, as the Holy
immunity of the sovereign is recognized only with regard to public acts or acts jure imperii of a state, but
See, was considered a subject of International Law. With the loss of the Papal States and the limitation of
not with regard to private acts or acts jure gestionis
the territory under the Holy See to an area of 108.7 acres, the position of the Holy See in International
(United States of America v. Ruiz, 136 SCRA 487 [1987]; Coquia and Defensor-Santiago, Public
Law became controversial (Salonga and Yap, Public International Law 36-37 [1992]).
International Law 194 [1984]).

In 1929, Italy and the Holy See entered into the Lateran Treaty, where Italy recognized the exclusive
Some states passed legislation to serve as guidelines for the executive or judicial determination when an
dominion and sovereign jurisdiction of the Holy See over the Vatican City. It also recognized the right of
act may be considered as jure gestionis. The United States passed the Foreign Sovereign Immunities Act
the Holy See to receive foreign diplomats, to send its own diplomats to foreign countries, and to enter
of 1976, which defines a commercial activity as "either a regular course of commercial conduct or a
into treaties according to International Law (Garcia, Questions and Problems In International Law, Public
particular commercial transaction or act." Furthermore, the law declared that the "commercial character
and Private 81 [1948]).
of the activity shall be determined by reference to the nature of the course of conduct or particular
transaction or act, rather than by reference to its purpose." The Canadian Parliament enacted in 1982 an
The Lateran Treaty established the statehood of the Vatican City "for the purpose of assuring to the Holy Act to Provide For State Immunity in Canadian Courts. The Act defines a "commercial activity" as any
See absolute and visible independence and of guaranteeing to it indisputable sovereignty also in the field particular transaction, act or conduct or any regular course of conduct that by reason of its nature, is of a
of international relations" (O'Connell, I International Law 311 [1965]). "commercial character."

In view of the wordings of the Lateran Treaty, it is difficult to determine whether the statehood is vested The restrictive theory, which is intended to be a solution to the host of problems involving the issue of
in the Holy See or in the Vatican City. Some writers even suggested that the treaty created two sovereign immunity, has created problems of its own. Legal treatises and the decisions in countries
international persons the Holy See and Vatican City (Salonga and Yap, supra, 37).
which follow the restrictive theory have difficulty in characterizing whether a contract of a sovereign residence of the Papal Nuncio. The right of a foreign sovereign to acquire property, real or personal, in a
state with a private party is an act jure gestionis or an act jure imperii. receiving state, necessary for the creation and maintenance of its diplomatic mission, is recognized in the
1961 Vienna Convention on Diplomatic Relations (Arts. 20-22). This treaty was concurred in by the
Philippine Senate and entered into force in the Philippines on November 15, 1965.
The restrictive theory came about because of the entry of sovereign states into purely commercial
activities remotely connected with the discharge of governmental functions. This is particularly true with
respect to the Communist states which took control of nationalized business activities and international In Article 31(a) of the Convention, a diplomatic envoy is granted immunity from the civil and
trading. administrative jurisdiction of the receiving state over any real action relating to private immovable
property situated in the territory of the receiving state which the envoy holds on behalf of the sending
state for the purposes of the mission. If this immunity is provided for a diplomatic envoy, with all the
This Court has considered the following transactions by a foreign state with private parties as acts jure
more reason should immunity be recognized as regards the sovereign itself, which in this case is the Holy
imperii: (1) the lease by a foreign government of apartment buildings for use of its military officers
See.
(Syquia v. Lopez, 84 Phil. 312 [1949]; (2) the conduct of public bidding for the repair of a wharf at a
United States Naval Station (United States of America v. Ruiz, supra.); and (3) the change of employment
status of base employees (Sanders v. Veridiano, 162 SCRA 88 [1988]). The decision to transfer the property and the subsequent disposal thereof are likewise clothed with a
governmental character. Petitioner did not sell Lot
5-A for profit or gain. It merely wanted to dispose off the same because the squatters living thereon
On the other hand, this Court has considered the following transactions by a foreign state with private
made it almost impossible for petitioner to use it for the purpose of the donation. The fact that squatters
parties as acts jure gestionis: (1) the hiring of a cook in the recreation center, consisting of three
have occupied and are still occupying the lot, and that they stubbornly refuse to leave the premises, has
restaurants, a cafeteria, a bakery, a store, and a coffee and pastry shop at the John Hay Air Station in
been admitted by private respondent in its complaint (Rollo, pp. 26, 27).
Baguio City, to cater to American servicemen and the general public (United States of America v.
Rodrigo, 182 SCRA 644 [1990]); and (2) the bidding for the operation of barber shops in Clark Air Base in
Angeles City (United States of America v. Guinto, 182 SCRA 644 [1990]). The operation of the restaurants The issue of petitioner's non-suability can be determined by the trial court without going to trial in the
and other facilities open to the general public is undoubtedly for profit as a commercial and not a light of the pleadings, particularly the admission of private respondent. Besides, the privilege of
governmental activity. By entering into the employment contract with the cook in the discharge of its sovereign immunity in this case was sufficiently established by the Memorandum and Certification of the
proprietary function, the United States government impliedly divested itself of its sovereign immunity Department of Foreign Affairs. As the department tasked with the conduct of the Philippines' foreign
from suit. relations (Administrative Code of 1987, Book IV, Title I, Sec. 3), the Department of Foreign Affairs has
formally intervened in this case and officially certified that the Embassy of the Holy See is a duly
accredited diplomatic mission to the Republic of the Philippines exempt from local jurisdiction and
In the absence of legislation defining what activities and transactions shall be considered "commercial"
entitled to all the rights, privileges and immunities of a diplomatic mission or embassy in this country
and as constituting acts jure gestionis, we have to come out with our own guidelines, tentative they may
(Rollo, pp. 156-157). The determination of the executive arm of government that a state or
be.
instrumentality is entitled to sovereign or diplomatic immunity is a political question that is conclusive
upon the courts (International Catholic Migration Commission v. Calleja, 190 SCRA 130 [1990]). Where
Certainly, the mere entering into a contract by a foreign state with a private party cannot be the ultimate the plea of immunity is recognized and affirmed by the executive branch, it is the duty of the courts to
test. Such an act can only be the start of the inquiry. The logical question is whether the foreign state is accept this claim so as not to embarrass the executive arm of the government in conducting the
engaged in the activity in the regular course of business. If the foreign state is not engaged regularly in a country's foreign relations (World Health Organization v. Aquino, 48 SCRA 242 [1972]). As
business or trade, the particular act or transaction must then be tested by its nature. If the act is in in International Catholic Migration Commission and in World Health Organization, we abide by the
pursuit of a sovereign activity, or an incident thereof, then it is an act jure imperii, especially when it is certification of the Department of Foreign Affairs.
not undertaken for gain or profit.
Ordinarily, the procedure would be to remand the case and order the trial court to conduct a hearing to
As held in United States of America v. Guinto, (supra): establish the facts alleged by petitioner in its motion. In view of said certification, such procedure would
however be pointless and unduly circuitous (Ortigas & Co. Ltd. Partnership v. Judge Tirso Velasco, G.R.
There is no question that the United States of America, like any other state, will be No. 109645, July 25, 1994).
deemed to have impliedly waived its non-suability if it has entered into a contract
in its proprietary or private capacity. It is only when the contract involves its IV
sovereign or governmental capacity that no such waiver may be implied.
Private respondent is not left without any legal remedy for the redress of its grievances. Under both
In the case at bench, if petitioner has bought and sold lands in the ordinary course of a real estate Public International Law and Transnational Law, a person who feels aggrieved by the acts of a foreign
business, surely the said transaction can be categorized as an act jure gestionis. However, petitioner has sovereign can ask his own government to espouse his cause through diplomatic channels.
denied that the acquisition and subsequent disposal of Lot 5-A were made for profit but claimed that it
acquired said property for the site of its mission or the Apostolic Nunciature in the Philippines. Private
Private respondent can ask the Philippine government, through the Foreign Office, to espouse its claims
respondent failed to dispute said claim.
against the Holy See. Its first task is to persuade the Philippine government to take up with the Holy See
the validity of its claims. Of course, the Foreign Office shall first make a determination of the impact of its
Lot 5-A was acquired by petitioner as a donation from the Archdiocese of Manila. The donation was espousal on the relations between the Philippine government and the Holy See (Young, Remedies of
made not for commercial purpose, but for the use of petitioner to construct thereon the official place of Private Claimants Against Foreign States, Selected Readings on Protection by Law of Private Foreign
Investments 905, 919 [1964]). Once the Philippine government decides to espouse the claim, the latter
ceases to be a private cause.

According to the Permanent Court of International Justice, the forerunner of the International Court of
Justice:

By taking up the case of one of its subjects and by reporting to diplomatic action or
international judicial proceedings on his behalf, a State is in reality asserting its
own rights its right to ensure, in the person of its subjects, respect for the rules
of international law (The Mavrommatis Palestine Concessions, 1 Hudson, World
Court Reports 293, 302 [1924]).

WHEREFORE, the petition for certiorari is GRANTED and the complaint in Civil Case No. 90-183 against
petitioner is DISMISSED.

SO ORDERED.
Republic of the Philippines a.) immunity from legal process with respect to acts performed by them in their
SUPREME COURT official capacity except when the Bank waives the immunity.
Manila
the immunity mentioned therein is not absolute, but subject to the exception that the acts was done in
FIRST DIVISION "official capacity." It is therefore necessary to determine if petitioner's case falls within the ambit of
Section 45(a). Thus, the prosecution should have been given the chance to rebut the DFA protocol and it
must be accorded the opportunity to present its controverting evidence, should it so desire.
G.R. No. 125865 January 28, 2000

Third, slandering a person could not possibly be covered by the immunity agreement because our laws
JEFFREY LIANG (HUEFENG), petitioner,
do not allow the commission of a crime, such as defamation, in the name of official duty. The imputation
vs.
of theft is ultra vires and cannot be part of official functions. It is well-settled principle of law that a
PEOPLE OF THE PHILIPPINES, respondent.
public official may be liable in his personal private capacity for whatever damage he may have caused by
his act done with malice or in bad faith or beyond the scope of his authority or jurisdiction.4 It appears
YNARES-SANTIAGO, J.: that even the government's chief legal counsel, the Solicitor General, does not support the stand taken
by petitioner and that of the DFA.
Petitioner is an economist working with the Asian Development Bank (ADB). Sometime in 1994, for
allegedly uttering defamatory words against fellow ADB worker Joyce Cabal, he was charged before the Fourth, under the Vienna Convention on Diplomatic Relations, a diplomatic agent, assuming petitioner is
Metropolitan Trial Court (MeTC) of Mandaluyong City with two counts of grave oral defamation such, enjoys immunity from criminal jurisdiction of the receiving state except in the case of an action
docketed as Criminal Cases Nos. 53170 and 53171. Petitioner was arrested by virtue of a warrant issued relating to any professional or commercial activity exercised by the diplomatic agent in the receiving
by the MeTC. After fixing petitioner's bail at P2,400.00 per criminal charge, the MeTC released him to the state outside his official functions.5 As already mentioned above, the commission of a crime is not part of
custody of the Security Officer of ADB. The next day, the MeTC judge received an "office of protocol" official duty.
from the Department of Foreign Affairs (DFA) stating that petitioner is covered by immunity from legal
process under Section 45 of the Agreement between the ADB and the Philippine Government regarding
Finally, on the contention that there was no preliminary investigation conducted, suffice it to say that
the Headquarters of the ADB (hereinafter Agreement) in the country. Based on the said protocol
preliminary investigation is not a matter of right in cases cognizable by the MeTC such as the one at
communication that petitioner is immune from suit, the MeTC judge without notice to the prosecution
bar.6 Being purely a statutory right, preliminary investigation may be invoked only when specifically
dismissed the two criminal cases. The latter filed a motion for reconsideration which was opposed by the
granted by law.7 The rule on the criminal procedure is clear that no preliminary investigation is required
DFA. When its motion was denied, the prosecution filed a petition for certiorari and mandamus with the
in cases falling within the jurisdiction of the MeTC.8 Besides the absence of preliminary investigation
Regional Trial Court (RTC) of Pasig City which set aside the MeTC rulings and ordered the latter court to
does not affect the court's jurisdiction nor does it impair the validity of the information or otherwise
enforce the warrant of arrest it earlier issued. After the motion for reconsideration was denied,
render it defective.9
petitioner elevated the case to this Court via a petition for review arguing that he is covered by immunity
under the Agreement and that no preliminary investigation was held before the criminal cases were filed
in court.1wphi1.nt WHEREFORE, the petition is DENIED.

The petition is not impressed with merit. SO ORDERED.1wphi1.nt

First, courts cannot blindly adhere and take on its face the communication from the DFA that petitioner Davide, Jr., C.J., Puno, Kapunan and Pardo, JJ., concur.
is covered by any immunity. The DFA's determination that a certain person is covered by immunity is
only preliminary which has no binding effect in courts. In receiving ex-parte the DFA's advice and in motu
propio dismissing the two criminal cases without notice to the prosecution, the latter's right to due
process was violated. It should be noted that due process is a right of the accused as much as it is of the
prosecution. The needed inquiry in what capacity petitioner was acting at the time of the alleged
utterances requires for its resolution evidentiary basis that has yet to be presented at the proper
time.1 At any rate, it has been ruled that the mere invocation of the immunity clause does not ipso
facto result in the dropping of the charges.2

Second, under Section 45 of the Agreement which provides:

Officers and staff of the Bank including for the purpose of this Article experts and consultants
performing missions for the Bank shall enjoy the following privileges and immunities:
Republic of the Philippines On January 2, 1991, private respondent IRRI, through counsel, wrote the Labor Arbiter to inform him
SUPREME COURT that the Institute enjoys immunity from legal process by virtue of Article 3 of Presidential Decree No.
Manila 1620, 5 and that it invokes such diplomatic immunity and privileges as an international organization in
the instant case filed by petitioner, not having waived the same. 6
THIRD DIVISION
IRRI likewise wrote in the same tenor to the Regional Director of the Department of Labor and
Employment. 7

While admitting IRRI's defense of immunity, the Labor Arbiter, nonetheless, cited an Order issued by the
G.R. No. 106483 May 22, 1995
Institute on August 13, 1991 to the effect that "in all cases of termination, respondent IRRI waives its
immunity," 8 and, accordingly, considered the defense of immunity no longer a legal obstacle in resolving
ERNESTO L. CALLADO, petitioner, the case. The dispositive portion of the Labor arbiter's decision dated October 31, 1991, reads:
vs.
INTERNATIONAL RICE RESEARCH INSTITUTE, respondent.
WHEREFORE, premises considered, judgment is hereby rendered ordering
respondent to reinstate complainant to his former position without loss or (sic)
seniority rights and privileges within five (5) days from receipt hereof and to pay
his full backwages from March 7, 1990 to October 31, 1991, in the total amount of
ROMERO, J.: P83,048.75 computed on the basis of his last monthly salary. 9

Did the International Rice Research Institute (IRRI) waive its immunity from suit in this dispute which The NLRC found merit in private respondent' s appeal and, finding that IRRI did not waive its immunity,
arose from an employer-employee relationship? ordered the aforesaid decision of the Labor Arbiter set aside and the complaint dismissed. 10

We rule in the negative and vote to dismiss the petition. Hence, this petition where it is contended that the immunity of the IRRI as an international organization
granted by Article 3 of Presidential Decree No. 1620 may not be invoked in the case at bench inasmuch
as it waived the same by virtue of its Memorandum on "Guidelines on the handling of dismissed
Ernesto Callado, petitioner, was employed as a driver at the IRRI from April 11, 1983 to December 14, employees in relation to P.D. 1620." 11
1990. On February 11, 1990, while driving an IRRI vehicle on an official trip to the Ninoy Aquino
International Airport and back to the IRRI, petitioner figured in an accident.
It is also petitioner's position that a dismissal of his complaint before the Labor Arbiter leaves him no
other remedy through which he can seek redress. He further states that since the investigation of his
Petitioner was informed of the findings of a preliminary investigation conducted by the IRRI's Human case was not referred to the Council of IRRI Employees and Management (CIEM), he was denied his
Resource Development Department Manager in a Memorandum dated March 5, 1990. 1 In view of the constitutional right to due process.
aforesaid findings, he was charged with:

We find no merit in petitioner's arguments.


(1) Driving an institute vehicle while on official duty under the influence of liquor;

IRRI's immunity from suit is undisputed.


(2) Serious misconduct consisting of your failure to report to your supervisors the
failure of your vehicle to start because of a problem with the car battery which,
you alleged, required you to overstay in Manila for more than six (6) hours, Presidential Decree No. 1620, Article 3 provides:
whereas, had you reported the matter to IRRI, Los Baos by telephone, your
problem could have been solved within one or two hours; Art. 3. Immunity from Legal Process. The Institute shall enjoy immunity from any
penal, civil and administrative proceedings, except insofar as that immunity has
(3) Gross and habitual neglect of your duties. 2 been expressly waived by the Director-General of the Institute or his authorized
representatives.

In a Memorandum dated March 9, 1990, petitioner submitted his answer and defenses to the charges
against him.3 After evaluating petitioner's answer, explanations and other evidence, IRRI issued a Notice In the case of International Catholic Migration Commission v. Hon. Calleja, et al. and Kapisanan ng
of Termination to petitioner on December 7, 1990. 4 Manggagawa at TAC sa IRRI v. Secretary of Labor and Employment and IRRI, 12 the Court upheld the
constitutionality of the aforequoted law. After the Court noted the letter of the Acting Secretary of
Foreign Affairs to the Secretary of Labor dated June 17, 1987, where the immunity of IRRI from the
Thereafter, petitioner filed a complaint on December 19, 1990 before the Labor Arbiter for illegal jurisdiction of the Department of Labor and Employment was sustained, the Court stated that this
dismissal, illegal suspension and indemnity pay with moral and exemplary damages and attorney's fees. opinion constituted "a categorical recognition by the Executive Branch of the Government that . . . IRRI
enjoy(s) immunities accorded to international organizations, which determination has been held to be a
political question conclusive upon the Courts in order not to embarass a political department of If and when a dismissed employee files a complaint against the Institute contesting the legality of
Government. 13 We cited the Court's earlier pronouncement in WHO v. Hon. Benjamin Aquino, et al., 14 to dismissal, IRRI's answer to the complaint will:
wit:
1. Indicate in the identification of IRRI that it is an international organization
It is a recognized principle of international law and under our system of separation operating under the laws of the Philippines including P.D. 1620. and
of powers that diplomatic immunity is essentially a political question and courts 2. Base the defense on the merits and facts of the case as well as the legality of the
should refuse to look beyond a determination by the executive branch of the cause or causes for termination.
government, and where the plea of diplomatic immunity is recognized and
affirmed by the executive branch of the government as in the case at bar, it is then
3) Waiving immunity under P.D. 1620
the duty of the courts to accept the claim of immunity upon appropriate
suggestion by the principal law officer of the government . . . or other officer
acting under his direction. Hence, in adherence to the settled principle that courts If the plaintiff's attorney or the arbiter, asks if IRRI will waive its immunity
may not so exercise their jurisdiction . . . as to embarass the executive arm of the we may reply that the Institute will be happy to do so, as it has in the past in the
government in conducting foreign relations, it is accepted doctrine that in such formal manner required thereby reaffirming our commitment to abide by the laws
cases the judicial department of (this) government follows the action of the of the Philippines and our full faith in the integrity and impartially of the legal
political branch and will not embarrass the latter by assuming an antagonistic system. 17 (Emphasis in this paragraphs ours)
jurisdiction. 15
From the last paragraph of the foregoing quotation, it is clear that in cases involving dismissed
Further, we held that "(t)he raison d'etre for these immunities is the assurance of unimpeded employees, the Institute may waive its immunity, signifying that such waiver is discretionary on its part.
performance of their functions by the agencies concerned.
We agree with private respondent IRRI that this memorandum cannot, by any stretch of the imagination,
The grant of immunity from local jurisdiction to . . . and IRRI is clearly necessitated be considered the express waiver by the Director-General. Respondent Commission has quoted IRRI's
by their international character and respective purposes. The objective is to avoid reply thus:
the danger of partiality and interference by the host country in their internal
workings. The exercise of jurisdiction by the Department of Labor in these The 1983 . . . is an internal memo addressed to Personnel and Legal Office and was
instances would defeat the very purpose of immunity, which is to shield the affairs issued for its guidance in handling those cases where IRRI opts to waive its
of international organizations, in accordance with international practice, from immunity. It is not a declaration of waiver for all cases. This is apparent from the
political pressure or control by the host country to the prejudice of member States use of the permissive term "may" rather than the mandatory term "shall" in the
of the organization, and to ensure the unhampered the performance of their last paragraph of the memo. Certainly the memo cannot be considered as the
functions. 16 express waiver by the Director General as contemplated by P.D. 1620, especially
since the memo was issued by a former Director-General. At the very least, the
The grant of immunity to IRRI is clear and unequivocal and an express waiver by its Director-General is express declaration of the incumbent Director-general supersedes the 1983 memo
the only way by which it may relinquish or abandon this immunity. and should be accorded greater respect. It would be equally important to point
out that the Personnel and Legal Office has been non-existent since 1988 as a
result of major reorganization of the IRRI. Cases of IRRI before DOLE are handled
On the matter of waiving its immunity from suit, IRRI had, early on, made its position clear. Through
by an external Legal Counsel as in this particular
counsel, the Institute wrote the Labor Arbiter categorically informing him that the Institute will not waive
case. 18 (Emphasis supplied)
its diplomatic immunity. In the second place, petitioner's reliance on the Memorandum with "Guidelines
in handling cases of dismissal of employees in relation to P.D. 1620" dated July 26, 1983, is misplaced.
The Memorandum reads, in part: The memorandum, issued by the former Director-General to a now-defunct division of the IRRI, was
meant for internal circulation and not as a pledge of waiver in all cases arising from dismissal of
employees. Moreover, the IRRI's letter to the Labor Arbiter in the case at bench made in 1991 declaring
Time and again the Institute has reiterated that it will not use its immunity under
that it has no intention of waiving its immunity, at the very least, supplants any pronouncement of
P.D. 1620 for the purpose of terminating the services of any of its employees.
alleged waiver issued in previous cases.
Despite continuing efforts on the part of IRRI to live up to this undertaking, there
appears to be apprehension in the minds of some IRRI employees. To help allay
these fears the following guidelines will be followed hereafter by the Petitioner's allegation that he was denied due process is unfounded and has no basis.
Personnel/Legal Office while handling cases of dismissed employees.
It is not denied that he was informed of the findings and charges resulting from an investigation
xxx xxx xxx conducted of his case in accordance with IRRI policies and procedures. He had a chance to comment
thereon in a Memorandum he submitted to the Manager of the Human Resource and Development
Department. Therefore, he was given proper notice and adequate opportunity to refute the charges and
2. Notification/manifestation to MOLE or labor arbiter
findings, hereby fulfilling the basic requirements of due process.
Finally, on the issue of referral to the Council of IRRI Employees and Management (CIEM), petitioner
similarly fails to persuade the Court.

The Court, in the Kapisanan ng mga Manggagawa at TAC sa IRRI case, 19 held:

Neither are the employees of IRRI without remedy in case of dispute with
management as, in fact, there had been organized a forum for better
management-employee relationship as evidenced by the formation of the Council
of IRRI Employees and Management (CIEM) wherein "both management and
employees were and still are represented for purposes of maintaining mutual and
beneficial cooperation between IRRI and its employees." The existence of this
Union factually and tellingly belies the argument that Pres. Decree No. Decree No.
1620, which grants to IRRI the status, privileges and immunities of an international
organization, deprives its employees of the right to self-organization.

We have earlier concluded that petitioner was not denied due process, and this, notwithstanding the
non-referral to the Council of IRRI Employees and Management. Private respondent correctly pointed
out that petitioner, having opted not to seek the help of the CIEM Grievance Committee, prepared his
answer by his own self. 20 He cannot now fault the Institute for not referring his case to the CIEM.

IN VIEW OF THE FOREGOING, the petition for certiorari is DISMISSED. No costs.

SO ORDERED.
Republic of the Philippines Union. However, the aforesaid defendants filed a Third Party Complaint against the petitioner and the
SUPREME COURT driver of a dump truck of petitioner.
Manila
Thereafter, the case was subsequently transferred to Branch IV, presided over by respondent judge and
FIRST DIVISION was subsequently docketed as Civil Case No. 107-Bg. By virtue of a court order dated May 7, 1975, the
private respondents amended the complaint wherein the petitioner and its regular employee, Alfredo
Bislig were impleaded for the first time as defendants. Petitioner filed its answer and raised affirmative
G.R. No. L-52179 April 8, 1991
defenses such as lack of cause of action, non-suability of the State, prescription of cause of action and
the negligence of the owner and driver of the passenger jeepney as the proximate cause of the collision.
MUNICIPALITY OF SAN FERNANDO, LA UNION, petitioner
vs.
In the course of the proceedings, the respondent judge issued the following questioned orders, to wit:
HON. JUDGE ROMEO N. FIRME, JUANA RIMANDO-BANIA, IAUREANO BANIA, JR., SOR MARIETA
BANIA, MONTANO BANIA, ORJA BANIA, AND LYDIA R. BANIA, respondents.
(1) Order dated November 4, 1975 dismissing the cross-claim against Bernardo Balagot;
Mauro C. Cabading, Jr. for petitioner.
Simeon G. Hipol for private respondent. (2) Order dated July 13, 1976 admitting the Amended Answer of the Municipality of San
Fernando, La Union and Bislig and setting the hearing on the affirmative defenses only with
respect to the supposed lack of jurisdiction;

(3) Order dated August 23, 1976 deferring there resolution of the grounds for the Motion to
MEDIALDEA, J.:
Dismiss until the trial;

This is a petition for certiorari with prayer for the issuance of a writ of preliminary mandatory injunction
(4) Order dated February 23, 1977 denying the motion for reconsideration of the order of July
seeking the nullification or modification of the proceedings and the orders issued by the respondent
13, 1976 filed by the Municipality and Bislig for having been filed out of time;
Judge Romeo N. Firme, in his capacity as the presiding judge of the Court of First Instance of La Union,
Second Judicial District, Branch IV, Bauang, La Union in Civil Case No. 107-BG, entitled "Juana Rimando
Bania, et al. vs. Macario Nieveras, et al." dated November 4, 1975; July 13, 1976; August 23,1976; (5) Order dated March 16, 1977 reiterating the denial of the motion for reconsideration of the
February 23, 1977; March 16, 1977; July 26, 1979; September 7, 1979; November 7, 1979 and December order of July 13, 1976;
3, 1979 and the decision dated October 10, 1979 ordering defendants Municipality of San Fernando, La
Union and Alfredo Bislig to pay, jointly and severally, the plaintiffs for funeral expenses, actual damages
(6) Order dated July 26, 1979 declaring the case deemed submitted for decision it appearing
consisting of the loss of earning capacity of the deceased, attorney's fees and costs of suit and dismissing
that parties have not yet submitted their respective memoranda despite the court's direction;
the complaint against the Estate of Macario Nieveras and Bernardo Balagot.
and

The antecedent facts are as follows:


(7) Order dated September 7, 1979 denying the petitioner's motion for reconsideration
and/or order to recall prosecution witnesses for cross examination.
Petitioner Municipality of San Fernando, La Union is a municipal corporation existing under and in
accordance with the laws of the Republic of the Philippines. Respondent Honorable Judge Romeo N.
On October 10, 1979 the trial court rendered a decision, the dispositive portion is hereunder quoted as
Firme is impleaded in his official capacity as the presiding judge of the Court of First Instance of La Union,
follows:
Branch IV, Bauang, La Union. While private respondents Juana Rimando-Bania, Laureano Bania, Jr., Sor
Marietta Bania, Montano Bania, Orja Bania and Lydia R. Bania are heirs of the deceased Laureano
Bania Sr. and plaintiffs in Civil Case No. 107-Bg before the aforesaid court. IN VIEW OF ALL OF (sic) THE FOREGOING, judgment is hereby rendered for the plaintiffs, and
defendants Municipality of San Fernando, La Union and Alfredo Bislig are ordered to pay
jointly and severally, plaintiffs Juana Rimando-Bania, Mrs. Priscilla B. Surell, Laureano Bania
At about 7 o'clock in the morning of December 16, 1965, a collision occurred involving a passenger
Jr., Sor Marietta Bania, Mrs. Fe B. Soriano, Montano Bania, Orja Bania and Lydia B. Bania
jeepney driven by Bernardo Balagot and owned by the Estate of Macario Nieveras, a gravel and sand
the sums of P1,500.00 as funeral expenses and P24,744.24 as the lost expected earnings of
truck driven by Jose Manandeg and owned by Tanquilino Velasquez and a dump truck of the Municipality
the late Laureano Bania Sr., P30,000.00 as moral damages, and P2,500.00 as attorney's fees.
of San Fernando, La Union and driven by Alfredo Bislig. Due to the impact, several passengers of the
Costs against said defendants.
jeepney including Laureano Bania Sr. died as a result of the injuries they sustained and four (4) others
suffered varying degrees of physical injuries.
The Complaint is dismissed as to defendants Estate of Macario Nieveras and Bernardo
Balagot.
On December 11, 1966, the private respondents instituted a compliant for damages against the Estate of
Macario Nieveras and Bernardo Balagot, owner and driver, respectively, of the passenger jeepney, which
was docketed Civil Case No. 2183 in the Court of First Instance of La Union, Branch I, San Fernando, La SO ORDERED. (Rollo, p. 30)
Petitioner filed a motion for reconsideration and for a new trial without prejudice to another motion Municipal corporations, for example, like provinces and cities, are agencies of the State when they are
which was then pending. However, respondent judge issued another order dated November 7, 1979 engaged in governmental functions and therefore should enjoy the sovereign immunity from suit.
denying the motion for reconsideration of the order of September 7, 1979 for having been filed out of Nevertheless, they are subject to suit even in the performance of such functions because their charter
time. provided that they can sue and be sued. (Cruz, Philippine Political Law, 1987 Edition, p. 39)

Finally, the respondent judge issued an order dated December 3, 1979 providing that if defendants A distinction should first be made between suability and liability. "Suability depends on the consent of
municipality and Bislig further wish to pursue the matter disposed of in the order of July 26, 1979, such the state to be sued, liability on the applicable law and the established facts. The circumstance that a
should be elevated to a higher court in accordance with the Rules of Court. Hence, this petition. state is suable does not necessarily mean that it is liable; on the other hand, it can never be held liable if
it does not first consent to be sued. Liability is not conceded by the mere fact that the state has allowed
itself to be sued. When the state does waive its sovereign immunity, it is only giving the plaintiff the
Petitioner maintains that the respondent judge committed grave abuse of discretion amounting to
chance to prove, if it can, that the defendant is liable." (United States of America vs. Guinto, supra, p.
excess of jurisdiction in issuing the aforesaid orders and in rendering a decision. Furthermore, petitioner
659-660)
asserts that while appeal of the decision maybe available, the same is not the speedy and adequate
remedy in the ordinary course of law.
Anent the issue of whether or not the municipality is liable for the torts committed by its employee, the
test of liability of the municipality depends on whether or not the driver, acting in behalf of the
On the other hand, private respondents controvert the position of the petitioner and allege that the
municipality, is performing governmental or proprietary functions. As emphasized in the case of Torio vs.
petition is devoid of merit, utterly lacking the good faith which is indispensable in a petition
Fontanilla (G. R. No. L-29993, October 23, 1978. 85 SCRA 599, 606), the distinction of powers becomes
for certiorari and prohibition. (Rollo, p. 42.) In addition, the private respondents stress that petitioner has
important for purposes of determining the liability of the municipality for the acts of its agents which
not considered that every court, including respondent court, has the inherent power to amend and
result in an injury to third persons.
control its process and orders so as to make them conformable to law and justice. (Rollo, p. 43.)

Another statement of the test is given in City of Kokomo vs. Loy, decided by the Supreme Court of
The controversy boils down to the main issue of whether or not the respondent court committed grave
Indiana in 1916, thus:
abuse of discretion when it deferred and failed to resolve the defense of non-suability of the State
amounting to lack of jurisdiction in a motion to dismiss.
Municipal corporations exist in a dual capacity, and their functions are twofold. In one they
exercise the right springing from sovereignty, and while in the performance of the duties
In the case at bar, the respondent judge deferred the resolution of the defense of non-suability of the
pertaining thereto, their acts are political and governmental. Their officers and agents in such
State amounting to lack of jurisdiction until trial. However, said respondent judge failed to resolve such
capacity, though elected or appointed by them, are nevertheless public functionaries
defense, proceeded with the trial and thereafter rendered a decision against the municipality and its
performing a public service, and as such they are officers, agents, and servants of the state. In
driver.
the other capacity the municipalities exercise a private, proprietary or corporate right, arising
from their existence as legal persons and not as public agencies. Their officers and agents in
The respondent judge did not commit grave abuse of discretion when in the exercise of its judgment it the performance of such functions act in behalf of the municipalities in their corporate or
arbitrarily failed to resolve the vital issue of non-suability of the State in the guise of the municipality. individual capacity, and not for the state or sovereign power." (112 N.E., 994-995) (Ibid, pp.
However, said judge acted in excess of his jurisdiction when in his decision dated October 10, 1979 he 605-606.)
held the municipality liable for the quasi-delict committed by its regular employee.
It has already been remarked that municipal corporations are suable because their charters grant them
The doctrine of non-suability of the State is expressly provided for in Article XVI, Section 3 of the the competence to sue and be sued. Nevertheless, they are generally not liable for torts committed by
Constitution, to wit: "the State may not be sued without its consent." them in the discharge of governmental functions and can be held answerable only if it can be shown that
they were acting in a proprietary capacity. In permitting such entities to be sued, the State merely gives
Stated in simple parlance, the general rule is that the State may not be sued except when it gives consent the claimant the right to show that the defendant was not acting in its governmental capacity when the
to be sued. Consent takes the form of express or implied consent. injury was committed or that the case comes under the exceptions recognized by law. Failing this, the
claimant cannot recover. (Cruz, supra, p. 44.)

Express consent may be embodied in a general law or a special law. The standing consent of the State to
be sued in case of money claims involving liability arising from contracts is found in Act No. 3083. A In the case at bar, the driver of the dump truck of the municipality insists that "he was on his way to the
special law may be passed to enable a person to sue the government for an alleged quasi-delict, as in Naguilian river to get a load of sand and gravel for the repair of San Fernando's municipal streets." (Rollo,
Merritt v. Government of the Philippine Islands (34 Phil 311). (see United States of America v. Guinto, p. 29.)
G.R. No. 76607, February 26, 1990, 182 SCRA 644, 654.)
In the absence of any evidence to the contrary, the regularity of the performance of official duty is
Consent is implied when the government enters into business contracts, thereby descending to the level presumed pursuant to Section 3(m) of Rule 131 of the Revised Rules of Court. Hence, We rule that the
of the other contracting party, and also when the State files a complaint, thus opening itself to a driver of the dump truck was performing duties or tasks pertaining to his office.
counterclaim. (Ibid)
We already stressed in the case of Palafox, et. al. vs. Province of Ilocos Norte, the District Engineer, and
the Provincial Treasurer (102 Phil 1186) that "the construction or maintenance of roads in which the
truck and the driver worked at the time of the accident are admittedly governmental activities."

After a careful examination of existing laws and jurisprudence, We arrive at the conclusion that the
municipality cannot be held liable for the torts committed by its regular employee, who was then
engaged in the discharge of governmental functions. Hence, the death of the passenger tragic and
deplorable though it may be imposed on the municipality no duty to pay monetary compensation.

All premises considered, the Court is convinced that the respondent judge's dereliction in failing to
resolve the issue of non-suability did not amount to grave abuse of discretion. But said judge exceeded
his jurisdiction when it ruled on the issue of liability.

ACCORDINGLY, the petition is GRANTED and the decision of the respondent court is hereby modified,
absolving the petitioner municipality of any liability in favor of private respondents.

SO ORDERED.
Republic of the Philippines and as constructed the stage for the "zarzuela" was "5- meters by 8 meters in size, had a wooden floor
SUPREME COURT high at the rear and was supported by 24 bamboo posts 4 in a row in front, 4 in the rear and 5 on each
Manila side with bamboo braces." 1

FIRST DIVISION The "zarzuela" entitled "Midas Extravaganza" was donated by an association of Malasiqui employees of
the Manila Railroad Company in Caloocan, Rizal. The troupe arrived in the evening of January 22 for the
performance and one of the members of the group was Vicente Fontanilla. The program started at about
G.R. No. L-29993 October 23, 1978
10:15 o'clock that evening with some speeches, and many persons went up the stage. The "zarzuela"
then began but before the dramatic part of the play was reached, the stage collapsed and Vicente
LAUDENCIO TORIO, GUILLERMO EVANGELISTA, MANUEL DE GUZMAN, ALFONSO R. MAGSANOC, JESUS Fontanilla who was at the rear of the stage was pinned underneath. Fontanilia was taken to tile San
MACARANAS, MAXIMO MANANGAN, FIDEL MONTEMAYOR, MELCHOR VIRAY, RAMON TULAGAN, all Carlos General Hospital where he died in the afternoon of the following day.
Members of the Municipal Council of Malasiqui in 1959, Malasiqui, Pangasinan, petitioners,
vs.
The heirs of Vicente Fontanilia filed a complaint with the Court of First Instance of Manila on September
ROSALINA, ANGELINA, LEONARDO, EDUARDO, ARTEMIO, ANGELITA, ANITA, ERNESTO, NORMA,
11, 1959 to recover damages. Named party-defendants were the Municipality of Malasiqui, the
VIRGINIA, REMEDIOS and ROBERTO, all surnamed FONTANILLA, and THE HONORABLE COURT OF
Municipal Council of Malasiqui and all the individual members of the Municipal Council in 1959.
APPEALS,respondents.

Answering the complaint defendant municipality invoked inter alia the principal defense that as a legally
G.R. No. L-30183 October 23, 1978
and duly organized public corporation it performs sovereign functions and the holding of a town fiesta
was an exercise of its governmental functions from which no liability can arise to answer for the
MUNICIPALITY OF MALASIQUI, petitioner, negligence of any of its agents.
vs.
ROSALINA, ANGELINA, LEONARDO, EDUARDO, ARTEMIO, ANGELITA, ANITA, ERNESTO, NORMA,
The defendant councilors inturn maintained that they merely acted as agents of the municipality in
VIRGINIA, REMEDIOS and ROBERTO, all surnamed FONTANILLA, and the Honorable COURT OF
carrying out the municipal ordinance providing for the management of the town fiesta celebration and
APPEALS,respondents.
as such they are likewise not liable for damages as the undertaking was not one for profit; furthermore,
they had exercised due care and diligence in implementing the municipal ordinance. 2
Julian M. Armas, Assistant Provincial Fiscal for petitioners.
After trial, the Presiding Judge, Hon. Gregorio T. Lantin narrowed the issue to whether or not the
Isidro L. Padilla for respondents. defendants exercised due diligence 'm the construction of the stage. From his findings he arrived at the
conclusion that the Executive Committee appointed by the municipal council had exercised due diligence
and care like a good father of the family in selecting a competent man to construct a stage strong
enough for the occasion and that if it collapsed that was due to forces beyond the control of the
committee on entertainment, consequently, the defendants were not liable for damages for the death of
MUOZ PALMA, J.: Vicente Fontanilla. The complaint was accordingly dismissed in a decision dated July 10, 1962. 3

These Petitions for review present the issue of whether or not the celebration of a town fiesta The Fontanillas appealed to the Court of Appeals. In a decision Promulgated on October 31, 1968, the
authorized by a municipal council under Sec. 2282 of the Municipal Law as embodied in the Revised Court of Appeals through its Fourth Division composed at the time of Justices Salvador V. Esguerra,
Administrative Code is a governmental or a corporate or proprietary function of the municipality. Nicasio A. Yatco and Eulogio S. Serrano reversed the trial court's decision and ordered all the defendants-
appellees to pay jointly and severally the heirs of Vicente Fontanilla the sums of P12,000.00 by way of
A resolution of that issue will lead to another, viz the civil liability for damages of the Municipality of moral and actual damages: P1200.00 its attorney's fees; and the costs. 4
Malasiqui, and the members of the Municipal Council of Malasiqui, province of Pangasinan, for a death
which occurred during the celebration of the town fiesta on January 22, 1959, and which was attributed The case is now before Us on various assignments of errors all of which center on the proposition stated
to the negligence of the municipality and its council members. at the sentence of this Opinion and which We repeat:

The following facts are not in dispute: Is the celebration of a town fiesta an undertaking in the excercise of a municipality's governmental or
public function or is it or a private or proprietary character?
On October 21, 1958, the Municipal Council of Malasiqui, Pangasinan, passed Resolution No. 159
whereby "it resolved to manage the 1959 Malasiqui town fiesta celebration on January 21, 22, and 23, 1. Under Philippine laws municipalities are political bodies corporate and as such ag endowed with the
1959." Resolution No. 182 was also passed creating the "1959 Malasiqui 'Town Fiesta Executive faculties of municipal corporations to be exercised by and through their respective municipal
Committee" which in turn organized a sub-committee on entertainment and stage, with Jose Macaraeg governments in conformity with law, and in their proper corporate name, they may inter alia sue and be
as Chairman. the council appropriated the amount of P100.00 for the construction of 2 stages, one for sued, and contract and be contracted with.5
the "zarzuela" and another for the cancionan Jose Macaraeg supervised the construction of the stage
The powers of a municipality are twofold in character public, governmental or political on the one hand, tragic and deplorable it may be, the death of Palafox imposed on the province no duty to pay monetary
and corporate, private, or proprietary on the other. Governmental powers are those exercised by the consideration. 12
corporation in administering the powers of the state and promoting the public welfare and they include
the legislative, judicial public, and political Municipal powers on the other hand are exercised for the
With respect to proprietary functions, the settled rule is that a municipal corporation can be held liable
special benefit and advantage of the community and include those which are ministerial private and
to third persons ex contract 13 or ex delicto. 14
corporate. 6

Municipal corporations are subject to be sued upon contracts and in tort. ...
As to when a certain activity is governmental and when proprietary or private, that is generally a difficult
matter to determine. The evolution of the municipal law in American Jurisprudence, for instance, has
shown that; none of the tests which have evolved and are stated in textbooks have set down a xxx xxx xxx
conclusive principle or rule, so that each case will have to be determined on the basis of attending
circumstances. The rule of law is a general one, that the superior or employer must answer civilly
for the negligence or want of skill of its agent or servant in the course or fine of his
In McQuillin on Municipal Corporations, the rule is stated thus: "A municipal corporation proper has ... a employment, by which another, who is free from contributory fault, is injured.
public character as regards the state at large insofar as it is its agent in government, and private (so- Municipal corporations under the conditions herein stated, fall within the
called) insofar as it is to promote local necessities and conveniences for its own community. 7 operation of this rule of law, and are liable, accordingly, to civil actions for
damages when the requisite elements of liability co-exist. ... (Dillon on Municipal
Corporations, 5th ed. Sec. 1610,1647, cited in Mendoza v. de Leon, supra. 514)
Another statement of the test is given in City of Kokomo v. Loy, decided by the Supreme Court of Indiana
in 1916, thus:
3. Coming to the cam before Us, and applying the general tests given above, We hold that the ho of the
town fiesta in 1959 by the municipality of Malsiqui Pangasinan was an exercise of a private or
Municipal corporations exist in a dual capacity, and their functions are two fold. In
proprietary function of the municipality.
one they exercise the right springing from sovereignty, and while in the
performance of the duties pertaining thereto, their acts are political and
governmental Their officers and agents in such capacity, though elected or Section 2282 of the Chatter on Municipal Law of the Revised Administrative Code provides:
appointed by the are nevertheless public functionaries performing a public service,
and as such they are officers, agents, and servants of the state. In the other Section 2282. Celebration of fiesta. fiesta may be held in each municipality not
capacity the municipalities exercise a private. proprietary or corporate right, oftener than once a year upon a date fixed by the municipal council A fiesta s not
arising from their existence as legal persons and not as public agencies. Their be held upon any other date than that lawfully fixed therefor, except when, for
officers and agents in the performance of such functions act in behalf of the weighty reasons, such as typhoons, foundations, earthquakes, epidemics, or other
municipalities in their corporate or in. individual capacity, and not for the state or public ties, the fiesta cannot be hold in the date fixed in which case it may be held
sovereign power. (112 N. E 994-995) at a later date in the same year, by resolution of the council.

In the early Philippine case of Mendoza v. de Leon 1916, the Supreme Court, through Justice Grant T. This provision simply gives authority to the municipality to accelebrate a yearly fiesta but it does not
Trent, relying mainly on American Jurisprudence classified certain activities of the municipality as impose upon it a duty to observe one. Holding a fiesta even if the purpose is to commemorate a religious
governmental, e.g.: regulations against fire, disease, preservation of public peace, maintenance of or historical event of the town is in essence an act for the special benefit of the community and not for
municipal prisons, establishment of schools, post-offices, etc. while the following are corporate or the general welfare of the public performed in pursuance of a policy of the state. The mere fact that the
proprietary in character, viz: municipal waterwork, slaughter houses, markets, stables, bathing celebration, as claimed was not to secure profit or gain but merely to provide entertainment to the town
establishments, wharves, ferries, and fisheries. 8 Maintenance of parks, golf courses, cemeteries and inhabitants is not a conclusive test. For instance, the maintenance of parks is not a source of income for
airports among others, are also recognized as municipal or city activities of a proprietary character. 9 the nonetheless it is private undertaking as distinguished from the maintenance of public schools, jails,
and the like which are for public service.
2. This distinction of powers becomes important for purposes of determining the liability of the
municipality for the acts of its agents which result in an injury to third persons. As stated earlier, there can be no hard and fast rule for purposes of determining the true nature of an
undertaking or function of a municipality; the surrounding circumstances of a particular case are to be
If the injury is caused in the course of the performance of a governmental function or duty no recovery, considered and will be decisive. The basic element, however beneficial to the public the undertaking may
as a rule, can be. had from the municipality unless there is an existing statute on the matter, 10 nor from be, is that it is governmental in essence, otherwise. the function becomes private or proprietary in
its officers, so long as they performed their duties honestly and in good faith or that they did not act character. Easily, no overnmental or public policy of the state is involved in the celebration of a town
wantonly and maliciously. 11 In Palafox, et al., v. Province of Ilocos Norte, et al., 1958, a truck driver fiesta. 15
employed by the provincial government of Ilocos Norte ran over Proceto Palafox in the course of his
work at the construction of a road. The Supreme Court in affirming the trial court's dismissal of the 4. It follows that under the doctrine of respondent superior, petitioner-municipality is to be held liable
complaint for damages held that the province could not be made liable because its employee was in the for damages for the death of Vicente Fontanilia if that was at- tributable to the negligence of the
performance of a governmental function the construction and maintenance of roads and however municipality's officers, employees, or agents.
Art. 2176, Civil Code: Whoever by act or omission causes damage to another, there Thus, private respondents argue that the "Midas Extravaganza" which was to be performed during the
being fault or negligence, is obliged to pay for the damage done. . . town fiesta was a "donation" offered by an association of Malasiqui employees of the Manila Railroad
Co. in Caloocan, and that when the Municipality of Malasiqui accepted the donation of services and
constructed precisely a "zarzuela stage" for the purpose, the participants in the stage show had the right
Art. 2180, Civil Code: The obligation imposed by article 2176 is demandable not
to expect that the Municipality through its "Committee on entertainment and stage" would build or put
only for one's own acts or omission, but also for those of persons for whom one is
up a stage or platform strong enough to sustain the weight or burden of the performance and take the
responsible. . .
necessary measures to insure the personal safety of the participants. 20 We agree.

On this point, the Court of Appeals found and held that there was negligence.
Quite relevant to that argument is the American case of Sanders v. City of Long Beach, 1942, which was
an action against the city for injuries sustained from a fall when plaintiff was descending the steps of the
The trial court gave credence to the testimony of Angel Novado, a witness of the defendants (now city auditorium. The city was conducting a "Know your City Week" and one of the features was the
petitioners), that a member of the "extravaganza troupe removed two principal braces located on the showing of a motion picture in the city auditorium to which the general public was invited and plaintiff
front portion of the stage and u them to hang the screen or "telon", and that when many people went Sanders was one of those who attended. In sustaining the award for Damages in favor of plaintiff, the
up the stage the latter collapsed. This testimony was not believed however by respondent appellate District Court of Appeal, Second district, California, heldinter alia that the "Know your City Week" was a
court, and rightly so. According to said defendants, those two braces were "mother" or "principal" "proprietary activity" and not a "governmental one" of the city, that defendant owed to plaintiff, an
braces located semi-diagonally from the front ends of the stage to the front posts of the ticket booth invitee the duty of exercising ordinary care for her safety, and plaintiff was entitled to assume that she
located at the rear of the stage and were fastened with a bamboo twine. 16 That being the case, it would not be exposed to a danger (which in this case consisted of lack of sufficient illumination of the
becomes incredible that any person in his right mind would remove those principal braces and leave the premises) that would come to her through a violation of defendant duty. 21
front portion of the stage practically unsuported Moreover, if that did happen, there was indeed
negligence as there was lack of suspension over the use of the stage to prevent such an occurrence.
We can say that the deceased Vicente Fontanilla was similarly situated as Sander The Municipality of
Malasiqui resolved to celebrate the town fiesta in January of 1959; it created a committee in charge of
At any rate, the guitarist who was pointed to by Novado as the person who removed the two bamboo the entertainment and stage; an association of Malasiqui residents responded to the call for the
braces denied having done go. The Court of Appeals said "Amor by himself alone could not have festivities and volunteered to present a stage show; Vicente Fontanilla was one of the participants who
removed the two braces which must be about ten meters long and fastened them on top of the stags for like Sanders had the right to expect that he would be exposed to danger on that occasion.
the curtain. The stage was only five and a half meters wide. Surely, it, would be impractical and unwieldy
to use a ten meter bamboo pole, much more two poles for the stage curtain. 17
Lastly, petitioner or appellant Municipality cannot evade ability and/or liability under the c that it was
Jose Macaraeg who constructed the stage. The municipality acting through its municipal council
The appellate court also found that the stage was not strong enough considering that only P100.00 was appointed Macaraeg as chairman of the sub-committee on entertainment and in charge of the
appropriate for the construction of two stages and while the floor of the "zarzuela" stage was of wooden construction of the "zarzuela" stage. Macaraeg acted merely as an agent of the Municipality. Under the
planks, the Post and braces used were of bamboo material We likewise observe that although the stage doctrine of respondent superior mentioned earlier, petitioner is responsible or liable for the negligence
was described by the Petitioners as being supported by "24" posts, nevertheless there were only 4 in of its agent acting within his assigned tasks. 22
front, 4 at the rear, and 5 on each side. Where were the rest?
... when it is sought to render a municipal corporation liable for the act of servants or agents, a cardinal
The Court of Appeals thus concluded inquiry is, whether they are the servants or agents of the corporation. If the corporation appoints or
elects them, can control them in the discharge of their duties, can continue or remove the can hold them
The court a quo itself attributed the collapse of the stage to the great number of responsible for the manner in which they discharge their trust, and if those duties relate to the exercise
onlookers who mounted the stage. The municipality and/or its agents had the of corporate powers, and are for the benefit of the corporation in its local or special interest, they may
necessary means within its command to prevent such an occurrence. Having filed justly be regarded as its agents or servants, and the maxim of respondent superior applies." ... (Dillon on
to take the necessary steps to maintain the safety of the stage for the use of the Municipal Corporations, 5th Ed., Vol IV, p. 2879)
participants in the stage presentation prepared in connection with the celebration
of the town fiesta, particularly, in preventing non participants or spectators from 5. The remaining question to be resolved centers on the liability of the municipal councilors who enacted
mounting and accumulating on the stage which was not constructed to meet the the ordinance and created the fiesta committee.
additional weight- the defendant-appellees were negligent and are liable for the
death of Vicente Fontanilla . (pp. 30-31, rollo, L-29993)
The Court of Appeals held the councilors jointly and solidarity liable with the municipality for damages
under Article 27 of the Civil Code which provides that d any person suffering ing material or moral loss
The findings of the respondent appellate court that the facts as presented to it establish negligence as a because a public servant or employee refuses or neglects, without just cause to perform his official duty
matter of law and that the Municipality failed to exercise the due diligence of a good father of the family, may file an action for damages and other relief at the latter. 23
will not disturbed by Us in the absence of a clear showing of an abuse of discretion or a gross
misapprehension of facts." 18
In their Petition for review the municipal councilors allege that the Court of Appeals erred in ruling that
the holding of a town fiesta is not a governmental function and that there was negligence on their part
Liability rests on negligence which is "the want of such care as a person of ordinary prudence would for not maintaining and supervising the safe use of the stage, in applying Article 27 of the Civil Code
exercise under the circumstances of the case." 19
against them and in not holding Jose Macaraeg liable for the collapse of the stage and the consequent 6. One last point We have to resolve is on the award of attorney's fees by respondent court. Petitioner-
death of Vicente Fontanilla. 24 municipality assails the award.

We agree with petitioners that the Court of Appeals erred in applying Article 27 of the Civil Code against Under paragraph 11, Art. 2208 of the Civil Code attorney's fees and expenses of litigation may be
the for this particular article covers a case of nonfeasance or non-performance by a public officer of his granted when the court deems it just and equitable. In this case of Vicente Fontanilla, although
official duty; it does not apply to a case of negligence or misfeasance in carrying out an official duty. respondent appellate court failed to state the grounds for awarding attorney's fees, the records show
however that attempts were made by plaintiffs, now private respondents, to secure an extrajudicial
compensation from the municipality: that the latter gave prorases and assurances of assistance but
If We are led to set aside the decision of the Court of Appeals insofar as these petitioners are concerned,
failed to comply; and it was only eight month after the incident that the bereaved family of Vicente
it is because of a plain error committed by respondent court which however is not invoked in petitioners'
Fontanilla was compelled to seek relief from the courts to ventilate what was believed to be a just
brief.
cause. 28

In Miguel v. The Court of appeal. et al., the Court, through Justice, now Chief Justice, Fred Ruiz Castro,
We hold, therefore, that there is no error committed in the grant of attorney's fees which after all is a
held that the Supreme Court is vested with ample authority to review matters not assigned as errors in
matter of judicial discretion. The amount of P1,200.00 is fair and reasonable.
an appeal if it finds that their consideration and resolution are indispensable or necessary in arriving at a
just decision in a given case, and that tills is author under Sec. 7, Rule 51 of the Rules of Court. 25 We
believe that this pronouncement can well be applied in the instant case. PREMISES CONSIDERED, We AFFIRM in toto the decision of the Court of Appeals insofar as the
Municipality of Malasiqui is concerned (L-30183), and We absolve the municipal councilors from liability
and SET ASIDE the judgment against them (L-9993).
The Court of Appeals in its decision now under review held that the celebration of a town fiesta by the
Municipality of Malasiqui was not a governmental function. We upheld that ruling. The legal
consequence thereof is that the Municipality stands on the same footing as an ordinary private Without pronouncement as to costs.
corporation with the municipal council acting as its board of directors. It is an elementary principle that a
corporation has a personality, separate and distinct from its officers, directors, or persons composing
SO ORDERED,
it 26 and the latter are not as a rule co-responsible in an action for damages for tort or negligence culpa
aquilla committed by the corporation's employees or agents unless there is a showing of bad faith or
gross or wanton negligence on their part. 27

xxx xxx xxx

The ordinary doctrine is that a director, merely by reason of his office, is not
personally Stable for the torts of his corporation; he Must be shown to have
personally voted for or otherwise participated in them ... Fletcher Encyclopedia
Corporations, Vol 3A Chapt 11, p. 207)

Officers of a corporation 'are not held liable for the negligence of the corporation
merely because of their official relation to it, but because of some wrongful or
negligent act by such officer amounting to a breach of duty which resulted in an
injury ... To make an officer of a corporation liable for the negligence of the
corporation there must have been upon his part such a breach of duty as
contributed to, or helped to bring about, the injury; that is to say, he must be a
participant in the wrongful act. ... (pp. 207-208, Ibid.)

xxx xxx xxx

Directors who merely employ one to give a fireworks Ambition on the corporate
are not personally liable for the negligent acts of the exhibitor. (p. 211, Ibid.)

On these people We absolve Use municipal councilors from any liability for the death of Vicente
Fontanilla. The records do not show that said petitioners directly participated in the defective
construction of the "zarzuela" stage or that they personally permitted spectators to go up the platform.
THIRD DIVISION 4. The construction of the Sports Center shall commence within a period of one (1) year from 09 March
1984 and shall be completed within a period of five (5) years from 09 March 1984.

xxx xxx xxx


[G.R. No. 97882. August 28, 1996]
6. The properties donated (which is more than five (5) percent of the total land area of the DONORs
subdivision) shall constitute the entire open space for DONORs subdivision and all other lands or areas
previously reserved or designated, including Lot 1 and Lot 2A of Block 72 and the whole Block 29 are
dispensed with, and rendered free, as open spaces, and the DONEE hereby agrees to execute and deliver
THE CITY OF ANGELES, Hon. ANTONIO ABAD SANTOS, in his capacity as MAYOR of Angeles City, and all necessary consents, approvals, endorsements, and authorizations to effect the foregoing.
the SANGGUNIANG PANLUNGSOD OF THE CITY OF ANGELES, petitioners, vs. COURT OF
APPEALS and TIMOG SILANGAN DEVELOPMENT CORPORATION, respondents.
7. The properties donated are devoted and described as open spaces of the DONORs subdivision, and to
this effect, the DONEE, upon acceptance of this donation, releases the DONOR and/or assumes any and
DECISION all obligations and liabilities appertaining to the properties donated.

PANGANIBAN, J.:
8. Any substantial breach of the foregoing provisos shall entitle the DONOR to revoke or rescind this
Deed of Donation, and in such eventuality, the DONEE agrees to vacate and return the premises,
In resolving this petition, the Court addressed the questions of whether a donor of open spaces in together with all improvements, to the DONOR peacefully without necessity of judicial action.
a residential subdivision can validly impose conditions on the said donation; whether the city
government as donee can build and operate a drug rehabilitation center on the donated land intended
On July 19, 1988, petitioners started the construction of a drug rehabilitation center on a portion
for open space; and whether the said donation may be validly rescinded by the donor.
of the donated land. Upon learning thereof, private respondent protested such action for being violative
Petitioners claim they have the right to construct and operate a drug rehabilitation center on the of the terms and conditions of the amended deed and prejudicial to its interest and to those of its clients
donated land in question, contrary to the provisions stated in the amended Deed of Donation. and residents. Private respondent also offered another site for the rehabilitation center. However,
petitioners ignored the protest, maintaining that the construction was not violative of the terms of the
On the other hand, private respondent, owner/developer of the Timog Park residential donation. The alternative site was rejected because, according to petitioners, the site was too isolated
subdivision in Angeles City, opposed the construction and now, the operation of the said center on the and had no electric and water facilities.
donated land, which is located within said residential subdivision.
On August 8, 1988, private respondent filed a complaint with the Regional Trial Court, Branch 56,
Before us is a petition for review on certiorari assailing the Decision[1] of the Court of in Angeles City against the petitioners, alleging breach of the conditions imposed in the amended deed
Appeals[2] dated October 31, 1990, which affirmed the decision[3] of the Regional Trial Court of Angeles of donation and seeking the revocation of the donation and damages, with preliminary injunction and/or
City Branch 56,[4] dated February 15, 1989. temporary restraining order to halt the construction of the said center.

On August 10, 1988, the trial court issued a temporary restraining order to enjoin the petitioners
from further proceeding with the construction of the center, which at that time was already 40%
The Antecedents complete.

However, the trial court denied the prayer for preliminary injunction based on the prohibition in
Presidential Decree No. 1818.
In a Deed of Donation dated March 9, 1984, subsequently superseded by a Deed of Donation
dated September 27, 1984, which in turn was superseded by an Amended Deed of Donation dated In their Answer with counterclaim, petitioners admitted the commencement of the construction
November 26, 1984, private respondent donated to the City of Angeles, 51 parcels of land situated in but alleged inter alia that the conditions imposed in the amended deed were contrary to Municipal
Barrio Pampang, City of Angeles, with an aggregate area of 50,676 square meters, more or less, part of a Ordinance No. 1, Series of 1962, otherwise known as the Subdivision Ordinance of the Municipality of
bigger area also belonging to private respondent. The amended deed[5]provided, among others, that: Angeles.[6]

On October 15, 1988, private respondent filed a Motion for Partial Summary Judgment on the
2. The properties donated shall be devoted and utilized solely for the site of the Angeles City Sports ground that the main defense of the petitioners was anchored on a pure question of law and that their
Center (which excludes cockfighting) pursuant to the plans to be submitted within six (6) months by the legal position was untenable.
DONEE to the DONOR for the latters approval, which approval shall not be unreasonably withheld as
long as entire properties donated are developed as a Sports Complex. Any change or modification in the The petitioners opposed, contending that they had a meritorious defense as (1) private
basic design or concept of said Sports Center must have the prior written consent of the DONOR. respondents had no right to dictate upon petitioners what to do with the donated land and how to do it
so long as the purpose remains for public use; and (2) the cause of action of the private respondent
3. No commercial building, commercial complex, market or any other similar complex, mass or tenament became moot and academic when the Angeles City Council repealed the resolution providing for the
(sic) housing/buildings(s) shall be constructed in the properties donated nor shall cockfighting, be construction of said drug rehabilitation center and adopted a new resolution changing
allowed in the premises.
the purpose and usage of said center to a sports development and youth center in order to conform with On April 26, 1991, the respondent Court rendered the assailed Decision affirming the ruling of the
the sports complex project constructed on the donated land. trial court. Subsequently, the petitioners motion for reconsideration was also denied for lack of merit.

On February 15, 1989, the trial court rendered its decision, in relevant part reading as follows: Consequently, this Petition for Review.

x x x the Court finds no inconsistency between the conditions imposed in the Deeds of Donation and the
provision of the Subdivision Ordinance of the City of Angeles requiring subdivisions in Angeles City to
The Issues
reserve at least one (1) hectare in the subdivision as suitable sites known as open spaces for parks,
playgrounds, playlots and/or other areas to be dedicated to public use. On the contrary, the condition
requiring the defendant city of Angeles to devote and utilize the properties donated to it by the plaintiff
for the site of the Angeles City Sports Center conforms with the requirement in the Subdivision The key issues[8] raised by petitioners may be restated as follows:
Ordinance that the subdivision of the plaintiff shall be provided with a playground or playlot, among
others. I. Whether a subdivision owner/developer is legally bound under Presidential Decree No. 1216 to donate
to the city or municipality the open space allocated exclusively for parks, playground and recreational
On the other hand the term public use in the Subdivision Ordinance should not be construed to include a use.
Drug Rehabilitation Center as that would be contrary to the primary purpose of the Subdivision
Ordinance requiring the setting aside of a portion known as Open Space for park, playground and II. Whether the percentage of the open space allocated exclusively for parks, playgrounds and
playlots, since these are intended primarily for the benefit of the residents of the subdivision. While recreational use is to be based on the gross area of the subdivision or on the total area reserved for open
laudable to the general public, a Drug Rehabilitation Center in a subdivision will be a cause of concern space.
and constant worry to its residents.
III. Whether private respondent as subdivision owner/developer may validly impose conditions in the
As to the third issue in paragraph (3), the passage of the Ordinance changing the purpose of the building Amended Deed of Donation regarding the use of the open space allocated exclusively for parks and
constructed in the donated properties from a Drug Rehabilitation Center to a Sports Center comes too playgrounds.
late. It should have been passed upon the demand of the plaintiff to the defendant City of Angeles to
stop the construction of the Drug Rehabilitation Center, not after the complaint was filed.
IV. Whether or not the construction of the Drug Rehabilitation Center on the donated open space may
be enjoined.
Besides, in seeking the revocation of the Amended Deed of Donation, plaintiff also relies on the failure of
the defendant City of Angeles to submit the plan of the proposed Sports Center within six (6) months and
construction of the same within five years from March 9, 1984, which are substantial violations of the V. Whether the donation by respondent as subdivision owner/developer of the open space of its
conditions imposed in the Amended Deed of Donation. subdivision in favor of petitioner City of Angeles may be revoked for alleged violation of the Amended
Deed of Donation.

The dispositive portion of the RTC decision reads:


Central to this entire controversy is the question of whether the donation of the open space may
be revoked at all.
WHEREFORE, judgment is hereby rendered:

(1) Enjoining defendants, its officers, employees and all persons acting on their behalf to perpetually
cease and desist from constructing a Drug Rehabilitation Center or any other building or improvement First Issue: Developer Legally Bound to Donate Open Space
on the Donated Land.

The law involved in the instant case is Presidential Decree No. 1216, dated October 14,
(2) Declaring the amended Deed of Donation revoked and rescinded and ordering defendants to
1977,[9] which reads:
peacefully vacate and return the Donated Land to plaintiff, together with all the improvements existing
thereon.And,
PRESIDENTIAL DECREE NO. 1216
(3) Denying the award of compensatory or actual and exemplary damages including attorneys fees.
Defining Open Space In Residential Subdivisions And Amending Section 31 Of Presidential Decree No.
957 Requiring Subdivision Owners To Provide Roads, Alleys, Sidewalks And Reserve Open Space For
NO PRONOUNCEMENT AS TO COST.
Parks Or Recreational Use.

In March 1989, petitioners filed their Notice of Appeal. On April 15, 1989, while the appeal was
WHEREAS, there is a compelling need to create and maintain a healthy environment in human
pending, petitioners inaugurated the Drug Rehabilitation Center.[7]
settlements by providing open spaces, roads, alleys and sidewalks as may be deemed suitable to
enhance the quality of life of the residents therein;
WHEREAS, such open spaces, roads, alleys and sidewalks in residential subdivisions are for public use and SECTION 3. Sections 2 and 5 of Presidential Decree No. 953 are hereby repealed and other laws, decrees,
are, therefore, beyond the commerce of men; executive orders, institutions, rules and regulations or parts thereof inconsistent with these provisions
are also repealed or amended accordingly.
WHEREAS, pursuant to Presidential Decree No. 953 at least thirty per cent (30%) of the total area of a
subdivision must be reserved, developed and maintained as open space for parks and recreational areas, SECTION 4. This Decree shall take effect immediately.
the cost of which will ultimately be borne by the lot buyers which thereby increase the acquisition price
of subdivision lots beyond the reach of the common mass;
Pursuant to the wording of Sec. 31 of P.D. 957 as above amended by the aforequoted P.D. No.
1216, private respondent is under legal obligation to donate the open space exclusively allocated for
WHEREAS, thirty percent (30%) required open space can be reduced to a level that will make the parks, playgrounds and recreational use to the petitioner.
subdivision industry viable and the price of residential lots within the means of the low income group at
the same time preserve the environmental and ecological balance through rational control of land use This can be clearly established by referring to the original provision of Sec. 31 of P.D. 957, which
and proper design of space and facilities; reads as follows:

WHEREAS, pursuant to Presidential Decree No. 757, government efforts in housing, including resources, SECTION 31. Donation of roads and open spaces to local government. The registered owner or developer
functions and activities to maximize results have been concentrated into one single agency, namely, the of the subdivision or condominium project, upon completion of the development of said projectmay, at
National Housing Authority; his option, convey by way of donation the roads and open spaces found within the project to the city or
municipality wherein the project is located. Upon acceptance of the donation by the city or municipality
concerned, no portion of the area donated shall thereafter be converted to any other purpose or
NOW, THEREFORE, I, FERDINAND E. MARCOS, President of the Philippines, by virtue of the powers purposes unless after hearing, the proposed conversion is approved by the Authority. (Italics supplied)
vested in me by the Constitution, do hereby order and decree:

It will be noted that under the aforequoted original provision, it was optional on the part of the
SECTION 1. For purposes of this Decree, the term open space shall mean an area reserved exclusively for owner or developer to donate the roads and open spaces found within the project to the city or
parks, playgrounds, recreational uses, schools, roads, places of worship, hospitals, health centers, municipality where the project is located. Elsewise stated, there was no legal obligation to make the
barangay centers and other similar facilities and amenities. donation.

SECTION 2. Section 31 of Presidential Decree No. 957 is hereby amended to read as follows: However, said Sec. 31 as amended now states in its last paragraph:

Section 31. Roads, Alleys, Sidewalks and Open Spaces The owner as developer of a subdivision shall Upon their completion x x x, the roads, alleys, sidewalks and playgrounds shall be donated by the owner
provide adequate roads, alleys and sidewalks. For subdivision projects one (1) hectare or more, the or developer to the city or municipality and it shall be mandatory for the local government to accept;
owner or developer shall reserve thirty per cent (30%) of the gross area for open space. Such open provided, however, that the parks and playgrounds may be donated to the Homeowners Association of
space shall have the following standards allocated exclusively for parks, playgrounds and recreational the project with the consent of the city or municipality concerned. x x x.
use:
It is clear from the aforequoted amendment that it is no longer optional on the part of the subdivision
a. 9% of gross area for high density or social housing (66 to 100 family lots per gross hectare). owner/developer to donate the open space for parks and playgrounds; rather there is now a legal
obligation to donate the same. Although there is a proviso that the donation of the parks and
playgrounds may be made to the homeowners association of the project with the consent of the city of
b. 7% of gross area for medium-density or economic housing (21 to 65 family lots per gross hectare). municipality concerned, nonetheless, the owner/developer is still obligated under the law to
donate. Such option does not change the mandatory character of the provision. The donation has to be
c. 3.5% of gross area for low-density or open market housing (20 family lots and below per gross made regardless of which donee is picked by the owner/developer. The consent requirement before the
hectare). same can be donated to the homeowners association emphasizes this point.

These areas reserved for parks, playgrounds and recreational use shall be non-alienable public lands, and
non-buildable. The plans of the subdivision project shall include tree planting on such parts of the Second Issue: Percentage of Area for Parks and Playgrounds
subdivision as may be designated by the Authority.

Upon their completion certified to by the Authority, the roads, alleys, sidewalks and playgrounds shall be Petitioners contend that the 3.5% to 9% allotted by Sec. 31 for parks, playgrounds and
donated by the owner or developer to the city or municipality and it shall be mandatory for the local recreational uses should be based on the gross area of the entire subdivision, and not merely on the area
governments to accept provided, however, that the parks and playgrounds may be donated to the of the open space alone, as contended by private respondent and as decided by the respondent Court.[10]
Homeowners Association of the project with the consent of the city or municipality concerned. No
portion of the parks and playgrounds donated thereafter shall be converted to any other purpose or The petitioners are correct. The language of Section 31 of P.D. 957 as amended by Section 2 of
purposes. P.D. 1216 is wanting in clarity and exactitude, but it can be easily inferred that the phrase gross area
refers to the entire subdivision area. The said phrase was used four times in the same section in two We hold that any condition may be imposed in the donation, so long as the same is not contrary
sentences, the first of which reads: to law, morals, good customs, public order or public policy. The contention of petitioners that the
donation should be unconditional because it is mandatory has no basis in law. P.D. 1216 does not
provide that the donation of the open space for parks and playgrounds should be unconditional. To rule
x x x For subdivision projects one (1) hectare or more, the owner or developer shall reserve thirty per
that it should be so is tantamount to unlawfully expanding the provisions of the decree.[12]
cent (30%) of the gross area for open space. x x x.
In the case at bar, one of the conditions imposed in the Amended Deed of Donation is that the
Here, the phrase 30% of the gross area refers to the total area of the subdivision, not of the open donee should build a sports complex on the donated land. Since P.D. 1216 clearly requires that the 3.5%
space. Otherwise, the definition of open space would be circular. Thus, logic dictates that the same basis to 9% of the gross area allotted for parks and playgrounds is non-buildable, then the obvious question
be applied in the succeeding instances where the phrase open space is used, i.e., 9% of gross area . . . 7% arises whether or not such condition was validly imposed and is binding on the donee. It is clear that the
of gross area . . . 3.5% of gross area . . .Moreover, we agree with petitioners that construing the 3.5% to non-buildable character applies only to the 3.5% to 9% area set by law. If there is any excess land over
9% as applying to the totality of the open space would result in far too small an area being devoted for and above the 3.5% to 9% required by the decree, which is also used or allocated for parks, playgrounds
parks, playgrounds, etc., thus rendering meaningless and defeating the purpose of the statute. This and recreational purposes, it is obvious that such excess area is not covered by the non-buildability
becomes clear when viewed in the light of the original requirement of P.D. 953 (Requiring the Planting of restriction. In the instant case, if there be an excess, then the donee would not be barred from
Trees in Certain Places, etc.), Section 2 of which reads: developing and operating a sports complex thereon, and the condition in the amended deed would then
be considered valid and binding.
Sec. 2. Every owner of land subdivided into residential/commercial/industrial lots after the effectivity of To determine if the over 50,000 square meter area donated pursuant to the amended deed would
this Decree shall reserve, develop and maintain not less than thirty percent (30%) of the total area of the yield an excess over the area required by the decree, it is necessary to determine under which density
subdivision, exclusive of roads, service streets and alleys, as open space for parks and recreational areas. category the Timog Park subdivision falls.

If the subdivision falls under the low density or open market housing category, with 20 family lots
No plan for a subdivision shall be approved by the Land Registration Commission or any office or agency
or below per gross hectare, the developer will need to allot only 3.5% of gross area for parks and
of the government unless at least thirty percent (30%) of the total area of the subdivision, exclusive of
playgrounds, and since the donated land constitutes more than five (5) percent of the total land area of
roads, service streets and alleys, is reserved as open space for parks and recreational areas x x x.
the subdivision,[13] there would therefore be an excess of over 1.5% of gross area which would not be
non-buildable. Petitioners, on the other hand, alleged (and private respondent did not controvert) that
To our mind, it is clear that P.D. 1216 was an attempt to achieve a happy compromise and a the subdivision in question is a medium-density or economic housing subdivision based on the sizes of
realistic balance between the imperatives of environmental planning and the need to maintain economic the family lots donated in the amended deed,[14] for which category the decree mandates that not less
feasibility in subdivision and housing development, by reducing the required area for parks, playgrounds than 7% of gross area be set aside. Since the donated land constitutes only a little more than 5% of the
and recreational uses from thirty percent (30%) to only 3.5% - 9% of the entire area of the subdivision. gross area of the subdivision, which is less than the area required to be allocated for non-buildable open
space, therefore there is no excess land to speak of. This then means that the condition to build a sports
complex on the donated land is contrary to law and should be considered as not imposed.

Third Issue: Imposition of Conditions in Donation of Open Space

Fourth Issue: Injunction vs. Construction of the Drug Rehabilitation Center


Petitioners argue that since the private respondent is required by law to donate the parks and
playgrounds, it has no right to impose the condition in the Amended Deed of Donation that the
properties donated shall be devoted and utilized solely for the site of the Angeles City Sports Center. It Petitioners argue that the court cannot enjoin the construction of the drug rehabilitation center
cannot prescribe any condition as to the use of the area donated because the use of the open spaces is because the decision of the trial court came only after the construction of the center was completed
already governed by P.D. 1216. In other words, the donation should be absolute. Consequently, the and, based on jurisprudence, there can be no injunction of events that have already transpired.[15]
conditions in the amended deed which were allegedly violated are deemed not written. Such being the
case, petitioners cannot be considered to have committed any violation of the terms and conditions of Private respondent, on the other hand, counters that the operation of the center is a continuing
the said amended deed, as the donation is deemed unconditional, and it follows that there is no basis for act which would clearly cause injury to private respondent, its clients, and residents of the subdivision,
revocation of the donation. and thus, a proper subject of injunction.[16] Equity should move in to warrant the granting of the
injunctive relief if persistent repetition of the wrong is threatened.[17]
However, the general law on donations does not prohibit the imposition of conditions on a
donation so long as the conditions are not illegal or impossible.[11] In light of Sec. 31 of P.D. 957, as amended, declaring the open space for parks, playgrounds and
recreational area as non-buildable, it appears indubitable that the construction and operation of a drug
In regard to donations of open spaces, P.D. 1216 itself requires among other things that the rehabilitation center on the land in question is a continuing violation of the law and thus should be
recreational areas to be donated be based, as aforementioned, on a percentage (3.5%, 7%, or 9%) of the enjoined.
total area of the subdivision depending on whether the subdivision is low -, medium -, or high-density. It
further declares that such open space devoted to parks, playgrounds and recreational areas are non- Furthermore, the factual background of this case warrants that this Court rule against petitioners
alienable public land and non-buildable. However, there is no prohibition in either P.D. 957 or P.D. 1216 on this issue. We agree with and affirm the respondent Courts finding that petitioners committed acts
against imposing conditions on such donation. mocking the judicial system.[18]
x x x When a writ of preliminary injunction was sought for by the appellee [private respondent] to enjoin (1) When the fault is on the part of both contracting parties, neither may recover what he has given by
the appellants [petitioners herein] from further continuing with the construction of the said center, the virtue of the contract, or demand the performance of the others undertaking;
latter resisted and took refuge under the provisions of Presidential Decree No. 1818 (which prohibits
writs of preliminary injunction) to continue with the construction of the building. Yet, the appellants also
comes into play here. Both petitioners and private respondents are in violation of P.D. 957 as amended,
presented City Council Resolution No. 227 which allegedly repealed the previous Resolution authorizing
for donating and accepting a donation of open space less than that required by law, and for agreeing to
the City Government to construct a Drug Rehabilitation Center on the donated property, by changing the
build and operate a sports complex on the non-buildable open space so donated; and petitioners, for
purpose and usage of the Drug Rehabilitation Center to Sports Development and Youth Center to make it
constructing a drug rehabilitation center on the same non-buildable area.
conform to the Sports Complex Project therein. Under this Resolution No. 227, the appellants claimed
that they have abandoned all plans for the construction of the Drug Rehabilitation Center. Nonetheless, Moreover, since the condition to construct a sports complex on the donated land has previously
when judgment was finally rendered on February 15, 1989, the appellants were quick to state that they been shown to be contrary to law, therefore, stipulation No. 8 of the amended deed cannot be
have not after all abandoned their plans for the center as they have in fact inaugurated the same on implemented because (1) no valid stipulation of the amended deed had been breached, and (2) it is
April 15, 1989. In plain and simple terms, this act is a mockery of our judicial system perpetrated by the highly improbable that the decree would have allowed the return of the donated land for open space
appellants. For them to argue that the court cannot deal on their Drug Rehabilitation Center is not only under any circumstance, considering the non-alienable character of such open space, in the light of the
preposterous but also ridiculous. second Whereas clause of P.D. 1216 which declares that xxx such open spaces, roads, alleys and
sidewalks in residential subdivisions are for public use and are, therefore, beyond the commerce of men.
It is interesting to observe that under the appealed decision the appellants and their officers, employees
Further, as a matter of public policy, private respondent cannot be allowed to evade its statutory
and all other persons acting on their behalf were perpetually enjoined to cease and desist from
obligation to donate the required open space through the expediency of invoking petitioners breach of
constructing a Drug Rehabilitation Center on the donated property. Under Section 4 of Rule 39 of the
the aforesaid condition. It is a familiar principle that the courts will not aid either party to enforce an
Rules of Court, it is provided that:
illegal contract, but will leave them both where they find them.Neither party can recover damages from
the other arising from the act contrary to law, or plead the same as a cause of action or as a
Section 4 A judgment in an action for injunction shall not be stayed after its rendition and before an defense. Each must bear the consequences of his own acts.[19]
appeal is taken or during the pendency of an appeal.
There is therefore no legal basis whatsoever to revoke the donation of the subject open space and
to return the donated land to private respondent. The donated land should remain with the donee as the
Accordingly, a judgment restraining a party from doing a certain act is enforceable and shall remain in law clearly intended such open spaces to be perpetually part of the public domain, non-alienable and
full force and effect even pending appeal. In the case at bar, the cease and desist order therefore still permanently devoted to public use as such parks, playgrounds or recreation areas.
stands. Appellants persistence and continued construction and, subsequent, operation of the Drug
Rehabilitation Center violate the express terms of the writ of injunction lawfully issued by the lower
court.
Removal/Demolition of Drug Rehabilitation Center
This Court finds no cogent reason to reverse the above mentioned findings of the respondent
court. The allegation of the petitioners that the construction of the center was finished before the
judgment of the trial court was rendered deserves scant consideration because it is self-serving and is Inasmuch as the construction and operation of the drug rehabilitation center has been established
completely unsupported by other evidence. to be contrary to law, the said center should be removed or demolished. At this juncture, we hasten to
add that this Court is and has always been four-square behind the governments efforts to eradicate the
The fact remains that the trial court rendered judgment enjoining the construction of the drug drug scourge in this country. But the end never justifies the means, and however laudable the purpose of
rehabilitation center, revoking the donation and ordering the return of the donated land. In spite of such the construction in question, this Court cannot and will not countenance an outright and continuing
injunction, petitioners publicly flaunted their disregard thereof with the subsequent inauguration of the violation of the laws of the land, especially when committed by public officials.
center on August 15, 1989. The operation of the center, after inauguration, is even more censurable.
In theory, the cost of such demolition, and the reimbursement of the public funds expended in the
construction thereof, should be borne by the officials of the City of Angeles who ordered and directed
such construction. This Court has time and again ruled that public officials are not immune from
Fifth Issue: Revocation of a Mandatory Donation Because of Non-compliance With an Illegal Condition damages in their personal capacities arising from acts done in bad faith. Otherwise stated, a public
official may be liable in his personal capacity for whatever damage he may have caused by his act done
with malice and in bad faith or beyond the scope of his authority or jurisdiction.[20] In the instant case,
The private respondent contends that the building of said drug rehabilitation center is violative of the public officials concerned deliberately violated the law and persisted in their violations, going so far
the Amended Deed of Donation. Therefore, under Article 764 of the New Civil Code and stipulation no. 8 as attempting to deceive the courts by their pretended change of purpose and usage for the center, and
of the amended deed, private respondent is empowered to revoke the donation when the donee has making a mockery of the judicial system. Indisputably, said public officials acted beyond the scope of
failed to comply with any of the conditions imposed in the deed. their authority and jurisdiction and with evident bad faith. However, as noted by the trial court,[21] the
petitioners mayor and members of the Sangguniang Panlungsod of Angeles City were sued only in
We disagree. Article 1412 of the Civil Code which provides that: their official capacities, hence, they could not be held personally liable without first giving them their day
in court. Prevailing jurisprudence[22] holding that public officials are personally liable for damages arising
If the act in which the unlawful or forbidden cause consists does not constitute a criminal offense, the from illegal acts done in bad faith are premised on said officials having been sued both in their official
following rules shall be observed: and personal capacities.
After due consideration of the circumstances, we believe that the fairest and most equitable
solution is to have the City of Angeles, donee of the subject open space and, ostensibly, the main
beneficiary of the construction and operation of the proposed drug rehabilitation center, undertake the
demolition and removal of said center, and if feasible, recover the cost thereof from the city officials
concerned.

WHEREFORE, the assailed Decision of the Court of Appeals is hereby MODIFIED as follows:

(1) Petitioners are hereby ENJOINED perpetually from operating the drug rehabilitation center or
any other such facility on the donated open space.

(2) Petitioner City of Angeles is ORDERED to undertake the demolition and removal of said drug
rehabilitation center within a period of three (3) months from finality of this Decision, and thereafter, to
devote the said open space for public use as a park, playground or other recreational use.

(3) The Amended Deed of Donation dated November 26, 1984 is hereby declared valid and
subsisting, except that the stipulations or conditions therein concerning the construction of the Sports
Center or Complex are hereby declared void and as if not imposed, and therefore of no force and effect.

No costs.

SO ORDERED.
Republic of the Philippines further information or to report noted or suspected irregularities. Exhibits E & E-1.
SUPREME COURT (Rollo, pp. 11-12)
Manila
The private respondent was the only one who was named "Auring" in the Office of the Provost Marshal.
THIRD DIVISION That the private respondent was the same "Auring" referred to in the POD was conclusively proven when
on February 7, 1978, petitioner M. H. Wylie wrote her a letter of apology for the "inadvertent"
publication. The private respondent then commenced an action for damages in the Court of First
Instance of Zambales (now Regional Trial Court) against
M. H. Wylie, Capt. James Williams and the U. S. Naval Base. She alleged that the article constituted false,
G.R. No. 74135 May 28, 1992 injurious, and malicious defamation and libel tending to impeach her honesty, virtue and reputation
exposing her to public hatred, contempt and ridicule; and that the libel was published and circulated in
M. H. WYLIE and CAPT. JAMES WILLIAMS, petitioners, the English language and read by almost all the U. S. Naval Base personnel. She prayed that she be
vs. awarded P300,000.00 as moral damages; exemplary damages which the court may find proper; and
AURORA I. RARANG and THE HONORABLE INTERMEDIATE APPELLATE COURT, respondents. P50,000.00 as attorney's fees.

In response to the complaint, the defendants filed a motion to dismiss anchored on three grounds:

GUTIERREZ, JR., J.: 1. Defendants M. H. Wylie and Capt. James Williams acted in the performance of
their official functions as officers of the United States Navy and are, therefore,
immune from suit;
The pivotal issue in this petition centers on the extent of the "immunity from suit" of the officials of a
United States Naval Base inside Philippine territory.
2. The United States Naval Base is an instrumentality of the US government which
cannot be sued without its consent; and
In February, 1978, petitioner M. H. Wylie was the assistant administrative officer while petitioner Capt.
James Williams was the commanding officer of the U. S. Naval Base in Subic Bay, Olongapo City. Private
respondent Aurora I. Rarang was an employee in the office of the Provost Marshal assigned as 3. This Court has no jurisdiction over the subject matter as well as the parties in
merchandise control guard. this case. (Record on Appeal, pp. 133-134)

M. H. Wylie, in his capacity as assistant administrative officer of the U.S. Naval Station supervised the The motion was, however, denied.
publication of the "Plan of the Day" (POD) which was published daily by the US Naval Base station. The
POD featured important announcements, necessary precautions, and general matters of interest to In their answer, the defendants reiterated the lack of jurisdiction of the court over the case.
military personnel. One of the regular features of the POD was the "action line inquiry." On February 3,
1978, the POD published, under the "NAVSTA ACTION LINE INQUIRY" the following:
In its decision, the trial court ruled that the acts of defendants M. H. Wylie and Cpt. James Williams were
not official acts of the government of the United States of America in the operation and control of the
Question: I have observed that Merchandise Control inspector/inspectress are (sic) Base but personal and tortious acts which are exceptions to the general rule that a sovereign country
consuming for their own benefit things they have confiscated from Base cannot be sued in the court of another country without its consent. In short, the trial court ruled that the
Personnel. The observation is even more aggravated by consuming such acts and omissions of the two US officials were not imputable against the US government but were done
confiscated items as cigarettes and food stuffs PUBLICLY. This is not to mention in the individual and personal capacities of the said officials. The trial court dismissed the suit against the
"Auring" who is in herself, a disgrace to her division and to the Office of the US Naval Base. The dispositive portion of the decision reads as follows:
Provost Marshal. In lieu of this observation, may I therefore, ask if the head of the
Merchandise Control Division is aware of this malpractice?
WHEREFORE, judgment is hereby rendered in favor of the plaintiff and against the
defendants jointly and severally, as follows:
Answer: Merchandise Control Guards and all other personnel are prohibited from
appropriating confiscated items for their own consumption or use. Two locked
1) Ordering defendants M. H. Wylie and Capt. James Williams to pay the plaintiff
containers are installed at the Main Gate area for deposit of confiscated items and
Aurora Rarang the sum of one hundred thousand (P100,000.00) pesos by way of
the OPM evidence custodian controls access to these containers.
moral and exemplary damages;

Merchandise Control Guards are permitted to eat their meals at their worksite due
2) Ordering defendants M. H. Wylie and Capt. James Williams to pay the plaintiff
to heavy workload. Complaints regarding merchandise control guards procedure
the sum of thirty thousand (P30,000.00) pesos by way of attorney's fees and
or actions may be made directly at the Office of the Provost Marshal for
expenses of litigation; and
immediate and necessary action. Specific dates and time along with details of
suspected violations would be most appreciated. Telephone 4-3430/4-3234 for
3) To pay the costs of this suit. state is automatically obligated to comply with these principles in its relations with
other states.
Counterclaims are dismissed.
As applied to the local state, the doctrine of state immunity is based on the
justification given by Justice Holmes that "there can be no legal right against the
Likewise, the suit against the U.S. Naval Base is ordered dismissed. (Record on
authority which makes the law on which the right depends." (Kawanakoa v.
Appeal, p. 154)
Polybank, 205 U.S. 349) There are other practical reasons for the enforcement of
the doctrine. In the case of the foreign state sought to be impleaded in the local
On appeal, the petitioners reiterated their stance that they are immune from suit since the subject jurisdiction, the added inhibition is expressed in the maxim par in parem, non
publication was made in their official capacities as officers of the U. S. Navy. They also maintained that habet imperium. All states are sovereign equals and cannot assert jurisdiction over
they did not intentionally and maliciously cause the questioned publication. one another. A contrary disposition would, in the language of a celebrated case,
"unduly vex the peace of nations." (Da Haber v. Queen of Portugal, 17 Q. B. 171)
The private respondent, not satisfied with the amount of damages awarded to her, also appealed the
trial court's decision. While the doctrine appears to prohibit only suits against the state without its
consent, it is also applicable to complaints filed against officials of the state for
Acting on these appeals, the Intermediate Appellate Court, now Court of Appeals, modified the trial acts allegedly performed by them in the discharge of their duties. The rule is that if
court's decision, to wit: the judgment against such officials will require the state itself to perform an
affirmative act to satisfy the same, such as the appropriation of the amount
needed to pay the damages awarded against them, the suit must be regarded as
WHEREFORE, the judgment of the court below is modified so that the defendants against the state itself although it has not been formally impleaded. (Garcia v.
are ordered to pay the plaintiff, jointly and severally, the sum of P175,000.00 as Chief of Staff, 16 SCRA 120) In such a situation, the state may move to dismiss the
moral damages and the sum of P60,000.00 as exemplary damages. The rest of the complaint on the ground that it has been filed without its consent.
judgment appealed from is hereby affirmed in toto. Costs against the defendants-
appellants. (Rollo, p. 44)
The doctrine is sometimes derisively called "the royal prerogative of dishonesty"
because of the privilege it grants the state to defeat any legitimate claim against it
The appellate court denied a motion for reconsideration filed by the petitioners. by simply invoking its non-suability. That is hardly fair, at least in democratic
societies, for the state is not an unfeeling tyrant unmoved by the valid claims of its
Hence, this petition. citizens. In fact, the doctrine is not absolute and does not say the state may not be
sued under any circumstance. On the contrary, the rule says that the state may not
be sued without its consent, which clearly imports that it may be sued if it
In a resolution dated March 9, 1987, we gave due course to the petition. consents.

The petitioners persist that they made the questioned publication in the performance of their official The consent of the state to be sued may be manifested expressly or impliedly.
functions as administrative assistant, in the case of M. H. Wylie, and commanding officer, in the case of Express consent may be embodied in a general law or a special law. Consent is
Capt. James Williams of the US Navy assigned to the U. S. Naval Station, Subic Bay, Olongapo City and implied when the state enters into a contract it itself commences litigation.
were, therefore, immune from suit for their official actions.

xxx xxx xxx


In the case of United States of America v. Guinto (182 SCRA 644 [1990]), we discussed the principle of the
state immunity from suit as follows:
The above rules are subject to qualification. Express consent is effected only by the
will of the legislature through the medium of a duly enacted statute. (Republic v.
The rule that a state may not be sued without its consent, now expressed in Article Purisima, 78 SCRA 470) We have held that not all contracts entered into by the
XVI, Section 3, of the 1987 Constitution, is one of the generally accepted principles government will operate as a waiver of its non-suability; distinction must be made
of international law that we have adopted as part of the law of our land under between its sovereign and proprietary acts. (United States of America v. Ruiz, 136
Article II, Section 2. SCRA 487) As for the filing of a complaint by the government, suability will result
only where the government is claiming affirmative relief from the defendant. (Lim
xxx xxx xxx v. Brownell, 107 Phil. 345) (at pp. 652-655)

Even without such affirmation, we would still be bound by the generally accepted In the same case we had opportunity to discuss extensively the nature and extent of immunity from suit
principles of international law under the doctrine of incorporation. Under this of United States personnel who are assigned and stationed in Philippine territory, to wit:
doctrine, as accepted by the majority of states, such principles are deemed
incorporated in the law of every civilized state as a condition and consequence of
its membership in the society of nations. Upon its admission to such society, the
In the case of the United States of America, the customary rule of international law over the present case for unlawful detainer. The question of
on state immunity is expressed with more specificity in the RP-US Bases Treaty. lack of jurisdiction was raised and interposed at the very
Article III thereof provides as follows: beginning of the action. The U.S. Government has not given
its consent to the filing of this suit which is essentially
against her, though not in name. Moreover, this is not only a
It is mutually agreed that the United States shall have the
case of a citizen filing a suit against his own Government
rights, power and authority within the bases which are
without the latter's consent but it is of a citizen filing an
necessary for the establishment, use, operation and defense
action against a foreign government without said
thereof or appropriate for the control thereof and all the
government's consent, which renders more obvious the lack
rights, power and authority within the limits of the territorial
of jurisdiction of the courts of his country. The principles of
waters and air space adjacent to, or in the vicinity of, the
law behind this rule are so elementary and of such general
bases which are necessary to provide access to them or
acceptance that we deem it unnecessary to cite authorities
appropriate for their control.
in support thereof."

The petitioners also rely heavily on Baer v. Tizon, (57 SCRA 1) along with several
xxx xxx xxx
other decisions, to support their position that they are not suable in the cases
below, the United States not having waived its sovereign immunity from suit. It is
emphasized that in Baer, the Court held: It bears stressing at this point that the above observations do not confer on the
United States of America a blanket immunity for all acts done by it or its agents in
the Philippines. Neither may the other petitioners claim that they are also
The invocation of the doctrine of immunity from suit of a
insulated from suit in this country merely because they have acted as agents of the
foreign state without its consent is appropriate. More
United States in the discharge of their official functions.
specifically, insofar as alien armed forces is concerned, the
starting point is Raquiza v. Bradford, a 1945 decision. In
dismissing a habeas corpus petition for the release of There is no question that the United States of America, like any other state, will be
petitioners confined by American army authorities, Justice deemed to have impliedly waived its non-suability if it has entered into a contract
Hilado, speaking for the Court, cited Coleman in its proprietary or private capacity. It is only when the contract involves its
v. Tennessee, where it was explicitly declared: "It is well sovereign or governmental capacity that no such waiver may be implied. This was
settled that a foreign army, permitted to march through a our ruling in United States of America v. Ruiz, (136 SCRA 487) where the
friendly country or to be stationed in it, by permission of its transaction in question dealt with the improvement of the wharves in the naval
government or sovereign, is exempt from the civil and installation at Subic Bay. As this was a clearly governmental function, we held that
criminal jurisdiction of the place." Two years later, in Tubb the contract did not operate to divest the United States of its sovereign immunity
and Tedrow v. Griess, this Court relied on the ruling from suit. In the words of Justice Vicente Abad Santos:
in Raquiza v. Bradford and cited in support thereof excerpts
from the works of the following authoritative writers: Vattel,
The traditional rule of immunity excepts a State from being
Wheaton, Hall, Lawrence, Oppenheim, Westlake, Hyde, and
sued in the courts of another State without its consent or
McNair and Lauterpacht. Accuracy demands the clarification
waiver. This rule is a necessary consequence of the
that after the conclusion of the Philippine-American Military
principles of independence and equality of States. However,
Bases Agreement, the treaty provisions should control on
the rules of International Law are not petrified; they are
such matter, the assumption being that there was a
constantly developing and evolving. And because the
manifestation of the submission to jurisdiction on the part of
activities of states have multiplied, it has been necessary to
the foreign power whenever appropriate. More to the point
distinguish them between sovereign and governmental
is Syquia v. Almeda Lopez, where plaintiffs as lessors sued
acts (jure imperii) and private, commercial and proprietary
the Commanding General of the United States Army in the
acts (jure gestionis). The result is that State immunity now
Philippines, seeking the restoration to them of the
extends only to acts jure imperii. The restrictive application
apartment buildings they owned leased to the United States
of State immunity is now the rule in the United States, the
armed forces station in the Manila area. A motion to dismiss
United Kingdom and other states in Western Europe.
on the ground of non-suability was filed and upheld by
respondent Judge. The matter was taken to this Court in
a mandamus proceeding. It failed. It was the ruling that xxx xxx xxx
respondent Judge acted correctly considering that the
"action must be considered as one against the U.S. The restrictive application of State immunity is proper only
Government." The opinion of Justice Montemayor when the proceedings arise out of commercial transactions
continued: "It is clear that the courts of the Philippines of the foreign sovereign, its commercial activities or
including the Municipal Court of Manila have no jurisdiction economic affairs. Stated differently, a State may be said to
have descended to the level of an individual and can thus be The question, therefore, arises are American naval officers who commit a crime or tortious act while
deemed to have tacitly given its consent to be sued only discharging official functions still covered by the principle of state immunity from suit? Pursuing the
when it enters into business contracts. It does not apply question further, does the grant of rights, power, and authority to the United States under the RP-US
where the contract relates to the exercise of its sovereign Bases Treaty cover immunity of its officers from crimes and torts? Our answer is No.
functions. In this case the projects are an integral part of the
naval base which is devoted to the defense of both the
Killing a person in cold blood while on patrol duty, running over a child while driving with reckless
United States and the Philippines, indisputably a function of
imprudence on an official trip, or slandering a person during office hours could not possibly be covered
the government of the highest order; they are not utilized
by the immunity agreement. Our laws and, we presume, those of the United States do not allow the
for nor dedicated to commercial or business purposes.
commission of crimes in the name of official duty.

The other petitioners in the cases before us all aver they have acted in the
The case of Chavez v. Sandiganbayan, 193 SCRA 282 [1991] gives the law on immunity from suit of public
discharge of their official functions as officers or agents of the United States.
officials:
However, this is a matter of evidence. The charges against them may not be
summarily dismissed on their mere assertion that their acts are imputable to the
United States of America, which has not given its consent to be sued. In fact, the The general rule is that public officials can be held personally accountable for acts
defendants are sought to be held answerable for personal torts in which the claimed to have been performed in connection with official duties where they
United States itself is not involved. If found liable, they and they alone must satisfy have acted ultra vires or where there is showing of bad faith.
the judgment. (At pp. 655-658)
xxx xxx xxx
In the light of these precedents, we proceed to resolve the present case.
Moreover, the petitioner's argument that the immunity proviso under Section 4(a)
The POD was published under the direction and authority of the commanding officer, U.S. Naval Station of Executive Order No. 1 also extends to him is not well-taken. A mere invocation
Subic Bay. The administrative assistant, among his other duties, is tasked to prepare and distribute the of the immunity clause does not ipso facto result in the charges being
POD. On February 3, 1978, when the questioned article was published in the POD, petitioner Capt. James automatically dropped.
Williams was the commanding officer while petitioner M.H. Wylie was the administrative assistant of the
US Naval Station at Subic bay. In the case of Presidential Commission on Good Government v. Pea (159 SCRA
556 [1988] then Chief Justice Claudio Teehankee, added a clarification of the
The NAVSTA ACTION LINE INQUIRY is a regular feature of the POD. It is a telephone answering device in immunity accorded PCGG officials under Section 4(a) of Executive Order No. 1 as
the office of the Administrative Assistant. The Action Line is intended to provide personnel access to the follows:
Commanding Officer on matters they feel should be brought to his attention for correction or
investigation. The matter of inquiry may be phoned in or mailed to the POD. (TSN, September 9, 1980, With respect to the qualifications expressed by Mr. Justice
pp. 12-13, Jerry Poblon) According to Feliciano in his separate opinion, I just wish to point out two
M. H. Wylie, the action line naming "Auring" was received about three (3) weeks prior to its being things: First, the main opinion does not claim absolute
published in the POD on February 3, 1978. It was forwarded to Rarang's office of employment, the immunity for the members of the Commission. The cited
Provost Marshal, for comment. The Provost Marshal office's response ". . . included a short note stating section of Executive Order No. 1 provides the Commission's
that if the article was published, to remove the name." (Exhibit 8-A, p. 5) The Provost Marshal's response members immunity from suit thus: "No civil action shall lie
was then forwarded to the executive officer and to the commanding officer for approval. The approval of against the Commission or any member thereof for anything
the Commanding officer was forwarded to the office of the Administrative Assistant for inclusion in the done or omitted in the discharge of the task contemplated
POD. A certain Mrs. Dologmodin, a clerk typist in the office of the Administrative Assistant prepared the by this order." No absolute immunity like that sought by
smooth copy of the POD. Finally, M. H. Wylie, the administrative assistant signed the smooth copy of the Mr. Marcos in his Constitution for himself and his
POD but failed to notice the reference to "Auring" in the action line inquiry. (Exh. 8-A, pp. 4-5, Questions subordinates is herein involved. It is understood that the
Nos. 14-15). immunity granted the members of the Commission by virtue
of the unimaginable magnitude of its task to recover the
There is no question, therefore, that the two (2) petitioners actively participated in screening the plundered wealth and the State's exercise of police power
features and articles in the POD as part of their official functions. Under the rule that U.S. officials in the was immunity from liability for damages in the official
performance of their official functions are immune from suit, then it should follow that the petitioners discharge of the task granted the members of the
may not be held liable for the questioned publication. Commission much in the same manner that judges are
immune from suit in the official discharge of the functions of
their office.
It is to be noted, however, that the petitioners were sued in their personal capacities for their alleged
. . . (at pp. 581-582)
tortious acts in publishing a libelous article.

xxx xxx xxx


Immunity from suit cannot institutionalize irresponsibility and non-accountability payment of moral damages, considering the wounded feelings and besmirched reputation of the
nor grant a privileged status not claimed by any other official of the Republic. defendants.
(id., at page 586)
Indeed the imputation of theft contained in the POD dated February 3, 1978 is a defamation against the
Where the petitioner exceeds his authority as Solicitor General, acts in bad faith, character and reputation of the private respondent. Petitioner Wylie himself admitted that the Office of
or, as contended by the private respondent, "maliciously conspir(es) with the the Provost Marshal explicitly recommended the deletion of the name Auring if the article were
PCGG commissioners in persecuting respondent Enrile by filing against him an published. The petitioners, however, were negligent because under their direction they issued the
evidently baseless suit in derogation of the latter's constitutional rights and publication without deleting the name "Auring." Such act or omission is ultra vires and cannot be part of
liberties" (Rollo, p. 417), there can be no question that a complaint for damages official duty. It was a tortious act which ridiculed the private respondent. As a result of the petitioners'
does not confer a license to persecute or recklessly injure another. The actions act, the private respondent, according to the record, suffered besmirched reputation, serious anxiety,
governed by Articles 19, 20, 21, and 32 of the Civil Code on Human Relations may wounded feelings and social humiliation, specially so, since the article was baseless and false. The
be taken against public officers or private citizens alike. . . . (pp. 289-291) petitioners, alone, in their personal capacities are liable for the damages they caused the private
respondent.
We apply the same ruling to this case.
WHEREFORE, the petition is hereby DISMISSED. The questioned decision and resolution of the then
Intermediate Appellate Court, now Court of Appeals, are AFFIRMED.
The subject article in the US Newsletter POD dated February 3, 1978 mentions a certain "Auring" as ". . a
disgrace to her division and to the Office of the Provost Marshal." The same article explicitly implies that
Auring was consuming and appropriating for herself confiscated items like cigarettes and foodstuffs.
There is no question that the Auring alluded to in the Article was the private respondent as she was the
only Auring in the Office of the Provost Marshal. Moreover, as a result of this article, the private
respondent was investigated by her supervisor. Before the article came out, the private respondent had
been the recipient of commendations by her superiors for honesty in the performance of her duties.

It may be argued that Captain James Williams as commanding officer of the naval base is far removed in
the chain of command from the offensive publication and it would be asking too much to hold him
responsible for everything which goes wrong on the base. This may be true as a general rule. In this
particular case, however, the records show that the offensive publication was sent to the commanding
officer for approval and he approved it. The factual findings of the two courts below are based on the
records. The petitioners have shown no convincing reasons why our usual respect for the findings of the
trial court and the respondent court should be withheld in this particular case and why their decisions
should be reversed.

Article 2176 of the Civil Code prescribes a civil liability for damages caused by a person's act or omission
constituting fault or negligence, to wit:

Art. 2176. Whoever by act or omission, causes damage to another, there being
fault or negligence is obliged to pay for the damage done. Such fault or negligence,
if there is no pre-existing contractual relation between the parties, is called
a quasi-delict and is governed by the provisions of this Chapter.

"Fault" or "negligence" in this Article covers not only acts "not punishable by law" but also acts criminal
in character, whether intentional or voluntary or negligent." (Andamo v. Intermediate Appellate Court,
191 SCRA 195 [1990]).

Moreover, Article 2219(7) of the Civil Code provides that moral damages may be recovered in case of
libel, slander or any other form of defamation. In effect, the offended party in these cases is given the
right to receive from the guilty party moral damages for injury to his feelings and reputation in addition
to punitive or exemplary damages. (Occena v. Icamina, 181 SCRA 328 [1990]). In another case, Heirs of
Basilisa Justiva v. Gustilo, 7 SCRA 72 [1963], we ruled that the allegation of forgery of documents could
be a defamation, which in the light of Article 2219(7) of the Civil Code could by analogy be ground for
Republic of the Philippines The within petition is thus an off-shot of the action (Civil Case No. SJC-56) instituted by petitioners-
SUPREME COURT spouses on April 17, 1978 against respondent NIA before the then Court of First Instance of Nueva Ecija,
Manila Branch VIII at San Jose City, for damages in connection with the death of their son resulting from the
aforestated accident.
SECOND DIVISION
After trial, the trial court rendered judgment on March 20, 1980 which directed respondent National
Irrigation Administration to pay damages (death benefits) and actual expenses to petitioners. The
G.R. No. L-55963 December 1, 1989
dispositive portion of the decision reads thus:

SPOUSES JOSE FONTANILLA AND VIRGINIA FONTANILLA, petitioners,


. . . . . Judgment is here rendered ordering the defendant National Irrigation
vs.
Administration to pay to the heirs of the deceased P12,000.00 for the death of
HONORABLE INOCENCIO D. MALIAMAN and NATIONAL IRRIGATION ADMINISTRATION, respondents.
Francisco Fontanilla; P3,389.00 which the parents of the deceased had spent for
the hospitalization and burial of the deceased Francisco Fontanilla; and to pay the
G.R. No. L-61045 December 1, 1989 costs. (Brief for the petitioners spouses Fontanilla, p. 4; Rollo, p. 132)

NATIONAL IRRIGATION ADMINISTRATION, appellant, Respondent National Irrigation Administration filed on April 21, 1980, its motion for reconsideration of
vs. the aforesaid decision which respondent trial court denied in its Order of June 13, 1980. Respondent
SPOUSES JOSE FONTANILLA and VIRGINIA FONTANILLA, appellees. National Irrigation Administration thus appealed said decision to the Court of Appeals (C.A.-G.R. No.
67237- R) where it filed its brief for appellant in support of its position.
Cecilio V. Suarez, Jr. for Spouses Fontanilla.
Instead of filing the required brief in the aforecited Court of Appeals case, petitioners filed the instant
Felicisimo C. Villaflor for NIA. petition with this Court.

The sole issue for the resolution of the Court is: Whether or not the award of moral damages, exemplary
damages and attorney's fees is legally proper in a complaint for damages based on quasi-delict which
resulted in the death of the son of herein petitioners.
PARAS, J.:

Petitioners allege:
In G.R. No. L-55963, the petition for review on certiorari seeks the affirmance of the decision dated
March 20, 1980 of the then Court of First Instance of Nueva Ecija, Branch VIII, at San Jose City and its
modification with respect to the denial of petitioner's claim for moral and exemplary damages and 1. The award of moral damages is specifically allowable. under paragraph 3 of
attorneys fees. Article 2206 of the New Civil Code which provides that the spouse, legitimate and
illegitimate descendants and ascendants of the deceased may demand moral
damages for mental anguish by reason of the death of the deceased. Should moral
In G.R. No. 61045, respondent National Irrigation Administration seeks the reversal of the aforesaid damages be granted, the award should be made to each of petitioners-
decision of the lower court. The original appeal of this case before the Court of Appeals was certified to spouses individually and in varying amounts depending upon proof of mental and
this Court and in the resolution of July 7, 1982, it was docketed with the aforecited number. And in the depth of intensity of the same, which should not be less than P50,000.00 for each
resolution of April 3, this case was consolidated with G.R. No. 55963. of them.

It appears that on August 21, 1976 at about 6:30 P.M., a pickup owned and operated by respondent 2. The decision of the trial court had made an impression that respondent National
National Irrigation Administration, a government agency bearing Plate No. IN-651, then driven officially Irrigation Administration acted with gross negligence because of the accident and
by Hugo Garcia, an employee of said agency as its regular driver, bumped a bicycle ridden by Francisco the subsequent failure of the National Irrigation Administration personnel
Fontanilla, son of herein petitioners, and Restituto Deligo, at Maasin, San Jose City along the Maharlika including the driver to stop in order to give assistance to the, victims. Thus, by
Highway. As a result of the impact, Francisco Fontanilla and Restituto Deligo were injured and brought to reason of the gross negligence of respondent, petitioners become entitled to
the San Jose City Emergency Hospital for treatment. Fontanilla was later transferred to the Cabanatuan exemplary damages under Arts. 2231 and 2229 of the New Civil Code.
Provincial Hospital where he died.

3. Petitioners are entitled to an award of attorney's fees, the amount of which


Garcia was then a regular driver of respondent National Irrigation Administration who, at the time of the (20%) had been sufficiently established in the hearing of May 23, 1979.
accident, was a licensed professional driver and who qualified for employment as such regular driver of
respondent after having passed the written and oral examinations on traffic rules and maintenance of
vehicles given by National Irrigation Administration authorities. 4. This petition has been filed only for the purpose of reviewing the findings of the
lower court upon which the disallowance of moral damages, exemplary damages
and attorney's fees was based and not for the purpose of disturbing the other The sole legal question on whether or not petitioners may be entitled to an award of moral and
findings of fact and conclusions of law. exemplary damages and attorney's fees can very well be answered with the application of Arts. 2176 and
2180 of theNew Civil Code.
The Solicitor General, taking up the cudgels for public respondent National Irrigation Administration,
contends thus: Art. 2176 thus provides:

1. The filing of the instant petition is rot proper in view of the appeal taken by Whoever by act omission causes damage to another, there being fault or
respondent National Irrigation Administration to the Court of Appeals against the negligence, is obliged to pay for damage done. Such fault or negligence, if there is
judgment sought to be reviewed. The focal issue raised in respondent's appeal to no pre-existing cotractual relation between the parties, is called a quasi-delict and
the Court of Appeals involves the question as to whether or not the driver of the is governed by the provisions of this Chapter
vehicle that bumped the victims was negligent in his operation of said vehicle. It
thus becomes necessary that before petitioners' claim for moral and exemplary
Paragraphs 5 and 6 of Art. 21 80 read as follows:
damages could be resolved, there should first be a finding of negligence on the
part of respondent's employee-driver. In this regard, the Solicitor General alleges
that the trial court decision does not categorically contain such finding. Employers shall be liable for the damages caused by their employees and
household helpers acting within the scope of their assigned tasks, even the though
the former are not engaged in any business or industry.
2. The filing of the "Appearance and Urgent Motion For Leave to File Plaintiff-
Appellee's Brief" dated December 28, 1981 by petitioners in the appeal (CA-G.R.
No. 67237-R; and G. R. No.61045) of the respondent National Irrigation The State is responsible in like manner when it acts through a special agent.; but
Administration before the Court of Appeals, is an explicit admission of said not when the damage has been caused by the official to whom the task done
petitioners that the herein petition, is not proper. Inconsistent procedures are properly pertains, in which case what is provided in Art. 2176 shall be applicable.
manifest because while petitioners question the findings of fact in the Court of
Appeals, they present only the questions of law before this Court which posture The liability of the State has two aspects. namely:
confirms their admission of the facts.

1. Its public or governmental aspects where it is liable for the tortious acts of
3. The fact that the parties failed to agree on whether or not negligence caused special agents only.
the vehicular accident involves a question of fact which petitioners should have
brought to the Court of Appeals within the reglementary period. Hence, the
decision of the trial court has become final as to the petitioners and for this reason 2. Its private or business aspects (as when it engages in private enterprises) where
alone, the petition should be dismissed. it becomes liable as an ordinary employer. (p. 961, Civil Code of the Philippines;
Annotated, Paras; 1986 Ed. ).

4. Respondent Judge acted within his jurisdiction, sound discretion and in


conformity with the law. In this jurisdiction, the State assumes a limited liability for the damage caused by the tortious acts or
conduct of its special agent.

5. Respondents do not assail petitioners' claim to moral and exemplary damages


by reason of the shock and subsequent illness they suffered because of the death Under the aforequoted paragrah 6 of Art. 2180, the State has voluntarily assumed liability for acts done
of their son. Respondent National Irrigation Administration, however, avers that it through special agents. The State's agent, if a public official, must not only be specially commissioned to
cannot be held liable for the damages because it is an agency of the State do a particular task but that such task must be foreign to said official's usual governmental functions. If
performing governmental functions and driver Hugo Garcia was a regular driver of the State's agent is not a public official, and is commissioned to perform non-governmental functions,
the vehicle, not a special agent who was performing a job or act foreign to his then the State assumes the role of an ordinary employer and will be held liable as such for its agent's
usual duties. Hence, the liability for the tortious act should. not be borne by tort. Where the government commissions a private individual for a special governmental task, it is acting
respondent government agency but by driver Garcia who should answer for the through a special agent within the meaning of the provision. (Torts and Damages, Sangco, p. 347, 1984
consequences of his act. Ed.)

6. Even as the trial court touched on the failure or laxity of respondent National Certain functions and activities, which can be performed only by the government, are more or less
Irrigation Administration in exercising due diligence in the selection and generally agreed to be "governmental" in character, and so the State is immune from tort liability. On
supervision of its employee, the matter of due diligence is not an issue in this case the other hand, a service which might as well be provided by a private corporation, and particularly
since driver Garcia was not its special agent but a regular driver of the vehicle. when it collects revenues from it, the function is considered a "proprietary" one, as to which there may
be liability for the torts of agents within the scope of their employment.

The National Irrigation Administration is an agency of the government exercising proprietary functions,
by express provision of Rep. Act No. 3601. Section 1 of said Act provides:
Section 1. Name and domicile.-A body corporate is hereby created which shall be It should be emphasized that the accident happened along the Maharlika National Road within the city
known as the National Irrigation Administration, hereinafter called the NIA for limits of San Jose City, an urban area. Considering the fact that the victim was thrown 50 meters away
short, which shall be organized immediately after the approval of this Act. It shall from the point of impact, there is a strong indication that driver Garcia was driving at a high speed. This
have its principal seat of business in the City of Manila and shall have is confirmed by the fact that the pick-up suffered substantial and heavy damage as above-described and
representatives in all provinces for the proper conduct of its business. the fact that the NIA group was then "in a hurry to reach the campsite as early as possible", as shown by
their not stopping to find out what they bumped as would have been their normal and initial reaction.
Section 2 of said law spells out some of the NIA's proprietary functions. Thus-
Evidently, there was negligence in the supervision of the driver for the reason that they were travelling
at a high speed within the city limits and yet the supervisor of the group, Ely Salonga, failed to caution
Sec. 2. Powers and objectives.-The NIA shall have the following powers and
and make the driver observe the proper and allowed speed limit within the city. Under the situation,
objectives:
such negligence is further aggravated by their desire to reach their destination without even checking
whether or not the vehicle suffered damage from the object it bumped, thus showing imprudence and
(a) x x x x x x x x x x x x x x x x x x reckelessness on the part of both the driver and the supervisor in the group.

(b) x x x x x x x x x x x x x x x x x x Significantly, this Court has ruled that even if the employer can prove the diligence in the selection and
supervision (the latter aspect has not been established herein) of the employee, still if he ratifies the
(c) To collect from the users of each irrigation system constructed by it such fees wrongful acts, or take no step to avert further damage, the employer would still be liable. (Maxion vs.
as may be necessary to finance the continuous operation of the system and Manila Railroad Co., 44 Phil. 597).
reimburse within a certain period not less than twenty-five years cost of
construction thereof; and Thus, too, in the case of Vda. de Bonifacio vs. B.L.T. Bus Co. (L-26810, August 31, 1970, 34 SCRA 618), this
Court held that a driver should be especially watchful in anticipation of others who may be using the
(d) To do all such other tthings and to transact all such business as are directly or highway, and his failure to keep a proper look out for reasons and objects in the line to be traversed
indirectly necessary, incidental or conducive to the attainment of the above constitutes negligence.
objectives.
Considering the foregoing, respondent NIA is hereby directed to pay herein petitioners-spouses the
Indubitably, the NIA is a government corporation with juridical personality and not a mere agency of the amounts of P12,000.00 for the death of Francisco Fontanilla; P3,389.00 for hospitalization and burial
government. Since it is a corporate body performing non-governmental functions, it now becomes liable expenses of the aforenamed deceased; P30,000.00 as moral damages; P8,000.00 as exemplary damages
for the damage caused by the accident resulting from the tortious act of its driver-employee. In this and attorney's fees of 20% of the total award.
particular case, the NIA assumes the responsibility of an ordinary employer and as such, it becomes
answerable for damages. SO ORDERED.

This assumption of liability, however, is predicated upon the existence of negligence on the part of
respondent NIA. The negligence referred to here is the negligence of supervision.

At this juncture, the matter of due diligence on the part of respondent NIA becomes a crucial issue in
determining its liability since it has been established that respondent is a government agency performing
proprietary functions and as such, it assumes the posture of an ordinary employer which, under Par. 5 of
Art. 2180, is responsible for the damages caused by its employees provided that it has failed to observe
or exercise due diligence in the selection and supervision of the driver.

It will be noted from the assailed decision of the trial court that "as a result of the impact, Francisco
Fontanilla wasthrown to a distance 50 meters away from the point of impact while Restituto Deligo was
thrown a little bit further away. The impact took place almost at the edge of the cemented portion of the
road." (Emphasis supplied,) [page 26, Rollo]

The lower court further declared that "a speeding vehicle coming in contact with a person causes force
and impact upon the vehicle that anyone in the vehicle cannot fail to notice. As a matter of fact, the
impact was so strong as shown by the fact that the vehicle suffered dents on the right side of the radiator
guard, the hood, the fender and a crack on the radiator as shown by the investigation report (Exhibit "E").
(Emphasis supplied) [page 29, Rollo]
Republic of the Philippines pensions of retirees, pay and allowances of military and civilian personnel and for maintenance and
SUPREME COURT operations of the Armed Forces of the Philippines, as per Certification dated July 3, 1969 by the AFP
Manila Controller,..." 2. The paragraph immediately succeeding in such petition then alleged: "12. Respondent
Judge, Honorable Guillermo P. Villasor, acted in excess of jurisdiction [or] with grave abuse of discretion
amounting to lack of jurisdiction in granting the issuance of an alias writ of execution against the
SECOND DIVISION
properties of the Armed Forces of the Philippines, hence, the Alias Writ of Execution and notices of
garnishment issued pursuant thereto are null and void." 3 In the answer filed by respondents, through
counsel Andres T. Velarde and Marcelo B. Fernan, the facts set forth were admitted with the only
qualification being that the total award was in the amount of P2,372,331.40. 4
G.R. No. L-30671 November 28, 1973
The Republic of the Philippines, as mentioned at the outset, did right in filing this certiorari and
REPUBLIC OF THE PHILIPPINES, petitioner, prohibition proceeding. What was done by respondent Judge is not in conformity with the dictates of the
vs. Constitution. .
HON. GUILLERMO P. VILLASOR, as Judge of the Court of First Instance of Cebu, Branch I, THE
PROVINCIAL SHERIFF OF RIZAL, THE SHERIFF OF QUEZON CITY, and THE SHERIFF OF THE CITY OF It is a fundamental postulate of constitutionalism flowing from the juristic concept of sovereignty that
MANILA, THE CLERK OF COURT, Court of First Instance of Cebu, P. J. KIENER CO., LTD., GAVINO the state as well as its government is immune from suit unless it gives its consent. It is readily
UNCHUAN, AND INTERNATIONAL CONSTRUCTION CORPORATION, respondents. understandable why it must be so. In the classic formulation of Holmes: "A sovereign is exempt from
suit, not because of any formal conception or obsolete theory, but on the logical and practical ground
Office of the Solicitor General Felix V. Makasiar and Solicitor Bernardo P. Pardo for petitioner. that there can be no legal right as against the authority that makes the law on which the right
depends." 5 Sociological jurisprudence supplies an answer not dissimilar. So it was indicated in a recent
decision, Providence Washington Insurance Co. v. Republic of the Philippines, 6 with its affirmation that "a
Andres T. Velarde and Marcelo B. Fernan for respondents. continued adherence to the doctrine of non-suability is not to be deplored for as against the
inconvenience that may be caused private parties, the loss of governmental efficiency and the obstacle
to the performance of its multifarious functions are far greater if such a fundamental principle were
abandoned and the availability of judicial remedy were not thus restricted. With the well known
propensity on the part of our people to go to court, at the least provocation, the loss of time and energy
FERNANDO, J.: required to defend against law suits, in the absence of such a basic principle that constitutes such an
effective obstacle, could very well be imagined." 7
The Republic of the Philippines in this certiorari and prohibition proceeding challenges the validity of an
order issued by respondent Judge Guillermo P. Villasor, then of the Court of First Instance of Cebu, This fundamental postulate underlying the 1935 Constitution is now made explicit in the revised charter.
Branch I, 1 declaring a decision final and executory and of an alias writ of execution directed against the It is therein expressly provided: "The State may not be sued without its consent." 8 A corollary, both
funds of the Armed Forces of the Philippines subsequently issued in pursuance thereof, the alleged dictated by logic and sound sense from a basic concept is that public funds cannot be the object of a
ground being excess of jurisdiction, or at the very least, grave abuse of discretion. As thus simply and garnishment proceeding even if the consent to be sued had been previously granted and the state
tersely put, with the facts being undisputed and the principle of law that calls for application liability adjudged. Thus in the recent case of Commissioner of Public Highways v. San Diego, 9 such a well-
indisputable, the outcome is predictable. The Republic of the Philippines is entitled to the writs prayed settled doctrine was restated in the opinion of Justice Teehankee: "The universal rule that where the
for. Respondent Judge ought not to have acted thus. The order thus impugned and the alias writ of State gives its consent to be sued by private parties either by general or special law, it may limit
execution must be nullified. claimant's action 'only up to the completion of proceedings anterior to the stage of execution' and that
the power of the Courts ends when the judgment is rendered, since government funds and properties
In the petition filed by the Republic of the Philippines on July 7, 1969, a summary of facts was set forth may not be seized under writs of execution or garnishment to satisfy such judgments, is based on
thus: "7. On July 3, 1961, a decision was rendered in Special Proceedings No. 2156-R in favor of obvious considerations of public policy. Disbursements of public funds must be covered by the
respondents P. J. Kiener Co., Ltd., Gavino Unchuan, and International Construction Corporation, and corresponding appropriation as required by law. The functions and public services rendered by the State
against the petitioner herein, confirming the arbitration award in the amount of P1,712,396.40, subject cannot be allowed to be paralyzed or disrupted by the diversion of public funds from their legitimate and
of Special Proceedings. 8. On June 24, 1969, respondent Honorable Guillermo P. Villasor, issued an Order specific objects, as appropriated by law." 10 Such a principle applies even to an attempted garnishment of
declaring the aforestated decision of July 3, 1961 final and executory, directing the Sheriffs of Rizal a salary that had accrued in favor of an employee. Director of Commerce and Industry v.
Province, Quezon City [as well as] Manila to execute the said decision. 9. Pursuant to the said Order Concepcion, 11 speaks to that effect. Justice Malcolm as ponente left no doubt on that score. Thus: "A
dated June 24, 1969, the corresponding Alias Writ of Execution [was issued] dated June 26, 1969, .... 10. rule which has never been seriously questioned, is that money in the hands of public officers, although it
On the strength of the afore-mentioned Alias Writ of Execution dated June 26, 1969, the Provincial may be due government employees, is not liable to the creditors of these employees in the process of
Sheriff of Rizal (respondent herein) served notices of garnishment dated June 28, 1969 with several garnishment. One reason is, that the State, by virtue of its sovereignty, may not be sued in its own courts
Banks, specially on the "monies due the Armed Forces of the Philippines in the form of deposits except by express authorization by the Legislature, and to subject its officers to garnishment would be to
sufficient to cover the amount mentioned in the said Writ of Execution"; the Philippine Veterans Bank permit indirectly what is prohibited directly. Another reason is that moneys sought to be garnished, as
received the same notice of garnishment on June 30, 1969 .... 11. The funds of the Armed Forces of the long as they remain in the hands of the disbursing officer of the Government, belong to the latter,
Philippines on deposit with the Banks, particularly, with the Philippine Veterans Bank and the Philippine although the defendant in garnishment may be entitled to a specific portion thereof. And still another
National Bank [or] their branches are public funds duly appropriated and allocated for the payment of reason which covers both of the foregoing is that every consideration of public policy forbids it." 12
In the light of the above, it is made abundantly clear why the Republic of the Philippines could rightfully
allege a legitimate grievance.

WHEREFORE, the writs of certiorari and prohibition are granted, nullifying and setting aside both the
order of June 24, 1969 declaring executory the decision of July 3, 1961 as well as the alias writ of
execution issued thereunder. The preliminary injunction issued by this Court on July 12, 1969 is hereby
made permanent.
Republic of the Philippines occupation of the premises from 1970 up to and including 1975, plus interest
SUPREME COURT thereon at the legal rate from January 1970 until fully paid;
Manila
4. ordering the restoration of ownership and possession over the five lots in
FIRST DIVISION question in favor of the plaintiffs in the same proportion aforementioned;

G.R. No. L-61744 June 25, 1984 5. ordering the defendant to pay the plaintiffs the sum of P3,000.00 for attomey's
fees; and to pay the cost of suit.
MUNICIPALITY OF SAN MIGUEL, BULACAN, petitioner,
vs. The counterclaim of the defendant is hereby ordered dismissed for lack of
HONORABLE OSCAR C. FERNANDEZ, in his capacity as the Presiding Judge, Branch IV, Baliuag, Bulacan, evidence presented to substantiate the same.
The PROVINCIAL SHERIFF of Bulacan, MARGARITA D. VDA. DE IMPERIO, ADORACION IMPERIO,
RODOLFO IMPERIO, CONRADO IMPERIO, ERNESTO IMPERIO, ALFREDO IMPERIO, CARLOS IMPERIO, JR.,
SO ORDERED. (pp. 11-12, Rollo)
JUAN IMPERIO and SPOUSES MARCELO PINEDA and LUCILA PONGCO, respondents.

The foregoing judgment became final when herein petitioner's appeal was dismissed due to its failure to
Pascual C. Liatchko for petitioner.
file the record on appeal on time. The dismissal was affirmed by the then Court of Appeals in CA-G.R. No.
SP-12118 and by this Court in G.R. No. 59938. Thereafter, herein private respondents moved for issuance
The Solicitor General and Marcelo Pineda for respondents. of a writ of execution for the satisfaction of the judgment. Respondent judge, on July 27, 1982, issued an
order, to wit:

Considering that an entry of judgment had already been made on June 14, 1982 in
G. R. No. L-59938 and;
RELOVA, J.:

Considering further that there is no opposition to plaintiffs' motion for execution


In Civil Case No. 604-B, entitled "Margarita D. Vda. de Imperio, et al. vs. Municipal Government of San
dated July 23, 1983;
Miguel, Bulacan, et al.", the then Court of First Instance of Bulacan, on April 28, 1978, rendered
judgment holding herein petitioner municipality liable to private respondents, as follows:
Let a writ of execution be so issued, as prayed for in the aforestated motion. (p.
10, Rollo)
WHEREFORE, premises considered, judgment is hereby rendered in favor of the
plaintiffs and against the defendant Municipal Government of San Miguel Bulacan,
represented by Mayor Mar Marcelo G. Aure and its Municipal Treasurer: Petitioner, on July 30, 1982, filed a Motion to Quash the writ of execution on the ground that the
municipality's property or funds are all public funds exempt from execution. The said motion to quash
was, however, denied by the respondent judge in an order dated August 23, 1982 and the alias writ of
1. ordering the partial revocation of the Deed of Donation signed by the deceased
execution stands in full force and effect.
Carlos Imperio in favor of the Municipality of San Miguel Bulacan, dated October
27, 1947 insofar as Lots Nos. 1, 2, 3, 4 and 5, Block 11 of Subdivision Plan Psd-
20831 are concerned, with an aggregate total area of 4,646 square meters, which On September 13, 1982, respondent judge issued an order which in part, states:
lots are among those covered and described under TCT No. T-1831 of the Register
of Deeds of Bulacan in the name of the Municipal Government of San Miguel
It is clear and evident from the foregoing that defendant has more than enough
Bulacan,
funds to meet its judgment obligation. Municipal Treasurer Miguel C, Roura of San
Miguel, Bulacan and Provincial Treasurer of Bulacan Agustin O. Talavera are
2. ordering the defendant to execute the corresponding Deed of Reconveyance therefor hereby ordered to comply with the money judgment rendered by Judge
over the aforementioned five lots in favor of the plaintiffs in the proportion of the Agustin C. Bagasao against said municipality. In like manner, the municipal
undivided one-half () share in the name of plaintiffs Margarita D. Vda. de authorities of San Miguel, Bulacan are likewise ordered to desist from plaintiffs'
Imperio, Adoracion, Rodolfo, Conrado, Ernesto, Alfredo, Carlos, Jr. and Juan, all legal possession of the property already returned to plaintiffs by virtue of the alias
surnamed Imperio, and the remaining undivided one-half () share in favor of writ of execution.
plaintiffs uses Marcelo E. Pineda and Lucila Pongco;
Finally, defendants are hereby given an inextendible period of ten (10) days from
3. ordering the defendant municipality to pay to the plaintiffs in the proportion receipt of a copy of this order by the Office of the Provincial Fiscal of Bulacan
mentioned in the immediately preceding paragraph the sum of P64,440.00 within which to submit their written compliance, (p. 24, Rollo)
corresponding to the rentals it has collected from the occupants for their use and
When the treasurers (provincial and municipal) failed to comply with the order of September 13, 1982, (d) By delivering to the judgment-debtor the excess, if any, unless otherwise,
respondent judge issued an order for their arrest and that they will be release only upon compliance directed by judgment or order of the court.
thereof.
The foregoing has not been followed in the case at bar.
Hence, the present petition on the issue whether the funds of the Municipality of San Miguel, Bulacan, in
the hands of the provincial and municipal treasurers of Bulacan and San Miguel, respectively, are public
ACCORDINGLY, the petition is granted and the order of respondent judge, dated July 27, 1982, granting
funds which are exempt from execution for the satisfaction of the money judgment in Civil Case No. 604-
issuance of a writ of execution; the alias writ of execution, dated July 27, 1982; and the order of
B.
respondent judge, dated September 13, 1982, directing the Provincial Treasurer of Bulacan and the
Municipal Treasurer of San Miguel, Bulacan to comply with the money judgments, are SET ASIDE; and
Well settled is the rule that public funds are not subject to levy and execution. The reason for this was respondents are hereby enjoined from implementing the writ of execution.
explained in the case of Municipality of Paoay vs. Manaois, 86 Phil. 629 "that they are held in trust for
the people, intended and used for the accomplishment of the purposes for which municipal corporations
SO ORDERED.
are created, and that to subject said properties and public funds to execution would materially impede,
even defeat and in some instances destroy said purpose." And, in Tantoco vs. Municipal Council of Iloilo,
49 Phil. 52, it was held that "it is the settled doctrine of the law that not only the public property but also Teehankee (Chairm
the taxes and public revenues of such corporations Cannot be seized under execution against them,
either in the treasury or when in transit to it. Judgments rendered for taxes, and the proceeds of such
judgments in the hands of officers of the law, are not subject to execution unless so declared by statute."
Thus, it is clear that all the funds of petitioner municipality in the possession of the Municipal Treasurer
of San Miguel, as well as those in the possession of the Provincial Treasurer of Bulacan, are also public
funds and as such they are exempt from execution.

Besides, Presidential Decree No. 477, known as "The Decree on Local Fiscal Administration", Section 2
(a), provides:

SEC. 2. Fundamental Principles. Local government financial affairs, transactions,


and operations shall be governed by the fundamental principles set forth
hereunder:

(a) No money shall be paid out of the treasury except in pursuance of a lawful
appropriation or other specific statutory authority.

xxx xxx xxx

Otherwise stated, there must be a corresponding appropriation in the form of an ordinance duly passed
by the Sangguniang Bayan before any money of the municipality may be paid out. In the case at bar, it
has not been shown that the Sangguniang Bayan has passed an ordinance to this effect.

Furthermore, Section 15, Rule 39 of the New Rules of Court, outlines the procedure for the enforcement
of money judgment:

(a) By levying on all the property of the debtor, whether real or personal, not
otherwise exempt from execution, or only on such part of the property as is
sufficient to satisfy the judgment and accruing cost, if he has more than sufficient
property for the purpose;

(b) By selling the property levied upon;

(c) By paying the judgment-creditor so much of the proceeds as will satisfy the
judgment and accruing costs; and
Republic of the Philippines How many of the present Senators can say without remorse in their conscience and with
SUPREME COURT serenity of mind, that they do not owe their victory to electoral robbery? How may?
Manila
The author or authors of the robbery of the records from the said iron safe of the Senate
EN BANC have, perhaps, but followed the example of certain Senators who secured their election
through fraud and robbery.
G.R. No. L-18463 October 4, 1922
The Philippine Senate, in its session of September 9, 1920, adopted a resolution authorizing its
committee on elections and privileges to report as to the action which should be taken with reference to
THE PEOPLE OF THE PHILIPPINE ISLANDS, plaintiff-appellee,
the article published inLa Nacion. On September 15, 1920, the Senate adopted a resolution authorizing
vs.
the President of the Senate to indorse to the Attorney-General, for his study and corresponding action,
GREGORIO PERFECTOR, defendant-appellant.
all the papers referring to the case of the newspaper La Nacion and its editor, Mr. Gregorio Perfecto. As
a result, an information was filed in the municipal court of the City of Manila by an assistant city fiscal, in
Alfonso E. Mendoza and the appellant in behalf of the latter. which the editorial in question was set out and in which it was alleged that the same constituted a
Attorney-General Villa-Real for appellee. violation of article 256 of the Penal Code. The defendant Gregorio Perfecto was found guilty in the
municipal court and again in the Court of First Instance of Manila.

During the course of the trial in the Court of First Instance, after the prosecution had rested, the defense
MALCOLM, J.: moved for the dismissal of the case. On the subject of whether or not article 256 of the Penal Code,
under which the information was presented, is in force, the trial judge, the Honorable George R. Harvey,
said:
The important question is here squarely presented of whether article 256 of the Spanish Penal Code,
punishing "Any person who, by . . . writing, shall defame, abuse, or insult any Minister of the Crown or
other person in authority . . .," is still in force. This antiquated provision was doubtless incorporated into the Penal Code of Spain for the
protection of the Ministers of the Crown and other representatives of the King against free
speech and action by Spanish subjects. A severe punishment was prescribed because it was
About August 20, 1920, the Secretary of the Philippine Senate, Fernando M. Guerrero, discovered that doubtless considered a much more serious offense to insult the King's representative than to
certain documents which constituted the records of testimony given by witnesses in the investigation of insult an ordinary individual. This provision, with almost all the other articles of that Code,
oil companies, had disappeared from his office. Shortly thereafter, the Philippine Senate, having been was extended to the Philippine Islands when under the dominion of Spain because the King's
called into special session by the Governor-General, the Secretary for the Senate informed that body of subject in the Philippines might defame, abuse or insult the Ministers of the Crown or other
the loss of the documents and of the steps taken by him to discover the guilty party. The day following representatives of His Majesty. We now have no Ministers of the Crown or other persons in
the convening of the Senate, September 7, 1920, the newspaper La Nacion, edited by Mr. Gregorio authority in the Philippines representing the King of Spain, and said provision, with other
Perfecto, published an article reading as follows: articles of the Penal Code, had apparently passed into "innocuous desuetude," but the
Supreme Corut of the Philippine Islands has, by a majority decision, held that said article 256
Half a month has elapsed since the discovery, for the first time, of the scandalous robbery of is the law of the land to-day. . . .
records which were kept and preserved in the iron safe of the Senate, yet up to this time
there is not the slightest indication that the author or authors of the crime will ever be The Helbig case is a precedent which, by the rule of stare decisis, is binding upon this court
discovered. until otherwise determined by proper authority.

To find them, it would not, perhaps, be necessary to go out of the Sente itself, and the In the decision rendered by the same judge, he concluded with the following language:
persons in charge of the investigation of the case would not have to display great skill in order
to succeed in their undertaking, unless they should encounter the insuperable obstacle of
offical concealment. In the United States such publications are usually not punishable as criminal offense, and little
importance is attached to them, because they are generally the result of political controversy
and are usually regarded as more or less colored or exaggerated. Attacks of this character
In that case, every investigation to be made would be but a mere comedy and nothing more. upon a legislative body are not punishable, under the Libel Law. Although such publications
are reprehensible, yet this court feels some aversion to the application of the provision of law
After all, the perpetration of the robbery, especially under the circumstances that have under which this case was filed. Our Penal Code has come to us from the Spanish regime.
surrounded it, does not surprise us at all. Article 256 of that Code prescribes punishment for persons who use insulting language about
Ministers of the Crown or other "authority." The King of Spain doubtless left the need of such
protection to his ministers and others in authority in the Philippines as well as in Spain.
The execution of the crime was but the natural effect of the environment of the place in Hence, the article referred to was made applicable here. Notwithstanding the change of
which it was committed. sovereignty, our Supreme Court, in a majority decision, has held that this provision is still in
force, and that one who made an insulting remark about the President of the United States
was punishable under it. (U.S. vs. Helbig, supra.) If it applicable in that case, it would appear expressed either in writing, printing, or by signs or pictures, or the like, or public theatrical
to be applicable in this case. Hence, said article 256 must be enforced, without fear or favor, exhibitions, tending to blacken the memory of one who is dead or to impeach the honesty,
until it shall be repealed or superseded by other legislation, or until the Supreme Court shall virtue, or reputation, or publish the alleged or natural deffects of one who is alive, and
otherwise determine. thereby expose him to public hatred, contempt or ridicule." Section 13 provides that "All laws
and parts of laws now in force, so far as the same may be in conflict herewith, are hereby
repealed. . . ."
In view of the foregoing considerations, the court finds the defendant guilty as charged in the
information and under article 256 of their Penal Code sentences him to suffer two months
and one day of arresto mayorand the accessory penalties prescribed by law, and to pay the That parts of laws in force in 1901 when the Libel Law took effect, were in conflict therewith, and that
costs of both instances. the Libel Law abrogated certain portion of the Spanish Penal Code, cannot be gainsaid. Title X of Book II
of the Penal Code, covering the subjects of calumny and insults, must have been particularly affected by
the Libel Law. Indeed, in the early case of Pardo de Tavera vs. Garcia Valdez ([1902], 1. Phil., 468), the
The fifteen errors assigned by the defendant and appellant, reenforced by an extensive brief, and
Supreme Court spoke of the Libel Law as "reforming the preexisting Spanish law on the subject
eloquent oral argument made in his own behalf and by his learned counsel, all reduce themselves to the
of calumnia and injuria." Recently, specific attention was given to the effect of the Libel Law on the
pertinent and decisive question which was announced in the beginning of this decision.
provisions of the Penal Code, dealing with calumny and insults, and it was found that those provisions of
the Penal Code on the subject of calumny and insults in which the elements of writing an publicity
It will be noted in the first place that the trial judge considered himself bound to follow the rule entered, were abrogated by the Libel Law. (People vs. Castro [1922], p. 842, ante.)
announced in the case of United States vs. Helbig (R. G. No. 14705, 1 not published). In that case, the
accused was charged with having said, "To hell with the President and his proclamations, or words to
The Libel Law must have had the same result on other provisions of the Penal Code, as for instance
that effect," in violation of article 256 of the Penal Code. He was found guilty in a judgment rendered by
article 256.
the Court of First Instance of Manila and again on appeal to the Supreme Court, with the writer of the
instant decision dissenting on two principal grounds: (1) That the accused was deprived of the
constitutional right of cross-examination, and (2) that article 256 of the Spanish Penal Code is no longer The facts here are that the editor of a newspaper published an article, naturally in writing, which may
in force. Subsequently, on a motion of reconsideration, the court, being of the opinion that the Court of have had the tendency to impeach the honesty, virtue, or reputation of members of the Philippine
First Instance had committed a prejudicial error in depriving the accused of his right to cross-examine a Senate, thereby possibly exposing them to public hatred, contempt, or ridicule, which is exactly libel, as
principal witness, set aside the judgment affirming the judgment appealed from and ordered the return defined by the Libel Law. Sir J. F. Stephen is authority for the statement that a libel is indictable when
of the record to the court of origin for the celebration of a new trial. Whether such a trial was actually defaming a "body of persons definite and small enough for individual members to be recognized as such,
had, is not known, but at least, the record in the Helbig case has never again been elevated to this court. in or by means of anything capable of being a libel." (Digest of Criminal Law, art. 267.) But in the United
States, while it may be proper to prosecute criminally the author of a libel charging a legislator with
corruption, criticisms, no matter how severe, on a legislature, are within the range of the liberty of the
There may perchance exist some doubt as to the authority of the decision in the Helbig case, in view of
press, unless the intention and effect be seditious. (3 Wharton's Criminal Law, p. 2131.) With these facts
the circumstances above described. This much, however, is certain: The facts of the Helbig case and the
and legal principles in mind, recall that article 256 begins: Any person who, by . . . writing, shall defame,
case before us, which we may term the Perfecto case, are different, for in the first case there was an oral
abuse, or insult any Minister of the Crown or other person in authority," etc.
defamation, while in the second there is a written defamation. Not only this, but a new point which,
under the facts, could not have been considered in the Helbig case, is, in the Perfecto case, urged upon
the court. And, finally, as is apparent to all, the appellate court is not restrained, as was the trial court, by The Libel Law is a complete and comprehensive law on the subject of libel. The well-known rule of
strict adherence to a former decision. We much prefer to resolve the question before us unhindered by statutory construction is, that where the later statute clearly covers the old subject-matter of antecedent
references to the Helbig decision. acts, and it plainly appears to have been the purpose of the legislature to give expression in it to the
whole law on the subject, previous laws are held to be repealed by necessary implication. (1 Lewis'
Sutherland Statutory Construction, p. 465.) For identical reasons, it is evident that Act No. 277 had the
This is one of those cases on which a variety of opinions all leading to the same result can be had. A
effect so much of this article as punishes defamation, abuse, or insults by writing.
majority of the court are of the opinion that the Philippine Libel Law, Act No. 277, has had the effect of
repealing so much of article 256 of the Penal Code as relates to written defamation, abuse, or insult, and
that under the information and the facts, the defendant is neither guilty of a violation of article 256 of Act No. 292 of the Philippine Commission, the Treason and Sedition Law, may also have affected article
the Penal Code, nor of the Libel Law. The view of the Chief Justice is that the accused should be 256, but as to this point, it is not necessary to make a pronouncement.
acquitted for the reason that the facts alleged in the information do not constitute a violation of article
156 of the Penal Code. Three members of the court believe that article 256 was abrogated completely by
2. Effect of the change from Spanish to Amercian sevoreignty over the Philippine son article
the change from Spanish to American sovereignty over the Philippines and is inconsistent with
256 of the Spanish Penal Code. Appellant's main proposition in the lower court and again
democratic principles of government.
energetically pressed in the appellate court was that article 256 of the Spanish Penal Code is
not now in force because abrogated by the change from Spanish to American sovereignty
Without prejudice to the right of any member of the court to explain his position, we will discuss the two over the Philippines and because inconsistent with democratic principles of government. This
main points just mentioned. view was indirectly favored by the trial judge, and, as before stated, is the opinion of three
members of this court.
1. Effect of the Philippine Libel Law, Act No. 277, on article 256 of the Spanish Penal Code.
The Libel Law, Act No. 277, was enacted by the Philippine Commission shortly after Article 256 is found in Chapter V of title III of Book II of the Spanish Penal Code. Title I of Book II punishes
organization of this legislative body. Section 1 defines libel as a "malicious defamation, the crimes of treason, crimes that endanger the peace or independence of the state, crimes against
international law, and the crime of piracy. Title II of the same book punishes the crimes of lese majeste, codal provisions and a determination of the extent to which they accorded with or were repugnant to
crimes against the Cortesand its members and against the council of ministers, crimes against the form the "'great principles of liberty and law' which had been 'made the basis of our governmental system.' "
of government, and crimes committed on the occasion of the exercise of rights guaranteed by the But when the question has been squarely raised, the appellate court has been forced on occasion to hold
fundamental laws of the state, including crime against religion and worship. Title III of the same Book, in certain portions of the Spanish codes repugnant t democratic institutions and American constitutional
which article 256 is found, punishes the crimes of rebellion, sedition, assaults upon persons in authority, principles. (U.S. vs. Sweet [1901], 1 Phil., 18; U.S. vs. Balcorta [1913], 25 Phil., 273; U.S. vs. Balcorta
and their agents, and contempts, insults, injurias, and threats against persons in authority, and [1913], 25 Phil., 533; Weems vs. U.S., supra.)
insults, injurias, and threats against their agents and other public officers, the last being the title to
Chapter V. The first two articles in Chapter V define and punish the offense of contempt committed by
The nature of the government which has been set up in the Philippines under American sovereignty was
any one who shall be word or deed defame, abuse, insult, or threathen a minister of the crown, or any
outlined by President McKinley in that Magna Charta of Philippine liberty, his instructions to the
person in authority. The with an article condemning challenges to fight duels intervening, comes article
Commission, of April 7, 1900. In part, the President said:
256, now being weighed in the balance. It reads as follows: "Any person who, by word, deed, or writing,
shall defame, abuse, or insult any Minister of the Crown or other person in authority, while engaged in
the performance of official duties, or by reason of such performance, provided that the offensive In all the forms of government and administrative provisions which they are authorized to
minister or person, or the offensive writing be not addressed to him, shall suffer the penalty of arresto prescribe, the Commission should bear in mind that he government which they are
mayor," that is, the defamation, abuse, or insult of any Minister of the Crown of the Monarchy of establishing is designed not for our satisfaction or for the expression of our theoretical views,
Spain (for there could not be a Minister of the Crown in the United States of America), or other person in but for the happiness, peace, and prosperity of the people of the Philippine Islands, and the
authority in the Monarchy of Spain. measures adopted should be made to conform to their customs, their habits, and even their
prejudices, to the fullest extent consistent with the accomplishment of the indispensable
requisites of just and effective government. At the same time the Commission should bear in
It cannot admit of doubt that all those provisions of the Spanish Penal Code having to do with such
mind, and the people of the Islands should be made plainly to understand, that there are
subjects as treason, lese majeste, religion and worship, rebellion, sedition, and contempts of ministers of
certain great principles of government which have been made the basis of our governmental
the crown, are not longer in force. Our present task, therefore, is a determination of whether article 256
system, which we deem essential to the rule of law and the maintenance of individual
has met the same fate, or, more specifically stated, whether it is in the nature of a municipal law or
freedom, and of which they have, unfortunately, been denied the experience possessed by us;
political law, and is consistent with the Constitution and laws of the United States and the characteristics
that there are also certain practical rules of government which we have found to be essential
and institutions of the American Government.
to the preservation of these great principles of liberty and law, and that these principles and
these rules of government must be established and maintained in their islands for the sake of
It is a general principle of the public law that on acquisition of territory the previous political relations of their liberty and happiness, however much they may conflict with the customs or laws of
the ceded region are totally abrogated. "Political" is here used to denominate the laws regulating the procedure with which they are familiar. It is evident that the most enligthened thought of the
relations sustained by the inhabitants to the sovereign. (American Insurance Co. vs. Canter [1828], 1 Pet., Philippine Islands fully appreciates the importance of these principles and rules, and they will
511; Chicago, Rock Island and Pacific Railway Co. vs. McGlinn [1885], 114 U.S., 542; Roa vs. Collector of inevitably within a short time command universal assent.
Customs [1912], 23 Phil., 315.) Mr. Justice Field of the United States Supreme Court stated the obvious
when in the course of his opinion in the case of Chicago, Rock Island and Pacific Railway Co. vs. McGlinn,
The courts have naturally taken the same view. Mr. Justice Elliott, speaking for our Supreme Court, in the
supra, he said: "As a matter of course, all laws, ordinances and regulations in conflict with the political
case of United States vs. Bull ([1910], 15 Phil., 7), said: "The President and Congress framed the
character, institutions and Constitution of the new government are at once displaced. Thus, upon a
government on the model with which American are familiar, and which has proven best adapted for the
cession of political jurisdiction and legislative power and the latter is involved in the former to the
advancement of the public interests and the protection of individual rights and privileges."
United States, the laws of the country in support of an established religion or abridging the freedom of
the press, or authorizing cruel and unusual punishments, and he like, would at once cease to be of
obligatory force without any declaration to that effect." To quote again from the United States Supreme Therefore, it has come with somewhat of a shock to hear the statement made that the happiness, peace,
Court: "It cannot be admitted that the King of Spain could, by treaty or otherwise, impart to the United and prosperity of the people of the Philippine Islands and their customs, habits, and prejudices, to follow
States any of his royal prerogatives; and much less can it be admitted that they have capacity to receive the language of President McKinley, demand obeisance to authority, and royal protection for that
or power to exercise them. Every nation acquiring territory, by treaty or otherwise, must hold it subject authority.
to the Constitution and laws of its own government, and not according to those of the government
ceding it." (Pollard vs. Hagan [1845], 3 Hos., 210.) According to our view, article 256 of the Spanish Penal Code was enacted by the Government of Spain to
protect Spanish officials who were the representatives of the King. With the change of sovereignty, a
On American occupation of the Philippines, by instructions of the President to the Military Commander new government, and a new theory of government, as set up in the Philippines. It was in no sense a
dated May 28, 1898, and by proclamation of the latter, the municipal laws of the conquered territory continuation of the old, although merely for convenience certain of the existing institutions and laws
affecting private rights of person and property and providing for the punishment of crime were were continued. The demands which the new government made, and makes, on the individual citizen
nominally continued in force in so far as they were compatible with the new order of things. But are likewise different. No longer is there a Minister of the Crown or a person in authority of such exalted
President McKinley, in his instructions to General Merritt, was careful to say: "The first effect of the position that the citizen must speak of him only with bated breath. "In the eye of our Constitution and
military occupation of the enemy's territory is the severance of the former political relation of the laws, every man is a sovereign, a ruler and a freeman, and has equal rights with every other man. We
inhabitants and the establishment of a new political power." From that day to this, the ordinarily it has have no rank or station, except that of respectability and intelligence as opposed to indecency and
been taken for granted that the provisions under consideration were still effective. To paraphrase the ignorance, and the door to this rank stands open to every man to freely enter and abide therein, if he is
language of the United States Supreme Court in Weems vs. United States ([1910], 217 U. S., 349), there qualified, and whether he is qualified or not depends upon the life and character and attainments and
was not and could not be, except as precise questions were presented, a careful consideration of the conduct of each person for himself. Every man may lawfully do what he will, so long as it is not malum in
se ormalum prohibitum or does not infringe upon the qually sacred rights of others." (State vs. Shepherd
[1903], 177 Mo., 205; 99 A. S. R., 624.)

It is true that in England, from which so many of the laws and institutions of the United States are
derived, there were once statutes of scandalum magnatum, under which words which would not be
actionable if spoken of an ordinary subject were made actionable if spoken of a peer of the realm or of
any of the great officers of the Crown, without proof of any special damage. The Crown of England,
unfortunately, took a view less tolerant that that of other sovereigns, as for instance, the Emperors
Augustus, Caesar, and Tiberius. These English statutes have, however, long since, become obsolete,
while in the United States, the offense of scandalum magnatum is not known. In the early days of the
American Republic, a sedition law was enacted, making it an offense to libel the Government, the
Congress, or the President of the United States, but the law met with so much popular disapproval, that
it was soon repealed. "In this country no distinction as to persons is recognized, and in practice a person
holding a high office is regarded as a target at whom any person may let fly his poisonous words. High
official position, instead of affording immunity from slanderous and libelous charges, seems rather to be
regarded as making his character free plunder for any one who desires to create a senation by attacking
it." (Newell, Slander and Libel, 3d ed., p. 245; Sillars vs. Collier [1890], 151 Mass., 50; 6 L.R.A., 680.)

Article 256 of the Penal Code is contrary to the genius and fundamental principles of the American
character and system of government. The gulf which separates this article from the spirit which inspires
all penal legislation of American origin, is as wide as that which separates a monarchy from a democratic
Republic like that of the United States. This article was crowded out by implication as soon as the United
States established its authority in the Philippine Islands. Penalties out of all proportion to the gravity of
the offense, grounded in a distorted monarchical conception of the nature of political authority, as
opposed to the American conception of the protection of the interests of the public, have been
obliterated by the present system of government in the Islands. 1awph!l.net

From an entirely different point of view, it must be noted that this article punishes contempts against
executive officials, although its terms are broad enough to cover the entire official class. Punishment for
contempt of non-judicial officers has no place in a government based upon American principles. Our
official class is not, as in monarchies, an agent of some authority greater than the people but it is an
agent and servant of the people themselves. These officials are only entitled to respect and obedience
when they are acting within the scope of their authority and jurisdiction. The American system of
government is calculated to enforce respect and obedience where such respect and obedience is due,
but never does it place around the individual who happens to occupy an official position by mandate of
the people any official halo, which calls for drastic punishment for contemptuous remarks.

The crime of lese majeste disappeared in the Philippines with the ratification of the Treaty of Paris.
Ministers of the Crown have no place under the American flag.

To summarize, the result is, that all the members of the court are of the opinion, although for different
reasons, that the judgment should be reversed and the defendant and appellant acquitted, with costs de
officio. So ordered.
Republic of the Philippines Bakunawa, Anacorita Reyes, Ruperto Reyes, Adela Reyes and
SUPREME COURT Priscilla Reyes as the only children legitimated by the
Manila subsequent marriage of Francisco Reyes Diaz to Irene
Ondez; (2) Declaring the plaintiff Sinforosa R. Bales to have
been an illegitimate child of Francisco Reyes Diaz; (3)
EN BANC
Declaring Lots Nos. 4474, 4475, 4892, 5265, 4803, 4581,
4506 and 1/4 of Lot 1145 as belonging to the conjugal
A.M. No. 133-J May 31, 1982 partnership of the spouses Francisco Reyes Diaz and Felisa
Espiras; (4) Declaring Lot No. 2304 and 1/4 of Lot No. 3416
BERNARDITA R. MACARIOLA, complainant, as belonging to the spouses Francisco Reyes Diaz and Irene
vs. Ondez in common partnership; (5) Declaring that 1/2 of Lot
HONORABLE ELIAS B. ASUNCION, Judge of the Court of First Instance of Leyte, respondent. No. 1184 as belonging exclusively to the deceased Francisco
Reyes Diaz; (6) Declaring the defendant Bernardita R.
Macariola, being the only legal and forced heir of her
mother Felisa Espiras, as the exclusive owner of one-half of
each of Lots Nos. 4474, 4475, 4892, 5265, 4803, 4581, 4506;
MAKASIAR, J: and the remaining one-half (1/2) of each of said Lots Nos.
4474, 4475, 4892, 5265, 4803, 4581, 4506 and one-half (1/2)
of one-fourth (1/4) of Lot No. 1154 as belonging to the
In a verified complaint dated August 6, 1968 Bernardita R. Macariola charged respondent Judge Elias B. estate of Francisco Reyes Diaz; (7) Declaring Irene Ondez to
Asuncion of the Court of First Instance of Leyte, now Associate Justice of the Court of Appeals, with "acts be the exclusive owner of one-half (1/2) of Lot No. 2304 and
unbecoming a judge." one-half (1/2) of one-fourth (1/4) of Lot No. 3416; the
remaining one-half (1/2) of Lot 2304 and the remaining one-
The factual setting of the case is stated in the report dated May 27, 1971 of then Associate Justice Cecilia half (1/2) of one-fourth (1/4) of Lot No. 3416 as belonging to
Muoz Palma of the Court of Appeals now retired Associate Justice of the Supreme Court, to whom this the estate of Francisco Reyes Diaz; (8) Directing the division
case was referred on October 28, 1968 for investigation, thus: or partition of the estate of Francisco Reyes Diaz in such a
manner as to give or grant to Irene Ondez, as surviving
widow of Francisco Reyes Diaz, a hereditary share of. one-
Civil Case No. 3010 of the Court of First Instance of Leyte was a complaint for
twelfth (1/12) of the whole estate of Francisco Reyes Diaz
partition filed by Sinforosa R. Bales, Luz R. Bakunawa, Anacorita Reyes, Ruperto
(Art. 996 in relation to Art. 892, par 2, New Civil Code), and
Reyes, Adela Reyes, and Priscilla Reyes, plaintiffs, against Bernardita R. Macariola,
the remaining portion of the estate to be divided among the
defendant, concerning the properties left by the deceased Francisco Reyes, the
plaintiffs Sinforosa R. Bales, Luz R. Bakunawa, Anacorita
common father of the plaintiff and defendant.
Reyes, Ruperto Reyes, Adela Reyes, Priscilla Reyes and
defendant Bernardita R. Macariola, in such a way that the
In her defenses to the complaint for partition, Mrs. Macariola alleged among other extent of the total share of plaintiff Sinforosa R. Bales in the
things that; a) plaintiff Sinforosa R. Bales was not a daughter of the deceased hereditary estate shall not exceed the equivalent of two-
Francisco Reyes; b) the only legal heirs of the deceased were defendant Macariola, fifth (2/5) of the total share of any or each of the other
she being the only offspring of the first marriage of Francisco Reyes with Felisa plaintiffs and the defendant (Art. 983, New Civil Code), each
Espiras, and the remaining plaintiffs who were the children of the deceased by his of the latter to receive equal shares from the hereditary
second marriage with Irene Ondez; c) the properties left by the deceased were all estate, (Ramirez vs. Bautista, 14 Phil. 528; Diancin vs. Bishop
the conjugal properties of the latter and his first wife, Felisa Espiras, and no of Jaro, O.G. [3rd Ed.] p. 33); (9) Directing the parties, within
properties were acquired by the deceased during his second marriage; d) if there thirty days after this judgment shall have become final to
was any partition to be made, those conjugal properties should first be partitioned submit to this court, for approval a project of partition of the
into two parts, and one part is to be adjudicated solely to defendant it being the hereditary estate in the proportion above indicated, and in
share of the latter's deceased mother, Felisa Espiras, and the other half which is such manner as the parties may, by agreement, deemed
the share of the deceased Francisco Reyes was to be divided equally among his convenient and equitable to them taking into consideration
children by his two marriages. the location, kind, quality, nature and value of the
properties involved; (10) Directing the plaintiff Sinforosa R.
On June 8, 1963, a decision was rendered by respondent Judge Asuncion in Civil Bales and defendant Bernardita R. Macariola to pay the
Case 3010, the dispositive portion of which reads: costs of this suit, in the proportion of one-third (1/3) by the
first named and two-thirds (2/3) by the second named; and
(I 1) Dismissing all other claims of the parties [pp 27-29 of
IN VIEW OF THE FOREGOING CONSIDERATIONS, the Court, Exh. C].
upon a preponderance of evidence, finds and so holds, and
hereby renders judgment (1) Declaring the plaintiffs Luz R.
The decision in civil case 3010 became final for lack of an appeal, and on October While the Court thought it more desirable for all the parties
16, 1963, a project of partition was submitted to Judge Asuncion which is marked to have signed this Project of Partition, nevertheless, upon
Exh. A. Notwithstanding the fact that the project of partition was not signed by the assurance of both counsels of the respective parties to this
parties themselves but only by the respective counsel of plaintiffs and defendant, Court that the Project of Partition, as above- quoted, had
Judge Asuncion approved it in his Order dated October 23, 1963, which for been made after a conference and agreement of the
convenience is quoted hereunder in full: plaintiffs and the defendant approving the above Project of
Partition, and that both lawyers had represented to the
Court that they are given full authority to sign by themselves
The parties, through their respective counsels, presented to
the Project of Partition, the Court, therefore, finding the
this Court for approval the following project of partition:
above-quoted Project of Partition to be in accordance with
law, hereby approves the same. The parties, therefore, are
COMES NOW, the plaintiffs and the defendant in the above- directed to execute such papers, documents or instrument
entitled case, to this Honorable Court respectfully submit sufficient in form and substance for the vesting of the rights,
the following Project of Partition: interests and participations which were adjudicated to the
respective parties, as outlined in the Project of Partition and
l. The whole of Lots Nos. 1154, 2304 and 4506 shall belong the delivery of the respective properties adjudicated to each
exclusively to Bernardita Reyes Macariola; one in view of said Project of Partition, and to perform such
other acts as are legal and necessary to effectuate the said
Project of Partition.
2. A portion of Lot No. 3416 consisting of 2,373.49 square
meters along the eastern part of the lot shall be awarded
likewise to Bernardita R. Macariola; SO ORDERED.

3. Lots Nos. 4803, 4892 and 5265 shall be awarded to Given in Tacloban City, this 23rd day of October, 1963.
Sinforosa Reyes Bales;
(SGD) ELIAS B. ASUNCION Judge
4. A portion of Lot No. 3416 consisting of 1,834.55 square
meters along the western part of the lot shall likewise be EXH. B.
awarded to Sinforosa Reyes-Bales;
The above Order of October 23, 1963, was amended on November 11, 1963, only
5. Lots Nos. 4474 and 4475 shall be divided equally among for the purpose of giving authority to the Register of Deeds of the Province of
Luz Reyes Bakunawa, Anacorita Reyes, Ruperto Reyes, Adela Leyte to issue the corresponding transfer certificates of title to the respective
Reyes and Priscilla Reyes in equal shares; adjudicatees in conformity with the project of partition (see Exh. U).

6. Lot No. 1184 and the remaining portion of Lot No. 3416 One of the properties mentioned in the project of partition was Lot 1184 or rather
after taking the portions awarded under item (2) and (4) one-half thereof with an area of 15,162.5 sq. meters. This lot, which according to
above shall be awarded to Luz Reyes Bakunawa, Anacorita the decision was the exclusive property of the deceased Francisco Reyes, was
Reyes, Ruperto Reyes, Adela Reyes and Priscilla Reyes in adjudicated in said project of partition to the plaintiffs Luz, Anacorita Ruperto,
equal shares, provided, however that the remaining portion Adela, and Priscilla all surnamed Reyes in equal shares, and when the project of
of Lot No. 3416 shall belong exclusively to Priscilla Reyes. partition was approved by the trial court the adjudicatees caused Lot 1184 to be
subdivided into five lots denominated as Lot 1184-A to 1184-E inclusive (Exh. V).
WHEREFORE, it is respectfully prayed that the Project of
Partition indicated above which is made in accordance with Lot 1184-D was conveyed to Enriqueta D. Anota, a stenographer in Judge
the decision of the Honorable Court be approved. Asuncion's court (Exhs. F, F-1 and V-1), while Lot 1184-E which had an area of
2,172.5556 sq. meters was sold on July 31, 1964 to Dr. Arcadio Galapon (Exh. 2)
Tacloban City, October 16, 1963. who was issued transfer certificate of title No. 2338 of the Register of Deeds of the
city of Tacloban (Exh. 12).

(SGD) BONIFACIO RAMO Atty. for the Defendant Tacloban


City On March 6, 1965, Dr. Arcadio Galapon and his wife Sold a portion of Lot 1184-E
with an area of around 1,306 sq. meters to Judge Asuncion and his wife, Victoria S.
Asuncion (Exh. 11), which particular portion was declared by the latter for taxation
(SGD) ZOTICO A. TOLETE Atty. for the Plaintiff Tacloban City purposes (Exh. F).
On August 31, 1966, spouses Asuncion and spouses Galapon conveyed their On November 2, 1970, Judge Jose D. Nepomuceno of the Court of First Instance of Leyte, who was
respective shares and interest in Lot 1184-E to "The Traders Manufacturing and directed and authorized on June 2, 1969 by the then Secretary (now Minister) of Justice and now
Fishing Industries Inc." (Exit 15 & 16). At the time of said sale the stockholders of Minister of National Defense Juan Ponce Enrile to hear and decide Civil Case No. 4234, rendered a
the corporation were Dominador Arigpa Tan, Humilia Jalandoni Tan, Jaime Arigpa decision, the dispositive portion of which reads as follows:
Tan, Judge Asuncion, and the latter's wife, Victoria S. Asuncion, with Judge
Asuncion as the President and Mrs. Asuncion as the secretary (Exhs. E-4 to E-7).
A. IN THE CASE AGAINST JUDGE ELIAS B. ASUNCION
The Articles of Incorporation of "The Traders Manufacturing and Fishing Industries,
Inc." which we shall henceforth refer to as "TRADERS" were registered with the
Securities and Exchange Commission only on January 9, 1967 (Exh. E) [pp. 378-385, (1) declaring that only Branch IV of the Court of First Instance of Leyte has
rec.]. jurisdiction to take cognizance of the issue of the legality and validity of the Project
of Partition [Exhibit "B"] and the two Orders [Exhibits "C" and "C- 3"] approving the
partition;
Complainant Bernardita R. Macariola filed on August 9, 1968 the instant complaint dated August 6, 1968
alleging four causes of action, to wit: [1] that respondent Judge Asuncion violated Article 1491,
paragraph 5, of the New Civil Code in acquiring by purchase a portion of Lot No. 1184-E which was one of (2) dismissing the complaint against Judge Elias B. Asuncion;
those properties involved in Civil Case No. 3010 decided by him; [2] that he likewise violated Article 14,
paragraphs I and 5 of the Code of Commerce, Section 3, paragraph H, of R.A. 3019, otherwise known as (3) adjudging the plaintiff, Mrs. Bernardita R. Macariola to pay defendant Judge
the Anti-Graft and Corrupt Practices Act, Section 12, Rule XVIII of the Civil Service Rules, and Canon 25 of Elias B. Asuncion,
the Canons of Judicial Ethics, by associating himself with the Traders Manufacturing and Fishing
Industries, Inc., as a stockholder and a ranking officer while he was a judge of the Court of First Instance
of Leyte; [3] that respondent was guilty of coddling an impostor and acted in disregard of judicial (a) the sum of FOUR HUNDRED THOUSAND PESOS
decorum by closely fraternizing with a certain Dominador Arigpa Tan who openly and publicly advertised [P400,000.00] for moral damages;
himself as a practising attorney when in truth and in fact his name does not appear in the Rolls of
Attorneys and is not a member of the Philippine Bar; and [4] that there was a culpable defiance of the (b) the sum of TWO HUNDRED THOUSAND PESOS
law and utter disregard for ethics by respondent Judge (pp. 1-7, rec.). [P200,000.001 for exemplary damages;

Respondent Judge Asuncion filed on September 24, 1968 his answer to which a reply was filed on (c) the sum of FIFTY THOUSAND PESOS [P50,000.00] for
October 16, 1968 by herein complainant. In Our resolution of October 28, 1968, We referred this case to nominal damages; and
then Justice Cecilia Muoz Palma of the Court of Appeals, for investigation, report and recommendation.
After hearing, the said Investigating Justice submitted her report dated May 27, 1971 recommending
that respondent Judge should be reprimanded or warned in connection with the first cause of action (d) he sum of TEN THOUSAND PESOS [PI0,000.00] for
alleged in the complaint, and for the second cause of action, respondent should be warned in case of a Attorney's Fees.
finding that he is prohibited under the law to engage in business. On the third and fourth causes of
action, Justice Palma recommended that respondent Judge be exonerated. B. IN THE CASE AGAINST THE DEFENDANT MARIQUITA
VILLASIN, FOR HERSELF AND FOR THE HEIRS OF THE
The records also reveal that on or about November 9 or 11, 1968 (pp. 481, 477, rec.), complainant herein DECEASED GERARDO VILLASIN
instituted an action before the Court of First Instance of Leyte, entitled "Bernardita R. Macariola,
plaintiff, versus Sinforosa R. Bales, et al., defendants," which was docketed as Civil Case No. 4235, (1) Dismissing the complaint against the defendants Mariquita Villasin and the
seeking the annulment of the project of partition made pursuant to the decision in Civil Case No. 3010 heirs of the deceased Gerardo Villasin;
and the two orders issued by respondent Judge approving the same, as well as the partition of the estate
and the subsequent conveyances with damages. It appears, however, that some defendants were
(2) Directing the plaintiff to pay the defendants Mariquita Villasin and the heirs of
dropped from the civil case. For one, the case against Dr. Arcadio Galapon was dismissed because he was
Gerardo Villasin the cost of the suit.
no longer a real party in interest when Civil Case No. 4234 was filed, having already conveyed on March
6, 1965 a portion of lot 1184-E to respondent Judge and on August 31, 1966 the remainder was sold to
the Traders Manufacturing and Fishing Industries, Inc. Similarly, the case against defendant Victoria C. IN THE CASE AGAINST THE DEFENDANT SINFOROSA R.
Asuncion was dismissed on the ground that she was no longer a real party in interest at the time the BALES, ET AL., WHO WERE PLAINTIFFS IN CIVIL CASE NO.
aforesaid Civil Case No. 4234 was filed as the portion of Lot 1184 acquired by her and respondent Judge 3010
from Dr. Arcadio Galapon was already sold on August 31, 1966 to the Traders Manufacturing and Fishing
industries, Inc. Likewise, the cases against defendants Serafin P. Ramento, Catalina Cabus, Ben Barraza (1) Dismissing the complaint against defendants Sinforosa R. Bales, Adela R.
Go, Jesus Perez, Traders Manufacturing and Fishing Industries, Inc., Alfredo R. Celestial and Pilar P. Herrer, Priscilla R. Solis, Luz R. Bakunawa, Anacorita R. Eng and Ruperto O. Reyes.
Celestial, Leopoldo Petilla and Remedios Petilla, Salvador Anota and Enriqueta Anota and Atty. Zotico A.
Tolete were dismissed with the conformity of complainant herein, plaintiff therein, and her counsel.
D. IN THE CASE AGAINST DEFENDANT BONIFACIO RAMO
(1) Dismissing the complaint against Bonifacio Ramo; Galapon for which he was issued TCT No. 2338 by the Register of Deeds of Tacloban City, and on March
6, 1965 he sold a portion of said lot to respondent Judge and his wife who declared the same for taxation
purposes only. The subsequent sale on August 31, 1966 by spouses Asuncion and spouses Galapon of
(2) Directing the plaintiff to pay the defendant Bonifacio Ramo the cost of the suit.
their respective shares and interest in said Lot 1184-E to the Traders Manufacturing and Fishing
Industries, Inc., in which respondent was the president and his wife was the secretary, took place long
SO ORDERED [pp. 531-533, rec.] after the finality of the decision in Civil Case No. 3010 and of the subsequent two aforesaid orders
therein approving the project of partition.
It is further disclosed by the record that the aforesaid decision was elevated to the Court of Appeals
upon perfection of the appeal on February 22, 1971. While it appears that complainant herein filed on or about November 9 or 11, 1968 an action before the
Court of First Instance of Leyte docketed as Civil Case No. 4234, seeking to annul the project of partition
I and the two orders approving the same, as well as the partition of the estate and the subsequent
conveyances, the same, however, is of no moment.

WE find that there is no merit in the contention of complainant Bernardita R. Macariola, under her first
cause of action, that respondent Judge Elias B. Asuncion violated Article 1491, paragraph 5, of the New The fact remains that respondent Judge purchased on March 6, 1965 a portion of Lot 1184-E from Dr.
Civil Code in acquiring by purchase a portion of Lot No. 1184-E which was one of those properties Arcadio Galapon; hence, after the finality of the decision which he rendered on June 8, 1963 in Civil Case
involved in Civil Case No. 3010. 'That Article provides: No. 3010 and his two questioned orders dated October 23, 1963 and November 11, 1963. Therefore, the
property was no longer subject of litigation.

Article 1491. The following persons cannot acquire by purchase, even at a public or
judicial action, either in person or through the mediation of another: The subsequent filing on November 9, or 11, 1968 of Civil Case No. 4234 can no longer alter, change or
affect the aforesaid facts that the questioned sale to respondent Judge, now Court of Appeals Justice,
was effected and consummated long after the finality of the aforesaid decision or orders.
xxx xxx xxx

Consequently, the sale of a portion of Lot 1184-E to respondent Judge having taken place over one year
(5) Justices, judges, prosecuting attorneys, clerks of superior and inferior courts, after the finality of the decision in Civil Case No. 3010 as well as the two orders approving the project of
and other officers and employees connected with the administration of justice, the partition, and not during the pendency of the litigation, there was no violation of paragraph 5, Article
property and rights in litigation or levied upon an execution before the court 1491 of the New Civil Code.
within whose jurisdiction or territory they exercise their respective functions; this
prohibition includes the act of acquiring by assignment and shall apply to lawyers,
with respect to the property and rights which may be the object of any litigation in It is also argued by complainant herein that the sale on July 31, 1964 of Lot 1184-E to Dr. Arcadio
which they may take part by virtue of their profession [emphasis supplied]. Galapon by Priscilla Reyes, Adela Reyes and Luz R. Bakunawa was only a mere scheme to conceal the
illegal and unethical transfer of said lot to respondent Judge as a consideration for the approval of the
project of partition. In this connection, We agree with the findings of the Investigating Justice thus:
The prohibition in the aforesaid Article applies only to the sale or assignment of the property which is the
subject of litigation to the persons disqualified therein. WE have already ruled that "... for the prohibition
to operate, the sale or assignment of the property must take place during the pendency of the litigation And so we are now confronted with this all-important question whether or not the
involving the property" (The Director of Lands vs. Ababa et al., 88 SCRA 513, 519 [1979], Rosario vda. de acquisition by respondent of a portion of Lot 1184-E and the subsequent transfer
Laig vs. Court of Appeals, 86 SCRA 641, 646 [1978]). of the whole lot to "TRADERS" of which respondent was the President and his wife
the Secretary, was intimately related to the Order of respondent approving the
project of partition, Exh. A.
In the case at bar, when the respondent Judge purchased on March 6, 1965 a portion of Lot 1184-E, the
decision in Civil Case No. 3010 which he rendered on June 8, 1963 was already final because none of the
parties therein filed an appeal within the reglementary period; hence, the lot in question was no longer Respondent vehemently denies any interest or participation in the transactions
subject of the litigation. Moreover, at the time of the sale on March 6, 1965, respondent's order between the Reyeses and the Galapons concerning Lot 1184-E, and he insists that
dated October 23, 1963 and the amended order dated November 11, 1963 approving the October 16, there is no evidence whatsoever to show that Dr. Galapon had acted, in the
1963 project of partition made pursuant to the June 8, 1963 decision, had long become final for there purchase of Lot 1184-E, in mediation for him and his wife. (See p. 14 of
was no appeal from said orders. Respondent's Memorandum).

Furthermore, respondent Judge did not buy the lot in question on March 6, 1965 directly from the xxx xxx xxx
plaintiffs in Civil Case No. 3010 but from Dr. Arcadio Galapon who earlier purchased on July 31, 1964 Lot
1184-E from three of the plaintiffs, namely, Priscilla Reyes, Adela Reyes, and Luz R. Bakunawa after the On this point, I agree with respondent that there is no evidence in the record
finality of the decision in Civil Case No. 3010. It may be recalled that Lot 1184 or more specifically one- showing that Dr. Arcadio Galapon acted as a mere "dummy" of respondent in
half thereof was adjudicated in equal shares to Priscilla Reyes, Adela Reyes, Luz Bakunawa, Ruperto acquiring Lot 1184-E from the Reyeses. Dr. Galapon appeared to this investigator
Reyes and Anacorita Reyes in the project of partition, and the same was subdivided into five lots as a respectable citizen, credible and sincere, and I believe him when he testified
denominated as Lot 1184-A to 1184-E. As aforestated, Lot 1184-E was sold on July 31, 1964 to Dr. that he bought Lot 1184-E in good faith and for valuable consideration from the
Reyeses without any intervention of, or previous understanding with Judge sold Lot 1154 on October 22, 1963, it was for no other reason than that she was
Asuncion (pp. 391- 394, rec.). wen aware of the distribution of the properties of her deceased father as per Exhs.
A and B. It is also significant at this point to state that Mrs. Macariola admitted
during the cross-examination that she went to Tacloban City in connection with
On the contention of complainant herein that respondent Judge acted illegally in approving the project
the sale of Lot 1154 to Dr. Decena (tsn p. 92, November 28, 1968) from which we
of partition although it was not signed by the parties, We quote with approval the findings of the
can deduce that she could not have been kept ignorant of the proceedings in civil
Investigating Justice, as follows:
case 3010 relative to the project of partition.

1. I agree with complainant that respondent should have required the signature of
Complainant also assails the project of partition because according to her the
the parties more particularly that of Mrs. Macariola on the project of partition
properties adjudicated to her were insignificant lots and the least valuable.
submitted to him for approval; however, whatever error was committed by
Complainant, however, did not present any direct and positive evidence to prove
respondent in that respect was done in good faith as according to Judge Asuncion
the alleged gross inequalities in the choice and distribution of the real properties
he was assured by Atty. Bonifacio Ramo, the counsel of record of Mrs. Macariola,
when she could have easily done so by presenting evidence on the area, location,
That he was authorized by his client to submit said project of partition, (See Exh. B
kind, the assessed and market value of said properties. Without such evidence
and tsn p. 24, January 20, 1969). While it is true that such written authority if there
there is nothing in the record to show that there were inequalities in the
was any, was not presented by respondent in evidence, nor did Atty. Ramo appear
distribution of the properties of complainant's father (pp. 386389, rec.).
to corroborate the statement of respondent, his affidavit being the only one that
was presented as respondent's Exh. 10, certain actuations of Mrs. Macariola lead
this investigator to believe that she knew the contents of the project of partition, Finally, while it is. true that respondent Judge did not violate paragraph 5, Article 1491 of the New Civil
Exh. A, and that she gave her conformity thereto. I refer to the following Code in acquiring by purchase a portion of Lot 1184-E which was in litigation in his court, it was,
documents: however, improper for him to have acquired the same. He should be reminded of Canon 3 of the Canons
of Judicial Ethics which requires that: "A judge's official conduct should be free from the appearance of
impropriety, and his personal behavior, not only upon the bench and in the performance of judicial
1) Exh. 9 Certified true copy of OCT No. 19520 covering Lot 1154 of the
duties, but also in his everyday life, should be beyond reproach." And as aptly observed by the
Tacloban Cadastral Survey in which the deceased Francisco Reyes holds a "1/4
Investigating Justice: "... it was unwise and indiscreet on the part of respondent to have purchased or
share" (Exh. 9-a). On tills certificate of title the Order dated November 11, 1963,
acquired a portion of a piece of property that was or had been in litigation in his court and caused it to
(Exh. U) approving the project of partition was duly entered and registered on
be transferred to a corporation of which he and his wife were ranking officers at the time of such
November 26, 1963 (Exh. 9-D);
transfer. One who occupies an exalted position in the judiciary has the duty and responsibility of
maintaining the faith and trust of the citizenry in the courts of justice, so that not only must he be truly
2) Exh. 7 Certified copy of a deed of absolute sale executed by Bernardita Reyes honest and just, but his actuations must be such as not give cause for doubt and mistrust in the
Macariola onOctober 22, 1963, conveying to Dr. Hector Decena the one-fourth uprightness of his administration of justice. In this particular case of respondent, he cannot deny that the
share of the late Francisco Reyes-Diaz in Lot 1154. In this deed of sale the vendee transactions over Lot 1184-E are damaging and render his actuations open to suspicion and distrust.
stated that she was the absolute owner of said one-fourth share, the same having Even if respondent honestly believed that Lot 1184-E was no longer in litigation in his court and that he
been adjudicated to her as her share in the estate of her father Francisco Reyes was purchasing it from a third person and not from the parties to the litigation, he should nonetheless
Diaz as per decision of the Court of First Instance of Leyte under case No. 3010 have refrained from buying it for himself and transferring it to a corporation in which he and his wife
(Exh. 7-A). The deed of sale was duly registered and annotated at the back of OCT were financially involved, to avoid possible suspicion that his acquisition was related in one way or
19520 on December 3, 1963 (see Exh. 9-e). another to his official actuations in civil case 3010. The conduct of respondent gave cause for the
litigants in civil case 3010, the lawyers practising in his court, and the public in general to doubt the
In connection with the abovementioned documents it is to be noted that in the honesty and fairness of his actuations and the integrity of our courts of justice" (pp. 395396, rec.).
project of partition dated October 16, 1963, which was approved by respondent
on October 23, 1963, followed by an amending Order on November 11, 1963, Lot II
1154 or rather 1/4 thereof was adjudicated to Mrs. Macariola. It is this 1/4 share in
Lot 1154 which complainant sold to Dr. Decena on October 22, 1963, several days
With respect to the second cause of action, the complainant alleged that respondent Judge violated
after the preparation of the project of partition.
paragraphs 1 and 5, Article 14 of the Code of Commerce when he associated himself with the Traders
Manufacturing and Fishing Industries, Inc. as a stockholder and a ranking officer, said corporation having
Counsel for complainant stresses the view, however, that the latter sold her one- been organized to engage in business. Said Article provides that:
fourth share in Lot 1154 by virtue of the decision in Civil Case 3010 and not
because of the project of partition, Exh. A. Such contention is absurd because from
Article 14 The following cannot engage in commerce, either in person or by
the decision, Exh. C, it is clear that one-half of one- fourth of Lot 1154 belonged to
proxy, nor can they hold any office or have any direct, administrative, or financial
the estate of Francisco Reyes Diaz while the other half of said one-fourth was the
intervention in commercial or industrial companies within the limits of the
share of complainant's mother, Felisa Espiras; in other words, the decision did not
districts, provinces, or towns in which they discharge their duties:
adjudicate the whole of the one-fourth of Lot 1154 to the herein complainant (see
Exhs. C-3 & C-4). Complainant became the owner of the entire one-fourth of Lot
1154 only by means of the project of partition, Exh. A. Therefore, if Mrs. Macariola
1. Justices of the Supreme Court, judges and officials of the department of public On such transfer (by cession) of territory, it has never been
prosecution in active service. This provision shall not be applicable to mayors, held that the relations of the inhabitants with each other
municipal judges, and municipal prosecuting attorneys nor to those who by chance undergo any change. Their relations with their former
are temporarily discharging the functions of judge or prosecuting attorney. sovereign are dissolved, and new relations are created
between them and the government which has acquired their
territory. The same act which transfers their country,
xxx xxx xxx
transfers the allegiance of those who remain in it; and the
law which may be denominated political, is necessarily
5. Those who by virtue of laws or special provisions may not engage in commerce changed, although that which regulates the intercourse and
in a determinate territory. general conduct of individuals, remains in force, until altered
by the newly- created power of the State.
It is Our considered view that although the aforestated provision is incorporated in the Code of
Commerce which is part of the commercial laws of the Philippines, it, however, partakes of the nature of Likewise, in People vs. Perfecto (43 Phil. 887, 897 [1922]), this Court stated that: "It is a general principle
a political law as it regulates the relationship between the government and certain public officers and of the public law that on acquisition of territory the previous political relations of the ceded region are
employees, like justices and judges. totally abrogated. "

Political Law has been defined as that branch of public law which deals with the organization and There appears no enabling or affirmative act that continued the effectivity of the aforestated provision
operation of the governmental organs of the State and define the relations of the state with the of the Code of Commerce after the change of sovereignty from Spain to the United States and then to
inhabitants of its territory (People vs. Perfecto, 43 Phil. 887, 897 [1922]). It may be recalled that political the Republic of the Philippines. Consequently, Article 14 of the Code of Commerce has no legal and
law embraces constitutional law, law of public corporations, administrative law including the law on binding effect and cannot apply to the respondent, then Judge of the Court of First Instance, now
public officers and elections. Specifically, Article 14 of the Code of Commerce partakes more of the Associate Justice of the Court of Appeals.
nature of an administrative law because it regulates the conduct of certain public officers and employees
with respect to engaging in business: hence, political in essence.
It is also argued by complainant herein that respondent Judge violated paragraph H, Section 3 of
Republic Act No. 3019, otherwise known as the Anti-Graft and Corrupt Practices Act, which provides
It is significant to note that the present Code of Commerce is the Spanish Code of Commerce of 1885, that:
with some modifications made by the "Commission de Codificacion de las Provincias de Ultramar," which
was extended to the Philippines by the Royal Decree of August 6, 1888, and took effect as law in this
Sec. 3. Corrupt practices of public officers. In addition to acts or omissions of
jurisdiction on December 1, 1888.
public officers already penalized by existing law, the following shall constitute
corrupt practices of any public officer and are hereby declared to be unlawful:
Upon the transfer of sovereignty from Spain to the United States and later on from the United States to
the Republic of the Philippines, Article 14 of this Code of Commerce must be deemed to have been
xxx xxx xxx
abrogated because where there is change of sovereignty, the political laws of the former sovereign,
whether compatible or not with those of the new sovereign, are automatically abrogated, unless they
are expressly re-enacted by affirmative act of the new sovereign. (h) Directly or indirectly having financial or pecuniary
interest in any business, contract or transaction in
connection with which he intervenes or takes part in his
Thus, We held in Roa vs. Collector of Customs (23 Phil. 315, 330, 311 [1912]) that:
official capacity, or in which he is prohibited by the
Constitution or by any Iaw from having any interest.
By well-settled public law, upon the cession of territory by one nation to another,
either following a conquest or otherwise, ... those laws which are political in their
Respondent Judge cannot be held liable under the aforestated paragraph because there is no showing
nature and pertain to the prerogatives of the former government immediately
that respondent participated or intervened in his official capacity in the business or transactions of the
cease upon the transfer of sovereignty. (Opinion, Atty. Gen., July 10, 1899).
Traders Manufacturing and Fishing Industries, Inc. In the case at bar, the business of the corporation in
which respondent participated has obviously no relation or connection with his judicial office. The
While municipal laws of the newly acquired territory not in conflict with the, laws business of said corporation is not that kind where respondent intervenes or takes part in his capacity as
of the new sovereign continue in force without the express assent or affirmative Judge of the Court of First Instance. As was held in one case involving the application of Article 216 of the
act of the conqueror, the political laws do not. (Halleck's Int. Law, chap. 34, par. Revised Penal Code which has a similar prohibition on public officers against directly or indirectly
14). However, such political laws of the prior sovereignty as are not in conflict with becoming interested in any contract or business in which it is his official duty to intervene, "(I)t is not
the constitution or institutions of the new sovereign, may be continued in force if enough to be a public official to be subject to this crime; it is necessary that by reason of his office, he
the conqueror shall so declare by affirmative act of the commander-in-chief during has to intervene in said contracts or transactions; and, hence, the official who intervenes in contracts or
the war, or by Congress in time of peace. (Ely's Administrator vs. United States, transactions which have no relation to his office cannot commit this crime.' (People vs. Meneses, C.A. 40
171 U.S. 220, 43 L. Ed. 142). In the case of American and Ocean Ins. Cos. vs. 356 O.G. 11th Supp. 134, cited by Justice Ramon C. Aquino; Revised Penal Code, p. 1174, Vol. 11 [1976]).
Bales of Cotton (1 Pet. [26 U.S.] 511, 542, 7 L. Ed. 242), Chief Justice Marshall said:
It does not appear also from the records that the aforesaid corporation gained any undue advantage in Under Section 67 of said law, the power to remove or dismiss judges was then vested in the President of
its business operations by reason of respondent's financial involvement in it, or that the corporation the Philippines, not in the Commissioner of Civil Service, and only on two grounds, namely, serious
benefited in one way or another in any case filed by or against it in court. It is undisputed that there was misconduct and inefficiency, and upon the recommendation of the Supreme Court, which alone is
no case filed in the different branches of the Court of First Instance of Leyte in which the corporation authorized, upon its own motion, or upon information of the Secretary (now Minister) of Justice to
was either party plaintiff or defendant except Civil Case No. 4234 entitled "Bernardita R. Macariola, conduct the corresponding investigation. Clearly, the aforesaid section defines the grounds and
plaintiff, versus Sinforosa O. Bales, et al.,"wherein the complainant herein sought to recover Lot 1184-E prescribes the special procedure for the discipline of judges.
from the aforesaid corporation. It must be noted, however, that Civil Case No. 4234 was filed only on
November 9 or 11, 1968 and decided on November 2, 1970 by CFI Judge Jose D. Nepomuceno when
And under Sections 5, 6 and 7, Article X of the 1973 Constitution, only the Supreme Court can discipline
respondent Judge was no longer connected with the corporation, having disposed of his interest therein
judges of inferior courts as well as other personnel of the Judiciary.
on January 31, 1967.

It is true that under Section 33 of the Civil Service Act of 1959: "The Commissioner may, for ... violation
Furthermore, respondent is not liable under the same paragraph because there is no provision in both
of the existing Civil Service Law and rules or of reasonable office regulations, or in the interest of the
the 1935 and 1973 Constitutions of the Philippines, nor is there an existing law expressly prohibiting
service, remove any subordinate officer or employee from the service, demote him in rank, suspend him
members of the Judiciary from engaging or having interest in any lawful business.
for not more than one year without pay or fine him in an amount not exceeding six months' salary."
Thus, a violation of Section 12 of Rule XVIII is a ground for disciplinary action against civil service officers
It may be pointed out that Republic Act No. 296, as amended, also known as the Judiciary Act of 1948, and employees.
does not contain any prohibition to that effect. As a matter of fact, under Section 77 of said law,
municipal judges may engage in teaching or other vocation not involving the practice of law after office
However, judges cannot be considered as subordinate civil service officers or employees subject to the
hours but with the permission of the district judge concerned.
disciplinary authority of the Commissioner of Civil Service; for, certainly, the Commissioner is not the
head of the Judicial Department to which they belong. The Revised Administrative Code (Section 89) and
Likewise, Article 14 of the Code of Commerce which prohibits judges from engaging in commerce is, as the Civil Service Law itself state that the Chief Justice is the department head of the Supreme Court (Sec.
heretofore stated, deemed abrogated automatically upon the transfer of sovereignty from Spain to 20, R.A. No. 2260) [1959]); and under the 1973 Constitution, the Judiciary is the only other or second
America, because it is political in nature. branch of the government (Sec. 1, Art. X, 1973 Constitution). Besides, a violation of Section 12, Rule XVIII
cannot be considered as a ground for disciplinary action against judges because to recognize the same as
applicable to them, would be adding another ground for the discipline of judges and, as aforestated,
Moreover, the prohibition in paragraph 5, Article 1491 of the New Civil Code against the purchase by
Section 67 of the Judiciary Act recognizes only two grounds for their removal, namely, serious
judges of a property in litigation before the court within whose jurisdiction they perform their duties,
misconduct and inefficiency.
cannot apply to respondent Judge because the sale of the lot in question to him took place after the
finality of his decision in Civil Case No. 3010 as well as his two orders approving the project of partition;
hence, the property was no longer subject of litigation. Moreover, under Section 16(i) of the Civil Service Act of 1959, it is the Commissioner of Civil Service who
has original and exclusive jurisdiction "(T)o decide, within one hundred twenty days, after submission to
it, all administrative cases against permanent officers and employees in the competitive service, and,
In addition, although Section 12, Rule XVIII of the Civil Service Rules made pursuant to the Civil Service
except as provided by law, to have final authority to pass upon their removal, separation, and suspension
Act of 1959 prohibits an officer or employee in the civil service from engaging in any private business,
and upon all matters relating to the conduct, discipline, and efficiency of such officers and employees;
vocation, or profession or be connected with any commercial, credit, agricultural or industrial
and prescribe standards, guidelines and regulations governing the administration of discipline"
undertaking without a written permission from the head of department, the same, however, may not fall
(emphasis supplied). There is no question that a judge belong to the non-competitive or unclassified
within the purview of paragraph h, Section 3 of the Anti-Graft and Corrupt Practices Act because the last
service of the government as a Presidential appointee and is therefore not covered by the aforesaid
portion of said paragraph speaks of a prohibition by the Constitution or law on any public officer from
provision. WE have already ruled that "... in interpreting Section 16(i) of Republic Act No. 2260, we
having any interest in any business and not by a mere administrative rule or regulation. Thus, a violation
emphasized that only permanent officers and employees who belong to the classified service come
of the aforesaid rule by any officer or employee in the civil service, that is, engaging in private business
under the exclusive jurisdiction of the Commissioner of Civil Service" (Villaluz vs. Zaldivar, 15 SCRA
without a written permission from the Department Head may not constitute graft and corrupt practice
710,713 [1965], Ang-Angco vs. Castillo, 9 SCRA 619 [1963]).
as defined by law.

Although the actuation of respondent Judge in engaging in private business by joining the Traders
On the contention of complainant that respondent Judge violated Section 12, Rule XVIII of the Civil
Manufacturing and Fishing Industries, Inc. as a stockholder and a ranking officer, is not violative of the
Service Rules, We hold that the Civil Service Act of 1959 (R.A. No. 2260) and the Civil Service Rules
provissions of Article 14 of the Code of Commerce and Section 3(h) of the Anti-Graft and Corrupt
promulgated thereunder, particularly Section 12 of Rule XVIII, do not apply to the members of the
Practices Act as well as Section 12, Rule XVIII of the Civil Service Rules promulgated pursuant to the Civil
Judiciary. Under said Section 12: "No officer or employee shall engage directly in any private business,
Service Act of 1959, the impropriety of the same is clearly unquestionable because Canon 25 of the
vocation, or profession or be connected with any commercial, credit, agricultural or industrial
Canons of Judicial Ethics expressly declares that:
undertaking without a written permission from the Head of Department ..."

A judge should abstain from making personal investments in enterprises which are
It must be emphasized at the outset that respondent, being a member of the Judiciary, is covered by
apt to be involved in litigation in his court; and, after his accession to the bench, he
Republic Act No. 296, as amended, otherwise known as the Judiciary Act of 1948 and by Section 7, Article
should not retain such investments previously made, longer than a period
X, 1973 Constitution.
sufficient to enable him to dispose of them without serious loss. It is desirable that
he should, so far as reasonably possible, refrain from all relations which would litigants in his court so as to avoid suspicion 'that his social or business relations or
normally tend to arouse the suspicion that such relations warp or bias his friendship constitute an element in determining his judicial course" (par. 30,
judgment, or prevent his impartial attitude of mind in the administration of his Canons of Judicial Ethics), but if a Judge does have social relations, that in itself
judicial duties. ... would not constitute a ground for disciplinary action unless it be clearly shown
that his social relations be clouded his official actuations with bias and partiality in
favor of his friends (pp. 403-405, rec.).
WE are not, however, unmindful of the fact that respondent Judge and his wife had withdrawn on
January 31, 1967 from the aforesaid corporation and sold their respective shares to third parties, and it
appears also that the aforesaid corporation did not in anyway benefit in any case filed by or against it in In conclusion, while respondent Judge Asuncion, now Associate Justice of the Court of Appeals, did not
court as there was no case filed in the different branches of the Court of First Instance of Leyte from the violate any law in acquiring by purchase a parcel of land which was in litigation in his court and in
time of the drafting of the Articles of Incorporation of the corporation on March 12, 1966, up to its engaging in business by joining a private corporation during his incumbency as judge of the Court of First
incorporation on January 9, 1967, and the eventual withdrawal of respondent on January 31, 1967 from Instance of Leyte, he should be reminded to be more discreet in his private and business activities,
said corporation. Such disposal or sale by respondent and his wife of their shares in the corporation only because his conduct as a member of the Judiciary must not only be characterized with propriety but
22 days after the incorporation of the corporation, indicates that respondent realized that early that must always be above suspicion.
their interest in the corporation contravenes the aforesaid Canon 25. Respondent Judge and his wife
therefore deserve the commendation for their immediate withdrawal from the firm after its
WHEREFORE, THE RESPONDENT ASSOCIATE JUSTICE OF THE COURT OF APPEALS IS HEREBY REMINDED
incorporation and before it became involved in any court litigation
TO BE MORE DISCREET IN HIS PRIVATE AND BUSINESS ACTIVITIES.

III
SO ORDERED.

With respect to the third and fourth causes of action, complainant alleged that respondent was guilty of
coddling an impostor and acted in disregard of judicial decorum, and that there was culpable defiance of
the law and utter disregard for ethics. WE agree, however, with the recommendation of the Investigating
Justice that respondent Judge be exonerated because the aforesaid causes of action are groundless, and
WE quote the pertinent portion of her report which reads as follows:

The basis for complainant's third cause of action is the claim that respondent
associated and closely fraternized with Dominador Arigpa Tan who openly and
publicly advertised himself as a practising attorney (see Exhs. I, I-1 and J) when in
truth and in fact said Dominador Arigpa Tan does not appear in the Roll of
Attorneys and is not a member of the Philippine Bar as certified to in Exh. K.

The "respondent denies knowing that Dominador Arigpa Tan was an "impostor"
and claims that all the time he believed that the latter was a bona fide member of
the bar. I see no reason for disbelieving this assertion of respondent. It has been
shown by complainant that Dominador Arigpa Tan represented himself publicly as
an attorney-at-law to the extent of putting up a signboard with his name and the
words "Attorney-at Law" (Exh. I and 1- 1) to indicate his office, and it was but
natural for respondent and any person for that matter to have accepted that
statement on its face value. "Now with respect to the allegation of complainant
that respondent is guilty of fraternizing with Dominador Arigpa Tan to the extent
of permitting his wife to be a godmother of Mr. Tan's child at baptism (Exh. M &
M-1), that fact even if true did not render respondent guilty of violating any canon
of judicial ethics as long as his friendly relations with Dominador A. Tan and family
did not influence his official actuations as a judge where said persons were
concerned. There is no tangible convincing proof that herein respondent gave any
undue privileges in his court to Dominador Arigpa Tan or that the latter benefitted
in his practice of law from his personal relations with respondent, or that he used
his influence, if he had any, on the Judges of the other branches of the Court to
favor said Dominador Tan.

Of course it is highly desirable for a member of the judiciary to refrain as much as


possible from maintaining close friendly relations with practising attorneys and
Republic of the Philippines under their legitimate Constitution. And he cites, in support of this last proposition, the decisions of the
SUPREME COURT Supreme Court of the United States in the cases of Texas vs. White (7 Wall., 700, 743); Horn vs. Lockart
Manila (17 Wall., 570, 581); United States vs. Home Insurance Co. (22 Wall., 99, 104); Sprott vs.United States (20
Wall., 459).
EN BANC
The City Fiscal of Manila appeared before this Court as amicus curiae. In his memorandum he submits
that the petition for habeas corpus be denied on the following grounds: That the Court of Special and
G.R. No. L-49 November 12, 1945
Exclusive Criminal Jurisdiction and the Acts, Ordinances and Executive Orders, creating it are not of a
political complexion, for said Court was created, and the crimes and offenses placed under its jurisdiction
WILLIAM F. PERALTA, petitioner, were penalized heavily, in response to an urgent necessity, according to the preamble of Ordinance No.
vs. 7; that the right to appeal in a criminal case is not a constitutional right; and that the summary
THE DIRECTOR OF PRISONS, respondent. procedure established in said Ordinance No. 7 is not violative of the provision of Article III, section 1 (18)
of the Constitution of the Commonwealth, to the effect that no person shall be compelled to be a
William F. Peralta in his own behalf. witness against himself, nor of the provision of section 1 (1) of the same Article that no person shall be
Office of the Solicitor General Taada for respondent. deprived of life, liberty, or property without due process of law.
City Fiscal Mabanag as amicus curiae.
The features of the summary procedure adopted by Ordinance No. 7, assailed by the petitioner and the
FERIA, J.: Solicitor General as impairing the constitutional rights of an accused are: that court may interrogate the
accused and witnesses before trial in order to clarify the points in dispute; that the refusal of the accused
to answer the questions may be considered unfavorable to him; that if from the facts admitted at the
Petitioner-defendant, a member of the Metropolitan Constabulary of Manila charged with the preliminary interrogatory it appears that the defendant is guilty, he may be immediately convicted; and
supervision and control of the production, procurement and distribution of goods and other necessaries that the sentence of the sentence of the court is not appealable, except in case of death penalty which
as defined in section 1 of Act No. 9 of the National Assembly of the so-called Republic of the Philippines, cannot be executed unless and until reviewed and affirmed by a special division of the Supreme Court
was prosecuted for the crime of robbery as defined and penalized by section 2 (a) of Act No. 65 of the composed of three Justices.
same Assembly. He was found guilty and sentenced to life imprisonment, which he commenced to serve
on August 21, 1944, by the Court of Special and Exclusive Criminal Jurisdiction, created in section 1 of
Ordinance No. 7 promulgated by the President of the so-called Republic of the Philippines, pursuant to Before proceeding further, and in order to determine the law applicable to the questions involved in the
the authority conferred upon him by the Constitution and laws of the said Republic. And the procedure present case, it is necessary to bear in mind the nature and status of the government established in
followed in the trial was the summary one established in Chapter II of Executive Order No. 157 of the these Islands by the Japanese forces of occupation under the designation of Republic of the Philippines.
Chairman of the Executive Commission, made applicable to the trial violations of said Act No. 65 by
section 9 thereof and section 5 of said Ordinance No. 7. In the case of Co Kim Cham vs. Valdez Tan Keh and Dizon (G. R. No. L-5, pp. 113, 127, ante), recently
decided, this Court, speaking through the Justice who pens this decision, held:
The petition for habeas corpus is based on the ground that the Court of Special and Executive Criminal
Jurisdiction created by Ordinance No. 7 "was a political instrumentality of the military forces of the In view of the foregoing, it is evident that the Philippines Executive Commission, which was
Japanese Imperial Army, the aims and purposes of which are repugnant to those aims and political organized by Order No. 1, issued on January 23, 1942, by the Commander of the Japanese
purposes of the Commonwealth of the Philippines, as well as those of the United States of America, and forces, was a civil government established by the military forces of occupation and therefore
therefore, null and void ab initio," that the provisions of said Ordinance No. 7 are violative of the a de facto government of the second kind. It was not different from the government
fundamental laws of the Commonwealth of the Philippines and "the petitioner has been deprived of his established by the British in Castine, Maine, or by the United States in Tanpico, Mexico. As
constitutional rights"; that the petitioner herein is being punished by a law created to serve the political Halleck says, "the government established over an enemy's territory during the military
purpose of the Japanese Imperial Army in the Philippines, and "that the penalties provided for are much occupation may exercise all the powers given by the laws of war to the conqueror over the
(more) severe than the penalties provided for in the Revised Penal Code." conquered, and is subject to all restrictions which that code imposes. It is of little
consequence whether such government be called a military or civil government. Its character
The Solicitor General, in his answer in behalf of the respondent, states that, in his own opinion, for the is the same and the source of its authority the same. In either case it is a government
reasons expressed in his brief in the case of People of the Philippines, plaintiff-appellant, vs. Benedicto imposed by the laws of war and so far as it concerns the inhabitants of such territory or the
Jose y Santos, defendant-appellee, G. R. No. L-22 (p. 612, post), the acts and proceedings taken and had rest of the world those laws alone determine the legality or illegality of its acts." (vol. 2 p.
before the said Court of Special and Exclusive Criminal Jurisdiction which resulted in the conviction and 466.) The fact that the Philippine Executive Commission was a civil and not a military
imprisonment of the herein petitioner, should now be denied force and efficacy, and therefore the government and was run by Filipinos and not by Japanese nationals is of no consequence.
petition for habeas corpus should be granted. The reasons advanced by the Solicitor General in said brief
and in his reply memorandum in support of his contention are, that the Court of Special and Exclusive And speaking of the so-called Republic of the Philippines in the same decision, this Court said:
Criminal Jurisdiction created, and the summary procedure prescribed therefor, by said Ordinance No. 7
in connection with Executive Order No. 157 of the Chairman of the Executive Commission are tinged
The so-called Republic of the Philippines, apparently established and organized as a sovereign
with political complexion; that the procedure prescribed in Ordinance No. 7 does not afford a fair trial,
state independent from any other government by the Filipino people, was, in truth and
violates the Constitution of the Commonwealth, and impairs the Constitutional rights of accused persons
reality, a government established by the belligerent occupant or the Japanese forces of
occupation. It was of the same character as the Philippine Executive Commission, and the the Confederate States, the constitution of each state and that of the United States or the Union
ultimate source of its authority was the same the Japanese military authority and continued in force in those states during the War of Secession; while the Constitution of the
government. As General MacArthur stated in his proclamation of October 23, 1944, a portion Commonwealth Government was suspended during the occupation of the Philippines by the Japanese
of which has been already quoted, "under enemy duress, a so-called government styled as forces of the belligerent occupant at regular war with the United States.
the 'Republic of the Philippines' was established on October 14, 1943, based upon neither the
free expression of the peoples" will nor the sanction of the Government of the United States.'
The question which we have to resolve in the present case in the light of the law of nations are, first, the
Japan had no legal power to grant independence to the Philippines or transfer the sovereignty
validity of the creation of the Court of Special and Exclusive Criminal Jurisdiction, and of the summary
of the United States to, or recognize the latent sovereignty of the Filipino people, before its
procedure adopted for that court; secondly, the validity of the sentence which imprisonment during the
military occupation and possession of the Islands had matured into an absolute and
Japanese military occupation; and thirdly, if they were then valid, the effect on said punitive sentence of
permanent dominion or sovereignty by a treaty of peace or other means recognized in the
the reoccupation of the Philippines and the restoration therein of the Commonwealth Government.
law of nations.

(1) As to the validity of the creation of the Court of Special and Exclusive Criminal Jurisdiction by
As the so-called Republic of the Philippines was a de facto government of the second kind (of paramount
Ordinance No. 7, the only factor to be considered is the authority of the legislative power which
force), as the government established in Castine, Maine, during its occupation by the British forces and
promulgated said law or ordinance. It is well established in International Law that "The criminal
as that of Tampico, Mexico, occupied during the war with that the country by the United State Army, the
jurisdiction established by the invader in the occupied territory finds its source neither in the laws of the
question involved in the present case cannot be decided in the light of the Constitution of the
conquering or conquered state, it is drawn entirely form the law martial as defined in the usages of
Commonwealth Government; because the belligerent occupant was totally independent of the
nations. The authority thus derived can be asserted either through special tribunals, whose authority and
constitution of the occupied territory in carrying out the administration over said territory; and the
procedure is defined in the military code of the conquering state, or through the ordinary courts and
doctrine laid down by the Supreme Court of the United States in the cases involving the validity of
authorities of the occupied district." (Taylor, International Public Law, p. 598.) The so-called Republic of
judicial and legislative acts of the Confederate States, considered as de factogovernments of the third
the Philippines, being a governmental instrumentality of the belligerent occupant, had therefore the
kind, does not apply to the acts of the so-called Republic of the Philippines which is a de
power or was competent to create the Court of Special and Exclusive Criminal Jurisdiction. No question
facto government of paramount force. The Constitution of the so-called Republic of the Philippines can
may arise as to whether or not a court is of political complexion, for it is mere a governmental agency
neither be applied, since the validity of an act of a belligerent occupant cannot be tested in the light of
charged with the duty of applying the law to cases falling within its jurisdiction. Its judgments and
another act of the same occupant, whose criminal jurisdiction is drawn entirely from the law martial as
sentences may be of political complexion, or not depending upon the nature or character of the law so
defined in the usages of nations.
applied. There is no room for doubt, therefore, as to the validity of the creation of the court in question.

In the case of United States vs. Rice (4 Wheaton, 246), the Supreme Court of the United States held that,
With respect to the Summary procedure adopted by Ordinance No. 7, and followed in the trial of the
by the military occupation of Castine, Maine, the sovereignty of the United States in the territory was, of
case which resulted in the conviction of the herein petitioner, there is also no question as to the power
course, suspended, and the laws of the United States could no longer be rightfully enforced there or be
or competence of the belligerent occupant to promulgate the law providing for such procedure. For "the
obligatory upon the inhabitants who remained and submitted to the belligerent occupant. By the
invader deals freely with the relations of the inhabitants of the occupied territory towards himself . . . for
surrender the inhabitants passed under a temporary allegiance to the British government, and were
his security also, he declares certain acts, not forbidden by the ordinary laws of the country, to be
bound by such laws, and such only, as it chose to recognize and impose. And Oppenheim, in his Treatise
punishable; and he so far suspends the laws which guard personal liberty as is required for the summary
on International Law, says that, in carrying out the administration over the occupied territory and its
punishment of any one doing such acts." (Hall's International Law, seventh ed., p. 5000). A belligerent
inhabitants, "the (belligerent) occupant is totally independent of the constitution and the laws of the
"occupant may where necessary, set up military courts instead of the ordinary courts; and in case, and in
territory, since occupation is an aim of warfare, and the maintenance and safety of his forces, and the
so far as, he admits the administration of justice by the ordinary courts, he may nevertheless, so far as is
purpose of war, stand in the foreground of his interest and must be promoted under all circumstances or
necessary for military purposes, or for the maintenance of public order and safetytemporarily alter the
conditions. (Vol. II, Sixth Edition, Revised, 1944, p. 342.)
laws, especially the Criminal Law, on the basis of which justice is administered as well as the laws
regarding procedure." (Oppenheim's International Law, Vol. II, sixth edition, 1944, p.349.)
The doctrine laid down in the decisions of the Supreme Court of the United States (in the cases of
Texas vs.White, 7 Wall., 700; Horn vs. Lockart, 17 Wall., 570; Williams vs. Bruffy, 96 U. S., 176 United
No objection can be set up to the legality of its provisions in the light of the precepts of our
States vs. Home Insurance Co., 20 Wall., 249; Sprott vs. United States, 20 Wall., 459, and others) that the
Commonwealth Constitution relating to the rights of accused under that Constitution, because the latter
judicial and legislative acts of the Confederate States which impaired the rights of the citizens under the
was not in force during the period of the Japanese military occupation, as we have already stated. Nor
Constitution of the United States or of the States, or were in conflict with those constitutions, were null
may said Constitution be applied upon its revival at the time of the re-occupation of the Philippines by
and void, is not applicable to the present case. Because that doctrine rests on the propositions that "the
virtue of the principle of postliminium because "a constitution should operate prospectively only, unless
concession (of belligerency) made to the Confederate Government . . . sanctioned no hostile legislation .
the words employed show a clear intention that it should have a retrospective effect" (Cooley's
. . and it impaired in no respect the rights of loyal and citizens as they existed at the commencement of
Constitutional Limitations, seventh edition, page 97, and cases quoted and cited in the footnote),
hostilities" (Williams vs. Bruffy, supra);that the Union is perpetual and indissoluble, and the obligation of
especially as regards laws of procedure applied to cases already terminated completely.
allegiance to the to the estate and obedience to her laws and the estate constitution, subject to the
Constitution of the United States, remained unimpaired during the War of Secession
(Texas vs. White, supra) and that the Confederate States "in most, if not in all instances, merely The only restrictions or limitations imposed upon the power of a belligerent occupant to alter the laws or
transferred the existing state organizations to the support of a new and different national head. the promulgate new ones, especially the criminal law as well as the laws regarding procedure, so far as it is
same constitution, the same laws for the protection of the property and personal rights remained and necessary for military purposes, that is, for his control of the territory and the safety and protection of
were administered by the same officers." (Sprott vs. United States, supra). In fine, because in the case of his army, are those imposed by the Hague Regulations, the usages established by civilized nations, the
laws of humanity and the requirements of public conscience. It is obvious that the summary procedure
under consideration does not violate those precepts. It cannot be considered as violating the laws of They are not the same ordinary offenses penalized by the Revised Penal Code. The criminal acts
humanity and public conscience, for it is less objectionable, even from the point of view of those who are penalized by said Act No. 65 are those committed by persons charged or connected with the supervision
used to the accusatory system of criminal procedure than the procedural laws based on the semi- and control of the production, procurement and distribution of foods and other necessaries; and the
inquisitorial or mixed system prevailing in France and other countries in continental Europe. penalties imposed upon the violators are different from and much heavier than those provided by the
Revised Penal Code for the same ordinary crimes. The acts penalized by said Act were taken out of the
territorial law or Revised Penal Code, and referred to what is called martial law by international jurists,
(2) The validity of the sentence rendered by the Court of Special and Exclusive Criminal Jurisdiction which
defined above by Hyde, in order, not only to prevent food and other necessaries from reaching the
imposes life imprisonment upon the herein petitioner, depends upon the competence or power of the
"guerrillas" which were harassing the belligerent occupant from every nook and corner of the country,
belligerent occupant to promulgate Act No. 65 which punishes the crime of which said petitioner was
but also to preserve the food supply and other necessaries in order that, in case of necessity, the
convicted.
Imperial Japanese forces could easily requisition them, as they did, and as they had the right to do in
accordance with the law of nations for their maintenance and subsistence (Art. LII, Sec. III, Hague
Westlake says that Article XLIII, Section III, of the Hague Conventions of 1907 "indicates that the laws to Conventions of 1907). Especially taking into consideration the fact, of which this court may take judicial
be enforced by the occupant consist of, first, the territorial law in general, as that which stands to the notice, that the Imperial Japanese Army had depended mostly for their supply upon the produce of this
public order and social and commercial life of the district in a relation of mutual adaptation, so that any country.
needless displacement of it would defeat the object which the invader is enjoined to have in view, and
secondly, such variations of the territorial law as may be required by real necessity and are not expressly
The crimes penalized by Act No. 65 as well as the crimes against national security and the law of
prohibited by any of the rules which will come before us. Such variations will naturally be greatest in
nations, to wit: treason, espionage, inciting war, violation of neutrality, correspondence with hostile
what concerns the relation of the communities and individuals within the district to the invading army
country, flight to enemy's country, piracy; and the crimes against public order, such as rebellion, sedition
and its followers, it being necessary for the protection of the latter, and for the unhindered prosecution
and disloyalty, illegal possession of firearms and other, penalized by Ordinance No. 7 and placed under
of the war by them, that acts committed to their detriment shall not only lose what justification the
jurisdiction of the Court of Special and Exclusive Criminal Jurisdiction are all of a political complexion,
territorial law might give them as committed against enemies, but shall be repressed more severely than
because the acts constituting those offenses were punished, as are all political offenses, for public rather
the territorial law would repress acts committed against fellow subjects. Indeed the entire relation
than private reasons, and were acts in aid or favor of the enemy and against the welfare, safety and
between the invaders and the invaded, so far as it may fall within the criminal department whether by
security of the belligerent occupant. While it is true that these offenses, when committed against the
the intrinsic nature of the acts done or in consequence of the regulations made by the invaders, may be
Commonwealth or United States Government, are defined and also penalized by the territorial law
considered as taken out of the territorial law and referred to what is called martial law." (Westlake,
Revised Penal Code, they became inapplicable as crimes against the occupier upon the occupation of the
International Law, Part II, War, p. 96.)
Islands by the Japanese forces. And they had to be taken out of the territorial law and made punishable
by said Ordinance No. 7, for they were not penalized before under the Revised Penal Code when
According to Hyde (International Law, Vol. II, p. 386), the term "martial law," in so far as it is used to committed against the belligerent occupant or the government established by him in these Island. They
describe any fact in relation to belligerent occupation, does not refer to a particular code or system of are also considered by some writers as war crimes in a broad sense. In this connection Wheaton
law, or to a special agency entrusted with its administration. The term merely signifies that the body of observes the following:
law actually applied, having the sanction of military authority, is essentially martial. All law, by
whomsoever administered, in an occupied district martial law; and it is none the less so when applied by
"Of 'war crimes' the number is naturally indefinite, depending as they do on the acts from time to time
civil courts in matters devoid of special interest to the occupant. The words "martial law" are doubtless
ordered to be done or forbidden to be done in the martial law proclamation or regulations of the
suggestive of the power of the occupant to share the law as he sees fit; that is, to determine what shall
invading or occupying commander. Thus, in the Anglo-Boer war, the British military authorities
be deemed lawful or unlawful acts, to establish tests for ascertaining the guilt of offenders, to fix
proclaimed the following to be offenses against their martial law; Being in possession of arms,
penalties, and generally to administer justice through such agencies as the found expedient.
ammunition, etc.; traveling without a permit; sending prohibited goods; holding meetings other than
those allowed; using seditious language; spreading alarmist reports; overcharging for goods; wearing
And the United States Rules of Land Warfare provide that the belligerent occupant may promulgate such uniforms without due authority; going out of doors between certain hours; injuring military animals or
new laws and regulations as military necessity demands, and in this class will be included those laws stores; being in possession, without a permit, of horses, vehicles, cycles, etc.; hindering those in
which come into being as a result of military rule; that is, those which establish new crimes and offenses execution of military orders; trespassing on defense works. Such offenses, together with several others,
incident to a state of war and are necessary for the control of the country and the protection of the were specified in the Japanese regulations made in the Russo-Japanese war." (Wheaton's International
army, for the principal object of the occupant is to provide for the security of the invading army and to Law, War, seventh edition, 1944, p. 242.)
contribute to its support and efficiency and the success of its operations. (Pub. 1940, pp. 76, 77.)
It is, therefore, evident that the sentence rendered by the Court of Special and Exclusive Criminal
From the above it appears clear that it was within the power and competence of the belligerent Jurisdiction against the petitioner, imposing upon him the penalty of life imprisonment, was good and
occupant to promulgate, through the National Assembly of the so-called Republic of the Philippines, Act valid, since it was within the admitted power or competence of the belligerent occupant to promulgate
No. 65 of the said Assembly, which penalizes the crimes of robbery and other offenses by imprisonment the law penalizing the crime of which petitioner was convicted.
ranging from the maximum period of the imprisonment prescribed by the laws and ordinances
promulgated by the President of the so-called Republic as minimum, to life imprisonment or death as
(3) The last question is the legal effect of the reoccupation of the Philippines and restoration of the
maximum. Although these crimes are defined in the Revised Penal Code, they were altered and
Commonwealth Government; that is whether or not, by the principle of postliminy, the punitive
penalized by said Act No. 65 with different and heavier penalties, as new crimes and offenses demanded
sentence which petitioner is now serving fell through or ceased to be valid from that time.
by military necessity, incident to a state of war, and necessary for the control of the country by the
belligerent occupant, the protection and safety of the army of occupation, its support and efficiency, and
the success of its operations.
In order to resolve this last question, it is not necessary to enter into an elaborate discussion on the petitioner under said law, a sentence which, before the proclamation, had already become null and of no
matter. It is sufficient to quote the opinion on the subject of several international jurists and our recent effect.
decision in the case ofCo Kim Cham vs. Valdez Tan Keh and Dizon, supra.
We therefore hold that the punitive sentence under consideration, although good and valid during the
Hall, commenting on the effect of the principle of postliminy upon sentences of the tribunals continued military occupation of the Philippines by the Japanese forces, ceased to be good and valid ipso
or created by the belligerent occupant, opines "that judicial acts done under this control, when they are facto upon the reoccupation of these Island and the restoration therein of the Commonwealth
not of a political complexion, administrative acts so done, to the extent that they take effect during the Government.
continuance of his control, and the various acts done during the same time by private persons under the
sanction of municipal law, remain good. . . . Political acts on the other hand fall through as of course,
In view of all the foregoing, the writ of habeas corpus prayed for is hereby granted and it is ordered that
whether they introduce any positive change into the organization of the country, or whether they only
the petitioner be released forthwith, without pronouncement as to costs. So ordered.
suspend the working of that already in existence. The execution also of punitive sentences ceases as of
course when they have had reference to acts not criminal by the municipal law of the state, such for
example as acts directed against the security or control of the invader." (Hall's International Law, seventh Jaranilla, Pablo and Bengzon, JJ., concur.
edition, p. 518.) Moran, C.J., concurs in the result.

Westlake, speaking of the duration of the validity of punitive sentences for offenses such as the one in
question, which is within the admitted power or competence of the belligerent occupant to punish, says
that: "To the extent to which the legal power of the occupant is admitted he can make law for the
duration of his occupation. Like any other legislator he is morally subject to the duty of giving sufficient
notice of his enactments or regulations, not indeed so as to be debarred from carrying out his will
without notice, when required by military necessity and so far as practically carrying out his will can be
distinguished from punishment, but always remembering that to punish for breach of a regulation a
person who was justifiably ignorant of it would be outrageous. But the law made by the occupant within
his admitted power, whether morally justifiable or not, will bind any member of the occupied population
as against any other member of it, and will bind as between them all and their national government, so
far as it produces an effect during the occupation. When the occupation comes to an end the authority
of the national government is restored, either by the progress of operations during the war or by the
conclusion of a peace, no redress can be had for what has been actually carried out but nothing further
can follow from the occupant's legislation. A prisoner detained under it must be released, and no civil
right conferred by it can be further enforced. The enemy's law depends on him for enforcement as well
as for enactment. The invaded state is not subject to the indignity of being obliged to execute his
commands. (Westlake, International Law, Part II, War, pp. 97, 98.)

And Wheaton, who, as above stated, considers as war crimes such offenses as those penalized in
Ordinance No. 7 and Act No. 65, says: "In general, the cast of the occupant possess legal validity, and
under international law should not be abrogated by the subsequent government. But this rule does not
necessarily apply to acts that exceed the occupant's power (e.g., alienation of the domains of the State
or the sovereign), to sentences for 'war treason' and 'war crimes,' to acts of a political character, and to
those that beyond the period of occupation. When occupation ceases, no reparation is legally due for
what has already been carried out." (Wheaton's International Law, supra, p. 245.)

We have already held in our recent decision in the case of Co Kim Cham vs. Valdez Tan Keh and Dizon,
supra, that all judgments of political complexion of the courts during the Japanese regime, ceased to be
valid upon the reoccupation of the islands by virtue of the principle or right of postliminium. Applying
that doctrine to the present case, the sentence which convicted the petitioner of a crime of a political
complexion must be considered as having ceased to be valid ipso facto upon the reoccupation or
liberation of the Philippines by General Douglas MacArthur.

It may not be amiss to say in this connection that it is not necessary and proper to invoke the
proclamation of General Douglas MacArthur declaring null and void all laws, among them Act No. 65, of
the so-called Republic of the Philippines under which petitioner was convicted, in order to give
retroactive effect to the nullification of said penal act and invalidate sentence rendered against
Republic of the Philippines court which were continued during the Japanese occupation, were good and valid and remain good and
SUPREME COURT valid, and therefore enforceable now after the liberation or occupation of the Philippines, provided that
Manila such judgments do not have a political complexion, as this court held in its decision in the
abovementioned case of Co Kim Cham vs. Valdez Tan Keh and Dizon supra, in accordance with the
authorities therein cited.
EN BANC

Obviously, the sentence which petitioner is now serving has no political complexion. He was charged
G.R. No. L-6 November 29, 1945
with and convicted of an offense punishable under the municipal law of the Commonwealth, the Revised
Penal Code. Therefore, the sentence of the Court of First Instance of Ilocos Sur, as modified by the Court
ANICETO ALCANTARA, petitioner, of Appeals of Northern Luzon, is valid and enforceable.
vs.
DIRECTOR OF PRISONS, respondent.
A punitive or penal sentence is said to of a political complexion when it penalizes either a new act not
defined in the municipal laws, or acts already penalized by the latter as a crime against the legitimate
Buenaventura B. Martinez for petitioner. government, but taken out of the territorial law and penalized as a new offenses committed against
Office of the Solicitor General Taada for respondent. belligerent occupant, incident to a state of a war and necessary for the control of the occupied territory
and the protection of the army of the occupier. They are acts penalized for public rather than private
FERIA, J.: reasons, acts which tend, directly or indirectly, to aid or favor the enemy and are directed against the
welfare, safety and security, of the belligerent occupant. As example, the crimes against national security
, such as treason, espionage, etc., and against public order, such as rebellion, sedition, etc., were crimes
This is a petition for the issuance of a writ of habeas corpus and for the release of the petitioner on the against the Commonwealth or United States Government under the Revised Penal Code, which were
ground that the latter is unlawfully imprisoned and restrained of his liberty by the respondent Director of made crimes against the belligerent occupant.
Prison in the provincial jail at Vigan, Ilocos Sur.

In view of the foregoing, the petitioner for the writ of habeas corpus is denied.
Petitioner was convicted by the Court First Instance of Ilocos Sur (Criminal case No. 23) of the crime of
illegal discharge of firearms with less serious physical injuries. Upon appeal, the Court of Appeals of
Northern Luzon at Baguio modified said sentence (CA- G.R. No. 790)and sentence the petitioner to an Moran, C.J., Ozaeta, Paras, Jaranilla, Pablo and Bengzon, JJ., concur.
indeterminate penalty of from four months four months and twenty-one days of arresto mayor to three
years, nine months and three days ofprison correccional. The sentence as modified became final on
September 12, 1944, and June 23, 1945, petitioner commenced serving his sentence.

Petitioner now questions the validity of the decision of the Court of Appeals of Northern Luzon, on the Separate Opinions
sole ground that said court was only a creation of the so-called Republic of the Philippines during the
Japanese military occupation of the Islands; that the Court of Appeals was not authorized by
DE JOYA, J., concurring:
Commonwealth Act No. 3 to hold sessions in Baguio, and that only the two Justices constituted the
majority which promulgated the decision in question. The petitioner does not question the validity of
said decision on the strength of the Proclamation of General Douglas McArthur of October 23, 1944, The principal question involved in this habeas corpus case is the validity of the judicial proceedings held,
which according to our decision in the case of Co Kim Cham vs. Valdez Tan Keh and Dizon, G.R. No. L-5 (p. during the Japanese occupation, in the Court First Instance of Ilocos Sur, in which herein petitioner was
113, ante), does not refer to judicial processes. accused of frustrated murder, and in the Court of Appeals of Northern Luzon, in which, on appeal, said
petitioner was found guilty of illegal discharge of firearms with less serious physical injuries, and
sentenced to a term of imprisonment ranging from four moths and twenty-one days of arresto mayor to
In the said case of Co Kim Cham vs. Valdez Tan Keh and Dizon, this Court ruled that the so-called Republic
three years, and nine months and three days ofprison correccional; and the effect on said proceedings of
of the Philippines and the Philippine Executive Commission established in the Philippines during the
the proclamation of General Douglas McArthur, dated October 24 1944. The decision of this questions
Japanese regime were governments de facto organized by the belligerent occupant by the judicial acts
requires the application of principles of International Law, in connection with the municipal law of this
thereof were good and valid and remained good and valid after the restoration of the Commonwealth
country.
Government, except those a political complexion. In that the same case this Court held that the Court of
Appeals which was continued throughout the Japanese occupation, was the same Court of Appeals
existed prior to the Japanese occupation and was lately abolished by Executive Order No. 37. The Under the Constitution Commonwealth of the Philippines, International Law is part of the Fundamental
division of the Court of Appeals into several District Court of Appeals, and the reduction of the number law of the land (Article II, sec. 3). As International Law is an integral part of our law, it must be
of Justices sitting in each division, the regime of the so-called Republic effected no substantial change in ascertained and administered by this Court, whenever question of right depending upon it are presented
its nature and jurisdiction. for our determination (Kansas vs.Colorado, 185 U.S. 146; 22 Sup. Ct., 552; 46 Law. ed., 838).

Even assuming that the Court of Appeals of Northern Luzon was a new court created by the belligerent Since International Law is a body of rules accepted by nations as regulating their mutual relations, the
occupant or the de facto governments established by him, the judgments of such court, like those of the proof of their existence is to be found in the consent of the nations to abide by them; and this consent is
evidenced chiefly by the usages and customs of nation, as found in the writings of publicist and in the acquired by a solemn act of incorporation which had not been gone through." (Hall, International Law,
decisions of the highest courts of the different countries of the world (The Habana, 175 U. S., 677; 20 6th ed., p. 461.)
Sup. Ct., 290; 44 Law. ed., 320.).
It is, therefore, evident that the establishment of the government under the name of the Philippine
But while usages and customs are the older original source of International Law, great international Executive Commission, or the so-called Philippine Republic, afterwards, during Japanese occupation,
treaties are a latter source of increasing importance, such as The Hogue Conventions of 1899 and 1907. respecting the laws in force in the country, and permitting our courts to function and administer said
laws, as proclaim in the City of Manila, by the commander in chief of the Japanese Imperial Forces, on
January 3, 1942, was in accordance with the rules and principles of International Law.
The Hague Conventions of 1899, respecting laws and customs of war on land, expressly declare that:

If the military occupant is thus in duty bound to establish in the territory under military occupation
ARTICLE XLII. Territory is considered occupied when it is actually placed under the authority of
governmental agencies for the preservation of peace and order and for the proper administration of
the hostile army.
justice, in accordance with the local laws, it must necessarily follow that the judicial proceeding
conducted before the courts established by the military occupant must be considered legal and valid,
The occupation applies only to the territory where such authority is established, and in a even after said government established by the military occupant had been displaced by the legitimate
position to assert itself. government of the territory.

ART. XLII. The authority of the legitimate power having actually passed into the hands of the Thus the judgments rendered by the Confederate Courts, during the American Civil War, merely setting
occupant, the latter shall take all steps in his power to reestablish and insure, as far as the rights of private parties actually within their jurisdiction, not only tending to defeat the legal rights of
possible, public order and safety, while respecting, unless absolutely prevented, the laws in citizens of the United States, nor in furtherance of laws passed in aid of the rebellion, had been declared
force in the country. (32 Stat., II, 1821.). valid and binding (Cook vs.Oliver, 1 Woods, 437; Fed. Cas., No. 3, 164; Coleman vs. Tennessee, 97 U.S.,
509;24 Law. ed., 1118; Williams vs.Bruffy, 96 U.S. 176; Horn vs. Lockhart, 17 Wall., 570; Sprott vs. United
The above provisions of the Hague Conventions have been adopted by the nations giving adherence to States, 20 Wall., 459; Texas vs. White, 7 Wall., 700; Ketchum vs. Buckley [1878], 99 U.S., 188); and the
them, among which is the United States of America (32 Stat., II, 1821). judgment of a court of Georgia rendered in November, 1861, for the purchase money slaves was held
valid judgment when entered, and enforceable in 1871 (French vs. Tumllin, 10 Am. Law. Reg. [N.S.], 641;
Fed. Case, No. 5104).
The commander in chief of the invading forces or military occupant may exercise governmental
authority, but only when in actual possession of the enemy's territory, and this authority will be
exercised upon principles of International Law (New Orleans vs. Steamship Co. [1874], 20 Wall., 387; The judgments by the courts of the states constituting the Confederate States of the America were
Kelly vs. Sanders [1878], 99 U.S., 441; MacLeod vs. United States 229 U.S. 416; 33 Sup Ct., 955; 57 Law. considered legal and valid and enforceable, even after the termination of the American Civil War,
ed., 1260; II Oppenheim on International Law, sec. 167). because they had been rendered by the courts of a de facto government. The Confederate States were
a de facto government, in the sense that its citizens were bound to render the government obedience in
civil matters, and did not become responsible, as wrong-doers, for such act of obedience
It will thus be readily seen that the civil laws of the invaded state continue in force, in so far as they do (Thorington vs. Smith, 8 Wall. [U.S.] 9; 19 Law ed., 361).
not affect the hostile occupant unfavorably. The regular judicial tribunals of the occupied territory
continue to act in cases not affecting the military occupation, and is not usual for the invader to take the
whole administration into his own hands, because it is easier to preserve order through the agency of In the more recent case of Ketchum vs. Buckley ([1878], 99 U.S., 188), the Supreme Court of the United
the native officials, and also because the latter are more competent to administer the laws of the States held-- "It is now settled law in this court that during the late civil war the same general law for the
territory; and the military occupant generally keeps in their posts such of the judicial and administrative administration of justice and the protection of private rights, which had existed in the States prior to the
officers as are willing to serve under him, subjecting them only to supervision by the military authorities, rebellion, remained during its continuance and afterwards. As far as the acts of the States did not impair
or by superior civil authorities appointed by him (Young vs. United States, 97 U.S. 39; 24 Law. ed 992; or tend to impair the supremacy of the national authority, or the just rights of the citizens, under the
Coleman vs. Tennessee, 97 U.S. 509; 24 Law ed., 1118; MacLeod vs. United States, 229 U.S. 416; 33 Sup Constitution, they are in general to be treated as valid and binding." (Williams vs. Bruffy, 96 U.S., 176;
Ct., 955; 57 Law. ed., 1260 Taylor, International Law, secs. 576, 578; Wilson, International Law, pp. 331- Horn vs. Lockhart, 17 Wall., 570; Sprott vs. United States, 20 Wall., 459; Texas vs. White 7 Wall., 700.)
337; Hall, International Law, 6th ed. [1909], pp. 464, 465, 475, 476; Lawrence, International Law, 7th ed.,
412, 413; Davis, Elements of International Law, 3d ed., pp. 330-332, 335; Holland, International Law, pp. The government established in the Philippines, during Japanese occupation, would seem to fall under
356, 357, 359; Westlake, International Law, Part II, 2d ed., pp. 121-123). the following definition of de facto government given by the Supreme Court of the United States:

In 1811, during the occupation of Catalonia, Spain, by the French army, a Frenchman, accused of the But there is another description of government de facto, called also by publicists a
murder of a Catalan in that province, was tried and convicted by the assize Court of the Department of government de facto, but which might, perhaps, he more aptly denominated a government of
the Pyrenees Orientales, France. Upon appeal to the French Court of Cassation, the conviction was paramount force. Its distinguishing characteristics (1) that its existence is maintained by
quashed, on the ground that the courts of the territory within which the crime had been committed had active military power within the territories, and against the rightful authority of an
exclusive jurisdiction to try the case and that "the occupation of Catalonia by French troops and its established and lawful government; and (2) that while it exists it must necessarily be obeyed
government by the French authorities had not communicated to its inhabitants the character of French in civil matters by private citizens who by acts of obedience rendered in submission to such
citizens, nor to their territory the character of French territory, and that such character could only be force, do not become responsible, as wrongdoers, for those acts though not warranted by the
laws of the rightful government. Actual governments of this sort are established over districts
differing greatly in extent and conditions. They are usually administered directly by military judicial proceedings conducted before our courts, during the Japanese occupation would be highly
authority, but they may be administered, also, by civil authority, supported more or less detrimental to public interests.
directly by military force. (MacLeod vs. United States [1913], 229 U.S., 416.)
For the forgoing reasons, I concur in the majority opinion, and the petition for habeas corpus filed in this
The government established in the Philippines, under the Philippine Executive Commission or under the case should, therefore, be denied.
so-called Philippine Republic, during Japanese occupation, was and should, therefor, be considered as
a de factogovernment; and that the judicial proceedings conducted before the courts has been
established in this country, during said Japanese occupation, and are should be considered as legal and
valid enforceable, even after the liberation of this country by the American forces, as a long a said
judicial proceedings had been conducted, in accordance with the law of the Commonwealth of the
Philippines.

The judicial proceedings involved in the case under consideration merely refer to the prosecution of the
petitioner in this case, for the crime of frustrated murder, which was reduced to illegal discharge of
firearms with less serious physical injuries, under the provisions of the Revised Penal Code, in force in
this country under the Commonwealth government, before and during Japanese occupation.

Now, petitioner contends that the judicial proceedings in question are null and void, and that the
accused should be immediately released from the custody, under the provisions of the proclamation
issued by General Douglas McArthur dated October 23, 1944; as said proclamation nullifies all the laws,
regulations and processes of any other government in the Philippines than that of the Commonwealth of
the Philippines.

In other words petition demands a literal interpretation of said proclamation issued by the General
Douglas McArthur, a contention which, in our opinion, is untenable, as it would inevitably produce
judicial chaos and uncertainties. When an act is susceptible of two or more constructions, one of which
will maintain and the others destroy it, the Courts will always adopt the former (United
States vs. Coombs [1838]], 12 Pet., 72; 9 Law. ed., 1004; Board of Supervisors of Grenada
County vs. Brown [1884], 112 U.S., 261; 28 Law. ed., 704; 5 Sup. Ct. Rep., 125; In re Guaria [1913], 24
Phil., 37; Fuentes vs. Director of Prisons [1924], 46 Phil., 22; Yu Cong Eng vs.Trinidad [1925], 47 Phil.,
385). The judiciary, always alive to the dictates of national welfare, can properly incline the scales of its
decisions in favor of that solution which will most effectively promote the public policy (Smith, Bell &
Co., Ltd. vs. Natividad [1919], 40 Phil., 136). All laws should receive a sensible construction as not to lead
it injustice, oppression or an absurd consequence. It will always, therefore, be presumed that the
legislature intended exception to its language, which would avoid results of this character. The reason of
the law in such cases should prevail over its letter (United States vs. Kirby, 7 Wall [U.S.], 482; 19 Law. ed,
278; Church of Holy Trinity vs. United States, 143 U.S., 461; 12 Sup. Ct., 511; 36 Law. ed., 226;
Jacobson vs. Massachussetts, 197 U.S., 39; 25 Sup. Ct., 358; 49 Law. ed., 643; 3 Ann. Cas., 765; In re Allen,
2 Phil., 630). The duty of the court in construing a statute, which is reasonably susceptible of two
constructions to adopt that which saves its constitutionality, includes the duty of a avoiding a
construction which raises grave and doubtful constitutional questions, if it can be avoided (United
States vs. Delaware & Hudson Co., 213 U.S., 366; 29 Sup. Ct. 527; 53 Law. ed., 836).

According to the rules and principles of International Law, and the legal doctrines cited above, the
judicial proceedings conducted before the court of the justice, established here during Japanese military
occupation, merely applying the provisions of the municipal law of the territory, as the provisions of the
Revised Penal Code in the instant case which have no political or military significance, are and should be
considered legal, valid and binding. It is to be presumed that General Douglas McArthur knows said rules
and principles of International Law, as International Law is an integral part of the fundamental law of the
land, in accordance with the provisions of the Constitution of the United States. And it is also to be
presumed that General Douglas McArthur has acted, in accordance with said principles of International
Law, which have been sanction by the Supreme Court of the United States, as the nullification of all
Republic of the Philippines 3. Certification, dated 12 October 1998, also issued by Elizabeth B. Cerezo,
SUPREME COURT showing that Ching was elected as a member of the Sangguniang Bayan of Tubao,
Manila La Union during the 12 May 1992 synchronized elections.

EN BANC On 5 April 1999, the results of the 1998 Bar Examinations were released and Ching was one of the
successful Bar examinees. The oath-taking of the successful Bar examinees was scheduled on 5 May
1999. However, because of the questionable status of Ching's citizenship, he was not allowed to take his
oath. Pursuant to the resolution of this Court, dated 20 April 1999, he was required to submit further
proof of his citizenship. In the same resolution, the Office of the Solicitor General (OSG) was required to
BAR MATTER No. 914 October 1, 1999 file a comment on Ching's petition for admission to the bar and on the documents evidencing his
Philippine citizenship.
RE: APPLICATION FOR ADMISSION TO THE PHILIPPINE BAR,
The OSG filed its comment on 8 July 1999, stating that Ching, being the "legitimate child of a Chinese
vs. father and a Filipino mother born under the 1935 Constitution was a Chinese citizen and continued to be
so, unless upon reaching the age of majority he elected Philippine citizenship" 1 in strict compliance with
the provisions of Commonwealth Act No. 625 entitled "An Act Providing for the Manner in which the
VICENTE D. CHING, applicant. Option to Elect Philippine Citizenship shall be Declared by a Person Whose Mother is a Filipino Citizen."
The OSG adds that "(w)hat he acquired at best was only an inchoate Philippine citizenship which he
RESOLUTION could perfect by election upon reaching the age of majority." 2 In this regard, the OSG clarifies that "two
(2) conditions must concur in order that the election of Philippine citizenship may be effective, namely:
(a) the mother of the person making the election must be a citizen of the Philippines; and (b) said
election must be made upon reaching the age of majority." 3 The OSG then explains the meaning of the
phrase "upon reaching the age of majority:"
KAPUNAN, J.:
The clause "upon reaching the age of majority" has been construed to mean a
Can a legitimate child born under the 1935 Constitution of a Filipino mother and an alien father validly reasonable time after reaching the age of majority which had been interpreted by
elect Philippine citizenship fourteen (14) years after he has reached the age of majority? This is the the Secretary of Justice to be three (3) years (VELAYO, supra at p. 51 citing Op.,
question sought to be resolved in the present case involving the application for admission to the Sec. of Justice No. 70, s. 1940, Feb. 27, 1940). Said period may be extended under
Philippine Bar of Vicente D. Ching. certain circumstances, as when a (sic) person concerned has always considered
himself a Filipino (ibid., citing Op. Nos. 355 and 422, s. 1955; 3, 12, 46, 86 and 97,
s. 1953). But in Cuenco, it was held that an election done after over seven (7) years
The facts of this case are as follows:
was not made within a reasonable time.

Vicente D. Ching, the legitimate son of the spouses Tat Ching, a Chinese citizen, and Prescila A. Dulay, a
In conclusion, the OSG points out that Ching has not formally elected Philippine citizenship and, if ever
Filipino, was born in Francia West, Tubao, La Union on 11 April 1964. Since his birth, Ching has resided in
he does, it would already be beyond the "reasonable time" allowed by present jurisprudence. However,
the Philippines.
due to the peculiar circumstances surrounding Ching's case, the OSG recommends the relaxation of the
standing rule on the construction of the phrase "reasonable period" and the allowance of Ching to elect
On 17 July 1998, Ching, after having completed a Bachelor of Laws course at the St. Louis University in Philippine citizenship in accordance with C.A. No. 625 prior to taking his oath as a member of the
Baguio City, filed an application to take the 1998 Bar Examinations. In a Resolution of this Court, dated 1 Philippine Bar.
September 1998, he was allowed to take the Bar Examinations, subject to the condition that he must
submit to the Court proof of his Philippine citizenship.
On 27 July 1999, Ching filed a Manifestation, attaching therewith his Affidavit of Election of Philippine
Citizenship and his Oath of Allegiance, both dated 15 July 1999. In his Manifestation, Ching states:
In compliance with the above resolution, Ching submitted on 18 November 1998, the following
documents:
1. I have always considered myself as a Filipino;

1. Certification, dated 9 June 1986, issued by the Board of Accountancy of the


2. I was registered as a Filipino and consistently declared myself as one in my
Professional Regulations Commission showing that Ching is a certified public
school records and other official documents;
accountant;

3. I am practicing a profession (Certified Public Accountant) reserved for Filipino


2. Voter Certification, dated 14 June 1997, issued by Elizabeth B. Cerezo, Election
citizens;
Officer of the Commission on Elections (COMELEC) in Tubao La Union showing that
Ching is a registered voter of the said place; and
4. I participated in electoral process[es] since the time I was eligible to vote; validity of election of Philippine citizenship, this dilemma was resolved by basing the time period on the
decisions of this Court prior to the effectivity of the 1935 Constitution. In these decisions, the proper
period for electing Philippine citizenship was, in turn, based on the pronouncements of the Department
5. I had served the people of Tubao, La Union as a member of the Sangguniang
of State of the United States Government to the effect that the election should be made within a
Bayan from 1992 to 1995;
"reasonable time" after attaining the age of majority. 10 The phrase "reasonable time" has been
interpreted to mean that the election should be made within three (3) years from reaching the age of
6. I elected Philippine citizenship on July 15, 1999 in accordance with majority. 11 However, we held in Cuenco vs. Secretary of Justice, 12 that the three (3) year period is not an
Commonwealth Act No. 625; inflexible rule. We said:

7. My election was expressed in a statement signed and sworn to by me before a It is true that this clause has been construed to mean a reasonable period after
notary public; reaching the age of majority, and that the Secretary of Justice has ruled that three
(3) years is the reasonable time to elect Philippine citizenship under the
8. I accompanied my election of Philippine citizenship with the oath of allegiance constitutional provision adverted to above, which period may be extended under
to the Constitution and the Government of the Philippines; certain circumstances, as when the person concerned has always considered
himself a Filipino. 13

9. I filed my election of Philippine citizenship and my oath of allegiance to (sic) the


Civil Registrar of Tubao La Union, and However, we cautioned in Cuenco that the extension of the option to elect Philippine citizenship is not
indefinite:

10. I paid the amount of TEN PESOS (Ps. 10.00) as filing fees.
Regardless of the foregoing, petitioner was born on February 16, 1923. He became
of age on February 16, 1944. His election of citizenship was made on May 15,
Since Ching has already elected Philippine citizenship on 15 July 1999, the question raised is whether he 1951, when he was over twenty-eight (28) years of age, or over seven (7) years
has elected Philippine citizenship within a "reasonable time." In the affirmative, whether his citizenship after he had reached the age of majority. It is clear that said election has not been
by election retroacted to the time he took the bar examination. made "upon reaching the age of majority." 14

When Ching was born in 1964, the governing charter was the 1935 Constitution. Under Article IV, Section In the present case, Ching, having been born on 11 April 1964, was already thirty-five (35) years old
1(3) of the 1935 Constitution, the citizenship of a legitimate child born of a Filipino mother and an alien when he complied with the requirements of C.A. No. 625 on 15 June 1999, or over fourteen (14) years
father followed the citizenship of the father, unless, upon reaching the age of majority, the child elected after he had reached the age of majority. Based on the interpretation of the phrase "upon reaching the
Philippine citizenship. 4 This right to elect Philippine citizenship was recognized in the 1973 Constitution age of majority," Ching's election was clearly beyond, by any reasonable yardstick, the allowable period
when it provided that "(t)hose who elect Philippine citizenship pursuant to the provisions of the within which to exercise the privilege. It should be stated, in this connection, that the special
Constitution of nineteen hundred and thirty-five" are citizens of the Philippines. 5 Likewise, this circumstances invoked by Ching, i.e., his continuous and uninterrupted stay in the Philippines and his
recognition by the 1973 Constitution was carried over to the 1987 Constitution which states that "(t)hose being a certified public accountant, a registered voter and a former elected public official, cannot vest in
born before January 17, 1973 of Filipino mothers, who elect Philippine citizenship upon reaching the age him Philippine citizenship as the law specifically lays down the requirements for acquisition of Philippine
of majority" are Philippine citizens. 6 It should be noted, however, that the 1973 and 1987 Constitutional citizenship by election.
provisions on the election of Philippine citizenship should not be understood as having a curative effect
on any irregularity in the acquisition of citizenship for those covered by the 1935 Constitution. 7 If the
citizenship of a person was subject to challenge under the old charter, it remains subject to challenge Definitely, the so-called special circumstances cannot constitute what Ching erroneously labels as
under the new charter even if the judicial challenge had not been commenced before the effectivity of informal election of citizenship. Ching cannot find a refuge in the case of In re:Florencio Mallare, 15 the
the new Constitution. 8 pertinent portion of which reads:

C.A. No. 625 which was enacted pursuant to Section 1(3), Article IV of the 1935 Constitution, prescribes And even assuming arguendo that Ana Mallare were (sic) legally married to an
the procedure that should be followed in order to make a valid election of Philippine citizenship. Under alien, Esteban's exercise of the right of suffrage when he came of age, constitutes
Section 1 thereof, legitimate children born of Filipino mothers may elect Philippine citizenship by a positive act of election of Philippine citizenship. It has been established that
expressing such intention "in a statement to be signed and sworn to by the party concerned before any Esteban Mallare was a registered voter as of April 14, 1928, and that as early as
officer authorized to administer oaths, and shall be filed with the nearest civil registry. The said party 1925 (when he was about 22 years old), Esteban was already participating in the
shall accompany the aforesaid statement with the oath of allegiance to the Constitution and the elections and campaigning for certain candidate[s]. These acts are sufficient to
Government of the Philippines." show his preference for Philippine citizenship. 16

However, the 1935 Constitution and C.A. No. 625 did not prescribe a time period within which the Ching's reliance on Mallare is misplaced. The facts and circumstances obtaining therein are very different
election of Philippine citizenship should be made. The 1935 Charter only provides that the election from those in the present case, thus, negating its applicability. First, Esteban Mallare was born before
should be made "upon reaching the age of majority." The age of majority then commenced upon the effectivity of the 1935 Constitution and the enactment of C.A. No. 625. Hence, the requirements and
reaching twenty-one (21) years. 9 In the opinions of the Secretary of Justice on cases involving the procedures prescribed under the 1935 Constitution and C.A. No. 625 for electing Philippine citizenship
would not be applicable to him. Second, the ruling in Mallare was an obiter since, as correctly pointed
out by the OSG, it was not necessary for Esteban Mallare to elect Philippine citizenship because he was We repeat that any election of Philippine citizenship on the part of the private
already a Filipino, he being a natural child of a Filipino mother. In this regard, the Court stated: respondent would not only have been superfluous but it would also have resulted
in an absurdity. How can a Filipino citizen elect Philippine citizenship? 19
Esteban Mallare, natural child of Ana Mallare, a Filipina, is therefore himself a
Filipino, and no other act would be necessary to confer on him all the rights and The Court, like the OSG, is sympathetic with the plight of Ching. However, even if we consider the special
privileges attached to Philippine citizenship (U.S. vs. Ong Tianse, 29 Phil. 332; circumstances in the life of Ching like his having lived in the Philippines all his life and his consistent
Santos Co vs. Government of the Philippine Islands, 42 Phil. 543, Serra vs. Republic, belief that he is a Filipino, controlling statutes and jurisprudence constrain us to disagree with the
L-4223, May 12, 1952, Sy Quimsuan vs. Republic, L-4693, Feb. 16, 1953; Pitallano recommendation of the OSG. Consequently, we hold that Ching failed to validly elect Philippine
vs. Republic, L-5111, June 28, 1954). Neither could any act be taken on the citizenship. The span of fourteen (14) years that lapsed from the time he reached the age of majority
erroneous belief that he is a non-filipino divest him of the citizenship privileges to until he finally expressed his intention to elect Philippine citizenship is clearly way beyond the
which he is rightfully entitled. 17 contemplation of the requirement of electing "upon reaching the age of majority." Moreover, Ching has
offered no reason why he delayed his election of Philippine citizenship. The prescribed procedure in
electing Philippine citizenship is certainly not a tedious and painstaking process. All that is required of
The ruling in Mallare was reiterated and further elaborated in Co vs. Electoral Tribunal of the House of
the elector is to execute an affidavit of election of Philippine citizenship and, thereafter, file the same
Representatives, 18 where we held:
with the nearest civil registry. Ching's unreasonable and unexplained delay in making his election cannot
be simply glossed over.
We have jurisprudence that defines "election" as both a formal and an informal
process.
Philippine citizenship can never be treated like a commodity that can be claimed when needed and
suppressed when convenient. 20 One who is privileged to elect Philippine citizenship has only an inchoate
In the case of In re: Florencio Mallare (59 SCRA 45 [1974]), the Court held that the right to such citizenship. As such, he should avail of the right with fervor, enthusiasm and promptitude.
exercise of the right of suffrage and the participation in election exercises Sadly, in this case, Ching slept on his opportunity to elect Philippine citizenship and, as a result. this
constitute a positive act of election of Philippine citizenship. In the exact golden privilege slipped away from his grasp.
pronouncement of the Court, we held:
IN VIEW OF THE FOREGOING, the Court Resolves to DENY Vicente D. Ching's application for admission to
Esteban's exercise of the right of suffrage when he came of the Philippine Bar.
age constitutes a positive act of Philippine citizenship. (p. 52:
emphasis supplied)
SO ORDERED.

The private respondent did more than merely exercise his right of suffrage. He has established his life
here in the Philippines.

For those in the peculiar situation of the respondent who cannot be excepted to
have elected Philippine citizenship as they were already citizens, we apply the In Re
Mallare rule.

xxx xxx xxx

The filing of sworn statement or formal declaration is a requirement for those who
still have to elect citizenship. For those already Filipinos when the time to elect
came up, there are acts of deliberate choice which cannot be less binding. Entering
a profession open only to Filipinos, serving in public office where citizenship is a
qualification, voting during election time, running for public office, and other
categorical acts of similar nature are themselves formal manifestations for these
persons.

An election of Philippine citizenship presupposes that the person electing is an


alien. Or his status is doubtful because he is a national of two countries. There is
no doubt in this case about Mr. Ong's being a Filipino when he turned twenty-one
(21).
Republic of the Philippines 2) Jose Ong, Jr. is not a resident of the second district of Northern Samar.
SUPREME COURT
Manila
The HRET in its decision dated November 6, 1989, found for the private respondent.

EN BANC
A motion for reconsideration was filed by the petitioners on November 12, 1989. This was, however,
denied by the HRET in its resolution dated February 22, 1989.

Hence, these petitions for certiorari.


G.R. Nos. 92191-92 July 30, 1991
We treat the comments as answers and decide the issues raised in the petitions.
ANTONIO Y. CO, petitioner,
vs.
ON THE ISSUE OF JURISDICTION
ELECTORAL TRIBUNAL OF THE HOUSE OF REPRESENTATIVES AND JOSE ONG, JR., respondents.

The first question which arises refers to our jurisdiction.


G.R. Nos. 92202-03 July 30, 1991

The Constitution explicitly provides that the House of Representatives Electoral Tribunal (HRET) and the
SIXTO T. BALANQUIT, JR., petitioner,
Senate Electoral Tribunal (SET) shall be the sole judges of all contests relating to the election, returns,
vs.
and qualificationsof their respective members. (See Article VI, Section 17, Constitution)
ELECTORAL TRIBUNAL OF THE HOUSE OF REPRESENTATIVES AND JOSE ONG, JR., respondents.

The authority conferred upon the Electoral Tribunal is full, clear and complete. The use of the
Hechanova & Associates for petitioner Co.
word soleemphasizes the exclusivity of the jurisdiction of these Tribunals.

Brillantes, Nachura, Navarro and Arcilla Law Offices for respondent Ong, Jr.
The Supreme Court in the case of Lazatin v. HRET (168 SCRA 391 [1988]) stated that under the 1987
Constitution, the jurisdiction of the Electoral Tribunal is original and exclusive, viz:

The use of the word "sole" emphasizes the exclusive character of the jurisdiction
GUTIERREZ, JR., J.:p conferred (Angara v. Electoral Commission, supra at p. 162). The exercise of power
by the Electoral Commission under the 1935 Constitution has been described as
"intended to be as complete and unimpaired as if it had originally remained in the
The petitioners come to this Court asking for the setting aside and reversal of a decision of the House of
legislature." (id., at p. 175) Earlier this grant of power to the legislature was
Representatives Electoral Tribunal (HRET).
characterized by Justice Malcolm as "full, clear and complete; (Veloso v. Board of
Canvassers of Leyte and Samar, 39 Phil. 886 [1919]) Under the amended 1935
The HRET declared that respondent Jose Ong, Jr. is a natural born Filipino citizen and a resident of Constitution, the power was unqualifiedly reposed upon the Electoral Tribunal and
Laoang, Northern Samar for voting purposes. The sole issue before us is whether or not, in making that it remained as full, clear and complete as that previously granted the Legislature
determination, the HRET acted with grave abuse of discretion. and the Electoral Commission, (Lachica v. Yap, 25 SCRA 140 [1968]) The same may
be said with regard to the jurisdiction of the Electoral Tribunal under the 1987
On May 11, 1987, the congressional election for the second district of Northern Samar was held. Constitution. (p. 401)

Among the candidates who vied for the position of representative in the second legislative district of The Court continued further, ". . . so long as the Constitution grants the HRET the power to be the sole
Northern Samar are the petitioners, Sixto Balinquit and Antonio Co and the private respondent, Jose judge of all contests relating to election, returns and qualifications of members of the House of
Ong, Jr. Representatives, any final action taken by the HRET on a matter within its jurisdiction shall, as a rule, not
be reviewed by this Court . . . the power granted to the Electoral Tribunal is full, clear and complete and
excludes the exercise of any authority on the part of this Court that would in any wise restrict it or curtail
Respondent Ong was proclaimed the duly elected representative of the second district of Northern it or even affect the same." (pp. 403-404)
Samar.

When may the Court inquire into acts of the Electoral Tribunals under our constitutional grants of
The petitioners filed election protests against the private respondent premised on the following grounds: power?

1) Jose Ong, Jr. is not a natural born citizen of the Philippines; and
In the later case of Robles v. HRET (181 SCRA 780 [1990]) the Supreme Court stated that the judgments The records show that in the year 1895, the private respondent's grandfather, Ong Te, arrived in the
of the Tribunal are beyond judicial interference save only "in the exercise of this Court's so-called Philippines from China. Ong Te established his residence in the municipality of Laoang, Samar on land
extraordinary jurisdiction, . . . upon a determination that the Tribunal's decision or resolution was which he bought from the fruits of hard work.
rendered without or in excess of its jurisdiction, or with grave abuse of discretion or paraphrasing
Morrero, upon a clear showing of such arbitrary and improvident use by the Tribunal of its power as
As a resident of Laoang, Ong Te was able to obtain a certificate of residence from the then Spanish
constitutes a denial of due process of law, or upon a demonstration of a very clear unmitigated ERROR,
colonial administration.
manifestly constituting such GRAVE ABUSE OF DISCRETION that there has to be a remedy for such
abuse." (at pp. 785-786)
The father of the private respondent, Jose Ong Chuan was born in China in 1905. He was brought by Ong
Te to Samar in the year 1915.
In the leading case of Morrero v. Bocar (66 Phil. 429 [1938]) the Court ruled that the power of the
Electoral Commission "is beyond judicial interference except, in any event, upon a clear showing of such
arbitrary and improvident use of power as will constitute a denial of due process." The Court does not Jose Ong Chuan spent his childhood in the province of Samar. In Laoang, he was able to establish an
venture into the perilous area of trying to correct perceived errors of independent branches of the enduring relationship with his neighbors, resulting in his easy assimilation into the community.
Government, It comes in only when it has to vindicate a denial of due process or correct an abuse of
discretion so grave or glaring that no less than the Constitution calls for remedial action. As Jose Ong Chuan grew older in the rural and seaside community of Laoang, he absorbed Filipino
cultural values and practices. He was baptized into Christianity. As the years passed, Jose Ong Chuan met
The Supreme Court under the 1987 Constitution, has been given an expanded jurisdiction, so to speak, a natural born-Filipino, Agripina Lao. The two fell in love and, thereafter, got married in 1932 according
to review the decisions of the other branches and agencies of the government to determine whether or to Catholic faith and practice.
not they have acted within the bounds of the Constitution. (See Article VIII, Section 1, Constitution)
The couple bore eight children, one of whom is the private respondent who was born in 1948.
Yet, in the exercise thereof, the Court is to merely check whether or not the governmental branch or
agency has gone beyond the Constitutional limits of its jurisdiction, not that it erred or has a different The private respondent's father never emigrated from this country. He decided to put up a hardware
view. In the absence of a showing that the HRET has committed grave abuse of discretion amounting to store and shared and survived the vicissitudes of life in Samar.
lack of jurisdiction, there is no occasion for the Court to exercise its corrective power; it will not decide a
matter which by its nature is for the HRET alone to decide. (See Marcos v. Manglapus, 177 SCRA 668
[1989]) It has no power to look into what it thinks is apparent error. The business prospered. Expansion became inevitable. As a result, a branch was set-up in Binondo,
Manila. In the meantime, the father of the private respondent, unsure of his legal status and in an
unequivocal affirmation of where he cast his life and family, filed with the Court of First Instance of
As constitutional creations invested with necessary power, the Electoral Tribunals, although not powers Samar an application for naturalization on February 15, 1954.
in the tripartite scheme of the government, are, in the exercise of their functions independent organs
independent of Congress and the Supreme Court. The power granted to HRET by the Constitution is
intended to be as complete and unimpaired as if it had remained originally in the legislature. (Angara v. On April 28, 1955, the CFI of Samar, after trial, declared Jose Ong Chuan a Filipino citizen.
Electoral Commission, 63 Phil. 139 [1936])
On May 15, 1957, the Court of First Instance of Samar issued an order declaring the decision of April 28,
In passing upon petitions, the Court with its traditional and careful regard for the balance of powers, 1955 as final and executory and that Jose Ong Chuan may already take his Oath of Allegiance.
must permit this exclusive privilege of the Tribunals to remain where the Sovereign authority has place
it. (See Veloso v. Boards of Canvassers of Leyte and Samar, 39 Phil. 886 [1919]) Pursuant to said order, Jose Ong Chuan took his Oath of Allegiance; correspondingly, a certificate of
naturalization was issued to him.
It has been argued that under Article VI, Section 17 of the present Constitution, the situation may exist
as it exists today where there is an unhealthy one-sided political composition of the two Electoral At the time Jose Ong Chuan took his oath, the private respondent then a minor of nine years was
Tribunals. There is nothing in the Constitution, however, that makes the HRET because of its composition finishing his elementary education in the province of Samar. There is nothing in the records to
any less independent from the Court or its constitutional functions any less exclusive. The degree of differentiate him from other Filipinos insofar as the customs and practices of the local populace were
judicial intervention should not be made to depend on how many legislative members of the HRET concerned.
belong to this party or that party. The test remains the same-manifest grave abuse of discretion.
Fortunes changed. The house of the family of the private respondent in Laoang, Samar was burned to
In the case at bar, the Court finds no improvident use of power, no denial of due process on the part of the ground.
the HRET which will necessitate the exercise of the power of judicial review by the Supreme Court.
Undaunted by the catastrophe, the private respondent's family constructed another one in place of their
ON THE ISSUE OF CITIZENSHIP ruined house. Again, there is no showing other than that Laoang was their abode and home.

After completing his elementary education, the private respondent, in search for better education, went
to Manila in order to acquire his secondary and college education.
In the meantime, another misfortune was suffered by the family in 1975 when a fire gutted their second Those who elect Philippine citizenship in accordance with paragraph 3 hereof shall
house in Laoang, Samar. The respondent's family constructed still another house, this time a 16-door be deemed natural-born citizens.
apartment building, two doors of which were reserved for the family.
The Court interprets Section 1, Paragraph 3 above as applying not only to those who elect Philippine
The private respondent graduated from college, and thereafter took and passed the CPA Board citizenship after February 2, 1987 but also to those who, having been born of Filipino mothers, elected
Examinations. citizenship before that date.

Since employment opportunities were better in Manila, the respondent looked for work here. He found The provision in Paragraph 3 was intended to correct an unfair position which discriminates against
a job in the Central Bank of the Philippines as an examiner. Later, however, he worked in the hardware Filipino women. There is no ambiguity in the deliberations of the Constitutional Commission, viz:
business of his family in Manila. In 1971, his elder brother, Emil, was elected as a delegate to the 1971
Constitutional Convention. His status as a natural born citizen was challenged. Parenthetically, the
Mr. Azcuna: With respect to the provision of section 4,
Convention which in drafting the Constitution removed the unequal treatment given to derived
would this refer only to those who elect Philippine
citizenship on the basis of the mother's citizenship formally and solemnly declared Emil Ong,
citizenship after the effectivity of the 1973 Constitution or
respondent's full brother, as a natural born Filipino. The Constitutional Convention had to be aware of
would it also cover those who elected it under the 1973
the meaning of natural born citizenship since it was precisely amending the article on this subject.
Constitution?

The private respondent frequently went home to Laoang, Samar, where he grew up and spent his
Fr. Bernas: It would apply to anybody who elected Philippine
childhood days.
citizenship by virtue of the provision of the 1935 Constitution
whether the election was done before or after January 17,
In 1984, the private respondent married a Filipina named Desiree Lim. 1973. (Records of the Constitutional Commission, Vol. 1, p.
228; Emphasis supplied)
For the elections of 1984 and 1986, Jose Ong, Jr. registered himself as a voter of Laoang, Samar, and
correspondingly, voted there during those elections. xxx xxx xxx

The private respondent after being engaged for several years in the management of their family business Mr. Trenas: The Committee on Citizenship, Bill of Rights,
decided to be of greater service to his province and ran for public office. Hence, when the opportunity Political Rights and Obligations and Human Rights has more
came in 1987, he ran in the elections for representative in the second district of Northern Samar. or less decided to extend the interpretation of who is a
natural-born citizen as provided in section 4 of the 1973
Constitution by adding that persons who have elected
Mr. Ong was overwhelmingly voted by the people of Northern Samar as their representative in Congress.
Philippine Citizenship under the 1935 Constitution shall be
Even if the total votes of the two petitioners are combined, Ong would still lead the two by more than
natural-born? Am I right Mr. Presiding Officer?
7,000 votes.

Fr. Bernas: yes.


The pertinent portions of the Constitution found in Article IV read:

xxx xxx xxx


SECTION 1, the following are citizens of the Philippines:

Mr. Nolledo: And I remember very well that in the Reverend


1. Those who are citizens of the Philippines at the time of the adoption of the
Father Bernas' well written book, he said that the decision
Constitution;
was designed merely to accommodate former delegate
Ernesto Ang and that the definition on natural-born has no
2. Those whose fathers or mothers are citizens of the Philippines; retroactive effect. Now it seems that the Reverend Father
Bernas is going against this intention by supporting the
3. Those born before January 17, 1973, of Filipino mothers, who elect Philippine amendment?
citizenship upon reaching the age of majority; and
Fr. Bernas: As the Commissioner can see, there has been an
4. Those who are naturalized in accordance with law. evolution in my thinking. (Records of the Constitutional
Commission, Vol. 1, p. 189)

SECTION 2, Natural-born Citizens are those who are citizens of the Philippines from
birth without having to perform any act to acquire or perfect their citizenship. xxx xxx xxx
Mr. Rodrigo: But this provision becomes very important of a Filipino mother and an alien father would still have to elect Philippine citizenship. If one so elected,
because his election of Philippine citizenship makes him not he was not, under earlier laws, conferred the status of a natural-born.
only a Filipino citizen but a natural-born Filipino citizen
entitling him to run for Congress. . .
Under the 1973 Constitution, those born of Filipino fathers and those born of Filipino mothers with an
alien father were placed on equal footing. They were both considered as natural-born citizens.
Fr. Bernas: Correct. We are quite aware of that and for that
reason we will leave it to the body to approve that provision
Hence, the bestowment of the status of "natural-born" cannot be made to depend on the fleeting
of section 4.
accident of time or result in two kinds of citizens made up of essentially the same similarly situated
members.
Mr. Rodrigo: I think there is a good basis for the provision
because it strikes me as unfair that the Filipino citizen who
It is for this reason that the amendments were enacted, that is, in order to remedy this accidental
was born a day before January 17, 1973 cannot be a Filipino
anomaly, and, therefore, treat equally all those born before the 1973 Constitution and who elected
citizen or a natural-born citizen. (Records of the
Philippine citizenship either before or after the effectivity of that Constitution.
Constitutional Commission, Vol. 1, p. 231)

The Constitutional provision in question is, therefore curative in nature. The enactment was meant to
xxx xxx xxx
correct the inequitable and absurd situation which then prevailed, and thus, render those acts valid
which would have been nil at the time had it not been for the curative provisions. (See Development
Mr. Rodrigo: The purpose of that provision is to remedy an Bank of the Philippines v. Court of Appeals, 96 SCRA 342 [1980])
inequitable situation. Between 1935 and 1973 when we
were under the 1935 Constitution, those born of Filipino
There is no dispute that the respondent's mother was a natural born Filipina at the time of her marriage.
fathers but alien mothers were natural-born Filipinos.
Crucial to this case is the issue of whether or not the respondent elected or chose to be a Filipino citizen.
However, those born of Filipino mothers but alien fathers
would have to elect Philippine citizenship upon reaching the
age of majority; and if they do elect, they become Filipino Election becomes material because Section 2 of Article IV of the Constitution accords natural born status
citizens but not natural-born Filipino citizens. (Records of the to children born of Filipino mothers before January 17, 1973, if they elect citizenship upon reaching the
Constitutional Commission, Vol. 1, p. 356) age of majority.

The foregoing significantly reveals the intent of the framers. To make the provision prospective from To expect the respondent to have formally or in writing elected citizenship when he came of age is to ask
February 3, 1987 is to give a narrow interpretation resulting in an inequitable situation. It must also be for the unnatural and unnecessary. The reason is obvious. He was already a citizen. Not only was his
retroactive. mother a natural born citizen but his father had been naturalized when the respondent was only nine (9)
years old. He could not have divined when he came of age that in 1973 and 1987 the Constitution would
be amended to require him to have filed a sworn statement in 1969 electing citizenship inspite of his
It should be noted that in construing the law, the Courts are not always to be hedged in by the literal
already having been a citizen since 1957. In 1969, election through a sworn statement would have been
meaning of its language. The spirit and intendment thereof, must prevail over the letter, especially
an unusual and unnecessary procedure for one who had been a citizen since he was nine years old.
where adherence to the latter would result in absurdity and injustice. (Casela v. Court of Appeals, 35
SCRA 279 [1970])
We have jurisprudence that defines "election" as both a formal and an informal process.
A Constitutional provision should be construed so as to give it effective operation and suppress the
mischief at which it is aimed, hence, it is the spirit of the provision which should prevail over the letter In the case of In Re: Florencio Mallare (59 SCRA 45 [1974]), the Court held that the exercise of the right of
thereof. (Jarrolt v. Mabberly, 103 U.S. 580) suffrage and the participation in election exercises constitute a positive act of election of Philippine
citizenship. In the exact pronouncement of the Court, we held:
In the words of the Court in the case of J.M. Tuason v. LTA (31 SCRA 413 [1970]:
Esteban's exercise of the right of suffrage when he came of age, constitutes a
positive act of election of Philippine citizenship (p. 52; emphasis supplied)
To that primordial intent, all else is subordinated. Our Constitution, any
constitution is not to be construed narrowly or pedantically for the prescriptions
therein contained, to paraphrase Justice Holmes, are not mathematical formulas The private respondent did more than merely exercise his right of suffrage. He has established his life
having their essence in their form but are organic living institutions, the here in the Philippines.
significance of which is vital not formal. . . . (p. 427)
For those in the peculiar situation of the respondent who cannot be expected to have elected citizenship
The provision in question was enacted to correct the anomalous situation where one born of a Filipino as they were already citizens, we apply the In Re Mallare rule.
father and an alien mother was automatically granted the status of a natural-born citizen while one born
The respondent was born in an outlying rural town of Samar where there are no alien enclaves and no citizenship of his father. The citizenship of the father is relevant only to determine whether or not the
racial distinctions. The respondent has lived the life of a Filipino since birth. His father applied for respondent "chose" to be a Filipino when he came of age. At that time and up to the present, both
naturalization when the child was still a small boy. He is a Roman Catholic. He has worked for a sensitive mother and father were Filipinos. Respondent Ong could not have elected any other citizenship unless he
government agency. His profession requires citizenship for taking the examinations and getting a license. first formally renounced Philippine citizenship in favor of a foreign nationality. Unlike other persons
He has participated in political exercises as a Filipino and has always considered himself a Filipino citizen. faced with a problem of election, there was no foreign nationality of his father which he could possibly
There is nothing in the records to show that he does not embrace Philippine customs and values, nothing have chosen.
to indicate any tinge of alien-ness no acts to show that this country is not his natural homeland. The
mass of voters of Northern Samar are frilly aware of Mr. Ong's parentage. They should know him better
There is another reason why we cannot declare the HRET as having committed manifest grave abuse of
than any member of this Court will ever know him. They voted by overwhelming numbers to have him
discretion. The same issue of natural-born citizenship has already been decided by the Constitutional
represent them in Congress. Because of his acts since childhood, they have considered him as a Filipino.
Convention of 1971 and by the Batasang Pambansa convened by authority of the Constitution drafted by
that Convention. Emil Ong, full blood brother of the respondent, was declared and accepted as a natural
The filing of sworn statement or formal declaration is a requirement for those who still have to elect born citizen by both bodies.
citizenship. For those already Filipinos when the time to elect came up, there are acts of deliberate
choice which cannot be less binding. Entering a profession open only to Filipinos, serving in public office
Assuming that our opinion is different from that of the Constitutional Convention, the Batasang
where citizenship is a qualification, voting during election time, running for public office, and other
Pambansa, and the respondent HRET, such a difference could only be characterized as error. There
categorical acts of similar nature are themselves formal manifestations of choice for these persons.
would be no basis to call the HRET decision so arbitrary and whimsical as to amount to grave abuse of
discretion.
An election of Philippine citizenship presupposes that the person electing is an alien. Or his status is
doubtful because he is a national of two countries. There is no doubt in this case about Mr. Ong's being a
What was the basis for the Constitutional Convention's declaring Emil Ong a natural born citizen?
Filipino when he turned twenty-one (21).

Under the Philippine Bill of 1902, inhabitants of the Philippines who were Spanish subjects on the 11th
We repeat that any election of Philippine citizenship on the part of the private respondent would not
day of April 1899 and then residing in said islands and their children born subsequent thereto were
only have been superfluous but it would also have resulted in an absurdity. How can a Filipino citizen
conferred the status of a Filipino citizen.
elect Philippine citizenship?

Was the grandfather of the private respondent a Spanish subject?


The respondent HRET has an interesting view as to how Mr. Ong elected citizenship. It observed that
"when protestee was only nine years of age, his father, Jose Ong Chuan became a naturalized Filipino.
Section 15 of the Revised Naturalization Act squarely applies its benefit to him for he was then a minor Article 17 of the Civil Code of Spain enumerates those who were considered Spanish Subjects, viz:
residing in this country. Concededly, it was the law itself that had already elected Philippine citizenship
for protestee by declaring him as such." (Emphasis supplied) ARTICLE 17. The following are Spaniards:

The petitioners argue that the respondent's father was not, validly, a naturalized citizen because of his 1. Persons born in Spanish territory.
premature taking of the oath of citizenship.

2. Children born of a Spanish father or mother, even though they were born out of
The Court cannot go into the collateral procedure of stripping Mr. Ong's father of his citizenship after his Spain.
death and at this very late date just so we can go after the son.

3. Foreigners who may have obtained naturalization papers.


The petitioners question the citizenship of the father through a collateral approach. This can not be
done. In our jurisdiction, an attack on a person's citizenship may only be done through a direct action for
its nullity. (See Queto v. Catolico, 31 SCRA 52 [1970]) 4. Those without such papers, who may have acquired domicile in any town in the
Monarchy. (Emphasis supplied)

To ask the Court to declare the grant of Philippine citizenship to Jose Ong Chuan as null and void would
run against the principle of due process. Jose Ong Chuan has already been laid to rest. How can he be The domicile of a natural person is the place of his habitual residence. This domicile, once established is
given a fair opportunity to defend himself. A dead man cannot speak. To quote the words of the HRET considered to continue and will not be deemed lost until a new one is established. (Article 50, NCC;
"Ong Chuan's lips have long been muted to perpetuity by his demise and obviously he could not use Article 40, Civil Code of Spain; Zuellig v. Republic, 83 Phil. 768 [1949])
beyond where his mortal remains now lie to defend himself were this matter to be made a central issue
in this case." As earlier stated, Ong Te became a permanent resident of Laoang, Samar around 1895. Correspondingly,
a certificate of residence was then issued to him by virtue of his being a resident of Laoang, Samar.
The issue before us is not the nullification of the grant of citizenship to Jose Ong Chuan. Our function is (Report of the Committee on Election Protests and Credentials of the 1971 Constitutional Convention,
to determine whether or not the HRET committed abuse of authority in the exercise of its powers. September 7, 1972, p. 3)
Moreover, the respondent traces his natural born citizenship through his mother, not through the
The domicile that Ong Te established in 1895 continued until April 11, 1899; it even went beyond the Since the execution of the document and the inability to produce were adequately established, the
turn of the 19th century. It is also in this place were Ong Te set-up his business and acquired his real contents of the questioned documents can be proven by a copy thereof or by the recollection of
property. witnesses.

As concluded by the Constitutional Convention, Ong Te falls within the meaning of sub-paragraph 4 of Moreover, to erase all doubts as to the authenticity of the documentary evidence cited in the Committee
Article 17 of the Civil Code of Spain. Report, the former member of the 1971 Constitutional Convention, Atty. Nolledo, when he was
presented as a witness in the hearing of the protest against the private respondent, categorically stated
that he saw the disputed documents presented during the hearing of the election protest against the
Although Ong Te made brief visits to China, he, nevertheless, always returned to the Philippines. The fact
brother of the private respondent. (TSN, February 1, 1989, pp. 8-9)
that he died in China, during one of his visits in said country, was of no moment. This will not change the
fact that he already had his domicile fixed in the Philippines and pursuant to the Civil Code of Spain, he
had become a Spanish subject. In his concurring opinion, Mr. Justice Sarmiento, a vice-president of the Constitutional Convention, states
that he was presiding officer of the plenary session which deliberated on the report on the election
protest against Delegate Emil Ong. He cites a long list of names of delegates present. Among them are
If Ong Te became a Spanish subject by virtue of having established his domicile in a town under the
Mr. Chief Justice Fernan, and Mr. Justice Davide, Jr. The petitioners could have presented any one of the
Monarchy of Spain, necessarily, Ong Te was also an inhabitant of the Philippines for an inhabitant has
long list of delegates to refute Mr. Ong's having been declared a natural-born citizen. They did not do so.
been defined as one who has actual fixed residence in a place; one who has a domicile in a place.
Nor did they demur to the contents of the documents presented by the private respondent. They merely
(Bouvier's Law Dictionary, Vol. II) Apriori, there can be no other logical conclusion but to educe that Ong
relied on the procedural objections respecting the admissibility of the evidence presented.
Te qualified as a Filipino citizen under the provisions of section 4 of the Philippine Bill of 1902.

The Constitutional Convention was the sole judge of the qualifications of Emil Ong to be a member of
The HRET itself found this fact of absolute verity in concluding that the private respondent was a natural-
that body. The HRET by explicit mandate of the Constitution, is the sole judge of the qualifications of Jose
born Filipino.
Ong, Jr. to be a member of Congress. Both bodies deliberated at length on the controversies over which
they were sole judges. Decisions were arrived at only after a full presentation of all relevant factors
The petitioners' sole ground in disputing this fact is that document presented to prove it were not in which the parties wished to present. Even assuming that we disagree with their conclusions, we cannot
compliance with the best the evidence rule. The petitioners allege that the private respondent failed to declare their acts as committed with grave abuse of discretion. We have to keep clear the line
present the original of the documentary evidence, testimonial evidence and of the transcript of the between error and grave abuse.
proceedings of the body which the aforesaid resolution of the 1971 Constitutional Convention was
predicated.
ON THE ISSUE OF RESIDENCE

On the contrary, the documents presented by the private respondent fall under the exceptions to the
The petitioners question the residence qualification of respondent Ong.
best evidence rule.

The petitioners lose sight of the meaning of "residence" under the Constitution. The term "residence"
It was established in the proceedings before the HRET that the originals of the Committee Report No. 12,
has been understood as synonymous with domicile not only under the previous Constitutions but also
the minutes of the plenary session of 1971 Constitutional Convention held on November 28, 1972 cannot
under the 1987 Constitution.
be found.

The deliberations of the Constitutional Commission reveal that the meaning of residence vis-a-vis the
This was affirmed by Atty. Ricafrente, Assistant Secretary of the 1971 Constitutional Convention; by Atty.
qualifications of a candidate for Congress continues to remain the same as that of domicile, to wit:
Nolledo, Delegate to the 1971 Constitutional Convention; and by Atty. Antonio Santos, Chief Librarian of
the U.P Law Center, in their respective testimonies given before the HRET to the effect that there is no
governmental agency which is the official custodian of the records of the 1971 Constitutional Mr. Nolledo: With respect to Section 5, I remember that in
Convention. (TSN, December 12, 1988, pp. 30-31; TSN, January 17, 1989, pp. 34-35; TSN, February 1, the 1971 Constitutional Convention, there was an attempt
1989, p. 44; TSN, February 6, 1989, pp. 28-29) to require residence in the place not less than one year
immediately preceding the day of the elections. So my
question is: What is the Committee's concept of residence of
The execution of the originals was established by Atty. Ricafrente, who as the Assistant Secretary of the
a candidate for the legislature? Is it actual residence or is it
1971 Constitutional Convention was the proper party to testify to such execution. (TSN, December 12,
the concept of domicile or constructive residence?
1989, pp. 11-24)

Mr. Davide: Madame President, in so far as the regular


The inability to produce the originals before the HRET was also testified to as aforestated by Atty.
members of the National Assembly are concerned, the
Ricafrente, Atty. Nolledo, and Atty. Santos. In proving the inability to produce, the law does not require
proposed section merely provides, among others, and a
the degree of proof to be of sufficient certainty; it is enough that it be shown that after a bona fide
resident thereof, that is, in the district, for a period of not
diligent search, the same cannot be found. (see Government of P.I. v. Martinez, 44 Phil. 817 [1918])
less than one year preceding the day of the election. This
was in effect lifted from the 1973 Constitution, the
interpretation given to it was domicile. (Records of the 1987 the co-owner thereof (as a co- heir), notwithstanding the fact that these were still in the names of his
Constitutional Convention, Vol. 11, July 22, 1986. p. 87) parents.

xxx xxx xxx Even assuming that the private respondent does not own any property in Samar, the Supreme Court in
the case ofDe los Reyes v. Solidum (61 Phil. 893 [1935]) held that it is not required that a person should
have a house in order to establish his residence and domicile. It is enough that he should live in the
Mrs. Rosario Braid: The next question is on Section 7, page
municipality or in a rented house or in that of a friend or relative. (Emphasis supplied)
2. I think Commissioner Nolledo has raised the same point
that "resident" has been interpreted at times as a matter of
intention rather than actual residence. To require the private respondent to own property in order to be eligible to run for Congress would be
tantamount to a property qualification. The Constitution only requires that the candidate meet the age,
citizenship, voting and residence requirements. Nowhere is it required by the Constitution that the
Mr. De los Reyes: Domicile.
candidate should also own property in order to be qualified to run. (see Maquera v. Borra, 122 Phil. 412
[1965])
Ms. Rosario Braid: Yes, So, would the gentlemen consider at
the proper time to go back to actual residence rather than
It has also been settled that absence from residence to pursue studies or practice a profession or
mere intention to reside?
registration as a voter other than in the place where one is elected, does not constitute loss of residence.
(Faypon v. Quirino, 96 Phil. 294 [1954])
Mr. De los Reyes: But we might encounter some difficulty
especially considering that a provision in the Constitution in
As previously stated, the private respondent stayed in Manila for the purpose of finishing his studies and
the Article on Suffrage says that Filipinos living abroad may
later to practice his profession, There was no intention to abandon the residence in Laoang, Samar. On
vote as enacted by law. So, we have to stick to the original
the contrary, the periodical journeys made to his home province reveal that he always had
concept that it should be by domicile and not physical and
the animus revertendi.
actual residence. (Records of the 1987 Constitutional
Commission, Vol. 11, July 22, 1986, p. 110)
The Philippines is made up not only of a single race; it has, rather, undergone an interracial evolution.
Throughout our history, there has been a continuing influx of Malays, Chinese, Americans, Japanese,
The framers of the Constitution adhered to the earlier definition given to the word "residence" which
Spaniards and other nationalities. This racial diversity gives strength to our country.
regarded it as having the same meaning as domicile.

Many great Filipinos have not been whole-blooded nationals, if there is such a person, for there is none.
The term "domicile" denotes a fixed permanent residence to which when absent for business or
To mention a few, the great Jose Rizal was part Chinese, the late Chief Justice Claudio Teehankee was
pleasure, one intends to return. (Ong Huan Tin v. Republic, 19 SCRA 966 [1967]) The absence of a person
part Chinese, and of course our own President, Corazon Aquino is also part Chinese. Verily, some
from said permanent residence, no matter how long, notwithstanding, it continues to be the domicile of
Filipinos of whom we are proud were ethnically more Chinese than the private respondent.
that person. In other words, domicile is characterized by animus revertendi (Ujano v. Republic, 17 SCRA
147 [1966])
Our citizens no doubt constitute the country's greatest wealth. Citizenship is a special privilege which
one must forever cherish.
The domicile of origin of the private respondent, which was the domicile of his parents, is fixed at
Laoang, Samar. Contrary to the petitioners' imputation, Jose Ong, Jr. never abandoned said domicile; it
remained fixed therein even up to the present. However, in order to truly revere this treasure of citizenship, we do not, on the basis of too harsh an
interpretation, have to unreasonably deny it to those who qualify to share in its richness.
The private respondent, in the proceedings before the HRET sufficiently established that after the fire
that gutted their house in 1961, another one was constructed. Under the overly strict jurisprudence surrounding our antiquated naturalization laws only the very
affluent backed by influential patrons, who were willing to suffer the indignities of a lengthy, sometimes
humiliating, and often corrupt process of clearances by minor bureaucrats and whose lawyers knew how
Likewise, after the second fire which again destroyed their house in 1975, a sixteen-door apartment was
to overcome so many technical traps of the judicial process were able to acquire citizenship. It is time for
built by their family, two doors of which were reserved as their family residence. (TSN, Jose Ong, Jr.,
the naturalization law to be revised to enable a more positive, affirmative, and meaningful examination
November 18,1988, p. 8)
of an applicant's suitability to be a Filipino. A more humane, more indubitable and less technical
approach to citizenship problems is essential.
The petitioners' allegation that since the private respondent owns no property in Laoang, Samar, he
cannot, therefore, be a resident of said place is misplaced.
WHEREFORE, the petitions are hereby DISMISSED. The questioned decision of the House of
Representatives Electoral Tribunal is AFFIRMED. Respondent Jose Ong, Jr. is declared a natural-born
The properties owned by the Ong Family are in the name of the private respondent's parents. Upon the citizen of the Philippines and a resident of Laoang, Northern Samar.
demise of his parents, necessarily, the private respondent, pursuant to the laws of succession, became
Republic of the Philippines country; And provided, finally, That any Filipino citizen who is rendering service to, or is
SUPREME COURT commissioned in, the armed forces of a foreign country under any of the circumstances
Manila mentioned in paragraph (a) or (b), shall not be Republic of the Philippines during the period of
his service to, or commission in, the armed forces of said country. Upon his discharge from
the service of the said foreign country, he shall be automatically entitled to the full enjoyment
EN BANC
of his civil and politically entitled to the full enjoyment of his civil political rights as a Filipino
citizen x x x.
G.R. No. 142840 May 7, 2001
Whatever doubt that remained regarding his loss of Philippine citizenship was erased by his
ANTONIO BENGSON III, petitioner, naturalization as a U.S. citizen on June 5, 1990, in connection with his service in the U.S. Marine Corps.
vs.
HOUSE OF REPRESENTATIVES ELECTORAL TRIBUNAL and TEODORO C. CRUZ, respondents.
On March 17, 1994, respondent Cruz reacquired his Philippine citizenship through repatriation under
Republic Act No. 2630.3 He ran for and was elected as the Representative of the Second District of
CONCURRING OPINION Pangasinan in the May 11, 1998 elections. He won by a convincing margin of 26,671 votes over
petitioner Antonio Bengson III, who was then running for reelection.1wphi1.nt
DISSENTING OPINION
Subsequently, petitioner filed a case for Quo Warranto Ad Cautelam with respondent House of
KAPUNAN, J.: Representatives Electoral Tribunal (HRET) claiming that respondent Cruz was not qualified to become a
member of the House of Representatives since he is not a natural-born citizen as required under Article
VI, section 6 of the Constitution.4
The citizenship of respondent Teodoro C. Cruz is at issue in this case, in view of the constitutional
requirement that "no person shall be a Member of the House of Representative unless he is a natural-
born citizen."1 On March 2, 2000, the HRET rendered its decision5 dismissing the petition for quo warranto and
declaring Cruz the duly elected Representative of the Second District of Pangasinan in the May 1998
elections. The HRET likewise denied petitioner's motion for reconsideration of the decision in its
Respondent Cruz was a natural-born citizen of the Philippines. He was born in San Clemente, Tarlac, on resolution dated April 27, 2000.6
April 27, 1960, of Filipino parents. The fundamental law then applicable was the 1935 Constitution.2

Petitioner thus filed the present petition for certiorari assailing the HRET's decision on the following
On November 5, 1985, however, respondent Cruz enlisted in the United States Marine Corps and grounds:
without the consent of the Republic of the Philippines, took an oath of allegiance to the United States. As
a Consequence, he lost his Filipino citizenship for under Commonwealth Act No. 63, section 1(4), a
Filipino citizen may lose his citizenship by, among other, "rendering service to or accepting commission 1. The HRET committed serious errors and grave abuse of discretion, amounting to excess of
in the armed forces of a foreign country." Said provision of law reads: jurisdiction, when it ruled that private respondent is a natural-born citizen of the
Philippines despite the fact that he had ceased being such in view of the loss and renunciation
of such citizenship on his part.
SECTION 1. How citizenship may be lost. A Filipino citizen may lose his citizenship in any of
the following ways and/or events:
2. The HRET committed serious errors and grave abuse of discretion, amounting to excess of
jurisdiction, when it considered private respondent as a citizen of the Philippines despite the
xxx fact he did not validly acquire his Philippine citizenship.

(4) By rendering services to, or accepting commission in, the armed of a foreign 3. Assuming that private respondent's acquisition of Philippine citizenship was invalid, the
country: Provided, That the rendering of service to, or the acceptance of such commission in, HRET committed serious errors and grave abuse of discretion, amounting to excess of
the armed forces of a foreign country, and the taking of an oath of allegiance incident jurisdiction, when it dismissed the petition despite the fact that such reacquisition could not
thereto, with the consent of the Republic of the Philippines, shall not divest a Filipino of his legally and constitutionally restore his natural-born status.7
Philippine citizenship if either of the following circumstances is present:

The issue now before us is whether respondent Cruz, a natural-born Filipino who became an American
(a) The Republic of the Philippines has a defensive and/or offensive pact of alliance with said citizen, can still be considered a natural-born Filipino upon his reacquisition of Philippine citizenship.
foreign country; or

Petitioner asserts that respondent Cruz may no longer be considered a natural-born Filipino since he lost
(b) The said foreign country maintains armed forces on Philippine territory with the consent h is Philippine citizenship when he swore allegiance to the United States in 1995, and had to reacquire
of the Republic of the Philippines: Provided, That the Filipino citizen concerned, at the time of the same by repatriation. He insists that Article citizens are those who are from birth with out having to
rendering said service, or acceptance of said commission, and taking the oath of allegiance perform any act to acquire or perfect such citizenship.
incident thereto, states that he does so only in connection with his service to said foreign
Respondent on the other hand contends that he reacquired his status as natural-born citizen when he II;20 (3) service in the Armed Forces of the United States at any other time, 21 (4) marriage of a Filipino
was repatriated since the phrase "from birth" in Article IV, Section 2 refers to the innate, inherent and woman to an alien;22 and (5) political economic necessity.23
inborn characteristic of being a natural-born citizen.
As distinguished from the lengthy process of naturalization, repatriation simply consists of the taking of
The petition is without merit. an oath of allegiance to the Republic of the Philippine and registering said oath in the Local Civil Registry
of the place where the person concerned resides or last resided.
The 1987 Constitution enumerates who are Filipino citizens as follow:
In Angat v. Republic,24 we held:
(1) Those who are citizens of the Philippines at the time of the adoption of this Constitution;
xxx. Parenthetically, under these statutes [referring to RA Nos. 965 and 2630], the person
desiring to reacquire Philippine citizenship would not even be required to file a petition in
(2) Those whose fathers or mothers are citizens of the Philippines;
court, and all that he had to do was to take an oath of allegiance to the Republic of the
Philippines and to register that fact with the civil registry in the place of his residence or
(3) Those born before January 17, 1973 of Filipino mother, who elect Philippine citizenship where he had last resided in the Philippines. [Italics in the original.25
upon reaching the age of majority, and
Moreover, repatriation results in the recovery of the original nationality.26 This means that a naturalized
(4) Those who are naturalized in accordance with law.8 Filipino who lost his citizenship will be restored to his prior status as a naturalized Filipino citizen. On the
other hand, if he was originally a natural-born citizen before he lost his Philippine citizenship, he will be
There are two ways of acquiring citizenship: (1) by birth, and (2) by naturalization. These ways of restored to his former status as a natural-born Filipino.
acquiring citizenship correspond to the two kinds of citizens: the natural-born citizen, and the
naturalized citizen. A person who at the time of his birth is a citizen of a particular country, is a natural- In respondent Cruz's case, he lost his Filipino citizenship when he rendered service in the Armed Forces
born citizen thereof.9 of the United States. However, he subsequently reacquired Philippine citizenship under R.A. No. 2630,
which provides:
As defined in the same Constitution, natural-born citizens "are those citizens of the Philippines from
birth without having to perform any act to acquire or perfect his Philippine citezenship."10 Section 1. Any person who had lost his Philippine citizenship by rendering service to, or
accepting commission in, the Armed Forces of the United States, or after separation from the
On the other hand, naturalized citizens are those who have become Filipino citizens through Armed Forces of the United States, acquired United States citizenship, may reacquire
naturalization, generally under Commonwealth Act No. 473, otherwise known as the Revised Philippine citizenship by taking an oath of allegiance to the Republic of the Philippines and
Naturalization Law, which repealed the former Naturalization Law (Act No. 2927), and by Republic Act registering the same with Local Civil Registry in the place where he resides or last resided in
No. 530.11 To be naturalized, an applicant has to prove that he possesses all the qualifications12 and none the Philippines. The said oath of allegiance shall contain a renunciation of any other
of the disqualification13 provided by law to become a Filipino citizen. The decision granting Philippine citizenship.
citizenship becomes executory only after two (2) years from its promulgation when the court is satisfied
that during the intervening period, the applicant has (1) not left the Philippines; (2) has dedicated Having thus taken the required oath of allegiance to the Republic and having registered the same in the
himself to a lawful calling or profession; (3) has not been convicted of any offense or violation of Civil Registry of Magantarem, Pangasinan in accordance with the aforecited provision, respondent Cruz is
Government promulgated rules; or (4) committed any act prejudicial to the interest of the nation or deemed to have recovered his original status as a natural-born citizen, a status which he acquired at
contrary to any Government announced policies.14 birth as the son of a Filipino father.27 It bears stressing that the act of repatriation allows him to recover,
or return to, his original status before he lost his Philippine citizenship.
Filipino citizens who have lost their citizenship may however reacquire the same in the manner provided
by law. Commonwealth Act. No. (C.A. No. 63), enumerates the three modes by which Philippine Petitioner's contention that respondent Cruz is no longer a natural-born citizen since he had to perform
citizenship may be reacquired by a former citizen: (1) by naturalization, (2) by repatriation, and (3) by an act to regain his citizenship is untenable. As correctly explained by the HRET in its decision, the term
direct act of Congress.15 "natural-born citizen" was first defined in Article III, Section 4 of the 1973 Constitution as follows:

Naturalization is mode for both acquisition and reacquisition of Philippine citizenship. As a mode of Sec. 4. A natural-born citizen is one who is a citizen of the Philippines from birth without
initially acquiring Philippine citizenship, naturalization is governed by Commonwealth Act No. 473, as having to perform any act to acquire or perfect his Philippine citizenship.
amended. On the other hand, naturalization as a mode for reacquiring Philippine citizenship is governed
by Commonwealth Act No. 63.16 Under this law, a former Filipino citizen who wishes to reacquire
Two requisites must concur for a person to be considered as such: (1) a person must be a Filipino citizen
Philippine citizenship must possess certain qualifications17 and none of the disqualification mentioned in
birth and (2) he does not have to perform any act to obtain or perfect his Philippine citizenship.
Section 4 of C.A. 473.18

Under the 1973 Constitution definition, there were two categories of Filipino citizens which were not
Repatriation, on the other hand, may be had under various statutes by those who lost their citizenship
considered natural-born: (1) those who were naturalized and (2) those born before January 17,
due to: (1) desertion of the armed forces;19 services in the armed forces of the allied forces in World War
1973,38 of Filipino mothers who, upon reaching the age of majority, elected Philippine citizenship. Those
"naturalized citizens" were not considered natural-born obviously because they were not Filipino at birth
and had to perform an act to acquire Philippine citizenship. Those born of Filipino mothers before the
effectively of the 1973 Constitution were likewise not considered natural-born because they also had to
perform an act to perfect their Philippines citizenship.

The present Constitution, however, now consider those born of Filipino mothers before the effectivity of
the 1973 Constitution and who elected Philippine citizenship upon reaching the majority age as natural-
born. After defining who re natural-born citizens, Section 2 of Article IV adds a sentence: "Those who
elect Philippine citizenship in accordance with paragraph (3), Section 1 hereof shall be deemed natural-
born citizens." Consequently, only naturalized Filipinos are considered not natural-born citizens. It is
apparent from the enumeration of who are citizens under the present Constitution that there are only
two classes of citizens: (1) those who are natural-born and (2) those who are naturalized in accordance
with law. A citizen who is not a naturalized Filipino, i.e., did not have to undergo the process of
naturalization to obtain Philippine citizenship, necessarily is natural-born Filipino. Noteworthy is the
absence in said enumeration of a separate category for persons who, after losing Philippine citizenship,
subsequently reacquire it. The reason therefor is clear: as to such persons, they would either be natural-
born or naturalized depending on the reasons for the loss of their citizenship and the mode prescribed
by the applicable law for the reacquisition thereof. As respondent Cruz was not required by law to go
through naturalization proceeding in order to reacquire his citizenship, he is perforce a natural-born
Filipino. As such, he possessed all the necessary qualifications to be elected as member of the House of
Representatives.

A final point. The HRET has been empowered by the Constitution to be the "sole judge" of all contests
relating to the election, returns, and qualifications of the members of the House. 29 The Court's
jurisdiction over the HRET is merely to check "whether or not there has been a grave abuse of discretion
amounting to lack or excess of jurisdiction" on the part of the latter.30 In the absence thereof, there is no
occasion for the Court to exercise its corrective power and annul the decision of the HRET nor to
substitute the Court's judgement for that of the latter for the simple reason that it is not the office of a
petition for certiorari to inquire into the correctness of the assailed decision.31 There is no such showing
of grave abuse of discretion in this case.

WHEREFORE, the petition is hereby DISMISSED.

SO ORDERED.
Republic of the Philippines considered in American citizen under US Laws. But notwithstanding his registration
SUPREME COURT as an American citizen, he did not lose his Filipino citizenship.
Manila
Judging from the foregoing facts, it would appear that respondent Manzano is
EN BANC born a Filipino and a US citizen. In other words, he holds dual citizenship.

The question presented is whether under our laws, he is disqualified from the
position for which he filed his certificate of candidacy. Is he eligible for the office
he seeks to be elected?
G.R. No. 135083 May 26, 1999

Under Section 40(d) of the Local Government Code, those holding dual citizenship
ERNESTO S. MERCADO, petitioner,
are disqualified from running for any elective local position.
vs.
EDUARDO BARRIOS MANZANO and the COMMISSION ON ELECTIONS, respondents.
WHEREFORE, the Commission hereby declares the respondent Eduardo Barrios
Manzano DISQUALIFIED as candidate for Vice-Mayor of Makati City.

On May 8, 1998, private respondent filed a motion for reconsideration. 3 The motion remained pending
MENDOZA, J.:
even until after the election held on May 11, 1998.

Petitioner Ernesto S. Mercado and private respondent Eduardo B. Manzano were candidates for vice
Accordingly, pursuant to Omnibus Resolution No. 3044, dated May 10, 1998, of the COMELEC, the board
mayor of the City of Makati in the May 11, 1998 elections. The other one was Gabriel V. Daza III. The
of canvassers tabulated the votes cast for vice mayor of Makati City but suspended the proclamation of
results of the election were as follows:
the winner.

Eduardo B. Manzano 103,853


On May 19, 1998, petitioner sought to intervene in the case for disqualification. 4 Petitioner's motion was
opposed by private respondent.
Ernesto S. Mercado 100,894
The motion was not resolved. Instead, on August 31, 1998, the COMELEC en banc rendered its
Gabriel V. Daza III 54,275 1 resolution. Voting 4 to 1, with one commissioner abstaining, the COMELEC en banc reversed the ruling of
its Second Division and declared private respondent qualified to run for vice mayor of the City of Makati
The proclamation of private respondent was suspended in view of a pending petition for disqualification in the May 11, 1998 elections. 5The pertinent portions of the resolution of the COMELEC en banc read:
filed by a certain Ernesto Mamaril who alleged that private respondent was not a citizen of the
Philippines but of the United States. As aforesaid, respondent Eduardo Barrios Manzano was born in San Francisco,
California, U.S.A. He acquired US citizenship by operation of the United States
In its resolution, dated May 7, 1998, 2 the Second Division of the COMELEC granted the petition of Constitution and laws under the principle ofjus soli.
Mamaril and ordered the cancellation of the certificate of candidacy of private respondent on the
ground that he is a dual citizen and, under 40(d) of the Local Government Code, persons with dual He was also a natural born Filipino citizen by operation of the 1935 Philippine
citizenship are disqualified from running for any elective position. The COMELEC's Second Division said: Constitution, as his father and mother were Filipinos at the time of his birth. At the
age of six (6), his parents brought him to the Philippines using an American
What is presented before the Commission is a petition for disqualification of passport as travel document. His parents also registered him as an alien with the
Eduardo Barrios Manzano as candidate for the office of Vice-Mayor of Makati City Philippine Bureau of Immigration. He was issued an alien certificate of registration.
in the May 11, 1998 elections. The petition is based on the ground that the This, however, did not result in the loss of his Philippine citizenship, as he did not
respondent is an American citizen based on the record of the Bureau of renounce Philippine citizenship and did not take an oath of allegiance to the
Immigration and misrepresented himself as a natural-born Filipino citizen. United States.

In his answer to the petition filed on April 27, 1998, the respondent admitted that It is an undisputed fact that when respondent attained the age of majority, he
he is registered as a foreigner with the Bureau of Immigration under Alien registered himself as a voter, and voted in the elections of 1992, 1995 and 1998,
Certificate of Registration No. B-31632 and alleged that he is a Filipino citizen which effectively renounced his US citizenship under American law. Under
because he was born in 1955 of a Filipino father and a Filipino mother. He was Philippine law, he no longer had U.S. citizenship.
born in the United States, San Francisco, California, September 14, 1955, and is
At the time of the May 11, 1998 elections, the resolution of the Second Division, We first consider the threshold procedural issue raised by private respondent Manzano whether
adopted on May 7, 1998, was not yet final. Respondent Manzano obtained the petitioner Mercado his personality to bring this suit considering that he was not an original party in the
highest number of votes among the candidates for vice-mayor of Makati City, case for disqualification filed by Ernesto Mamaril nor was petitioner's motion for leave to intervene
garnering one hundred three thousand eight hundred fifty three (103,853) votes granted.
over his closest rival, Ernesto S. Mercado, who obtained one hundred thousand
eight hundred ninety four (100,894) votes, or a margin of two thousand nine
I. PETITIONER'S RIGHT TO BRING THIS SUIT
hundred fifty nine (2,959) votes. Gabriel Daza III obtained third place with fifty four
thousand two hundred seventy five (54,275) votes. In applying election laws, it
would be far better to err in favor of the popular choice than be embroiled in Private respondent cites the following provisions of Rule 8 of the Rules of Procedure of the COMELEC in
complex legal issues involving private international law which may well be settled support of his claim that petitioner has no right to intervene and, therefore, cannot bring this suit to set
before the highest court (Cf. Frivaldo vs. Commission on Elections, 257 SCRA 727). aside the ruling denying his motion for intervention:

WHEREFORE, the Commission en banc hereby REVERSES the resolution of the Sec. 1. When proper and when may be permitted to intervene. Any person
Second Division, adopted on May 7, 1998, ordering the cancellation of the allowed to initiate an action or proceeding may, before or during the trial of an
respondent's certificate of candidacy. action or proceeding, be permitted by the Commission, in its discretion to
intervene in such action or proceeding, if he has legal interest in the matter in
litigation, or in the success of either of the parties, or an interest against both, or
We declare respondent Eduardo Luis Barrios Manzano to be QUALIFIED as a
when he is so situated as to be adversely affected by such action or proceeding.
candidate for the position of vice-mayor of Makati City in the May 11, 1998,
elections.
xxx xxx xxx
ACCORDINGLY, the Commission directs the Makati City Board of Canvassers, upon
proper notice to the parties, to reconvene and proclaim the respondent Eduardo Sec. 3. Discretion of Commission. In allowing or disallowing a motion for
Luis Barrios Manzano as the winning candidate for vice-mayor of Makati City. intervention, the Commission or the Division, in the exercise of its discretion, shall
consider whether or not the intervention will unduly delay or prejudice the
adjudication of the rights of the original parties and whether or not the
Pursuant to the resolution of the COMELEC en banc, the board of canvassers, on the evening of August
intervenor's rights may be fully protected in a separate action or proceeding.
31, 1998, proclaimed private respondent as vice mayor of the City of Makati.

Private respondent argues that petitioner has neither legal interest in the matter in litigation
This is a petition for certiorari seeking to set aside the aforesaid resolution of the COMELEC en banc and
nor an interest to protect because he is "a defeated candidate for the vice-mayoralty post of
to declare private respondent disqualified to hold the office of vice mayor of Makati City. Petitioner
Makati City [who] cannot be proclaimed as the Vice-Mayor of Makati City if the private
contends that
respondent be ultimately disqualified by final and executory judgment."

[T]he COMELEC en banc ERRED in holding that:


The flaw in this argument is it assumes that, at the time petitioner sought to intervene in the
proceedings before the COMELEC, there had already been a proclamation of the results of the election
A. Under Philippine law, Manzano was no longer a U.S. citizen when he: for the vice mayoralty contest for Makati City, on the basis of which petitioner came out only second to
private respondent. The fact, however, is that there had been no proclamation at that time. Certainly,
petitioner had, and still has, an interest in ousting private respondent from the race at the time he
1. He renounced his U.S. citizenship when he attained the
sought to intervene. The rule in Labo v. COMELEC, 6 reiterated in several cases, 7 only applies to cases in
age of majority when he was already 37 years old; and,
which the election of the respondent is contested, and the question is whether one who placed second
to the disqualified candidate may be declared the winner. In the present case, at the time petitioner filed
2. He renounced his U.S. citizenship when he (merely) a "Motion for Leave to File Intervention" on May 20, 1998, there had been no proclamation of the
registered himself as a voter and voted in the elections of winner, and petitioner's purpose was precisely to have private respondent disqualified "from running for
1992, 1995 and 1998. [an] elective local position" under 40(d) of R.A. No. 7160. If Ernesto Mamaril (who originally instituted
the disqualification proceedings), a registered voter of Makati City, was competent to bring the action,
B. Manzano is qualified to run for and or hold the elective office of Vice-Mayor of so was petitioner since the latter was a rival candidate for vice mayor of Makati City.
the City of Makati;
Nor is petitioner's interest in the matter in litigation any less because he filed a motion for intervention
C. At the time of the May 11, 1998 elections, the resolution of the Second Division only on May 20, 1998, after private respondent had been shown to have garnered the highest number of
adopted on 7 May 1998 was not yet final so that, effectively, petitioner may not be votes among the candidates for vice mayor. That petitioner had a right to intervene at that stage of the
declared the winner even assuming that Manzano is disqualified to run for and proceedings for the disqualification against private respondent is clear from 6 of R.A. No. 6646,
hold the elective office of Vice-Mayor of the City of Makati. otherwise known as the Electoral Reform Law of 1987, which provides:
Any candidate who his been declared by final judgment to be disqualified shall not There may be other situations in which a citizen of the Philippines may, without performing any act, be
be voted for, and the votes cast for him shall not be counted. If for any reason a also a citizen of another state; but the above cases are clearly possible given the constitutional provisions
candidate is not declared by final judgment before an election to be disqualified on citizenship.
and he is voted for and receives the winning number of votes in such election, the
Court or Commission shall continue with the trial and hearing of action, inquiry, or
Dual allegiance, on the other hand, refers to the situation in which a person simultaneously owes, by
protest and, upon motion of the complainant or any intervenor, may during the
some positive act, loyalty to two or more states. While dual citizenship is involuntary, dual allegiance is
pendency thereof order the suspension of the proclamation of such candidate
the result of an individual's volition.
whenever the evidence of guilt is strong.

With respect to dual allegiance, Article IV, 5 of the Constitution provides: "Dual allegiance of citizens is
Under this provision, intervention may be allowed in proceedings for disqualification even after election
inimical to the national interest and shall be dealt with by law." This provision was included in the 1987
if there has yet been no final judgment rendered.
Constitution at the instance of Commissioner Blas F. Ople who explained its necessity as follows: 10

The failure of the COMELEC en banc to resolve petitioner's motion for intervention was tantamount to a
. . . I want to draw attention to the fact that dual allegiance is not dual citizenship. I
denial of the motion, justifying petitioner in filing the instant petition for certiorari. As the COMELEC en
have circulated a memorandum to the Bernas Committee according to which a
banc instead decided the merits of the case, the present petition properly deals not only with the denial
dual allegiance and I reiterate a dual allegiance is larger and more
of petitioner's motion for intervention but also with the substantive issues respecting private
threatening than that of mere double citizenship which is seldom intentional and,
respondent's alleged disqualification on the ground of dual citizenship.
perhaps, never insidious. That is often a function of the accident of mixed
marriages or of birth on foreign soil. And so, I do not question double citizenship at
This brings us to the next question, namely, whether private respondent Manzano possesses dual all.
citizenship and, if so, whether he is disqualified from being a candidate for vice mayor of Makati City.
What we would like the Committee to consider is to take constitutional cognizance
II. DUAL CITIZENSHIP AS A GROUND FOR DISQUALIFICATION of the problem of dual allegiance. For example, we all know what happens in the
triennial elections of the Federation of Filipino-Chinese Chambers of Commerce
which consists of about 600 chapters all over the country. There is a Peking ticket,
The disqualification of private respondent Manzano is being sought under 40 of the Local Government
as well as a Taipei ticket. Not widely known is the fact chat the Filipino-Chinese
Code of 1991 (R.A. No. 7160), which declares as "disqualified from running for any elective local position:
community is represented in the Legislative Yuan of the Republic of China in
. . . (d) Those with dual citizenship." This provision is incorporated in the Charter of the City of Makati. 8
Taiwan. And until recently, sponsor might recall, in Mainland China in the People's
Republic of China, they have the Associated Legislative Council for overseas
Invoking the maxim dura lex sed lex, petitioner, as well as the Solicitor General, who sides with him in Chinese wherein all of Southeast Asia including some European and Latin countries
this case, contends that through 40(d) of the Local Government Code, Congress has "command[ed] in were represented, which was dissolved after several years because of diplomatic
explicit terms the ineligibility of persons possessing dual allegiance to hold local elective office." friction. At that time, the Filipino-Chinese were also represented in that Overseas
Council.
To begin with, dual citizenship is different from dual allegiance. The former arises when, as a result of the
concurrent application of the different laws of two or more states, a person is simultaneously considered When I speak of double allegiance, therefore, I speak of this unsettled kind of
a national by the said states. 9 For instance, such a situation may arise when a person whose parents are allegiance of Filipinos, of citizens who are already Filipinos but who, by their acts,
citizens of a state which adheres to the principle of jus sanguinis is born in a state which follows the may be said to be bound by a second allegiance, either to Peking or Taiwan. I also
doctrine of jus soli. Such a person, ipso factoand without any voluntary act on his part, is concurrently took close note of the concern expressed by some Commissioners yesterday,
considered a citizen of both states. Considering the citizenship clause (Art. IV) of our Constitution, it is including Commissioner Villacorta, who were concerned about the lack of
possible for the following classes of citizens of the Philippines to possess dual citizenship: guarantees of thorough assimilation, and especially Commissioner Concepcion
who has always been worried about minority claims on our natural resources.
(1) Those born of Filipino fathers and/or mothers in foreign countries which follow
the principle of jus soli; Dull allegiance can actually siphon scarce national capital to Taiwan, Singapore,
China or Malaysia, and this is already happening. Some of the great commercial
(2) Those born in the Philippines of Filipino mothers and alien fathers if by the laws places in downtown Taipei are Filipino-owned, owned by Filipino-Chinese it is of
of their father's' country such children are citizens of that country; common knowledge in Manila. It can mean a tragic capital outflow when we have
to endure a capital famine which also means economic stagnation, worsening
unemployment and social unrest.
(3) Those who marry aliens if by the laws of the latter's country the former are
considered citizens, unless by their act or omission they are deemed to have
renounced Philippine citizenship. And so, this is exactly what we ask that the Committee kindly consider
incorporating a new section, probably Section 5, in the article on Citizenship which
will read as follows: DUAL ALLEGIANCE IS INIMICAL TO CITIZENSHIP AND SHALL BE
DEALT WITH ACCORDING TO LAW.
In another session of the Commission, Ople spoke on the problem of these citizens with dual allegiance, On the assumption that this person would carry two passports, one belonging to
thus: 11 the country of his or her father and one belonging to the Republic of the
Philippines, may such a situation disqualify the person to run for a local
government position?
. . . A significant number of Commissioners expressed their concern about dual
citizenship in the sense that it implies a double allegiance under a double
sovereignty which some of us who spoke then in a freewheeling debate thought SENATOR PIMENTEL. To my mind, Mr. President, it only means that at the moment
would be repugnant to the sovereignty which pervades the Constitution and to when he would want to run for public office, he has to repudiate one of his
citizenship itself which implies a uniqueness and which elsewhere in the citizenships.
Constitution is defined in terms of rights and obligations exclusive to that
citizenship including, of course, the obligation to rise to the defense of the State
SENATOR ENRILE. Suppose he carries only a Philippine passport but the country of
when it is threatened, and back of this, Commissioner Bernas, is, of course, the
origin or the country of the father claims that person, nevertheless, as a citizen?
concern for national security. In the course of those debates, I think some noted
No one can renounce. There are such countries in the world.
the fact that as a result of the wave of naturalizations since the decision to
establish diplomatic relations with the People's Republic of China was made in
1975, a good number of these naturalized Filipinos still routinely go to Taipei every SENATOR PIMENTEL. Well, the very fact that he is running for public office would,
October 10; and it is asserted that some of them do renew their oath of allegiance in effect, be an election for him of his desire to be considered as a Filipino citizen.
to a foreign government maybe just to enter into the spirit of the occasion when
the anniversary of the Sun Yat-Sen Republic is commemorated. And so, I have SENATOR ENRILE. But, precisely, Mr. President, the Constitution does not require
detected a genuine and deep concern about double citizenship, with its attendant an election. Under the Constitution, a person whose mother is a citizen of the
risk of double allegiance which is repugnant to our sovereignty and national Philippines is, at birth, a citizen without any overt act to claim the citizenship.
security. I appreciate what the Committee said that this could be left to the
determination of a future legislature. But considering the scale of the problem, the
real impact on the security of this country, arising from, let us say, potentially SENATOR PIMENTEL. Yes. What we are saying, Mr. President, is: Under the
great numbers of double citizens professing double allegiance, will the Committee Gentleman's example, if he does not renounce his other citizenship, then he is
entertain a proposed amendment at the proper time that will prohibit, in effect, or opening himself to question. So, if he is really interested to run, the first thing he
regulate double citizenship? should do is to say in the Certificate of Candidacy that: "I am a Filipino citizen, and I
have only one citizenship."

Clearly, in including 5 in Article IV on citizenship, the concern of the Constitutional Commission was not
with dual citizens per se but with naturalized citizens who maintain their allegiance to their countries of SENATOR ENRILE. But we are talking from the viewpoint of Philippine law, Mr.
origin even after their naturalization. Hence, the phrase "dual citizenship" in R.A. No. 7160, 40(d) and in President. He will always have one citizenship, and that is the citizenship invested
R.A. No. 7854, 20 must be understood as referring to "dual allegiance." Consequently, persons with upon him or her in the Constitution of the Republic.
mere dual citizenship do not fall under this disqualification. Unlike those with dual allegiance, who must,
therefore, be subject to strict process with respect to the termination of their status, for candidates with SENATOR PIMENTEL. That is true, Mr. President. But if he exercises acts that will
dual citizenship, it should suffice if, upon the filing of their certificates of candidacy, they elect Philippine prove that he also acknowledges other citizenships, then he will probably fall
citizenship to terminate their status as persons with dual citizenship considering that their condition is under this disqualification.
the unavoidable consequence of conflicting laws of different states. As Joaquin G. Bernas, one of the
most perceptive members of the Constitutional Commission, pointed out: "[D]ual citizenship is just a
reality imposed on us because we have no control of the laws on citizenship of other countries. We This is similar to the requirement that an applicant for naturalization must renounce "all allegiance and
recognize a child of a Filipino mother. But whether she is considered a citizen of another country is fidelity to any foreign prince, potentate, state, or sovereignty" 14 of which at the time he is a subject or
something completely beyond our control." 12 citizen before he can be issued a certificate of naturalization as a citizen of the Philippines. In Parado
v. Republic, 15 it was held:

By electing Philippine citizenship, such candidates at the same time forswear allegiance to the other
country of which they are also citizens and thereby terminate their status as dual citizens. It may be that, [W]hen a person applying for citizenship by naturalization takes an oath that he
from the point of view of the foreign state and of its laws, such an individual has not effectively renounce, his loyalty to any other country or government and solemnly declares
renounced his foreign citizenship. That is of no moment as the following discussion on 40(d) between that he owes his allegiance to the Republic of the Philippines, the condition
Senators Enrile and Pimentel clearly shows:13 imposed by law is satisfied and compiled with. The determination whether such
renunciation is valid or fully complies with the provisions of our Naturalization Law
lies within the province and is an exclusive prerogative of our courts. The latter
SENATOR ENRILE. Mr. President, I would like to ask clarification of line 41, page 17: should apply the law duly enacted by the legislative department of the Republic.
"Any person with dual citizenship" is disqualified to run for any elective local No foreign law may or should interfere with its operation and application. If the
position. Under the present Constitution, Mr. President, someone whose mother is requirement of the Chinese Law of Nationality were to be read into our
a citizen of the Philippines but his father is a foreigner is a natural-born citizen of Naturalization Law, we would be applying not what our legislative department has
the Republic. There is no requirement that such a natural born citizen, upon deemed it wise to require, but what a foreign government has thought or intended
reaching the age of majority, must elect or give up Philippine citizenship. to exact. That, of course, is absurd. It must be resisted by all means and at all cost.
It would be a brazen encroachment upon the sovereign will and power of the It is not disputed that on January 20, 1983 Frivaldo became an American. Would
people of this Republic. the retroactivity of his repatriation not effectively give him dual citizenship, which
under Sec. 40 of the Local Government Code would disqualify him "from running
for any elective local position?" We answer this question in the negative, as there
III. PETITIONER'S ELECTION OF PHILIPPINE CITIZENSHIP
is cogent reason to hold that Frivaldo was really STATELESS at the time he took
said oath of allegiance and even before that, when he ran for governor in 1988. In
The record shows that private respondent was born in San Francisco, California on September 4, 1955, of his Comment, Frivaldo wrote that he "had long renounced and had long
Filipino parents. Since the Philippines adheres to the principle of jus sanguinis, while the United States abandoned his American citizenship long before May 8, 1995. At best, Frivaldo
follows the doctrine of jus soli, the parties agree that, at birth at least, he was a national both of the was stateless in the interim when he abandoned and renounced his US
Philippines and of the United States. However, the COMELEC en banc held that, by participating in citizenship but before he was repatriated to his Filipino citizenship."
Philippine elections in 1992, 1995, and 1998, private respondent "effectively renounced his U.S.
citizenship under American law," so that now he is solely a Philippine national.
On this point, we quote from the assailed Resolution dated December 19, 1995:

Petitioner challenges this ruling. He argues that merely taking part in Philippine elections is not sufficient
By the laws of the United States, petitioner Frivaldo lost his
evidence of renunciation and that, in any event, as the alleged renunciation was made when private
American citizenship when he took his oath of allegiance to
respondent was already 37 years old, it was ineffective as it should have been made when he reached
the Philippine Government when he ran for Governor in
the age of majority.
1988, in 1992, and in 1995. Every certificate of candidacy
contains an oath of allegiance to the Philippine Government.
In holding that by voting in Philippine elections private respondent renounced his American citizenship,
the COMELEC must have in mind 349 of the Immigration and Nationality Act of the United States, which
These factual findings that Frivaldo has lost his foreign nationality long before the
provided that "A person who is a national of the United States, whether by birth or naturalization, shall
elections of 1995 have not been effectively rebutted by Lee. Furthermore, it is
lose his nationality by: . . . (e) Voting in a political election in a foreign state or participating in an election
basic that such findings of the Commission are conclusive upon this Court, absent
or plebiscite to determine the sovereignty over foreign territory." To be sure this provision was declared
any showing of capriciousness or arbitrariness or abuse.
unconstitutional by the U.S. Supreme Court in Afroyim v. Rusk 16 as beyond the power given to the U.S.
Congress to regulate foreign relations. However, by filing a certificate of candidacy when he ran for his
present post, private respondent elected Philippine citizenship and in effect renounced his American There is, therefore, no merit in petitioner's contention that the oath of allegiance contained in private
citizenship. Private respondent's certificate of candidacy, filed on March 27, 1998, contained the respondent's certificate of candidacy is insufficient to constitute renunciation that, to be effective, such
following statements made under oath: renunciation should have been made upon private respondent reaching the age of majority since no law
requires the election of Philippine citizenship to be made upon majority age.
6. I AM A FILIPINO CITIZEN (STATE IF "NATURAL-BORN" OR
"NATURALIZED") NATURAL-BORN Finally, much is made of the fact that private respondent admitted that he is registered as an American
citizen in the Bureau of Immigration and Deportation and that he holds an American passport which he
used in his last travel to the United States on April 22, 1997. There is no merit in this. Until the filing of
xxx xxx xxx
his certificate of candidacy on March 21, 1998, he had dual citizenship. The acts attributed to him can be
considered simply as the assertion of his American nationality before the termination of his American
10. I AM A REGISTERED VOTER OF PRECINCT NO. 747-A, BARANGAY SAN LORENZO, citizenship. What this Court said in Aznar v.COMELEC 18 applies mutatis mundatis to private respondent
CITY/MUNICIPALITY OF MAKATI, PROVINCE OF NCR. in the case at bar:

11. I AM NOT A PERMANENT RESIDENT OF, OR IMMIGRANT TO, A FOREIGN . . . Considering the fact that admittedly Osmea was both a Filipino and an
COUNTRY. American, the mere fact that he has a Certificate staring he is an American does
not mean that he is not still a Filipino. . . . [T]he Certification that he is an American
12. I AM ELIGIBLE FOR THE OFFICE I SEEK TO BE ELECTED. I WILL SUPPORT AND does not mean that he is not still a Filipino, possessed as he is, of both nationalities
DEFEND THE CONSTITUTION OF THE PHILIPPINES AND WILL MAINTAIN TRUE FAITH or citizenships. Indeed, there is no express renunciation here of Philippine
AND ALLEGIANCE THERETO; THAT I WILL OBEY THE LAWS, LEGAL ORDERS AND citizenship; truth to tell, there is even no implied renunciation of said citizenship.
DECREES PROMULGATED BY THE DULY CONSTITUTED AUTHORITIES OF THE When We consider that the renunciation needed to lose Philippine citizenship
REPUBLIC OF THE PHILIPPINES; AND THAT I IMPOSE THIS OBLIGATION UPON must be "express," it stands to reason that there can be no such loss of Philippine
MYSELF VOLUNTARILY, WITHOUT MENTAL RESERVATION OR PURPOSE OF citizenship when there is no renunciation, either "express" or "implied."
EVASION. I HEREBY CERTIFY THAT THE FACTS STATED HEREIN ARE TRUE AND
CORRECT OF MY OWN PERSONAL KNOWLEDGE. To recapitulate, by declaring in his certificate of candidacy that he is a Filipino citizen; that he is not a
permanent resident or immigrant of another country; that he will defend and support the Constitution of
The filing of such certificate of candidacy sufficed to renounce his American citizenship, effectively the Philippines and bear true faith and allegiance thereto and that he does so without mental
removing any disqualification he might have as a dual citizen. Thus, in Frivaldo v. COMELEC it was held: 17 reservation, private respondent has, as far as the laws of this country are concerned, effectively
repudiated his American citizenship and anything which he may have said before as a dual citizen.
On the other hand, private respondent's oath of allegiance to the Philippines, when considered with the
fact that he has spent his youth and adulthood, received his education, practiced his profession as an
artist, and taken part in past elections in this country, leaves no doubt of his election of Philippine
citizenship.

His declarations will be taken upon the faith that he will fulfill his undertaking made under oath. Should
he betray that trust, there are enough sanctions for declaring the loss of his Philippine citizenship
through expatriation in appropriate proceedings. In Yu v. Defensor-Santiago, 19 we sustained the denial
of entry into the country of petitioner on the ground that, after taking his oath as a naturalized citizen,
he applied for the renewal of his Portuguese passport and declared in commercial documents executed
abroad that he was a Portuguese national. A similar sanction can be taken against any one who, in
electing Philippine citizenship, renounces his foreign nationality, but subsequently does some act
constituting renunciation of his Philippine citizenship.

WHEREFORE, the petition for certiorari is DISMISSED for lack of merit.1wphi1.nt

SO ORDERED.
Republic of the Philippines Sec. 5. No petition for quo warranto shall be given due course without the
SUPREME COURT payment of a filing fee in the amount of Three Hundred Pesos (P300.00) and the
Manila legal research fee as required by law.

EN BANC and stresses that there is abundant jurisprudence holding that the payment of the filing fee is essential
to the timeliness of the filling of the petition itself. He cites many rulings of the Court to this effect,
specifically Manchester v. Court of Appeals. 1
G.R. No. 86564 August 1, 1989

For his part, the private respondent denies that the filing fee was paid out of time. In fact he says, it was
RAMON L. LABO, JR., petitioner,
fliedahead of time. His point is that when he filed his "Petition for Quo Warranto with Prayer for
vs.
Immediate Annulment of Proclamation and Restraining Order or Injunction" on January 26, 1988, the
THE COMMISSION ON ELECTIONS (COMELEC) EN BANC AND LUIS L. LARDIZABAL, respondents
COMELEC treated it as a pre-proclamation controversy and docketed it as SPC Case No. 88-288. No
docket fee was collected although it was offered. It was only on February 8, 1988, that the COMELEC
Estelito P. Mendoza for petitioner. decided to treat his petition as solely for quo warranto and re-docketed it as EPC Case No. 88-19, serving
him notice on February 10, 1988. He immediately paid the filing fee on that date.
Rillera and Quintana for private respondent.
The private respondent argues further that during the period when the COMELEC regarded his petition
as a pre-proclamation controversy, the time for filing an election protest or quo warranto proceeding
was deemed suspended under Section 248 of the Omnibus Election Code. 2 At any rate, he says, Rule 36,
Section 5, of the COMELEC Rules of Procedure cited by the petitioner, became effective only on
CRUZ, J.: November 15, 1988, seven days after publication of the said Rules in the Official Gazette pursuant to
Section 4, Rule 44 thereof. 3 These rules could not retroact to January 26,1988, when he filed his petition
The petitioner asks this Court to restrain the Commission on Elections from looking into the question of with the COMELEC.
his citizenship as a qualification for his office as Mayor of Baguio City. The allegation that he is a
foreigner, he says, is not the issue. The issue is whether or not the public respondent has jurisdiction to In his Reply, the petitioner argues that even if the Omnibus Election Code did not require it, the payment
conduct any inquiry into this matter, considering that the petition for quo warranto against him was not of filing fees was still necessary under Res. No. 1996 and, before that, Res. No. 1450 of the respondent
filed on time. COMELEC, promulgated on January 12, 1988, and February 26, 1980, respectively. To this, the private
respondent counters that the latter resolution was intended for the local elections held on January 30,
It is noteworthy that this argument is based on the alleged tardiness not of the petition itself but of the 1980, and did not apply to the 1988 local elections, which were supposed to be governed by the first-
payment of the filing fee, which the petitioner contends was an indispensable requirement. The fee is, mentioned resolution. However, Res. No. 1996 took effect only on March 3, 1988, following the lapse of
curiously enough, all of P300.00 only. This brings to mind the popular verse that for want of a horse the seven days after its publication as required by RA No. 6646, otherwise known as the Electoral Reform
kingdom was lost. Still, if it is shown that the petition was indeed filed beyond the reglementary period, Law of 1987, which became effective on January 5, 1988. Its Section 30 provides in part:
there is no question that this petition must be granted and the challenge abated.
Sec. 30. Effectivity of Regulations and Orders of the Commission. The rules and
The petitioner's position is simple. He was proclaimed mayor-elect of Baguio City, on January 20, 1988. regulations promulgated by the Commission shall take effect on the seventh day
The petition for quo warranto was filed by the private respondent on January 26, 1988, but no filing fee after their publication in the Official Gazette or in at least (2) daily newspapers of
was paid on that date. This fee was finally paid on February 10, 1988, or twenty-one days after his general circulation in the Philippines.
proclamation. As the petition by itself alone was ineffectual without the filing fee, it should be deemed
filed only when the fee was paid. This was done beyond the reglementary period provided for under The Court has considered the arguments of the parties and holds that the petition for quo warranto was
Section 253 of the Omnibus Election Code reading as follows: filed on time. We agree with the respondents that the fee was paid during the ten-day period as
extended by the pendency of the petition when it was treated by the COMELEC as a pre-proclamation
SEC. 253. Petition for quo warranto. Any voter contesting the election of a proceeding which did not require the payment of a filing fee. At that, we reach this conclusion only on
Member of the Batasang Pambansa, regional, provincial, or city officer on the the assumption that the requirement for the payment of the fees in quo warranto proceedings was
ground of ineligibility or of disloyalty to the Republic of the Philippines shall file a already effective. There is no record that Res. No. 1450 was even published; and as for Res. No. 1996,
sworn petition for quo warranto with the Commission within ten days after the this took effect only on March 3, 1988, seven days after its publication in the February 25, 1988 issues of
proclamation of the result of the election. the Manila Chronicle and the Philippine Daily Inquirer, or after the petition was filed.

The petitioner adds that the payment of the filing fee is required under Rule 36, Section 5, of the The petitioner forgets Ta;ada v. Tuvera 4 when he argues that the resolutions became effective
Procedural Rules of the COMELEC providing that "immediately upon approval" simply because it was so provided therein. We held in that case that
publication was still necessary under the due process clause despite such effectivity clause.
In any event, what is important is that the filing fee was paid, and whatever delay there may have been is Appeals for appropriate action. Considering, however, the length of time that this
not imputable to the private respondent's fault or neglect. It is true that in the Manchester Case, we case has been pending, we apply the rule in the case of Del Castillo v. Jaymalin,
required the timely payment of the filing fee as a precondition for the timeliness of the filing of the case (112 SCRA 629) and follow the principle enunciated in Alger Electric, Inc. v. Court
itself. In Sun Insurance Office, Ltd. v. Asuncion, 5 however this Court, taking into account the special of Appeals, (135 SCRA 37) which states:
circumstances of that case, declared:
... it is a cherished rule of procedure for this Court to always
This Court reiterates the rule that the trial court acquires jurisdiction over a case strive to settle the entire controversy in a single proceeding
only upon the payment of the prescribed filing fee. However, the court may allow leaving no root or branch to bear the seeds of future
the payment of the said fee within a reasonable time. In the event of non- litigation. No useful purpose will be served if this case is
compliance therewith, the case shall be dismissed. remanded to the trial court only to have its decision raised
again to the Intermediate Appellate Court and from there to
this Court. (p. 43)
The same idea is expressed in Rule 42, Section 18, of the COMELEC Rules of Procedure adopted on June
20, 1988, thus:
Only recently in the case of Beautifont, Inc., et al. v. Court of Appeals, et al. (G.R.
No. 50141, January 29, 1988), we stated that:
Sec. 18. Non-payment of prescribed fees. If the fees above prescribed are not
paid, theCommission may refuse to take action thereon until they are paid and
may dismiss the action or the proceeding. (Emphasis supplied.) ... But all those relevant facts are now before this Court. And those facts dictate
the rendition of a verdict in the petitioner's favor. There is therefore no point in
referring the case back to the Court of Appeals. The facts and the legal
The Court notes that while arguing the technical point that the petition for quo warranto should be
propositions involved will not change, nor should the ultimate judgment.
dismissed for failure to pay the filing fee on time, the petitioner would at the same time minimize his
Considerable time has already elapsed and, to serve the ends of justice, it is time
alleged lack of citizenship as "a futile technicality," It is regrettable, to say the least, that the requirement
that the controversy is finally laid to rest. (See Sotto v. Samson, 5 SCRA 733;
of citizenship as a qualification for public office can be so demeaned. What is worse is that it is regarded
Republic v. Paredes, 108 Phil. 57; Lianga Lumber Co. v. Lianga Timber Co., Inc., 76
as an even less important consideration than the reglementary period the petitioner insists upon.
SCRA 197; Erico v. Heirs of Chigas, 98 SCRA 575; Francisco v. City of Davao, 12
SCRA 628; Valencia v. Mabilangan, 105 Phil. 162).lwph1.t Sound practice seeks
This matter should normally end here as the sole issue originally raised by the petitioner is the timeliness to accommodate the theory which avoids waste of time, effort and expense, both
of thequo warranto proceedings against him. However, as his citizenship is the subject of that to the parties and the government, not to speak of delay in the disposal of the
proceeding, and considering the necessity for an early resolution of that more important question clearly case (cf. Fernandez v. Garcia, 92 Phil. 592, 597). A marked characteristic of our
and urgently affecting the public interest, we shall directly address it now in this same action. judicial set-up is that where the dictates of justice so demand ... the Supreme
Court should act, and act with finality.' (Li Siu Liat v. Republic, 21 SCRA 1039, 1046,
The Court has similarly acted in a notable number of cases, thus: citing Samal v. CA, 99 Phil. 230 and U.S. v. Gimenez, 34 Phil. 74). In this case, the
dictates of justice do demand that this Court act, and act with finality. 7

From the foregoing brief statement of the nature of the instant case, it would
appear that our sole function in this proceeding should be to resolve the single xxx
issue of whether or not the Court of Appeals erred in ruling that the motion for
new trial of the GSIS in question should indeed be deemedpro forma. But going Remand of the case to the lower court for further reception of evidence is not
over the extended pleadings of both parties, the Court is immediately impressed necessary where the court is in a position to resolve the dispute based on the
that substantial justice may not be timely achieved, if we should decide this case records before it. On many occasions, the Court, in the public interest and the
upon such a technical ground alone. We have carefully read all the allegations and expeditious administration of justice, has resolved actions on the merits instead of
arguments of the parties, very ably and comprehensively expounded by evidently remanding them to the trial court for further proceedings, such as where the ends
knowledgeable and unusually competent counsel, and we feel we can better serve of justice would not be subserved by the remand of the case or when public
the interests of justice by broadening the scope of our inquiry, for as the record interest demands an early disposition of the case or where the trial court had
before us stands, we see that there is enough basis for us to end the basic already received all the evidence of the parties. 8
controversy between the parties here and now, dispensing, however, with
procedural steps which would not anyway affect substantially the merits of their
This course of action becomes all the more justified in the present case where, to repeat for stress, it is
respective claims. 6
claimed that a foreigner is holding a public office.

xxx
We also note in his Reply, the petitioner says:

While it is the fault of the petitioner for appealing to the wrong court and thereby
In adopting private respondent's comment, respondent COMELEC implicitly
allowing the period for appeal to lapse, the more correct procedure was for the
adopted as "its own" private respondent's repeated assertion that petitioner is no
respondent court to forward the case to the proper court which was the Court of
longer a Filipino citizen. In so doing, has not respondent COMELEC effectively C) The marriage was declared void in the Australian Federal Court in Sydney on 27
disqualified itself, by reason of prejudgment, from resolving the petition for quo June 1980 on the ground that the marriage had been bigamous.
warranto filed by private respondent still pending before it? 9
D) According to our records LABO is still an Australian citizen.
This is still another reason why the Court has seen fit to rule directly on the merits of this case.
E) Should he return to Australia, LABO may face court action in respect of Section
Going over the record, we find that there are two administrative decisions on the question of the 50 of Australian Citizenship Act 1948 which relates to the giving of false or
petitioner's citizenship. The first was rendered by the Commission on Elections on May 12, 1982, and misleading information of a material nature in respect of an application for
found the petitioner to be a citizen of the Philippines. 10 The second was rendered by the Commission on Australian citizenship. If such a prosecution was successful, he could be deprived
Immigration and Deportation on September 13, 1988, and held that the petitioner was not a citizen of of Australian citizenship under Section 21 of the Act.
the Philippines. 11
F) There are two further ways in which LABO could divest himself of Australian
The first decision was penned by then COMELEC Chigas, Vicente Santiago, Jr., with Commissioners citizenship:
Pabalate Savellano and Opinion concurring in full and Commissioner Bacungan concurring in the
dismissal of the petition "without prejudice to the issue of the respondent's citizenship being raised
(i) He could make a declaration of Renunciation of Australian citizenship under
anew in a proper case." Commissioner Sagadraca reserved his vote, while Commissioner Felipe was for
Section 18 of the Australian Citizenship Act, or
deferring decision until representations shall have been made with the Australian Embassy for official
verification of the petitioner's alleged naturalization as an Australian.
(ii) If he acquired another nationality, (for example, Filipino) by a formal and
voluntary act other than marriage, then he would automatically lose as Australian
The second decision was unanimously rendered by Chairman Miriam Defensor-Santiago and
citizenship under Section 17 of the Act.
Commissioners Alano and Geraldez of the Commission on Immigration and Deportation. It is important
to observe that in the proceeding before the COMELEC, there was no direct proof that the herein
petitioner had been formally naturalized as a citizen of Australia. This conjecture, which was eventually IN WITNESS WHEREOF, I HAVE HEREUNTO SET MAY HAND AND SEAL OF THE
rejected, was merely inferred from the fact that he had married an Australian citizen, obtained an AUSTRALIAN EMBASSY, MANILA, THIS 12th DAY OF APRIL 1984. DONE AT MANILA
Australian passport, and registered as an alien with the CID upon his return to this country in 1980. IN THE PHILIPPINES.

On the other hand, the decision of the CID took into account the official statement of the Australian (Signed) GRAHAM C. WEST Consul
Government dated August 12, 1984, through its Consul in the Philippines, that the petitioner was still an
Australian citizen as of that date by reason of his naturalization in 1976. That statement 12 is reproduced This was affirmed later by the letter of February 1, 1988, addressed to the private
in full as follows: respondent by the Department of Foreign Affairs reading as follows: 13

I, GRAHAM COLIN WEST, Consul of Australia in the Philippines, by virtue of a certificate of appointment Sir:
signed and sealed by the Australian Minister of State for Foreign Affairs on 19 October 1983, and
recognized as such by Letter of Patent signed and sealed by the Philippines Acting Minister of Foreign
Affairs on 23 November 1983, do hereby provide the following statement in response to the subpoena With reference to your letter dated 1 February 1988, I wish to inform you that
Testificandum dated 9 April 1984 in regard to the Petition for disqualification against RAMON LABO, JR. Y inquiry made with the Australian Government through the Embassy of the
LOZANO (SPC No. 84-73), and do hereby certify that the statement is true and correct. Philippines in Canberra has elicited the following information:

STATEMENT 1) That Mr. Ramon L. Labo, Jr. acquired Australian citizenship on 28 July 1976.

A) RAMON LABO, JR. Y LOZANO, date of birth 23 December 1934, was married in 2) That prior to 17 July 1986, a candidate for Australian citizenship had to either
the Philippines to an Australian citizen. As the spouse of an Australian citizen, he swear an oath of allegiance or make an affirmation of allegiance which carries a
was not required to meet normal requirements for the grant of citizenship and renunciation of "all other allegiance.
was granted Australian citizenship by Sydney on 28 July 1976.
Very truly yours, For the Secretary of Foreign Affairs: (SGD) RODOLFO SEVERINO,
B) Any person over the age of 16 years who is granted Australian citizenship must JR. Assistant Secretary
take an oath of allegiance or make an affirmation of allegiance. The wording of the
oath of affirmation is: "I ..., renouncing all other allegiance ..." etc. This need not The decision also noted the oath of allegiance taken by every naturalized Australian reading as follows:
necessarily have any effect on his former nationality as this would depend on the
citizenship laws of his former country.
OATH OF ALLEGIANCE
I, A.B., renouncing all other allegiance, swear by Almighty God that I will be faithful provisions of CA No. 63, which enumerates the modes by which Philippine citizenship may be lost.
and bear true allegiance to Her Majesty Elizabeth the Second, Queen of Australia, Among these are: (1) naturalization in a foreign country; (2) express renunciation of citizenship; and (3)
Her heirs and successors according to law, and that I will faithfully observe the subscribing to an oath of allegiance to support the Constitution or laws of a foreign country, all of which
laws of Australia and fulfill my duties as an Australian citizen. 14 are applicable to the petitioner. It is also worth mentioning in this connection that under Article IV,
Section 5, of the present Constitution, "Dual allegiance of citizens is inimical to the national interest and
shall be dealt with by law."
and the Affirmation of Allegiance, which declares:

Even if it be assumed that, as the petitioner asserts, his naturalization in Australia was annulled after it
AFFIRMATION OF ALLEGIANCE
was found that his marriage to the Australian citizen was bigamous, that circumstance alone did not
automatically restore his Philippine citizenship. His divestiture of Australian citizenship does not concern
I, A.B., renouncing all other allegiance, solemnly and sincerely promise and declare us here. That is a matter between him and his adopted country. What we must consider is the fact that
that I will be faithful and bear true allegiance to Her Majesty Elizabeth the Second, he voluntarily and freely rejected Philippine citizenship and willingly and knowingly embraced the
Queen of Australia, Her heirs and successors according to law, and that I will citizenship of a foreign country. The possibility that he may have been subsequently rejected by
faithfully observe the Laws of Australia and fulfill my duties as an Australian Australia, as he claims, does not mean that he has been automatically reinstated as a citizen of the
citizen. 15 Philippines.

The petitioner does not question the authenticity of the above evidence. Neither does he deny that he Under CA No. 63 as amended by PD No. 725, Philippine citizenship may be reacquired by direct act of
obtained Australian Passport No. 754705, which he used in coming back to the Philippines in 1980, when Congress, by naturalization, or by repatriation. It does not appear in the record, nor does the petitioner
he declared before the immigration authorities that he was an alien and registered as such under Alien claim, that he has reacquired Philippine citizenship by any of these methods. He does not point to any
Certificate of Registration No. B-323985. 16 He later asked for the change of his status from immigrant to judicial decree of naturalization as to any statute directly conferring Philippine citizenship upon him.
a returning former Philippine citizen and was granted Immigrant Certificate of Residence No. Neither has he shown that he has complied with PD No. 725, providing that:
223809. 17 He also categorically declared that he was a citizen of Australia in a number of sworn
statements voluntarily made by him and. even sought to avoid the jurisdiction of the barangay court on
... (2) natural-born Filipinos who have lost their Philippine citizenship may
the ground that he was a foreigner. 18
reacquire Philippine citizenship through repatriation by applying with the Special
Committee on Naturalization created by Letter of Instruction No. 270, and, if their
The decision of the COMELEC in 1982 quaintly dismisses all these acts as "mistakes" that did not divest applications are approved, taking the necessary oath of allegiance to the Republic
the petitioner of his citizenship, although, as earlier noted, not all the members joined in this finding. We of the Philippines, after which they shall be deemed to have reacquired Philippine
reject this ruling as totally baseless. The petitioner is not an unlettered person who was not aware of the citizenship. The Commission on Immigration and Deportation shall thereupon
consequences of his acts, let alone the fact that he was assisted by counsel when he performed these cancel their certificate of registration. (Emphasis supplied.)
acts.
That is why the Commission on Immigration and Deportation rejected his application for the cancellation
The private respondent questions the motives of the COMELEC at that time and stresses Labo's political of his alien certificate of registration. And that is also the reason we must deny his present claim for
affiliation with the party in power then, but we need not go into that now. recognition as a citizen of the Philippines.

There is also the claim that the decision can no longer be reversed because of the doctrine of res The petitioner is not now, nor was he on the day of the local elections on January 18, 1988, a citizen of
judicata, but this too must be dismissed. This doctrine does not apply to questions of citizenship, as the the Philippines. In fact, he was not even a qualified voter under the Constitution itself because of his
Court has ruled in several cases. 19 Moreover, it does not appear that it was properly and seasonably alienage. 21 He was therefore ineligible as a candidate for mayor of Baguio City, under Section 42 of the
pleaded, in a motion to dismiss or in the answer, having been invoked only when the petitioner filed his Local Government Code providing in material part as follows:
reply 20 to the private respondent's comment. Besides, one of the requisites of res judicata, to wit,
identity of parties, is not present in this case.
Sec. 42. Qualifications. An elective local official must be a citizen of the
Philippines, at least twenty-three years of age on election day, a qualified voter
The petitioner's contention that his marriage to an Australian national in 1976 did not automatically registered as such in the barangay, municipality, city or province where he
divest him of Philippine citizenship is irrelevant. There is no claim or finding that he automatically ceased proposes to be elected, a resident therein for at least one year at the time of the
to be a Filipino because of that marriage. He became a citizen of Australia because he was naturalized as filing of his certificate of candidacy, and able to read and write English, Filipino, or
such through a formal and positive process, simplified in his case because he was married to an any other local language or dialect.
Australian citizen. As a condition for such naturalization, he formally took the Oath of Allegiance and/or
made the Affirmation of Allegiance, both quoted above. Renouncing all other allegiance, he swore "to be
The petitioner argues that his alleged lack of citizenship is a "futile technicality" that should not frustrate
faithful and bear true allegiance to Her Majesty Elizabeth the Second, Queen of Australia ..." and to fulfill
the will of the electorate of Baguio City, who elected him by a "resonant and thunderous majority." To
his duties "as an Australian citizen."
be accurate, it was not as loud as all that, for his lead over the second-placer was only about 2,100 votes.
In any event, the people of that locality could not have, even unanimously, changed the requirements of
The petitioner now claims that his naturalization in Australia made him at worst only a dual national and the Local Government Code and the Constitution. The electorate had no power to permit a foreigner
did not divest him of his Philippine citizenship. Such a specious argument cannot stand against the clear owing his total allegiance to the Queen of Australia, or at least a stateless individual owing no allegiance
to the Republic of the Philippines, to preside over them as mayor of their city. Only citizens of the It remains to stress that the citizen of the Philippines must take pride in his status as such and cherish
Philippines have that privilege over their countrymen. this priceless gift that, out of more than a hundred other nationalities, God has seen fit to grant him.
Having been so endowed, he must not lightly yield this precious advantage, rejecting it for another land
that may offer him material and other attractions that he may not find in his own country. To be sure, he
The probability that many of those who voted for the petitioner may have done so in the belief that he
has the right to renounce the Philippines if he sees fit and transfer his allegiance to a state with more
was qualified only strengthens the conclusion that the results of the election cannot nullify the
allurements for him. 33 But having done so, he cannot expect to be welcomed back with open arms once
qualifications for the office now held by him. These qualifications are continuing requirements; once any
his taste for his adopted country turns sour or he is himself disowned by it as an undesirable alien.
of them is lost during incumbency, title to the office itself is deemed forfeited. In the case at bar, the
citizenship and voting requirements were not subsequently lost but were not possessed at all in the first
place on the day of the election. The petitioner was disqualified from running as mayor and, although Philippine citizenship is not a cheap commodity that can be easily recovered after its renunciation. It may
elected, is not now qualified to serve as such. be restored only after the returning renegade makes a formal act of re-dedication to the country he has
abjured and he solemnly affirms once again his total and exclusive loyalty to the Republic of the
Philippines. This may not be accomplished by election to public office.
Finally, there is the question of whether or not the private respondent, who filed the quo
warranto petition, can replace the petitioner as mayor. He cannot. The simple reason is that as he
obtained only the second highest number of votes in the election, he was obviously not the choice of the WHEREFORE, petitioner Ramon J. Labo, Jr. is hereby declared NOT a citizen of the Philippines and
people of Baguio city. therefore DISQUALIFIED from continuing to serve as Mayor of Baguio City. He is ordered to VACATE his
office and surrender the same to the Vice-Mayor of Baguio City, once this decision becomes final and
executory. The temporary restraining order dated January 31, 1989, is LIFTED.
The latest ruling of the Court on this issue is Santos v. Commission on Elections 22 decided in 1985. In that
case, the candidate who placed second was proclaimed elected after the votes for his winning rival, who
was disqualified as a turncoat and considered a non-candidate, were all disregarded as stray. In effect,
the second placer won by default. That decision was supported by eight members of the Court
then 23 with three dissenting 24 and another two reserving their vote. 25 One was on official leave. 26

Re-examining that decision, the Court finds, and so holds, that it should be reversed in favor of the
earlier case ofGeronimo v. Ramos, 27 Which represents the more logical and democratic rule. That case,
which reiterated the doctrine first announced in 1912 in Topacio vs. Paredes 28 was supported by ten
members of the Court 29 without any dissent, although one reserved his vote, 30 another took no
part 31 and two others were on leave. 32 There the Court held:

... it would be extremely repugnant to the basic concept of the constitutionally


guaranteed right to suffrage if a candidate who has not acquired the majority or
plurality of votes is proclaimed a winner and imposed as the representative of a
constituency, the majority of which have positively declared through their ballots
that they do not choose him.

Sound policy dictates that public elective offices are filled by those who have
received the highest number of votes cast in the election for that office, and it is a
fundamental Idea in all republican forms of government that no one can be
declared elected and no measure can be declared carried unless he or it receives a
majority or plurality of the legal votes cast in the election. (20 Corpus Juris 2nd, S
243, p. 676.)

The fact that the candidate who obtained the highest number of votes is later
declared to be disqualified or not eligible for the office to which he was elected
does not necessarily entitle the candidate who obtained the second highest
number of votes to be declared the winner of the elective office. The votes cast for
a dead, disqualified, or non-eligible person may not be valid to vote the winner
into office or maintain him there. However, in the absence of a statute which
clearly asserts a contrary political and legislative policy on the matter, if the votes
were cast in the sincere belief that the candidate was alive, qualified, or eligible,
they should not be treated as stray, void or meaningless.
Republic of the Philippines The lifting of the Temporary Restraining Order issued by the Court on 7 December 1988 is urgently
SUPREME COURT sought by respondent Commissioner who was ordered to cease and desist from immediately deporting
Manila petitioner Yu pending the conclusion of hearings before the Board of Special Inquiry, CID. To finally
dispose of the case, the Court will likewise rule on petitioner's motion for clarification with prayer for
restraining order dated 5 December 1988, 9urgent motion for release from arbitrary detention dated 13
EN BANC
December 1988, 10 the memorandum in furtherance of said motion for release dated 14 December
1988, 11 motion to set case for oral argument dated 8 December 1988. 12
G.R. No. L-83882 January 24, 1989
Acting on the motion to lift the temporary restraining order (issued on 7 December 1988) dated 9
IN RE PETITION FOR HABEAS CORPUS OF WILLIE YU, petitioner, December 1988,13 and the vigorous opposition to lift restraining order dated 15 December 1988, 14 the
vs. Court resolved to give petitioner Yu a non-extendible period of three (3) days from notice within which
MIRIAM DEFENSOR-SANTIAGO, BIENVENIDO P. ALANO, JR., MAJOR PABALAN, DELEO HERNANDEZ, to explain and prove why he should still be considered a citizen of the Philippines despite his acquisition
BLODDY HERNANDEZ, BENNY REYES and JUN ESPIRITU SANTO, respondent. and use of a Portuguese passport. 15

Pelaez, Adriano and Gregorio and Bonifacio A. Alentajan for petitioner. Petitioner filed his compliance with the resolution of 15 December 1988 on 20 December
1988 16 followed by an earnest request for temporary release on 22 December 1988. Respondent filed on
Chavez, Hechanova & Lim Law Offices collaborating counsel for petitioner. 2 January 1989 her comment reiterating her previous motion to lift temporary restraining order.
Petitioner filed a reply thereto on 6 January 1989.

Augusto Jose y. Arreza for respondents.


Petitioner's own compliance reveals that he was originally issued a Portuguese passport in 1971, 17 valid
for five (5) years and renewed for the same period upon presentment before the proper Portuguese
consular officer. Despite his naturalization as a Philippine citizen on 10 February 1978, on 21 July 1981,
petitioner applied for and was issued Portuguese Passport No. 35/81 serial N. 1517410 by the Consular
PADILLA, J.: Section of the Portuguese Embassy in Tokyo. Said Consular Office certifies that his Portuguese passport
expired on 20 July 1986. 18 While still a citizen of the Philippines who had renounced, upon his
naturalization, "absolutely and forever all allegiance and fidelity to any foreign prince, potentate, state or
The present controversy originated with a petition for habeas corpus filed with the Court on 4 July 1988 sovereignty" and pledged to "maintain true faith and allegiance to the Republic of the Philippines," 19 he
seeking the release from detention of herein petitioner. 1 After manifestation and motion of the Solicitor declared his nationality as Portuguese in commercial documents he signed, specifically, the Companies
General of his decision to refrain from filing a return of the writ on behalf of the CID, respondent registry of Tai Shun Estate Ltd.20 filed in Hongkong sometime in April 1980.
Commissioner thru counsel filed the return. 2 Counsel for the parties were heard in oral argument on 20
July 1988. The parties were allowed to submit marked exhibits, and to file memoranda. 3 An internal
resolution of 7 November 1988 referred the case to the Court en banc. In its 10 November 1988 To the mind of the Court, the foregoing acts considered together constitute an express renunciation of
resolution, denying the petition for habeas corpus, the Court disposed of the pending issues of (1) petitioner's Philippine citizenship acquired through naturalization. In Board of Immigration
jurisdiction of the CID over a naturalized Filipino citizen and (2) validity of warrantless arrest and Commissioners us, Go Gallano, 21express renunciation was held to mean a renunciation that is made
detention of the same person. known distinctly and explicitly and not left to inference or implication. Petitioner, with full knowledge,
and legal capacity, after having renounced Portuguese citizenship upon naturalization as a Philippine
citizen 22 resumed or reacquired his prior status as a Portuguese citizen, applied for a renewal of his
Petitioner filed a motion for reconsideration with prayer for restraining order dated 24 November Portuguese passport 23 and represented himself as such in official documents even after he had become
1988. 4 On 29 November 1988, the Court resolved to deny with finality the aforesaid motion for a naturalized Philippine citizen. Such resumption or reacquisition of Portuguese citizenship is grossly
reconsideration, and further resolved to deny the urgent motion for issuance of a restraining order dated inconsistent with his maintenance of Philippine citizenship.
28 November 1988. 5

This Court issued the aforementioned TRO pending hearings with the Board of Special Inquiry, CID.
Undaunted, petitioner filed a motion for clarification with prayer for restraining order on 5 December However, pleadings submitted before this Court after the issuance of said TRO have unequivocally
1988. shown that petitioner has expressly renounced his Philippine citizenship. The material facts are not only
established by the pleadings they are not disputed by petitioner. A rehearing on this point with the
Acting on said motion, a temporary restraining order was issued by the Court on 7 December CID would be unnecessary and superfluous. Denial, if any, of due process was obviated when petitioner
1988. 6 Respondent Commissioner filed a motion to lift TRO on 13 December 1988, the basis of which is a was given by the Court the opportunity to show proof of continued Philippine citizenship, but he has
summary judgment of deportation against Yu issued by the CID Board of Commissioners on 2 December failed.
1988. 7 Petitioner also filed a motion to set case for oral argument on 8 December 1988.
While normally the question of whether or not a person has renounced his Philippine citizenship should
In the meantime, an urgent motion for release from arbitrary detention 8 was filed by petitioner on 13 be heard before a trial court of law in adversary proceedings, this has become unnecessary as this Court,
December 1988. A memorandum in furtherance of said motion for release dated 14 December 1988 was no less, upon the insistence of petitioner, had to look into the facts and satisfy itself on whether or not
filed on 15 December 1988 together with a vigorous opposition to the lifting of the TRO. petitioner's claim to continued Philippine citizenship is meritorious.
Philippine citizenship, it must be stressed, is not a commodity or were to be displayed when required
and suppressed when convenient. This then resolves adverse to the petitioner his motion for clarification
and other motions mentioned in the second paragraph, page 3 of this Decision.

WHEREFORE, premises considered, petitioner's motion for release from detention is DENIED.
Respondent's motion to lift the temporary restraining order is GRANTED. This Decision is immediately
executory.

SO ORDERED.
Republic of the Philippines Division, 1 promulgated on December 19, 1995 2 and another Resolution of the Comelec en
SUPREME COURT banc promulgated February 23, 1996 3 denying petitioner's motion for reconsideration.
Manila
The Facts
EN BANC
On March 20, 1995, private respondent Juan G. Frivaldo filed his Certificate of Candidacy for the office of
Governor of Sorsogon in the May 8, 1995 elections. On March 23, 1995, petitioner Raul R. Lee, another
candidate, filed a petition 4 with the Comelec docketed as SPA No. 95-028 praying that Frivaldo "be
disqualified from seeking or holding any public office or position by reason of not yet being a citizen of
G.R. No. 120295 June 28, 1996
the Philippines", and that his Certificate of Candidacy be canceled. On May 1, 1995, the Second Division
of the Comelec promulgated a Resolution 5 granting the petition with the following disposition 6:
JUAN G. FRIVALDO, petitioner,
vs.
WHEREFORE, this Division resolves to GRANT the petition and declares that
COMMISSION ON ELECTIONS, and RAUL R. LEE, respondents.
respondent is DISQUALIFIED to run for the Office of Governor of Sorsogon on the
ground that he is NOT a citizen of the Philippines. Accordingly, respondent's
G.R. No. 123755 June 28, 1996 certificate of candidacy is canceled.

RAUL R. LEE, petitioner, The Motion for Reconsideration filed by Frivaldo remained unacted upon until after the May 8, 1995
vs. elections. So, his candidacy continued and he was voted for during the elections held on said date. On
COMMISSION ON ELECTIONS and JUAN G. FRIVALDO, respondents. May 11, 1995, the Comelec en banc 7 affirmed the aforementioned Resolution of the Second Division.

The Provincial Board of Canvassers completed the canvass of the election returns and a Certificate of
Votes 8dated May 27, 1995 was issued showing the following votes obtained by the candidates for the
PANGANIBAN, J.:p position of Governor of Sorsogon:

The ultimate question posed before this Court in these twin cases is: Who should be declared the rightful Antonio H. Escudero, Jr. 51,060
governor of Sorsogon -
Juan G. Frivaldo 73,440
(i) Juan G. Frivaldo, who unquestionably obtained the highest number of votes in three successive
elections but who was twice declared by this Court to be disqualified to hold such office due to his alien Raul R. Lee 53,304
citizenship, and who now claims to have re-assumed his lost Philippine citizenship thru repatriation;
Isagani P. Ocampo 1,925
(ii) Raul R. Lee, who was the second placer in the canvass, but who claims that the votes cast in favor of
Frivaldo should be considered void; that the electorate should be deemed to have intentionally thrown
On June 9, 1995, Lee filed in said SPA No. 95-028, a (supplemental) petition 9 praying for his proclamation
away their ballots; and that legally, he secured the most number of valid votes; or
as the duly-elected Governor of Sorsogon.

(iii) The incumbent Vice-Governor, Oscar G. Deri, who obviously was not voted directly to the position of
In an order 10 dated June 21, 1995, but promulgated according to the petition "only on June 29, 1995,"
governor, but who according to prevailing jurisprudence should take over the said post inasmuch as, by
the Comelec en banc directed "the Provincial Board of Canvassers of Sorsogon to reconvene for the
the ineligibility of Frivaldo, a "permanent vacancy in the contested office has occurred"?
purpose of proclaiming candidate Raul Lee as the winning gubernatorial candidate in the province of
Sorsogon on June 29, 1995 . . ." Accordingly, at 8:30 in the evening of June 30, 1995, Lee was proclaimed
In ruling for Frivaldo, the Court lays down new doctrines on repatriation, clarifies/reiterates/amplifies governor of Sorsogon.
existing jurisprudence on citizenship and elections, and upholds the superiority of substantial justice over
pure legalisms.
On July 6, 1995, Frivaldo filed with the Comelec a new petition, 11 docketed as SPC No. 95-317, praying
for the annulment of the June 30, 1995 proclamation of Lee and for his own proclamation. He alleged
G.R. No. 123755 that on June 30, 1995, at 2:00 in the afternoon, he took his oath of allegiance as a citizen of the
Philippines after "his petition for repatriation under P.D. 725 which he filed with the Special Committee
This is a special civil action under Rules 65 and 58 of the Rules of Court for certiorari and preliminary on Naturalization in September 1994 had been granted". As such, when "the said order (dated June 21,
injunction to review and annul a Resolution of the respondent Commission on Elections (Comelec), First 1995) (of the Comelec) . . . was released and received by Frivaldo on June 30, 1995 at 5:30 o'clock in the
evening, there was no more legal impediment to the proclamation (of Frivaldo) as governor . . ." In the
alternative, he averred that pursuant to the two cases of Labo vs. Comelec, 12 the Vice-Governor - not Lee Third -- The alleged repatriation of respondent was neither valid nor is the effect
- should occupy said position of governor. thereof retroactive as to cure his ineligibility and qualify him to hold the Office of
Governor; and
On December 19, 1995, the Comelec First Division promulgated the herein assailed Resolution 13 holding
that Lee, "not having garnered the highest number of votes," was not legally entitled to be proclaimed as Fourth -- Correctly read and applied, the Labo Doctrine fully supports the validity
duly-elected governor; and that Frivaldo, "having garnered the highest number of votes, of petitioner's proclamation as duly elected Governor of Sorsogon.
and . . . having reacquired his Filipino citizenship by repatriation on June 30, 1995 under the provisions of
Presidential Decree No. 725 . . . (is) qualified to hold the office of governor of Sorsogon"; thus:
G.R. No. 120295

PREMISES CONSIDERED, the Commission (First Division), therefore RESOLVES to


This is a petition to annul three Resolutions of the respondent Comelec, the first two of which are also at
GRANT the Petition.
issue in G.R. No. 123755, as follows:

Consistent with the decisions of the Supreme Court, the proclamation of Raul R.
1. Resolution 16 of the Second Division, promulgated on May 1, 1995, disqualifying
Lee as Governor of Sorsogon is hereby ordered annulled, being contrary to law, he
Frivaldo from running for governor of Sorsogon in the May 8, 1995 elections "on
not having garnered the highest number of votes to warrant his proclamation.
the ground that he is not a citizen of the Philippines";

Upon the finality of the annulment of the proclamation of Raul R. Lee, the
2. Resolution 17 of the Comelec en banc, promulgated on May 11, 1995; and
Provincial Board of Canvassers is directed to immediately reconvene and, on the
basis of the completed canvass, proclaim petitioner Juan G. Frivaldo as the duly
elected Governor of Sorsogon having garnered the highest number of votes, and 3. Resolution 18 of the Comelec en banc, promulgated also on May 11, 1995
he having reacquired his Filipino citizenship by repatriation on June 30, 1995 under suspending the proclamation of, among others, Frivaldo.
the provisions of Presidential Decree No. 725 and, thus, qualified to hold the office
of Governor of Sorsogon. The Facts and the Issue

Conformably with Section 260 of the Omnibus Election Code (B.P. Blg. 881), the The facts of this case are essentially the same as those in G.R. No. 123755. However, Frivaldo assails the
Clerk of the Commission is directed to notify His Excellency the President of the above-mentioned resolutions on a different ground: that under Section 78 of the Omnibus Election
Philippines, and the Secretary of the Sangguniang Panlalawigan of the Province of Code, which is reproduced hereinunder:
Sorsogon of this resolution immediately upon the due implementation thereof.

Sec. 78. Petition to deny due course or to cancel a certificate of candidacy. -- A


On December 26, 1995, Lee filed a motion for reconsideration which was denied by the Comelec en verified petition seeking to deny due course or to cancel a certificate of candidacy
banc in its Resolution 14 promulgated on February 23, 1996. On February 26, 1996, the present petition may be filed by any person exclusively on the ground that any material
was filed. Acting on the prayer for a temporary restraining order, this Court issued on February 27, 1996 representation contained therein as required under Section 74 hereof is false. The
a Resolution which inter alia directed the parties "to maintain the status quo prevailing prior to the filing petition may be filed at any time not later than twenty-five days from the time of
of this petition." the filing of the certificate of candidacy and shall be decided, after notice and
hearing, not later than fifteen days before the election. (Emphasis supplied.)
The Issues in G.R. No. 123755
the Comelec had no jurisdiction to issue said Resolutions because they were not rendered
Petitioner Lee's "position on the matter at hand may briefly be capsulized in the following "within the period allowed by law" i.e., "not later than fifteen days before the election."
propositions" 15:
Otherwise stated, Frivaldo contends that the failure of the Comelec to act on the petition for
First -- The initiatory petition below was so far insufficient in form and substance disqualification within the period of fifteen days prior to the election as provided by law is a jurisdictional
to warrant the exercise by the COMELEC of its jurisdiction with the result that, in defect which renders the said Resolutions null and void.
effect, the COMELEC acted without jurisdiction in taking cognizance of and
deciding said petition; By Resolution on March 12, 1996, the Court consolidated G.R. Nos. 120295 and 123755 since they are
intimately related in their factual environment and are identical in the ultimate question raised, viz., who
Second -- The judicially declared disqualification of respondent was a continuing should occupy the position of governor of the province of Sorsogon.
condition and rendered him ineligible to run for, to be elected to and to hold the
Office of Governor; On March 19, 1995, the Court heard oral argument from the parties and required them thereafter to file
simultaneously their respective memoranda.
The Consolidated Issues Under Philippine law, 21 citizenship may be reacquired by direct act of Congress, by naturalization or by
repatriation. Frivaldo told this Court in G.R. No. 104654 22 and during the oral argument in this case that
he tried to resume his citizenship by direct act of Congress, but that the bill allowing him to do so "failed
From the foregoing submissions, the consolidated issues may be restated as follows:
to materialize, notwithstanding the endorsement of several members of the House of Representatives"
due, according to him, to the "maneuvers of his political rivals." In the same case, his attempt
1. Was the repatriation of Frivaldo valid and legal? If so, did it seasonably cure his lack of citizenship as to at naturalization was rejected by this Court because of jurisdictional, substantial and procedural defects.
qualify him to be proclaimed and to hold the Office of Governor? If not, may it be given retroactive
effect? If so, from when?
Despite his lack of Philippine citizenship, Frivaldo was overwhelmingly elected governor by the electorate
of Sorsogon, with a margin of 27,000 votes in the 1988 elections, 57,000 in 1992, and 20,000 in 1995
2. Is Frivaldo's "judicially declared" disqualification for lack of Filipino citizenship a continuing bar to his over the same opponent Raul Lee. Twice, he was judicially declared a non-Filipino and thus twice
eligibility to run for, be elected to or hold the governorship of Sorsogon? disqualified from holding and discharging his popular mandate. Now, he comes to us a third time, with a
fresh vote from the people of Sorsogon and a favorable decision from the Commission on Elections to
3. Did the respondent Comelec have jurisdiction over the initiatory petition in SPC No. 95-317 boot. Moreover, he now boasts of having successfully passed through the third and last mode of
considering that said petition is not "a pre-proclamation case, an election protest or a quo reacquiring citizenship: by repatriation under P.D. No. 725, with no less than the Solicitor General
warranto case"? himself, who was the prime opposing counsel in the previous cases he lost, this time, as counsel for co-
respondent Comelec, arguing the validity of his cause (in addition to his able private counsel Sixto S.
Brillantes, Jr.). That he took his oath of allegiance under the provisions of said Decree at 2:00 p.m. on
4. Was the proclamation of Lee, a runner-up in the election, valid and legal in light of existing June 30, 1995 is not disputed. Hence, he insists that he -- not Lee -- should have been proclaimed as the
jurisprudence? duly-elected governor of Sorsogon when the Provincial Board of Canvassers met at 8:30 p.m. on the said
date since, clearly and unquestionably, he garnered the highest number of votes in the elections and
5. Did the respondent Commission on Elections exceed its jurisdiction in promulgating the assailed since at that time, he already reacquired his citizenship.
Resolutions, all of which prevented Frivaldo from assuming the governorship of Sorsogon, considering
that they were not rendered within the period referred to in Section 78 of the Omnibus Election En contrario, Lee argues that Frivaldo's repatriation is tainted with serious defects, which we shall now
Code, viz., "not later than fifteen days before the elections"? discuss in seriatim.

The First Issue: Frivaldo's Repatriation First, Lee tells us that P.D. No. 725 had "been effectively repealed", asserting that "then President
Corazon Aquino exercising legislative powers under the Transitory Provisions of the 1987 Constitution,
The validity and effectivity of Frivaldo's repatriation is the lis mota, the threshold legal issue in this case. forbade the grant of citizenship by Presidential Decree or Executive Issuances as the same poses a
All the other matters raised are secondary to this. serious and contentious issue of policy which the present government, in the exercise of prudence and
sound discretion, should best leave to the judgment of the first Congress under the 1987 Constitution",
adding that in her memorandum dated March 27, 1987 to the members of the Special Committee on
The Local Government Code of 1991 19 expressly requires Philippine citizenship as a qualification for Naturalization constituted for purposes of Presidential Decree No. 725, President Aquino directed them
elective local officials, including that of provincial governor, thus: "to cease and desist from undertaking any and all proceedings within your functional area of
responsibility as defined under Letter of Instructions (LOI) No. 270 dated April 11, 1975, as amended." 23
Sec. 39. Qualifications. -- (a) An elective local official must be a citizen of the
Philippines; a registered voter in the barangay, municipality, city, or province or, in This memorandum dated March 27, 1987 24 cannot by any stretch of legal hermeneutics be construed as
the case of a member of the sangguniang panlalawigan, sangguniang panlungsod, a law sanctioning or authorizing a repeal of P.D. No. 725. Laws are repealed only by subsequent
or sangguniang bayan, the district where he intends to be elected; a resident ones 25 and a repeal may be express or implied. It is obvious that no express repeal was made because
therein for at least one (1) year immediately preceding the day of the election; and then President Aquino in her memorandum -- based on the copy furnished us by Lee -- did not
able to read and write Filipino or any other local language or dialect. categorically and/or impliedly state that P.D. 725 was being repealed or was being rendered without any
legal effect. In fact, she did not even mention it specifically by its number or text. On the other hand, it is
(b) Candidates for the position of governor, vice governor or a basic rule of statutory construction that repeals by implication are not favored. An implied repeal will
member of the sangguniang panlalawigan, or mayor, vice not be allowed "unless it is convincingly and unambiguously demonstrated that the two laws are clearly
mayor or member of the sangguniang panlungsod of highly repugnant and patently inconsistent that they cannot co-exist". 26
urbanized cities must be at least twenty-three (23) years of
age on election day. The memorandum of then President Aquino cannot even be regarded as a legislative enactment, for not
every pronouncement of the Chief Executive even under the Transitory Provisions of the 1987
xxx xxx xxx Constitution can nor should be regarded as an exercise of her law-making powers. At best, it could be
treated as an executive policy addressed to the Special Committee to halt the acceptance and processing
of applications for repatriation pending whatever "judgment the first Congress under the 1987
Inasmuch as Frivaldo had been declared by this Court 20 as a non-citizen, it is therefore incumbent upon
Constitution" might make. In other words, the former President did not repeal P.D. 725 but left it to the
him to show that he has reacquired citizenship; in fine, that he possesses the qualifications prescribed
first Congress -- once created -- to deal with the matter. If she had intended to repeal such law, she
under the said statute (R.A. 7160).
should have unequivocally said so instead of referring the matter to Congress. The fact is she carefully
couched her presidential issuance in terms that clearly indicated the intention of "the present then the issue of when an aspirant for public office should be a citizen was NOT resolved at all by the
government, in the exercise of prudence and sound discretion" to leave the matter of repeal to the new Court. Which question we shall now directly rule on.
Congress. Any other interpretation of the said Presidential Memorandum, such as is now being proffered
to the Court by Lee, would visit unmitigated violence not only upon statutory construction but on
Under Sec. 39 of the Local Government Code, "(a)n elective local official must be:
common sense as well.

* a citizen of the Philippines;


Second, Lee also argues that "serious congenital irregularities flawed the repatriation proceedings,"
asserting that Frivaldo's application therefor was "filed on June 29, 1995 . . . (and) was approved in just
one day or on June 30, 1995 . . .", which "prevented a judicious review and evaluation of the merits * a registered voter in the barangay, municipality, city, or province . . . where he
thereof." Frivaldo counters that he filed his application for repatriation with the Office of the President in intends to be elected;
Malacaang Palace on August 17, 1994. This is confirmed by the Solicitor General. However, the Special
Committee was reactivated only on June 8, 1995, when presumably the said Committee started * a resident therein for at least one (1) year immediately preceding the day of the
processing his application. On June 29, 1995, he filled up and re-submitted the FORM that the election;
Committee required. Under these circumstances, it could not be said that there was "indecent haste" in
the processing of his application.
* able to read and write Filipino or any other local language or dialect.

Anent Lee's charge that the "sudden reconstitution of the Special Committee on Naturalization was
intended solely for the personal interest of respondent," 27 the Solicitor General explained during the * In addition, "candidates for the position of governor . . . must be at least twenty-
oral argument on March 19, 1996 that such allegation is simply baseless as there were many others who three (23) years of age on election day.
applied and were considered for repatriation, a list of whom was submitted by him to this Court, through
a Manifestation 28 filed on April 3, 1996. From the above, it will be noted that the law does not specify any particular date or time when the
candidate must possess citizenship, unlike that for residence (which must consist of at least one year's
On the basis of the parties' submissions, we are convinced that the presumption of regularity in the residency immediately preceding the day of election) and age (at least twenty three years of age on
performance of official duty and the presumption of legality in the repatriation of Frivaldo have not been election day).
successfully rebutted by Lee. The mere fact that the proceedings were speeded up is by itself not a
ground to conclude that such proceedings were necessarily tainted. After all, the requirements of Philippine citizenship is an indispensable requirement for holding an elective public office, 31 and the
repatriation under P.D. No. 725 are not difficult to comply with, nor are they tedious and cumbersome. purpose of the citizenship qualification is none other than to ensure that no alien, i.e., no person owing
In fact, P.D. allegiance to another nation, shall govern our people and our country or a unit of territory thereof. Now,
725 29 itself requires very little of an applicant, and even the rules and regulations to implement the said an official begins to govern or to discharge his functions only upon his proclamation and on the day the
decree were left to the Special Committee to promulgate. This is not unusual since, unlike in law mandates his term of office to begin. Since Frivaldo re-assumed his citizenship on June 30, 1995 --
naturalization where an alien covets a first-timeentry into Philippine political life, in repatriation the the very day 32 the term of office of governor (and other elective officials) began -- he was therefore
applicant is a former natural-born Filipino who is merely seeking to reacquire his previous citizenship. In already qualified to be proclaimed, to hold such office and to discharge the functions and responsibilities
the case of Frivaldo, he was undoubtedly a natural-born citizen who openly and faithfully served his thereof as of said date. In short, at that time, he was already qualified to govern his native Sorsogon. This
country and his province prior to his naturalization in the United States -- a naturalization he insists was is the liberal interpretation that should give spirit, life and meaning to our law on qualifications
made necessary only to escape the iron clutches of a dictatorship he abhorred and could not in consistent with the purpose for which such law was enacted. So too, even from a literal (as distinguished
conscience embrace -- and who, after the fall of the dictator and the re-establishment of democratic from liberal) construction, it should be noted that Section 39 of the Local Government Code speaks of
space, wasted no time in returning to his country of birth to offer once more his talent and services to "Qualifications" of "ELECTIVE OFFICIALS", not of candidates. Why then should such qualification be
his people. required at the time of election or at the time of the filing of the certificates of candidacies, as Lee
insists? Literally, such qualifications -- unless otherwise expressly conditioned, as in the case of age and
So too, the fact that ten other persons, as certified to by the Solicitor General, were granted repatriation residence -- should thus be possessed when the "elective [or elected] official" begins to govern, i.e., at
argues convincingly and conclusively against the existence of favoritism vehemently posited by Raul Lee. the time he is proclaimed and at the start of his term -- in this case, on June 30, 1995. Paraphrasing this
At any rate, any contest on the legality of Frivaldo's repatriation should have been pursued before the Court's ruling in Vasquez vs. Giap and Li Seng Giap & Sons, 33 if the purpose of the citizenship
Committee itself, and, failing there, in the Office of the President, pursuant to the doctrine of exhaustion requirement is to ensure that our people and country do not end up being governed by aliens,i.e.,
of administrative remedies. persons owing allegiance to another nation, that aim or purpose would not be thwarted but instead
achieved by construing the citizenship qualification as applying to the time of proclamation of the
elected official and at the start of his term.
Third, Lee further contends that assuming the assailed repatriation to be valid, nevertheless it could only
be effective as at 2:00 p.m. of June 30, 1995 whereas the citizenship qualification prescribed by the Local
Government Code "must exist on the date of his election, if not when the certificate of candidacy is But perhaps the more difficult objection was the one raised during the oral argument 34 to the effect that
filed," citing our decision in G.R. 104654 30 which held that "both the Local Government Code and the the citizenship qualification should be possessed at the time the candidate (or for that matter the
Constitution require that only Philippine citizens can run and be elected to public office." Obviously, elected official) registered as a voter. After all, Section 39, apart from requiring the official to be a citizen,
however, this was a mere obiter as the only issue in said case was whether Frivaldo's naturalization was also specifies as another item of qualification, that he be a "registered voter". And, under the law 35 a
valid or not -- and NOT the effective date thereof. Since the Court held his naturalization to be invalid, "voter" must be a citizen of the Philippines. So therefore, Frivaldo could not have been a voter -- much
less a validly registered one -- if he was not a citizen at the time of such registration.
The answer to this problem again lies in discerning the purpose of the requirement. If the law intended "healing acts . . . curing defects and adding to the means of enforcing existing obligations . . . (and) are
thecitizenship qualification to be possessed prior to election consistent with the requirement of being a intended to supply defects, abridge superfluities in existing laws, and curb certain evils. . . . By their very
registered voter, then it would not have made citizenship a SEPARATE qualification. The law abhors a nature, curative statutes are retroactive . . . (and) reach back to past events to correct errors or
redundancy. It therefore stands to reason that the law intended CITIZENSHIP to be a qualification distinct irregularities and to render valid and effective attempted acts which would be otherwise ineffective for
from being a VOTER, even if being a voter presumes being a citizen first. It also stands to reason that the the purpose the parties intended."
voter requirement was included as another qualification (aside from "citizenship"), not to reiterate the
need for nationality but to require that the official be registered as a voter IN THE AREA OR TERRITORY
On the other hand, remedial or procedural laws, i.e., those statutes relating to remedies or modes of
he seeks to govern, i.e., the law states: "a registered voter in the barangay, municipality, city, or province
procedure, which do not create new or take away vested rights, but only operate in furtherance of the
. . . where he intends to be elected." It should be emphasized that the Local Government Code requires
remedy or confirmation of such rights, ordinarily do not come within the legal meaning of a
an elective official to be a registered voter. It does not require him to vote actually. Hence, registration --
retrospective law, nor within the general rule against the retrospective operation of statutes. 43
not the actual voting -- is the core of this "qualification". In other words, the law's purpose in this second
requirement is to ensure that the prospective official is actually registered in the area he seeks to govern
-- and not anywhere else. A reading of P.D. 725 immediately shows that it creates a new right, and also provides for a new remedy,
thereby filling certain voids in our laws. Thus, in its preamble, P.D. 725 expressly recognizes the plight of
"many Filipino women (who) had lost their Philippine citizenship by marriage to aliens" and who could
Before this Court, Frivaldo has repeatedly emphasized -- and Lee has not disputed -- that he "was and is a
not, under the existing law (C.A. No. 63, as amended) avail of repatriation until "after the death of their
registered voter of Sorsogon, and his registration as a voter has been sustained as valid by judicial
husbands or the termination of their marital status" and who could neither be benefitted by the 1973
declaration . . . In fact, he cast his vote in his precinct on May 8, 1995." 36
Constitution's new provision allowing "a Filipino woman who marries an alien to retain her Philippine
citizenship . . ." because "such provision of the new Constitution does not apply to Filipino women who
So too, during the oral argument, his counsel steadfastly maintained that "Mr. Frivaldo has always been had married aliens before said constitution took effect." Thus, P.D. 725 granted a new right to these
a registered voter of Sorsogon. He has voted in 1987, 1988, 1992, then he voted again in 1995. In fact, women -- the right to re-acquire Filipino citizenship even during their marital coverture, which right did
his eligibility as a voter was questioned, but the court dismissed (sic) his eligibility as a voter and he was not exist prior to P.D. 725. On the other hand, said statute also provided a new remedyand a new right in
allowed to vote as in fact, he voted in all the previous elections including on May 8, 1995." 37 favor of other "natural born Filipinos who (had) lost their Philippine citizenship but now desire to re-
acquire Philippine citizenship", because prior to the promulgation of P.D. 725 such former Filipinos
would have had to undergo the tedious and cumbersome process of naturalization, but with the advent
It is thus clear that Frivaldo is a registered voter in the province where he intended to be elected.
of P.D. 725 they could now re-acquire their Philippine citizenship under the simplified procedure of
repatriation.
There is yet another reason why the prime issue of citizenship should be reckoned from the date of
proclamation, not necessarily the date of election or date of filing of the certificate of candidacy. Section
The Solicitor General 44 argues:
253 of the Omnibus Election Code 38 gives any voter, presumably including the defeated candidate, the
opportunity to question the ELIGIBILITY (or the disloyalty) of a candidate. This is the only provision of the
Code that authorizes a remedy on how to contest before the Comelec an incumbent's ineligibility arising By their very nature, curative statutes are retroactive, (DBP vs. CA, 96 SCRA 342),
from failure to meet the qualifications enumerated under Sec. 39 of the Local Government Code. Such since they are intended to supply defects, abridge superfluities in existing laws (Del
remedy of Quo Warranto can be availed of "within ten days after proclamation" of the winning Castillo vs. Securities and Exchange Commission, 96 Phil. 119) and curb certain
candidate. Hence, it is only at such time that the issue of ineligibility may be taken cognizance of by the evils (Santos vs. Duata, 14 SCRA 1041).
Commission. And since, at the very moment of Lee's proclamation (8:30 p.m., June 30, 1995), Juan G.
Frivaldo was already and indubitably a citizen, having taken his oath of allegiance earlier in the afternoon
In this case, P.D. No. 725 was enacted to cure the defect in the existing
of the same day, then he should have been the candidate proclaimed as he unquestionably garnered the
naturalization law, specifically C.A. No. 63 wherein married Filipino women are
highest number of votes in the immediately preceding elections and such oath had already cured his
allowed to repatriate only upon the death of their husbands, and natural-born
previous "judicially-declared" alienage. Hence, at such time, he was no longer ineligible.
Filipinos who lost their citizenship by naturalization and other causes faced the
difficulty of undergoing the rigid procedures of C.A. 63 for reacquisition of Filipino
But to remove all doubts on this important issue, we also hold that the repatriation of Frivaldo citizenship by naturalization.
RETROACTED to the date of the filing of his application on August 17, 1994.
Presidential Decree No. 725 provided a remedy for the aforementioned legal
It is true that under the Civil Code of the Philippines, 39 "(l)aws shall have no retroactive effect, unless the aberrations and thus its provisions are considered essentially remedial and
contrary is provided." But there are settled exceptions 40 to this general rule, such as when the statute is curative.
CURATIVE or REMEDIAL in nature or when it CREATES NEW RIGHTS.
In light of the foregoing, and prescinding from the wording of the preamble, it is unarguable that the
According to Tolentino, 41 curative statutes are those which undertake to cure errors and irregularities, legislative intent was precisely to give the statute retroactive operation. "(A) retrospective operation is
thereby validating judicial or administrative proceedings, acts of public officers, or private deeds and given to a statute or amendment where the intent that it should so operate clearly appears from a
contracts which otherwise would not produce their intended consequences by reason of some statutory consideration of the act as a whole, or from the terms thereof." 45 It is obvious to the Court that the
disability or failure to comply with some technical requirement. They operate on conditions already statute was meant to "reach back" to those persons, events and transactions not otherwise covered by
existing, and are necessarily retroactive in operation. Agpalo, 42 on the other hand, says that curative prevailing law and jurisprudence. And inasmuch as it has been held that citizenship is a political and civil
statutes are right equally as important as the freedom of speech, liberty of abode, the right against unreasonable
searches and seizures and other guarantees enshrined in the Bill of Rights, therefore the legislative (June 30, 1995) or the date of election (May 8, 1995) or date of filing his certificate of candidacy (March
intent to give retrospective operation to P.D. 725 must be given the fullest effect possible. "(I)t has been 20, 1995) would become moot.
said that a remedial statute must be so construed as to make it effect the evident purpose for which it
was enacted, so that if the reason of the statute extends to past transactions, as well as to those in the
Based on the foregoing, any question regarding Frivaldo's status as a registered voter would also be
future, then it will be so applied although the statute does not in terms so direct, unless to do so would
deemed settled. Inasmuch as he is considered as having been repatriated -- i.e., his Filipino citizenship
impair some vested right or violate some constitutional guaranty." 46 This is all the more true of P.D. 725,
restored -- as of August 17, 1994, his previous registration as a voter is likewise deemed validated as of
which did not specify any restrictions on or delimit or qualify the right of repatriation granted therein.
said date.

At this point, a valid question may be raised: How can the retroactivity of P.D. 725 benefit Frivaldo
It is not disputed that on January 20, 1983 Frivaldo became an American. Would the retroactivity of his
considering that said law was enacted on June 5, 1975, while Frivaldo lost his Filipino citizenship much
repatriation not effectively give him dual citizenship, which under Sec. 40 of the Local Government Code
later, on January 20, 1983, and applied for repatriation even later, on August 17, 1994?
would disqualify him "from running for any elective local position?" 49 We answer this question in the
negative, as there is cogent reason to hold that Frivaldo was really STATELESS at the time he took said
While it is true that the law was already in effect at the time that Frivaldo became an American citizen, oath of allegiance and even before that, when he ran for governor in 1988. In his Comment, Frivaldo
nevertheless, it is not only the law itself (P.D. 725) which is to be given retroactive effect, but even the wrote that he "had long renounced and had long abandoned his American citizenship -- long before May
repatriation granted under said law to Frivaldo on June 30, 1995 is to be deemed to have retroacted to 8, 1995. At best, Frivaldo was stateless in the interim -- when he abandoned and renounced his US
the date of his application therefor, August 17, 1994. The reason for this is simply that if, as in this case, citizenship but before he was repatriated to his Filipino citizenship." 50
it was the intent of the legislative authority that the law should apply to past events -- i.e., situations and
transactions existing even before the law came into being -- in order to benefit the greatest number of
On this point, we quote from the assailed Resolution dated December 19, 1995: 51
former Filipinos possible thereby enabling them to enjoy and exercise the constitutionally guaranteed
right of citizenship, and such legislative intention is to be given the fullest effect and expression,
then there is all the more reason to have the law apply in a retroactive or retrospective manner to By the laws of the United States, petitioner Frivaldo lost his American citizenship
situations, events and transactions subsequent to the passage of such law. That is, the repatriation when he took his oath of allegiance to the Philippine Government when he ran for
granted to Frivaldo on June 30, 1995 can and should be made to take effect as of date of his application. Governor in 1988, in 1992, and in 1995. Every certificate of candidacy contains an
As earlier mentioned, there is nothing in the law that would bar this or would show a contrary intention oath of allegiance to the Philippine Government."
on the part of the legislative authority; and there is no showing that damage or prejudice to anyone, or
anything unjust or injurious would result from giving retroactivity to his repatriation. Neither has Lee These factual findings that Frivaldo has lost his foreign nationality long before the elections of 1995 have
shown that there will result the impairment of any contractual obligation, disturbance of any vested not been effectively rebutted by Lee. Furthermore, it is basic that such findings of the Commission are
right or breach of some constitutional guaranty. conclusive upon this Court, absent any showing of capriciousness or arbitrariness or
abuse. 52
Being a former Filipino who has served the people repeatedly, Frivaldo deserves a liberal interpretation
of Philippine laws and whatever defects there were in his nationality should now be deemed mooted by The Second Issue: Is Lack of Citizenship
his repatriation. a Continuing Disqualification?

Another argument for retroactivity to the date of filing is that it would prevent prejudice to applicants. If Lee contends that the May 1, 1995 Resolution 53 of the Comelec Second Division in SPA No. 95-028 as
P.D. 725 were not to be given retroactive effect, and the Special Committee decides not to act, i.e., to affirmed in totoby Comelec En Banc in its Resolution of May 11, 1995 "became final and executory after
delay the processing of applications for any substantial length of time, then the former Filipinos who may five (5) days or on May 17, 1995, no restraining order having been issued by this Honorable
be stateless, as Frivaldo -- having already renounced his American citizenship -- was, may be prejudiced Court. 54 Hence, before Lee "was proclaimed as the elected governor on June 30, 1995, there was already
for causes outside their control. This should not be. In case of doubt in the interpretation or application a final and executory judgment disqualifying" Frivaldo. Lee adds that this Court's two rulings (which
of laws, it is to be presumed that the law-making body intended right and justice to prevail. 47 Frivaldo now concedes were legally "correct") declaring Frivaldo an alien have also become final and
executory way before the 1995 elections, and these "judicial pronouncements of his political status as an
And as experience will show, the Special Committee was able to process, act upon and grant applications American citizen absolutely and for all time disqualified (him) from running for, and holding any public
for repatriation within relatively short spans of time after the same were filed. 48 The fact that such office in the Philippines."
interregna were relatively insignificant minimizes the likelihood of prejudice to the government as a
result of giving retroactivity to repatriation. Besides, to the mind of the Court, direct prejudice to the We do not agree.
government is possible only where a person's repatriation has the effect of wiping out a liability of his to
the government arising in connection with or as a result of his being an alien, and accruing only during
the interregnum between application and approval, a situation that is not present in the instant case. It should be noted that our first ruling in G.R. No. 87193 disqualifying Frivaldo was rendered in
connection with the 1988 elections while that in G.R. No. 104654 was in connection with the 1992
elections. That he was disqualified for such elections is final and can no longer be changed. In the words
And it is but right and just that the mandate of the people, already twice frustrated, should now prevail. of the respondent Commission (Second Division) in its assailed Resolution: 55
Under the circumstances, there is nothing unjust or iniquitous in treating Frivaldo's repatriation as
having become effective as of the date of his application, i.e., on August 17, 1994. This being so, all
questions about his possession of the nationality qualification -- whether at the date of proclamation
The records show that the Honorable Supreme Court had decided that Frivaldo The Fourth Issue: Was Lee's Proclamation Valid?
was not a Filipino citizen and thus disqualified for the purpose of the 1988 and
1992 elections. However, there is no record of any "final judgment" of the
Frivaldo assails the validity of the Lee proclamation. We uphold him for the following reasons:
disqualification of Frivaldo as a candidate for the May 8, 1995 elections. What the
Commission said in its Order of June 21, 1995 (implemented on June 30, 1995),
directing the proclamation of Raul R. Lee, was that Frivaldo was not a Filipino First. To paraphrase this Court in Labo vs. COMELEC, 60 "the fact remains that he (Lee) was not the choice
citizen "having been declared by the Supreme Court in its Order dated March 25, of the sovereign will," and in Aquino vs. COMELEC, 61 Lee is "a second placer, . . . just that, a second
1995, not a citizen of the Philippines." This declaration of the Supreme Court, placer."
however, was in connection with the 1992 elections.
In spite of this, Lee anchors his claim to the governorship on the pronouncement of this Court in the
Indeed, decisions declaring the acquisition or denial of citizenship cannot govern a person's future status aforesaid Labo 62 case, as follows:
with finality. This is because a person may subsequently reacquire, or for that matter lose, his citizenship
under any of the modes recognized by law for the purpose. Hence, in Lee vs. Commissioner of The rule would have been different if the electorate fully aware in fact and in law
Immigration, 56 we held: of a candidate's disqualification so as to bring such awareness within the realm of
notoriety, would nonetheless cast their votes in favor of the ineligible candidate. In
Everytime the citizenship of a person is material or indispensable in a judicial or such case, the electorate may be said to have waived the validity and efficacy of
administrative case, whatever the corresponding court or administrative authority their votes by notoriously misapplying their franchise or throwing away their
decides therein as to such citizenship is generally not considered res judicata, votes, in which case, the eligible candidate obtaining the next higher number of
hence it has to be threshed out again and again, as the occasion demands. votes may be deemed elected.

The Third Issue: Comelec's Jurisdiction But such holding is qualified by the next paragraph, thus:
Over The Petition in SPC No. 95-317
But this is not the situation obtaining in the instant dispute. It has not been shown,
Lee also avers that respondent Comelec had no jurisdiction to entertain the petition in SPC No. 95-317 and none was alleged, that petitioner Labo was notoriously known as an ineligible
because the only "possible types of proceedings that may be entertained by the Comelec are a pre- candidate, much less the electorate as having known of such fact. On the contrary,
proclamation case, an election protest or a quo warranto case". Again, Lee reminds us that he was petitioner Labo was even allowed by no less than the Comelec itself in its
proclaimed on June 30, 1995 but that Frivaldo filed SPC No. 95-317 questioning his (Lee's) proclamation resolution dated May 10, 1992 to be voted for the office of the city Payor as its
only on July 6, 1995 -- "beyond the 5-day reglementary period." Hence, according to him, Frivaldo's resolution dated May 9, 1992 denying due course to petitioner Labo's certificate of
"recourse was to file either an election protest or a quo warranto action." candidacy had not yet become final and subject to the final outcome of this case.

This argument is not meritorious. The Constitution 57 has given the Comelec ample power to "exercise The last-quoted paragraph in Labo, unfortunately for Lee, is the ruling appropriate in this case because
exclusive original jurisdiction over all contests relating to the elections, returns and qualifications of all Frivaldo was in 1995 in an identical situation as Labo was in 1992 when the Comelec's cancellation of his
elective . . . provincial . . . officials." Instead of dwelling at length on the various petitions that Comelec, in certificate of candidacy was not yet final on election day as there was in both cases a pending motion for
the exercise of its constitutional prerogatives, may entertain, suffice it to say that this Court has reconsideration, for which reason Comelec issued an (omnibus) resolution declaring that Frivaldo (like
invariably recognized the Commission's authority to hear and decide petitions for annulment of Labo in 1992) and several others can still be voted for in the May 8, 1995 election, as in fact, he was.
proclamations -- of which SPC No. 95-317 obviously is one. 58 Thus, in Mentang vs. COMELEC, 59 we ruled:
Furthermore, there has been no sufficient evidence presented to show that the electorate of Sorsogon
The petitioner argues that after proclamation and assumption of office, a pre- was "fully aware in fact and in law" of Frivaldo's alleged disqualification as to "bring such awareness
proclamation controversy is no longer viable. Indeed, we are aware of cases within the realm of notoriety;" in other words, that the voters intentionally wasted their ballots knowing
holding that pre-proclamation controversies may no longer be entertained by the that, in spite of their voting for him, he was ineligible. If Labo has any relevance at all, it is that the vice-
COMELEC after the winning candidate has been proclaimed. (citing Gallardo vs. governor -- and not Lee -- should be pro- claimed, since in losing the election, Lee was, to
Rimando, 187 SCRA 463; Salvacion vs. COMELEC, 170 SCRA 513; Casimiro vs. paraphrase Labo again, "obviously not the choice of the people" of Sorsogon. This is the emphatic
COMELEC, 171 SCRA 468.) This rule, however, is premised on an assumption that teaching of Labo:
the proclamation is no proclamation at all and the proclaimed candidate's
assumption of office cannot deprive the COMELEC of the power to make such The rule, therefore, is: the ineligibility of a candidate receiving majority votes does
declaration of nullity. (citing Aguam vs. COMELEC, 23 SCRA 883; Agbayani vs. not entitle the eligible candidate receiving the next highest number of votes to be
COMELEC, 186 SCRA 484.) declared elected. A minority or defeated candidate cannot be deemed elected to
the office.
The Court however cautioned that such power to annul a proclamation must "be done within ten (10)
days following the proclamation." Inasmuch as Frivaldo's petition was filed only six (6) days after Lee's
proclamation, there is no question that the Comelec correctly acquired jurisdiction over the same.
Second. As we have earlier declared Frivaldo to have seasonably reacquired his citizenship and inasmuch Mr. Justice Davide also believes that Quo Warranto is not the sole remedy to question the ineligibility of
as he obtained the highest number of votes in the 1995 elections, he -- not Lee -- should be proclaimed. a candidate, citing the Comelec's authority under Section 78 of the Omnibus Election Code allowing the
Hence, Lee's proclamation was patently erroneous and should now be corrected. denial of a certificate of candidacy on the ground of a false material representation therein as required
by Section 74. Citing Loong, he then states his disagreement with our holding that Section 78 is merely
directory. We really have no quarrel. Our point is that Frivaldo was in error in his claim in G.R. No.
The Fifth Issue: Is Section 78 of the
120295 that the Comelec Resolutions promulgated on May 1, 1995 and May 11, 1995 were invalid
Election Code Mandatory?
because they were issued "not later than fifteen days before the election" as prescribed by Section 78. In
dismissing the petition in G.R. No. 120295, we hold that the Comelec did not commit grave abuse of
In G.R. No. 120295, Frivaldo claims that the assailed Resolution of the Comelec (Second Division) dated discretion because "Section 6 of R.A. 6646 authorizes the Comelec to try and decide disqualifications
May 1, 1995 and the confirmatory en banc Resolution of May 11, 1995 disqualifying him for want of even after the elections." In spite of his disagreement with us on this point, i.e., that Section 78 "is
citizenship should be annulled because they were rendered beyond the fifteen (15) day period merely directory", we note that just like us, Mr. Justice Davide nonetheless votes to "DISMISS G.R. No.
prescribed by Section 78, of the Omnibus Election Code which reads as follows: 120295". One other point. Loong, as quoted in the dissent, teaches that a petition to deny due course
under Section 78 must be filed within the 25-day period prescribed therein. The present case however
Sec. 78. Petition to deny due course or to cancel a certificate of candidacy. -- A deals with the period during which the Comelec may decide such petition. And we hold that it may be
verified petition seeking to deny due course or to cancel a certificate of candidacy decided even after thefifteen day period mentioned in Section 78. Here, we rule that a
may be filed by any person exclusively on the ground that any material decision promulgated by the Comelec even after the elections is valid but Loong held that a
representation contained therein as required under Section 74 hereof is false. The petition filed beyond the 25-day period is out of time. There is no inconsistency nor conflict.
petition may be filed at any time not later than twenty-five days from the time of
the filing of the certificate of candidacy and shall be decided after notice and Mr. Justice Davide also disagrees with the Court's holding that, given the unique factual circumstances of
hearing, not later than fifteen days before the election. (Emphasis supplied.) Frivaldo, repatriation may be given retroactive effect. He argues that such retroactivity "dilutes" our
holding in the first Frivaldo case. But the first (and even the second Frivaldo) decision did not directly
This claim is now moot and academic inasmuch as these resolutions are deemed superseded by the involve repatriation as a mode of acquiring citizenship. If we may repeat, there is no question that
subsequent ones issued by the Commission (First Division) on December 19, 1995, affirmed en banc 63 on Frivaldo was not a Filipino for purposes of determining his qualifications in the 1988 and 1992 elections.
February 23, 1996; which both upheld his election. At any rate, it is obvious that Section 78 is merely That is settled. But his supervening repatriation has changed his political status -- not in 1988 or 1992,
directory as Section 6 of R.A. No. 6646 authorizes the Commission to try and decide petitions for but only in the 1995 elections.
disqualifications even after the elections, thus:
Our learned colleague also disputes our holding that Frivaldo was stateless prior to his repatriation,
Sec. 6. Effect of Disqualification Case. -- Any candidate who has been declared by saying that "informal renunciation or abandonment is not a ground to lose American citizenship". Since
final judgment to be disqualified shall not be voted for, and the votes cast for him our courts are charged only with the duty of determining who are Philippine nationals, we cannot rule on
shall not be counted. If for any reason a candidate is not declared by final the legal question of who are or who are not Americans. It is basic in international law that a State
judgment before an election to be disqualified and he is voted for and receives the determines ONLY those who are its own citizens -- not who are the citizens of other countries. 65 The
winning number of votes in such election, the Court or Commission shall continue issue here is: the Comelec made a finding of fact that Frivaldo was stateless and such finding has not
with the trial and hearing of the action, inquiry or protest and upon motion of the been shown by Lee to be arbitrary or whimsical. Thus, following settled case law, such finding is binding
complainant or any intervenor, may during the pendency thereof order the and final.
suspension of the proclamation of such candidate whenever the evidence of his
guilt is strong. (emphasis supplied) The dissenting opinion also submits that Lee who lost by chasmic margins to Frivaldo in all three previous
elections, should be declared winner because "Frivaldo's ineligibility for being an American was publicly
Refutation of known". First, there is absolutely no empirical evidence for such "public" knowledge. Second, even if
Mr. Justice Davide's Dissent there is, such knowledge can be true post facto only of the last two previous elections. Third, even the
Comelec and now this Court were/are still deliberating on his nationality before, during and after the
1995 elections. How then can there be such "public" knowledge?
In his dissenting opinion, the esteemed Mr. Justice Hilario G. Davide, Jr. argues that President Aquino's
memorandum dated March 27, 1987 should be viewed as a suspension (not a repeal, as urged by Lee) of
P.D. 725. But whether it decrees a suspension or a repeal is a purely academic distinction because the Mr. Justice Davide submits that Section 39 of the Local Government Code refers to the qualifications
said issuance is not a statute that can amend or abrogate an existing law. of electivelocal officials, i.e., candidates, and not elected officials, and that the citizenship qualification
The existence and subsistence of P.D. 725 were recognized in the first Frivaldo case; 64 viz., "(u)nder CA [under par. (a) of that section] must be possessed by candidates, not merely at the commencement of
No. 63 as amended by CA No. 473 and P.D. No. 725, Philippine citizenship maybe reacquired by . . . the term, but by election day at the latest. We see it differently. Section 39, par. (a) thereof speaks of
repatriation". He also contends that by allowing Frivaldo to register and to remain as a registered voter, "elective local official" while par. (b) to (f) refer to "candidates". If the qualifications under par. (a) were
the Comelec and in effect this Court abetted a "mockery" of our two previous judgments declaring him a intended to apply to "candidates" and not elected officials, the legislature would have said so, instead of
non-citizen. We do not see such abetting or mockery. The retroactivity of his repatriation, as discussed differentiating par. (a) from the rest of the paragraphs. Secondly, if Congress had meant that the
earlier, legally cured whatever defects there may have been in his registration as a voter for the purpose citizenship qualification should be possessed at election day or prior thereto, it would have specifically
of the 1995 elections. Such retroactivity did not change his disqualifications in 1988 and 1992, which stated such detail, the same way it did in pars. (b) to (f) far other qualifications of candidates for
were the subjects of such previous rulings. governor, mayor, etc.
Mr. Justice Davide also questions the giving of retroactive effect to Frivaldo's repatriation on the ground, . . . (L)aws governing election contests must be liberally construed to the end that
among others, that the law specifically provides that it is only after taking the oath of allegiance that the will of the people in the choice of public officials may not be defeated by mere
applicants shall be deemed to have reacquired Philippine citizenship. We do not question what the technical objections (citations omitted). 67
provision states. We hold however that the provision should be understood thus: that after taking the
oath of allegiance the applicant is deemed to have reacquired Philippine citizenship, which reacquisition
The law and the courts must accord Frivaldo every possible protection, defense and refuge, in deference
(or repatriation) is deemed for all purposes and intents to have retroacted to the date of his application
to the popular will. Indeed, this Court has repeatedly stressed the importance of giving effect to the
therefor.
sovereign will in order to ensure the survival of our democracy. In any action involving the possibility of a
reversal of the popular electoral choice, this Court must exert utmost effort to resolve the issues in a
In any event, our "so too" argument regarding the literal meaning of the word "elective" in reference to manner that would give effect to the will of the majority, for it is merely sound public policy to cause
Section 39 of the Local Authority Code, as well as regarding Mr. Justice Davide's thesis that the very elective offices to be filled by those who are the choice of the majority. To successfully challenge a
wordings of P.D. 725 suggest non-retroactivity, were already taken up rather extensively earlier in this winning candidate's qualifications, the petitioner must clearly demonstrate that the ineligibility is so
Decision. patently antagonistic 68 to constitutional and legal principles that overriding such ineligibility and thereby
giving effect to the apparent will of the people, would ultimately create greater prejudice to the very
democratic institutions and juristic traditions that our Constitution and laws so zealously protect and
Mr. Justice Davide caps his paper with a clarion call: "This Court must be the first to uphold the Rule of
promote. In this undertaking, Lee has miserably failed.
Law." We agree -- we must all follow the rule of law. But that is NOT the issue here. The issue
is how should the law be interpreted and applied in this case so it can be followed, so it can rule!
In Frivaldo's case. it would have been technically easy to find fault with his cause. The Court could have
refused to grant retroactivity to the effects of his repatriation and hold him still ineligible due to his
At balance, the question really boils down to a choice of philosophy and perception of how to interpret
failure to show his citizenship at the time he registered as a voter before the 1995 elections. Or, it could
and apply laws relating to elections: literal or liberal; the letter or the spirit, the naked provision or its
have disputed the factual findings of the Comelec that he was stateless at the time of repatriation and
ultimate purpose; legal syllogism or substantial justice; in isolation or in the context of social conditions;
thus hold his consequent dual citizenship as a disqualification "from running for any elective local
harshly against or gently in favor of the voters' obvious choice. In applying election laws, it would be far
position." But the real essence of justice does not emanate from quibblings over patchwork legal
better to err in favor of popular sovereignty than to be right in complex but little understood legalisms.
technicality. It proceeds from the spirit's gut consciousness of the dynamic role of law as a brick in the
Indeed, to inflict a thrice rejected candidate upon the electorate of Sorsogon would constitute
ultimate development of the social edifice. Thus, the Court struggled against and eschewed the easy,
unmitigated judicial tyranny and an unacceptable assault upon this Court's conscience.
legalistic, technical and sometimes harsh anachronisms of the law in order to evoke substantial justice in
the larger social context consistent with Frivaldo's unique situation approximating venerability in
EPILOGUE Philippine political life. Concededly, he sought American citizenship only to escape the clutches of the
dictatorship. At this stage, we cannot seriously entertain any doubt about his loyalty and dedication to
In sum, we rule that the citizenship requirement in the Local Government Code is to be possessed by an this country. At the first opportunity, he returned to this land, and sought to serve his people once more.
elective official at the latest as of the time he is proclaimed and at the start of the term of office to which The people of Sorsogon overwhelmingly voted for him three times. He took an oath of allegiance to this
he has been elected. We further hold P.D. No. 725 to be in full force and effect up to the present, not Republic every time he filed his certificate of candidacy and during his failed naturalization bid. And let it
having been suspended or repealed expressly nor impliedly at any time, and Frivaldo's repatriation by not be overlooked, his demonstrated tenacity and sheer determination to re-assume his nationality of
virtue thereof to have been properly granted and thus valid and effective. Moreover, by reason of the birth despite several legal set-backs speak more loudly, in spirit, in fact and in truth than any legal
remedial or curative nature of the law granting him a new right to resume his political status and the technicality, of his consuming intention and burning desire to re-embrace his native Philippines even
legislative intent behind it, as well as his unique situation of having been forced to give up his citizenship now at the ripe old age of 81 years. Such loyalty to and love of country as well as nobility of purpose
and political aspiration as his means of escaping a regime he abhorred, his repatriation is to be given cannot be lost on this Court of justice and equity. Mortals of lesser mettle would have given up. After all,
retroactive effect as of the date of his application therefor, during the pendency of which he was Frivaldo was assured of a life of ease and plenty as a citizen of the most powerful country in the world.
stateless, he having given up his U.S. nationality. Thus, in contemplation of law, he possessed the vital But he opted, nay, single-mindedly insisted on returning to and serving once more his struggling but
requirement of Filipino citizenship as of the start of the term of office of governor, and should have been beloved land of birth. He therefore deserves every liberal interpretation of the law which can be applied
proclaimed instead of Lee. Furthermore, since his reacquisition of citizenship retroacted to August 17, in his favor. And in the final analysis, over and above Frivaldo himself, the indomitable people of
1994, his registration as a voter of Sorsogon is deemed to have been validated as of said date as well. Sorsogon most certainly deserve to be governed by a leader of their overwhelming choice.
The foregoing, of course, are precisely consistent with our holding that lack of the citizenship
requirement is not a continuing disability or disqualification to run for and hold public office. And once WHEREFORE, in consideration of the foregoing:
again, we emphasize herein our previous rulings recognizing the Comelec's authority and jurisdiction to
hear and decide petitions for annulment of proclamations.
(1) The petition in G.R. No. 123755 is hereby DISMISSED. The assailed Resolutions of the respondent
Commission are AFFIRMED.
This Court has time and again liberally and equitably construed the electoral laws of our country to give
fullest effect to the manifest will of our people, 66 for in case of doubt, political laws must be interpreted
(2) The petition in G.R. No. 120295 is also DISMISSED for being moot and academic. In any event, it has
to give life and spirit to the popular mandate freely expressed through the ballot. Otherwise stated, legal
no merit.
niceties and technicalities cannot stand in the way of the sovereign will. Consistently, we have held:

No costs.
EN BANC Please be advised that the undersigned attended the public hearing called by the Senate Committee on
electoral Reforms, Suffrage and Peoples Participation presided over by the Hon. Sen. Raul Roco, its
Committee Chairman to date at the Senate, New GSIS Headquarters Building, Pasay City. The main
agenda item is the request by youth organizations to hold additional two days of registration. Thus,
participating students and civic leaders along with Comelec Representatives were in agreement that is
[G.R. No. 147066. March 26, 2001] legally feasible to have a two-day additional registration of voters to be conducted preferably on
February 17 and 18, 2001 nationwide. The deadline for the continuing voters registration under R.A.
8189 is December 27, 2000.

AKBAYAN Youth, SCAP, UCSC, MASP, KOMPIL II Youth, ALYANSA, KALIPI, PATRICIA O. PICAR, MYLA To address the concern that this may open the flood parts for hakot system, certain restrictive
GAIL Z. TAMONDONG, EMMANUEL E. OMBAO, JOHNNY ACOSTA, ARCHIE JOHN TALAUE, parameters were discussed. The following guidelines to serve as safeguards against fraudulent
RYAN DAPITAN, CHRISTOPHER OARDE, JOSE MARI MODESTO, RICHARD M. VALENCIA, applicants:
EDBEN TABUCOL, petitioners, vs. COMMISSION ON ELECTIONS, respondents.
1. The applicants for the registration shall be 25 years of age or less and will be registering
for the first time on May 14, 2001;

2. The applicants shall register in their places of residences; and


[G.R. No. 147179. March 26, 2001]
3. The applicants shall present valid identification documents, like school records.

Preparatory to the registration days, the following activities are likewise agreed:
MICHELLE D. BETITO, petitioner, vs. CHAIRMAN ALFREDO BENIPAYO, COMMISSIONERS MEHOL
SADAIN, RUFINO JAVIER, LUZVIMINDA TANCANGCO, RALPH LANTION, FLORENTINO
1. Submission of the list of students and their addresses immediately prior to the actual
TUASON and RESURRECCION BORRA, all of the Commission on Election
registration of the applicants;
(COMELEC), respondents.
2. The Comelec field officers will be given the opportunity to verify the voters enumerators
DECISION list or conduct ocular inspection;

BUENA, J.: 3. Availability of funds for the purpose; and

4. Meetings with student groups to ensure orderly and honest conduct of the registration
At the helm of controversy in the instant consolidated petitions[1] before us is the exercise of a and drum up interest to register among the new voters.
right so indubitably cherished and accorded primacy, if not utmost reverence, no less than by the
fundamental law - the right of suffrage.
The rationale for the additional two-day registration is the renewed political awareness and interest to
Invoking this right, herein petitioners - representing the youth sector - seek to direct the participate in the political process generated by the recent political events in the country among our
Commission on Elections (COMELEC) to conduct a special registration before the May 14, 2001 General youth.Considering that they failed to register on December 27, 2000 deadline, they approved for special
Elections, of new voters ages 18 to 21. According to petitioners, around four million youth failed to registration days.
register on or before the December 27, 2000 deadline set by the respondent COMELEC under Republic
Act No. 8189.[2] In view of the foregoing, the Commission en banc has to discuss all aspects regarding this request with
directives to the Finance Services Department (FSD) to submit certified available funds for the
Acting on the clamor of the students and civic leaders, Senator Raul Roco, Chairman of the purpose, and for the Deputy Executive Director for Operations (DEDO) for the estimated costs of
Committee on Electoral Reforms, Suffrage, and Peoples Participation, through a Letter dated January 25, additional two days of registration.
2001, invited the COMELEC to a public hearing for the purpose of discussing the extension of the
registration of voters to accommodate those who were not able to register before the COMELEC
deadline.[3] The presence of REDs on January 30 can be used partly for consultation on the practical side and
logistical requirements of such additional registration days. The meeting will be set at 1:30 p.m. at the
Commissioners Luzviminda G. Tancangco and Ralph C. Lantion, together with Consultant Office of ED.[4]
Resurreccion Z. Borra (now Commissioner) attended the public hearing called by the Senate Committee
headed by Senator Roco, held at the Senate, New GSIS Headquarters Bldg., Pasay City.
Immediately, Commissioner Borra called a consultation meeting among regional heads and
On January 29, 2001, Commissioners Tancangco and Lantion submitted Memorandum No. 2001- representatives and a number of senior staff headed by Executive Director Mamasapunod Aguam. It was
027 on the Report on the Request for a Two-day Additional Registration of New Voters Only, excerpts of the consensus of the group, with the exception of Director Jose Tolentino, Jr. of the ASD, to disapprove
which are hereto quoted: the request for additional registration of voters on the ground that Section 8 of R.A. 8189 explicitly
provides that no registration shall be conducted during the period starting one hundred twenty (120)
days before a regular election and that the Commission has no more time left to accomplish all pre- the Legislature, which statutes for all intents and purposes, are crafted to effectively insulate such so
election activities.[5] cherished right from ravishment and preserve the democratic institutions our people have, for so long,
guarded against the spoils of opportunism, debauchery and abuse.
On February 8, 2001, the COMELEC issued Resolution No. 3584, the decretal portion of which
reads: To be sure, the right of suffrage ardently invoked by herein petitioners, is not at all
absolute. Needless to say, the exercise of the right of suffrage, as in the enjoyment of all other rights, is
subject to existing substantive and procedural requirements embodied in our Constitution, statute books
Deliberating on the foregoing memoranda, the Commission RESOLVED, as it hereby RESOLVES, to deny
and other repositories of law. Thus, as to the substantive aspect, Section 1, Article V of the Constitution
the request to conduct a two-day additional registration of new voters on February 17 and 18, 2001.
provides:

Commissioners Rufino S. B. Javier and Mehol K. Sadain voted to deny the request while
SECTION 1. SUFFRAGE MAY BE EXERCISED BY ALL CITIZENS OF THE PHILIPPINES NOT OTHERWISE
Commissioners Luzviminda Tancangco and Ralph Lantion voted to accommodate the students
DISQUALIFIED BY LAW, WHO ARE AT LEAST EIGHTEEN YEARS OF AGE, AND WHO SHALL HAVE RESIDED IN
request. With this impasse, the Commission construed its Resolution as having taken effect.
THE PHILIPPINES FOR AT LEAST ONE YEAR AND IN THE PLACE WHEREIN THEY PROPOSE TO VOTE FOR AT
Aggrieved by the denial, petitioners AKBAYAN-Youth, SCAP, UCSC, MASP, KOMPIL II (YOUTH) et al. LEAST SIX MONTHS IMMEDIATELY PRECEDING THE ELECTIONS. NO LITERACY, PROPERTY, OR OTHER
filed before this Court the instant Petition for Certiorari and Mandamus, docketed as G.R. No. 147066, SUBSTANTIVE REQUIREMENT SHALL BE IMPOSED ON THE EXERCISE OF SUFFRAGE.
which seeks to set aside and nullify respondent COMELECs Resolution and/or to declare Section 8 of R.
A. 8189 unconstitutional insofar as said provision effectively causes the disenfranchisement of As to the procedural limitation, the right of a citizen to vote is necessarily conditioned upon
petitioners and others similarly situated. Likewise, petitioners pray for the issuance of a writ of certain procedural requirements he must undergo: among others, the process of
mandamus directing respondent COMELEC to conduct a special registration of new voters and to admit registration. Specifically, a citizen in order to be qualified to exercise his right to vote, in addition to the
for registration petitioners and other similarly situated young Filipinos to qualify them to vote in the May minimum requirements set by the fundamental charter, is obliged by law to register, at present, under
14, 2001 General Elections. the provisions of Republic Act No. 8189, otherwise known as the Voters Registration Act of 1996.
On March 09, 2001, herein petitioner Michelle Betito, a student of the University of the Stated differently, the act of registration is an indispensable precondition to the right of
Philippines, likewise filed a Petition for Mandamus, docketed as G.R. No. 147179, praying that this Court suffrage. For registration is part and parcel of the right to vote and an indispensable element in the
direct the COMELEC to provide for another special registration day under the continuing registration election process.Thus, contrary to petitioners argument, registration cannot and should not be
provision under the Election Code. denigrated to the lowly stature of a mere statutory requirement. Proceeding from the significance of
registration as a necessary requisite to the right to vote, the State undoubtedly, in the exercise of its
On March 13, 2001, this Court resolved to consolidate the two petitions and further required
inherent police power, may then enact laws to safeguard and regulate the act of voters registration for
respondents to file their Comment thereon within a non-extendible period expiring at 10:00 A.M. of
the ultimate purpose of conducting honest, orderly and peaceful election, to the incidental yet generally
March 16, 2001. Moreover, this Court resolved to set the consolidated cases for oral arguments on
important end, that even pre-election activities could be performed by the duly constituted authorities
March 16, 2001.[6]
in a realistic and orderly manner one which is not indifferent and so far removed from the pressing order
On March 16, 2001, the Solicitor General, in its Manifestation and Motion in lieu of Comment, of the day and the prevalent circumstances of the times.
recommended that an additional continuing registration of voters be conducted at the soonest possible
Viewed broadly, existing legal proscription and pragmatic operational considerations bear great
time in order to accommodate that disenfranchised voters for purposes of the May 14, 2001 elections.
weight in the adjudication of the issues raised in the instant petitions.
In effect, the Court in passing upon the merits of the present petitions, is tasked to resolve a two-
On the legal score, Section 8, of the R.A. 8189, which provides a system of continuing registration,
pronged issue focusing on respondent COMELECs issuance of the assailed Resolution dated February 8,
is explicit, to wit:
2001, which Resolution, petitioners, by and large, argue to have undermined their constitutional right to
vote on the May 14, 2001 general elections and caused the disenfranchisement of around four (4)
million Filipinos of voting age who failed to register before the registration deadline set by the COMELEC. SEC. 8. System of Continuing Registration of Voters. The Personal filing of application of registration of
voters shall be conducted daily in the office of the Election Officer during regular office hours. No
Thus, this Court shall determine: registration shall, however, be conducted during the period starting one hundred twenty (120) days
before a regular election and ninety (90) days before a special election. (Emphasis Ours)
a) Whether or not respondent COMELEC committed grave abuse of discretion in issuing
COMELEC Resolution dated February 8, 2001;
Likewise, Section 35 of R.A. 8189, which among others, speaks of a prohibitive period within which
b) Whether or not this Court can compel respondent COMELEC, through the extraordinary to file a sworn petition for the exclusion of voters from the permanent voters list, provides:
writ of mandamus, to conduct a special registration of new voters during the period
between the COMELECs imposed December 27, 2000 deadline and the May 14, 2001
general elections. SEC. 35. Petition for Exclusion of Voters from the List Any registered voter, representative of a political
party x x x may file x x x except one hundred (100) days prior to a regular election xxx.
The petitions are bereft of merit.
As aptly observed and succinctly worded by respondent COMELEC in its Comment:
In a representative democracy such as ours, the right of suffrage, although accorded a prime niche
in the hierarchy of rights embodied in the fundamental law, ought to be exercised within the proper
bounds and framework of the Constitution and must properly yield to pertinent laws skillfully enacted by
x x x The petition for exclusion is a necessary component to registration since it is a safety mechanism Similarly, every new statute should be construed in connection with those already existing in
that gives a measure of protection against flying voters, non-qualified registrants, and the like. The relation to the same subject matter and all should be made to harmonize and stand together, if they can
prohibitive period, on the other hand serves the purpose of securing the voters substantive right to be be done by any fair and reasonable interpretation.[10] Interpretare et concordare legibus est optimus
included in the list of voters. interpretandi, which means that the best method of interpretation is that which makes laws consistent
with other laws. Accordingly, courts of justice, when confronted with apparently conflicting statutes,
should endeavor to reconcile them instead of declaring outright the invalidity of one against the other.
In real-world terms, this means that if a special voters registration is conducted, then the prohibitive
Courts should harmonize them, if this is possible, because they are equally the handiwork of the same
period for filing petitions for exclusion must likewise be adjusted to a later date. If we do not, then no
legislature.[11]
one can challenge the Voters list since we would already be well into the 100-day prohibitive
period. Aside from being a flagrant breach of the principles of due process, this would open the In light of the foregoing doctrine, we hold that Section 8 of R.A. 8189 applies in the present case,
registration process to abuse and seriously compromise the integrity of the voters list, and consequently, for the purpose of upholding the assailed COMELEC Resolution and denying the instant petitions,
that of the entire election. considering that the aforesaid law explicitly provides that no registration shall be conducted during the
period starting one hundred twenty (120) days before a regular election.
x x x It must be remembered that the period serve a vital role in protecting the integrity of the
registration process. Without the prohibitive periods, the COMELEC would be deprived of any time to Corollarily, it is specious for herein petitioners to argue that respondent COMELEC may validly and
evaluate the evidence on the application. We would be obliged to simply take them at face value. If we legally conduct a two-day special registration, through the expedient of the letter of Section 28 of R.A.
compromise on these safety nets, we may very well end up with a voters list full of flying voters, 8436. To this end, the provisions of Section 28, R.A. 8436 would come into play in cases where the pre-
overflowing with unqualified registrants, populated with shadows and ghosts x x x. election acts are susceptible of performance within the available period prior to election day. In more
categorical language, Section 28 of R.A. 8436 is, to our mind, anchored on the sound premise that these
certain pre-election acts are still capable of being reasonably performed vis-a-vis the remaining period
x x x The short cuts that will have to be adopted in order to fit the entire process of registration within before the date of election and the conduct of other related pre-election activities required under the
the last 60 days will give rise to haphazard list of voters, some of whom might not even be qualified to law.
vote. x x x the very possibility that we shall be conducting elections on the basis of an inaccurate list is
enough to cast a cloud of doubt over the results of the polls. If that happens, the unforgiving public will In its Comment, respondent COMELECwhich is the constitutional body tasked by no less than the
disown the results of the elections, regardless of who wins, and regardless of how many courts validate fundamental charter (Sec. 2, par. 3, Article IX-C of the Constitution) to decide, except those involving the
our own results. x x x right to vote, all questions affecting elections, including registration of voterspainstakingly and
thoroughly emphasized the operational impossibility[12] of conducting a special registration, which in its
on language, can no longer be accomplished within the time left to (us) the Commission.[13]
Perhaps undaunted by such scenario, petitioners invoke the so called standby powers or residual
powers of the COMELEC, as provided under the relevant provisions of Section 29, Republic Act No. Hence:
6646[7] and adopted verbatim in Section 28 of Republic Act No. 8436,[8] thus:

xxx xxx xxx.


SEC. 28. Designation of other Dates for Certain Pre-election Acts - If it should no longer be possible to
observe the periods and dates prescribed by law for certain pre-election acts, the Commission shall fix
other periods and dates in order to ensure accomplishments of the activities so voters shall not be 19) In any case, even without the legal obstacles, the last 60 days will not be a walk in the
deprived of their right to suffrage. park for the Comelec. Allow us to outline what the Commission has yet to do, and the
time to do it in:

On this matter, the act of registration is concededly, by its very nature, a pre-election act. Under 20) First we have to complete the Project of Precincts by the 19th of March. The Projects of
Section 3(a) of R.A. 8189, registration, as a process, has its own specific definition, precise meaning and Precincts Indicate the total number of established precincts and the number of
coverage, thus: registered voters per precincts in a city or municipality. Without the final Project of
Precincts, we cannot even determine the proper allocation of official ballots, election
returns and other election forms and paraphernalia. More succinctly said, without the
a) Registration refers to the act of accomplishing and filing of a sworn application for registration by a
Project of Precincts, we wont know how many forms to print and so were liable to
qualified voter before the election officer of the city or municipality wherein he resides and including the
come up short.
same in the book of registered voters upon approval by the Election Registration Board;
21) More Importantly, without a completed Project of Precincts, it will be impossible to
At this point, it bears emphasis that the provisions of Section 29 of R.A. 8436 invoked by herein complete the rest of the tasks that must be accomplished prior to the elections.
petitioners and Section 8 of R.A. 8189 volunteered by respondent COMELEC, far from contradicting each
22) Second, the Board of Elections Inspectors must be constituted on or before the 4th of
other, actually share some common ground. True enough, both provisions, although at first glance may
March. In addition, the list of the members of the BEI including the precinct where they
seem to be at war in relation to the other, are in a more circumspect perusal, necessarily capable of
are assigned and the barangay where that precinct is located - must be furnished by
being harmonized and reconciled.
the Election Officer to all the candidates and political candidates not later than the
Rudimentary is the principle in legal hermeneutics that changes made by the legislature in the 26th of March.
form of amendments to a statute should be given effect, together with other parts of the amended act.
It is not to be presumed that the legislature, in making such changes, was indulging in mere semantic
exercise. There must be some purpose in making them, which should be ascertained and given effect.[9]
23) Third, the Book of Voters, which contains the approved Voter Registration Records of 34) Assuming optimistically that we can then finish the inspection, verification, and sealing
registered voters in particular precinct, must be inspected, verified, and sealed of the Book if Voters by May 15, we will already have overshot the May 14, election
beginning March 30, until April 15. date, and still not have finished our election preparations.

24) Fourth, the Computerized Voters List must be finalized and printed out of use on 35) After this point, we could have to prepare the allocation of Official Ballots, Election
election day; and finally Returns, and other Non-Accountable Forms and Supplies to be used for the new
registrants. Once the allocation is ready, the contracts would be awarded, the various
25) Fifth, the preparation, bidding, printing, and distribution of the Voters Information forms printed, delivered, verified, and finally shipped out to the different
Sheet must be completed on or before April 15. municipalities. All told, this process would take approximately 26 days, from the
15th of May until June 10.
26) With this rigorous schedule of pre-election activities, the Comelec will have roughly a
month that will act as a buffer against any number of unforeseen occurrences that 36) Only then can we truly say that we are ready to hold the elections.
might delay the elections. This is the logic and the wisdom behind setting the 120-day
prohibitive period. After all, preparing for an election is no easy task.
xxx xxx xxx.[14]
27) To hold special registrations now would, aside from being Illegal, whittle that
approximately 30-day margin away to nothing. It is an accepted doctrine in administrative law that the determination of administrative agency as
28) When we say registration of voters, we do not - contrary to popular opinion - refer only to the operation, implementation and application of a law would be accorded great weight considering
to the act of going to the Election Officer and writing our names down. Registration is, that these specialized government bodies are, by their nature and functions, in the best position to know
In fact, a long process that takes about three weeks to complete not even counting what they can possibly do or not do, under prevailing circumstances.
how long it would take to prepare for the registration in the first place. Beyond this, it is likewise well-settled that the law does not require that the impossible be
29) In order to concretize, the senior Staff of the Comelec, the other Commissioners, done.[15] The law obliges no one to perform an impossibility, expressed in the maxim, nemo tenetur ad
prepared a time-table in order to see exactly how the superimposition of special impossible.[16] In other words, there is no obligation to do an impossible thing. Impossibilium nulla
registration would affect the on-going preparation for the May 14 elections. obligato est. Hence, a statute may not be so construed as to require compliance with what it prescribes
cannot, at the time, be legally, coincidentally[17], it must be presumed that the legislature did not at all
30) We assumed for the sake of argument that we were to hold the special registration on intend an interpretation or application of a law which is far removed from the realm of the
April 16 and 17. These are not arbitrary numbers, by the way it takes in account the possible. Truly, in the interpretation of statutes, the interpretation to be given must be such that it is in
fact that we only have about 800,000 Voters Registration Forms available, as against an accordance with logic, common sense, reasonableness and practicality. Thus, we are of the considered
estimated 4.5 million potential registrants, and it would take about 14 days If we were view that the stand-by power of the respondent COMELEC under Section 28 of R.A. 8436, presupposes
to declare special registrations today to print up the difference and to verify these the possibility of its being exercised or availed of, and not otherwise.
accountable forms. After printing and verification, the forms would have to be packed
and shipped - roughly taking up a further two and a half weeks. Only then can we get Further, petitioners bare allegation that they were disenfranchised when respondent COMELEC
on with registration. pegged the registration deadline on December 27, 2000 instead of January 13, 2001 the day before the
period before the May 14, 2001 regular elections commences is, to our mind, not sufficient. On this
31) The first step in registration is, of course, filling the application for registration with the matter, there is no allegation in the two consolidated petitions and the records are bereft of any showing
Election Officer. The application, according to Section 17 of R.A. 8189, is then set for that anyone of herein petitioners has filed an application to be registered as a voter which was denied by
hearing, with notice of that hearing being posted in the city or municipal bulletin board the COMELEC nor filed a complaint before the respondent COMELEC alleging that he or she proceeded
for at least one week prior. Thus, if we held registrations on the 16th and the 17th the to the Office of the Election Officer to register between the period starting from December 28, 2000 to
posting requirement would be completed by the 24th. Considering that time must be January 13, 2001, and that he or she was disallowed or barred by respondent COMELEC from filing his
allowed for the filling of oppositions, the earliest that the Election Registration Board application for registration. While it may be true that respondent COMELEC set the registration deadline
can be convened for hearing would be the May 1st and 2nd. on December 27, 2000, this Court is of the Firm view that petitioners were not totally denied the
opportunity to avail of the continuing registration under R.A. 8189. Stated in a different manner, the
32) Assuming and this is a big assumption that there are nit challenges to the applicants petitioners in the instant case are not without fault or blame. They admit in their petition[18] that they
right to register, the Election registration Board can immediately rule on the Applicants failed to register, for whatever reason, within the period of registration and came to this Court and
registration, and post notices of its action by the 2nd until the 7th of May. By the 10th, invoked its protective mantle not realizing, so to speak, the speck in their eyes. Impuris minibus nemo
copies of the notice of the action taken by the Board will have already been furnished accedat curiam. Let no one come to court with unclean hands.
to the applicants and the heads of registered political parties.
In a similar vein, well-entrenched is the rule in our jurisdiction that the law aids the vigilant and
33) Only at this point can our Election Officers once again focus on the business of getting not those who slumber on their rights. Vigilantis sed non dormientibus jura in re subveniunt.
ready for the elections. Once the results of the special registration are finalized, they
can be encoded and a new Computerized Voters List generated - at the earliest, by Applying the foregoing, this court is of the firm view that respondent COMELEC did not commit an
May 11, after which the new CVL would be posted. Incidentally, it we were to follow abuse of discretion, much less be adjudged to have committed the same in some patent, whimsical and
the letter of the law strictly, a May 11 posting date for the new CVL would be improper arbitrary manner, in issuing Resolution No. 3584 which, in respondents own terms, resolved to deny the
since the R.A. 8189 provides that the CVL be posted at least 90 days before the request to conduct a two-day additional registration of new voters on February 17 and 18, 2001.
election.
On this particular matter, grave abuse of discretion implies a capricious and whimsical exercise of
judgment as is equivalent to lack of jurisdiction, or, when the power is exercised in an arbitrary or
despotic manner by reason of passion or personal hostility, and it must be so patent and gross as to
amount to an evasion of positive duty enjoined or to act at all in contemplation of laws.[19]

Under these circumstances, we rule that the COMELEC, in denying the request of petitioners to
hold a special registration, acted within the bounds and confines of the applicable law on the matter --
Section 8 of RA 8189. In issuing the assailed Resolution, respondent COMELEC simply performed its
constitutional task to enforce and administer all laws and regulations relative to the conduct of an
election,[20] inter alia, questions relating to the registration of voters; evidently, respondent COMELEC
merely exercised a prerogative that chiefly pertains to it and one which squarely falls within the proper
sphere of its constitutionally-mandated powers. Hence, whatever action respondent takes in the
exercise of its wide latitude of discretion, specifically on matters involving voters registration, pertains to
the wisdom rather than the legality of the act. Accordingly, in the absence of clear showing of grave
abuse of power of discretion on the part of respondent COMELEC, this Court may not validly conduct an
incursion and meddle with affairs exclusively within the province of respondent COMELEC a body
accorded by no less than the fundamental law with independence.

As to petitioners prayer for the issuance of the writ of mandamus, we hold that this Court cannot,
in view of the very nature of such extraordinary writ, issue the same without transgressing the time-
honored principles in this jurisdiction.

As an extraordinary writ, the remedy of mandamus lies only to compel an officer to perform a
ministerial duty, not a discretionary one; mandamus will not issue to control the exercise of discretion of
a public officer where the law imposes upon him the duty to exercise his judgment in reference to any
manner in which he is required to act, because it is his judgment that is to be exercised and not that of
the court.[21]

Considering the circumstances where the writ of mandamus lies and the peculiarities of the
present case, we are of the firm belief that petitioners failed to establish, to the satisfaction of this Court,
that they are entitled to the issuance of this extraordinary writ so as to effectively compel respondent
COMELEC to conduct a special registration of voters. For the determination of whether or not the
conduct of a special registration of voters is feasible, possible or practical within the remaining period
before the actual date of election, involves the exercise of discretion and thus, cannot be controlled by
mandamus.

In Bayan vs. Executive Secretary Zamora and related cases,[22] we enunciated that the Courts
function, as sanctioned by Article VIII, Section 1, is merely (to) check whether or not the governmental
branch or agency has gone beyond the constitutional limits of its jurisdiction, not that it erred or has a
different view. In the absence of a showing...(of) grave abuse of discretion amounting to lack of
jurisdiction, there is no occasion for the Court to exercise its corrective power... It has no power to look
into what it thinks is apparent error.[23]

Finally, the Court likewise takes judicial notice of the fact that the President has issued
Proclamation No. 15 calling Congress to a Special Session on March 19, 2001, to allow the conduct of
Special Registration of new voters. House Bill No. 12930 has been filed before the Lower House, which
bill seeks to amend R.A. 8189 as to the 120-day prohibitive period provided for under said law. Similarly,
Senate Bill No. 2276[24] was filed before the Senate, with the same intention to amend the aforesaid law
and, in effect, allow the conduct of special registration before the May 14, 2001 General Elections.This
Court views the foregoing factual circumstances as a clear intimation on the part of both the executive
and legislative departments that a legal obstacle indeed stands in the way of the conduct by the
Commission on Elections of a special registration before the May 14, 2001 General Elections.

WHEREFORE, premises considered, the instant petitions for certiorari and mandamus are hereby
DENIED.

SO ORDERED.
Republic of the Philippines against the establishment or opening by aliens actually engaged in the retail business of additional stores
SUPREME COURT or branches of retail business, (6) a provision requiring aliens actually engaged in the retail business to
Manila present for registration with the proper authorities a verified statement concerning their businesses,
giving, among other matters, the nature of the business, their assets and liabilities and their offices and
principal offices of judicial entities; and (7) a provision allowing the heirs of aliens now engaged in the
EN BANC
retail business who die, to continue such business for a period of six months for purposes of liquidation.

G.R. No. L-7995 May 31, 1957


III. Grounds upon which petition is based-Answer thereto

LAO H. ICHONG, in his own behalf and in behalf of other alien residents, corporations and partnerships
Petitioner, for and in his own behalf and on behalf of other alien residents corporations and partnerships
adversely affected. by Republic Act No. 1180, petitioner,
adversely affected by the provisions of Republic Act. No. 1180, brought this action to obtain a judicial
vs.
declaration that said Act is unconstitutional, and to enjoin the Secretary of Finance and all other persons
JAIME HERNANDEZ, Secretary of Finance, and MARCELINO SARMIENTO, City Treasurer of
acting under him, particularly city and municipal treasurers, from enforcing its provisions. Petitioner
Manila,respondents.
attacks the constitutionality of the Act, contending that: (1) it denies to alien residents the equal
protection of the laws and deprives of their liberty and property without due process of law ; (2) the
Ozaeta, Lichauco and Picazo and Sycip, Quisumbing, Salazar and Associates for petitioner. subject of the Act is not expressed or comprehended in the title thereof; (3) the Act violates
Office of the Solicitor General Ambrosio Padilla and Solicitor Pacifico P. de Castro for respondent international and treaty obligations of the Republic of the Philippines; (4) the provisions of the Act
Secretary of Finance. against the transmission by aliens of their retail business thru hereditary succession, and those requiring
City Fiscal Eugenio Angeles and Assistant City Fiscal Eulogio S. Serrano for respondent City Treasurer. 100% Filipino capitalization for a corporation or entity to entitle it to engage in the retail business, violate
Dionisio Reyes as Amicus Curiae. the spirit of Sections 1 and 5, Article XIII and Section 8 of Article XIV of the Constitution.
Marcial G. Mendiola as Amicus Curiae.
Emiliano R. Navarro as Amicus Curiae.
In answer, the Solicitor-General and the Fiscal of the City of Manila contend that: (1) the Act was passed
in the valid exercise of the police power of the State, which exercise is authorized in the Constitution in
LABRADOR, J.: the interest of national economic survival; (2) the Act has only one subject embraced in the title; (3) no
treaty or international obligations are infringed; (4) as regards hereditary succession, only the form is
I. The case and issue, in general affected but the value of the property is not impaired, and the institution of inheritance is only of
statutory origin.

This Court has before it the delicate task of passing upon the validity and constitutionality of a legislative
enactment, fundamental and far-reaching in significance. The enactment poses questions of due process, IV. Preliminary consideration of legal principles involved
police power and equal protection of the laws. It also poses an important issue of fact, that is whether
the conditions which the disputed law purports to remedy really or actually exist. Admittedly springing a. The police power.
from a deep, militant, and positive nationalistic impulse, the law purports to protect citizen and country
from the alien retailer. Through it, and within the field of economy it regulates, Congress attempts to
There is no question that the Act was approved in the exercise of the police power, but petitioner claims
translate national aspirations for economic independence and national security, rooted in the drive and
that its exercise in this instance is attended by a violation of the constitutional requirements of due
urge for national survival and welfare, into a concrete and tangible measures designed to free the
process and equal protection of the laws. But before proceeding to the consideration and resolution of
national retailer from the competing dominance of the alien, so that the country and the nation may be
the ultimate issue involved, it would be well to bear in mind certain basic and fundamental, albeit
free from a supposed economic dependence and bondage. Do the facts and circumstances justify the
preliminary, considerations in the determination of the ever recurrent conflict between police power
enactment?
and the guarantees of due process and equal protection of the laws. What is the scope of police power,
and how are the due process and equal protection clauses related to it? What is the province and power
II. Pertinent provisions of Republic Act No. 1180 of the legislature, and what is the function and duty of the courts? These consideration must be clearly
and correctly understood that their application to the facts of the case may be brought forth with clarity
Republic Act No. 1180 is entitled "An Act to Regulate the Retail Business." In effect it nationalizes the and the issue accordingly resolved.
retail trade business. The main provisions of the Act are: (1) a prohibition against persons, not citizens of
the Philippines, and against associations, partnerships, or corporations the capital of which are not It has been said the police power is so far - reaching in scope, that it has become almost impossible to
wholly owned by citizens of the Philippines, from engaging directly or indirectly in the retail trade; (2) an limit its sweep. As it derives its existence from the very existence of the State itself, it does not need to
exception from the above prohibition in favor of aliens actually engaged in said business on May 15, be expressed or defined in its scope; it is said to be co-extensive with self-protection and survival, and as
1954, who are allowed to continue to engaged therein, unless their licenses are forfeited in accordance such it is the most positive and active of all governmental processes, the most essential, insistent and
with the law, until their death or voluntary retirement in case of natural persons, and for ten years after illimitable. Especially is it so under a modern democratic framework where the demands of society and
the approval of the Act or until the expiration of term in case of juridical persons; (3) an exception of nations have multiplied to almost unimaginable proportions; the field and scope of police power has
therefrom in favor of citizens and juridical entities of the United States; (4) a provision for the forfeiture become almost boundless, just as the fields of public interest and public welfare have become almost all-
of licenses (to engage in the retail business) for violation of the laws on nationalization, control weights embracing and have transcended human foresight. Otherwise stated, as we cannot foresee the needs
and measures and labor and other laws relating to trade, commerce and industry; (5) a prohibition and demands of public interest and welfare in this constantly changing and progressive world, so we
cannot delimit beforehand the extent or scope of police power by which and through which the State e. Legislative discretion not subject to judicial review.
seeks to attain or achieve interest or welfare. So it is that Constitutions do not define the scope or extent
of the police power of the State; what they do is to set forth the limitations thereof. The most important
Now, in this matter of equitable balancing, what is the proper place and role of the courts? It must not
of these are the due process clause and the equal protection clause.
be overlooked, in the first place, that the legislature, which is the constitutional repository of police
power and exercises the prerogative of determining the policy of the State, is by force of circumstances
b. Limitations on police power. primarily the judge of necessity, adequacy or reasonableness and wisdom, of any law promulgated in the
exercise of the police power, or of the measures adopted to implement the public policy or to achieve
public interest. On the other hand, courts, although zealous guardians of individual liberty and right,
The basic limitations of due process and equal protection are found in the following provisions of our
have nevertheless evinced a reluctance to interfere with the exercise of the legislative prerogative. They
Constitution:
have done so early where there has been a clear, patent or palpable arbitrary and unreasonable abuse of
the legislative prerogative. Moreover, courts are not supposed to override legitimate policy, and courts
SECTION 1.(1) No person shall be deprived of life, liberty or property without due process of never inquire into the wisdom of the law.
law, nor any person be denied the equal protection of the laws. (Article III, Phil. Constitution)
V. Economic problems sought to be remedied
These constitutional guarantees which embody the essence of individual liberty and freedom in
democracies, are not limited to citizens alone but are admittedly universal in their application, without
With the above considerations in mind, we will now proceed to delve directly into the issue involved. If
regard to any differences of race, of color, or of nationality. (Yick Wo vs. Hopkins, 30, L. ed. 220, 226.)
the disputed legislation were merely a regulation, as its title indicates, there would be no question that it
falls within the legitimate scope of legislative power. But it goes further and prohibits a group of
c. The, equal protection clause. residents, the aliens, from engaging therein. The problem becomes more complex because its subject is
a common, trade or occupation, as old as society itself, which from the immemorial has always been
The equal protection of the law clause is against undue favor and individual or class privilege, as well as open to residents, irrespective of race, color or citizenship.
hostile discrimination or the oppression of inequality. It is not intended to prohibit legislation, which is
limited either in the object to which it is directed or by territory within which is to operate. It does not a. Importance of retail trade in the economy of the nation.
demand absolute equality among residents; it merely requires that all persons shall be treated
alike, under like circumstances and conditions both as to privileges conferred and liabilities enforced. The
In a primitive economy where families produce all that they consume and consume all that they
equal protection clause is not infringed by legislation which applies only to those persons falling within a
produce, the dealer, of course, is unknown. But as group life develops and families begin to live in
specified class, if it applies alike to all persons within such class, and reasonable grounds exists for
communities producing more than what they consume and needing an infinite number of things they do
making a distinction between those who fall within such class and those who do not. (2 Cooley,
not produce, the dealer comes into existence. As villages develop into big communities and
Constitutional Limitations, 824-825.)
specialization in production begins, the dealer's importance is enhanced. Under modern conditions and
standards of living, in which man's needs have multiplied and diversified to unlimited extents and
d. The due process clause. proportions, the retailer comes as essential as the producer, because thru him the infinite variety of
articles, goods and needed for daily life are placed within the easy reach of consumers. Retail dealers
The due process clause has to do with the reasonableness of legislation enacted in pursuance of the perform the functions of capillaries in the human body, thru which all the needed food and supplies are
police power. Is there public interest, a public purpose; is public welfare involved? Is the Act reasonably ministered to members of the communities comprising the nation.
necessary for the accomplishment of the legislature's purpose; is it not unreasonable, arbitrary or
oppressive? Is there sufficient foundation or reason in connection with the matter involved; or has there There cannot be any question about the importance of the retailer in the life of the community. He
not been a capricious use of the legislative power? Can the aims conceived be achieved by the means ministers to the resident's daily needs, food in all its increasing forms, and the various little gadgets and
used, or is it not merely an unjustified interference with private interest? These are the questions that things needed for home and daily life. He provides his customers around his store with the rice or corn,
we ask when the due process test is applied. the fish, the salt, the vinegar, the spices needed for the daily cooking. He has cloths to sell, even the
needle and the thread to sew them or darn the clothes that wear out. The retailer, therefore, from the
The conflict, therefore, between police power and the guarantees of due process and equal protection of lowly peddler, the owner of a small sari-sari store, to the operator of a department store or, a
the laws is more apparent than real. Properly related, the power and the guarantees are supposed to supermarket is so much a part of day-to-day existence.
coexist. The balancing is the essence or, shall it be said, the indispensable means for the attainment of
legitimate aspirations of any democratic society. There can be no absolute power, whoever exercise it, b. The alien retailer's trait.
for that would be tyranny. Yet there can neither be absolute liberty, for that would mean license and
anarchy. So the State can deprive persons of life, liberty and property, provided there is due process of
The alien retailer must have started plying his trades in this country in the bigger centers of population
law; and persons may be classified into classes and groups, provided everyone is given the equal
(Time there was when he was unknown in provincial towns and villages). Slowly but gradually be invaded
protection of the law. The test or standard, as always, is reason. The police power legislation must be
towns and villages; now he predominates in the cities and big centers of population. He even pioneers, in
firmly grounded on public interest and welfare, and a reasonable relation must exist between purposes
far away nooks where the beginnings of community life appear, ministering to the daily needs of the
and means. And if distinction and classification has been made, there must be a reasonable basis for said
residents and purchasing their agricultural produce for sale in the towns. It is an undeniable fact that in
distinction.
many communities the alien has replaced the native retailer. He has shown in this trade, industry Chinese .......... 12,087 93,155,459 29.38 294,894,227
without limit, and the patience and forbearance of a slave.
Others .......... 422 10,514,675 3.32 9,995,402

Derogatory epithets are hurled at him, but he laughs these off without murmur; insults of ill-bred and 1949:
insolent neighbors and customers are made in his face, but he heeds them not, and he forgets and Filipino .......... 113,659 213,451,602 60.89 462,532,901
forgives. The community takes note of him, as he appears to be harmless and extremely useful.
Chinese .......... 16,248 125,223,336 35.72 392,414,875

c. Alleged alien control and dominance. Others .......... 486 12,056,365 3.39 10,078,364
1951:
There is a general feeling on the part of the public, which appears to be true to fact, about the Filipino ......... 119,352 224,053,620 61.09 466,058,052
controlling and dominant position that the alien retailer holds in the nation's economy. Food and other
essentials, clothing, almost all articles of daily life reach the residents mostly through him. In big cities Chinese .......... 17,429 134,325,303 36.60 404,481,384
and centers of population he has acquired not only predominance, but apparent control over Others .......... 347 8,614,025 2.31 7,645,327
distribution of almost all kinds of goods, such as lumber, hardware, textiles, groceries, drugs, sugar, flour,
garlic, and scores of other goods and articles. And were it not for some national corporations like the
Naric, the Namarco, the Facomas and the Acefa, his control over principal foods and products would
easily become full and complete.

AVERAGE
Petitioner denies that there is alien predominance and control in the retail trade. In one breath it is said ASSETS AND GROSS SALES PER ESTABLISHMENT
that the fear is unfounded and the threat is imagined; in another, it is charged that the law is merely the
result of radicalism and pure and unabashed nationalism. Alienage, it is said, is not an element of Item
control; also so many unmanageable factors in the retail business make control virtually impossible. The Year and Retailer's Gross Sales
Assets
first argument which brings up an issue of fact merits serious consideration. The others are matters of Nationality (Pesos)
(Pesos)
opinion within the exclusive competence of the legislature and beyond our prerogative to pass upon and
decide.
1941:

The best evidence are the statistics on the retail trade, which put down the figures in black and white. Filipino ............................................. 1,878 1,633
Between the constitutional convention year (1935), when the fear of alien domination and control of the
retail trade already filled the minds of our leaders with fears and misgivings, and the year of the Chinese .............................................. 7,707 9,691
enactment of the nationalization of the retail trade act (1954), official statistics unmistakably point out to
the ever-increasing dominance and control by the alien of the retail trade, as witness the following Others ............................................... 24,415 8,281
tables:
1947:
Assets Gross Sales
Filipino ............................................. 1,878 2,516
Year and Retailers No.- Per cent Per cent
Pesos Pesos
Nationality Establishments Distribution Distribution Chinese ........................................... 7,707 14,934
1941:
Others .............................................. 24,749 13,919
Filipino .......... 106,671 200,323,138 55.82 174,181,924 51.74
Chinese ........... 15,356 118,348,692 32.98 148,813,239 44.21 1948: (Census)
Others ............ 1,646 40,187,090 11.20 13,630,239 4.05
Filipino ............................................. 1,878 4,111
1947:
Filipino .......... 111,107 208,658,946 65.05 279,583,333 57.03 Chinese ............................................. 7,707 24,398
Chinese ........... 13,774 106,156,218 33.56 205,701,134 41.96
Others .............................................. 24,916 23,686
Others ........... 354 8,761,260 .49 4,927,168 1.01
1948: (Census) 1949:

Filipino .......... 113,631 213,342,264 67.30 467,161,667 60.51 Filipino ............................................. 1,878 4,069
assume the initiative, if not the leadership, in the struggle for the economic freedom of the
Chinese .............................................. 7,707 24,152
nation in somewhat the same way that it did in the crusade for political freedom. Thus . . . it
(the Constitution) envisages an organized movement for the protection of the nation not only
Others .............................................. 24,807 20,737 against the possibilities of armed invasion but also against its economic subjugation by alien
interests in the economic field. (Phil. Political Law by Sinco, 10th ed., p. 476.)
1951:

Filipino ............................................. 1,877 3,905 Belief in the existence of alien control and predominance is felt in other quarters. Filipino businessmen,
manufacturers and producers believe so; they fear the dangers coming from alien control, and they
Chinese ............................................. 7,707 33,207 express sentiments of economic independence. Witness thereto is Resolution No. 1, approved on July
18, 1953, of the Fifth National convention of Filipino Businessmen, and a similar resolution, approved on
March 20, 1954, of the Second National Convention of Manufacturers and Producers. The man in the
Others ............................................... 24,824 22,033
street also believes, and fears, alien predominance and control; so our newspapers, which have
editorially pointed out not only to control but to alien stranglehold. We, therefore, find alien domination
and control to be a fact, a reality proved by official statistics, and felt by all the sections and groups that
(Estimated Assets and Gross Sales of Retail Establishments, By Year and Nationality of
compose the Filipino community.
Owners, Benchmark: 1948 Census, issued by the Bureau of Census and Statistics, Department
of Commerce and Industry; pp. 18-19 of Answer.)
e. Dangers of alien control and dominance in retail.
The above statistics do not include corporations and partnerships, while the figures on Filipino
establishments already include mere market vendors, whose capital is necessarily small.. But the dangers arising from alien participation in the retail trade does not seem to lie in the
predominance alone; there is a prevailing feeling that such predominance may truly endanger the
national interest. With ample capital, unity of purpose and action and thorough organization, alien
The above figures reveal that in percentage distribution of assests and gross sales, alien participation has
retailers and merchants can act in such complete unison and concert on such vital matters as the fixing
steadily increased during the years. It is true, of course, that Filipinos have the edge in the number of
of prices, the determination of the amount of goods or articles to be made available in the market, and
retailers, but aliens more than make up for the numerical gap through their assests and gross sales which
even the choice of the goods or articles they would or would not patronize or distribute, that fears of
average between six and seven times those of the very many Filipino retailers. Numbers in retailers,
dislocation of the national economy and of the complete subservience of national economy and of the
here, do not imply superiority; the alien invests more capital, buys and sells six to seven times more, and
consuming public are not entirely unfounded. Nationals, producers and consumers alike can be placed
gains much more. The same official report, pointing out to the known predominance of foreign elements
completely at their mercy. This is easily illustrated. Suppose an article of daily use is desired to be
in the retail trade, remarks that the Filipino retailers were largely engaged in minor retailer enterprises.
prescribed by the aliens, because the producer or importer does not offer them sufficient profits, or
As observed by respondents, the native investment is thinly spread, and the Filipino retailer is practically
because a new competing article offers bigger profits for its introduction. All that aliens would do is to
helpless in matters of capital, credit, price and supply.
agree to refuse to sell the first article, eliminating it from their stocks, offering the new one as a
substitute. Hence, the producers or importers of the prescribed article, or its consumers, find the article
d. Alien control and threat, subject of apprehension in Constitutional convention. suddenly out of the prescribed article, or its consumers, find the article suddenly out of circulation.
Freedom of trade is thus curtailed and free enterprise correspondingly suppressed.
It is this domination and control, which we believe has been sufficiently shown to exist, that is the
legislature's target in the enactment of the disputed nationalization would never have been adopted. We can even go farther than theoretical illustrations to show the pernicious influences of alien
The framers of our Constitution also believed in the existence of this alien dominance and control when domination. Grave abuses have characterized the exercise of the retail trade by aliens. It is a fact within
they approved a resolution categorically declaring among other things, that "it is the sense of the judicial notice, which courts of justice may not properly overlook or ignore in the interests of truth and
Convention that the public interest requires the nationalization of the retail trade; . . . ." (II Aruego, The justice, that there exists a general feeling on the part of the public that alien participation in the retail
Framing of the Philippine Constitution, 662-663, quoted on page 67 of Petitioner.) That was twenty-two trade has been attended by a pernicious and intolerable practices, the mention of a few of which would
years ago; and the events since then have not been either pleasant or comforting. Dean Sinco of the suffice for our purposes; that at some time or other they have cornered the market of essential
University of the Philippines College of Law, commenting on the patrimony clause of the Preamble commodities, like corn and rice, creating artificial scarcities to justify and enhance profits to
opines that the fathers of our Constitution were merely translating the general preoccupation of Filipinos unreasonable proportions; that they have hoarded essential foods to the inconvenience and prejudice of
"of the dangers from alien interests that had already brought under their control the commercial and the consuming public, so much so that the Government has had to establish the National Rice and Corn
other economic activities of the country" (Sinco, Phil. Political Law, 10th ed., p. 114); and analyzing the Corporation to save the public from their continuous hoarding practices and tendencies; that they have
concern of the members of the constitutional convention for the economic life of the citizens, in violated price control laws, especially on foods and essential commodities, such that the legislature had
connection with the nationalistic provisions of the Constitution, he says: to enact a law (Sec. 9, Republic Act No. 1168), authorizing their immediate and automatic deportation
for price control convictions; that they have secret combinations among themselves to control prices,
But there has been a general feeling that alien dominance over the economic life of the cheating the operation of the law of supply and demand; that they have connived to boycott honest
country is not desirable and that if such a situation should remain, political independence merchants and traders who would not cater or yield to their demands, in unlawful restraint of freedom
alone is no guarantee to national stability and strength. Filipino private capital is not big of trade and enterprise. They are believed by the public to have evaded tax laws, smuggled goods and
enough to wrest from alien hands the control of the national economy. Moreover, it is but of money into and out of the land, violated import and export prohibitions, control laws and the like, in
recent formation and hence, largely inexperienced, timid and hesitant. Under such derision and contempt of lawful authority. It is also believed that they have engaged in corrupting public
conditions, the government as the instrumentality of the national will, has to step in and officials with fabulous bribes, indirectly causing the prevalence of graft and corruption in the
Government. As a matter of fact appeals to unscrupulous aliens have been made both by the differences are certainly a valid reason for the State to prefer the national over the alien in the retail
Government and by their own lawful diplomatic representatives, action which impliedly admits a trade. We would be doing violence to fact and reality were we to hold that no reason or ground for a
prevailing feeling about the existence of many of the above practices. legitimate distinction can be found between one and the other.

The circumstances above set forth create well founded fears that worse things may come in the future. b. Difference in alien aims and purposes sufficient basis for distinction.
The present dominance of the alien retailer, especially in the big centers of population, therefore,
becomes a potential source of danger on occasions of war or other calamity. We do not have here in this
The above objectionable characteristics of the exercise of the retail trade by the aliens, which are actual
country isolated groups of harmless aliens retailing goods among nationals; what we have are well
and real, furnish sufficient grounds for legislative classification of retail traders into nationals and aliens.
organized and powerful groups that dominate the distribution of goods and commodities in the
Some may disagree with the wisdom of the legislature's classification. To this we answer, that this is the
communities and big centers of population. They owe no allegiance or loyalty to the State, and the State
prerogative of the law-making power. Since the Court finds that the classification is actual, real and
cannot rely upon them in times of crisis or emergency. While the national holds his life, his person and
reasonable, and all persons of one class are treated alike, and as it cannot be said that the classification
his property subject to the needs of his country, the alien may even become the potential enemy of the
is patently unreasonable and unfounded, it is in duty bound to declare that the legislature acted within
State.
its legitimate prerogative and it can not declare that the act transcends the limit of equal protection
established by the Constitution.
f. Law enacted in interest of national economic survival and security.
Broadly speaking, the power of the legislature to make distinctions and classifications among persons is
We are fully satisfied upon a consideration of all the facts and circumstances that the disputed law is not not curtailed or denied by the equal protection of the laws clause. The legislative power admits of a wide
the product of racial hostility, prejudice or discrimination, but the expression of the legitimate desire and scope of discretion, and a law can be violative of the constitutional limitation only when the classification
determination of the people, thru their authorized representatives, to free the nation from the economic is without reasonable basis. In addition to the authorities we have earlier cited, we can also refer to the
situation that has unfortunately been saddled upon it rightly or wrongly, to its disadvantage. The law is case of Linsey vs. Natural Carbonic Fas Co. (1911), 55 L. ed., 369, which clearly and succinctly defined the
clearly in the interest of the public, nay of the national security itself, and indisputably falls within the application of equal protection clause to a law sought to be voided as contrary thereto:
scope of police power, thru which and by which the State insures its existence and security and the
supreme welfare of its citizens.
. . . . "1. The equal protection clause of the Fourteenth Amendment does not take from the
state the power to classify in the adoption of police laws, but admits of the exercise of the
VI. The Equal Protection Limitation wide scope of discretion in that regard, and avoids what is done only when it is without any
reasonable basis, and therefore is purely arbitrary. 2. A classification having some reasonable
basis does not offend against that clause merely because it is not made with mathematical
a. Objections to alien participation in retail trade. The next question that now poses solution is, Does
nicety, or because in practice it results in some inequality. 3. When the classification in such a
the law deny the equal protection of the laws? As pointed out above, the mere fact of alienage is the
law is called in question, if any state of facts reasonably can be conceived that would sustain
root and cause of the distinction between the alien and the national as a trader. The alien resident owes
it, the existence of that state of facts at the time the law was enacted must be assumed. 4.
allegiance to the country of his birth or his adopted country; his stay here is for personal convenience; he
One who assails the classification in such a law must carry the burden of showing that it does
is attracted by the lure of gain and profit. His aim or purpose of stay, we admit, is neither illegitimate nor
not rest upon any reasonable basis but is essentially arbitrary."
immoral, but he is naturally lacking in that spirit of loyalty and enthusiasm for this country where he
temporarily stays and makes his living, or of that spirit of regard, sympathy and consideration for his
Filipino customers as would prevent him from taking advantage of their weakness and exploiting them. c. Authorities recognizing citizenship as basis for classification.
The faster he makes his pile, the earlier can the alien go back to his beloved country and his beloved kin
and countrymen. The experience of the country is that the alien retailer has shown such utter disregard
The question as to whether or not citizenship is a legal and valid ground for classification has already
for his customers and the people on whom he makes his profit, that it has been found necessary to
been affirmatively decided in this jurisdiction as well as in various courts in the United States. In the case
adopt the legislation, radical as it may seem.
of Smith Bell & Co. vs. Natividad, 40 Phil. 136, where the validity of Act No. 2761 of the Philippine
Legislature was in issue, because of a condition therein limiting the ownership of vessels engaged in
Another objection to the alien retailer in this country is that he never really makes a genuine coastwise trade to corporations formed by citizens of the Philippine Islands or the United States, thus
contribution to national income and wealth. He undoubtedly contributes to general distribution, but the denying the right to aliens, it was held that the Philippine Legislature did not violate the equal protection
gains and profits he makes are not invested in industries that would help the country's economy and clause of the Philippine Bill of Rights. The legislature in enacting the law had as ultimate purpose the
increase national wealth. The alien's interest in this country being merely transient and temporary, it encouragement of Philippine shipbuilding and the safety for these Islands from foreign interlopers. We
would indeed be ill-advised to continue entrusting the very important function of retail distribution to held that this was a valid exercise of the police power, and all presumptions are in favor of its
his hands. constitutionality. In substance, we held that the limitation of domestic ownership of vessels engaged in
coastwise trade to citizens of the Philippines does not violate the equal protection of the law and due
process or law clauses of the Philippine Bill of Rights. In rendering said decision we quoted with approval
The practices resorted to by aliens in the control of distribution, as already pointed out above, their
the concurring opinion of Justice Johnson in the case of Gibbons vs. Ogden, 9 Wheat., I, as follows:
secret manipulations of stocks of commodities and prices, their utter disregard of the welfare of their
customers and of the ultimate happiness of the people of the nation of which they are mere guests,
which practices, manipulations and disregard do not attend the exercise of the trade by the nationals, "Licensing acts, in fact, in legislation, are universally restraining acts; as, for example, acts
show the existence of real and actual, positive and fundamental differences between an alien and a licensing gaming houses, retailers of spirituous liquors, etc. The act, in this instance, is
national which fully justify the legislative classification adopted in the retail trade measure. These distinctly of that character, and forms part of an extensive system, the object of which is to
encourage American shipping, and place them on an equal footing with the shipping of other In Fraser vs. McConway & Tarley Co., 82 Fed. 257 (Pennsylvania, 1897), a state law which imposed a tax
nations. Almost every commercial nation reserves to its own subjects a monopoly of its on every employer of foreign-born unnaturalized male persons over 21 years of age, was declared void
coasting trade; and a countervailing privilege in favor of American shipping is contemplated, because the court found that there was no reason for the classification and the tax was an arbitrary
in the whole legislation of the United States on this subject. It is not to give the vessel an deduction from the daily wage of an employee.
American character, that the license is granted; that effect has been correctly attributed to
the act of her enrollment. But it is to confer on her American privileges, as contra
d. Authorities contra explained.
distinguished from foreign; and to preserve the Government from fraud by foreigners; in
surreptitiously intruding themselves into the American commercial marine, as well as frauds
upon the revenue in the trade coastwise, that this whole system is projected." It is true that some decisions of the Federal court and of the State courts in the United States hold that
the distinction between aliens and citizens is not a valid ground for classification. But in this decision the
laws declared invalid were found to be either arbitrary, unreasonable or capricious, or were the result or
The rule in general is as follows:
product of racial antagonism and hostility, and there was no question of public interest involved or
pursued. In Yu Cong Eng vs. Trinidad, 70 L. ed. 1059 (1925), the United States Supreme Court declared
Aliens are under no special constitutional protection which forbids a classification otherwise invalid a Philippine law making unlawful the keeping of books of account in any language other than
justified simply because the limitation of the class falls along the lines of nationality. That English, Spanish or any other local dialect, but the main reasons for the decisions are: (1) that if Chinese
would be requiring a higher degree of protection for aliens as a class than for similar classes were driven out of business there would be no other system of distribution, and (2) that the Chinese
than for similar classes of American citizens. Broadly speaking, the difference in status would fall prey to all kinds of fraud, because they would be deprived of their right to be advised of their
between citizens and aliens constitutes a basis for reasonable classification in the exercise of business and to direct its conduct. The real reason for the decision, therefore, is the court's belief that no
police power. (2 Am., Jur. 468-469.) public benefit would be derived from the operations of the law and on the other hand it would deprive
Chinese of something indispensable for carrying on their business. In Yick Wo vs. Hopkins, 30 L. ed 220
(1885) an ordinance conferring powers on officials to withhold consent in the operation of laundries
In Commonwealth vs. Hana, 81 N. E. 149 (Massachusetts, 1907), a statute on the licensing of hawkers
both as to persons and place, was declared invalid, but the court said that the power granted was
and peddlers, which provided that no one can obtain a license unless he is, or has declared his intention,
arbitrary, that there was no reason for the discrimination which attended the administration and
to become a citizen of the United States, was held valid, for the following reason: It may seem wise to
implementation of the law, and that the motive thereof was mere racial hostility. In State vs.
the legislature to limit the business of those who are supposed to have regard for the welfare, good
Montgomery, 47 A. 165 (Maine, 1900), a law prohibiting aliens to engage as hawkers and peddlers was
order and happiness of the community, and the court cannot question this judgment and conclusion.
declared void, because the discrimination bore no reasonable and just relation to the act in respect to
In Bloomfield vs. State, 99 N. E. 309 (Ohio, 1912), a statute which prevented certain persons, among
which the classification was proposed.
them aliens, from engaging in the traffic of liquors, was found not to be the result of race hatred, or in
hospitality, or a deliberate purpose to discriminate, but was based on the belief that an alien cannot be
sufficiently acquainted with "our institutions and our life as to enable him to appreciate the relation of The case at bar is radically different, and the facts make them so. As we already have said, aliens do not
this particular business to our entire social fabric", and was not, therefore, invalid. In Ohio ex rel. Clarke naturally possess the sympathetic consideration and regard for the customers with whom they come in
vs. Deckebach, 274 U. S. 392, 71 L. ed. 115 (1926), the U.S. Supreme Court had under consideration an daily contact, nor the patriotic desire to help bolster the nation's economy, except in so far as it
ordinance of the city of Cincinnati prohibiting the issuance of licenses (pools and billiard rooms) to aliens. enhances their profit, nor the loyalty and allegiance which the national owes to the land. These
It held that plainly irrational discrimination against aliens is prohibited, but it does not follow that alien limitations on the qualifications of the aliens have been shown on many occasions and instances,
race and allegiance may not bear in some instances such a relation to a legitimate object of legislation as especially in times of crisis and emergency. We can do no better than borrow the language of Anton vs.
to be made the basis of permitted classification, and that it could not state that the legislation is clearly Van Winkle, 297 F. 340, 342, to drive home the reality and significance of the distinction between the
wrong; and that latitude must be allowed for the legislative appraisement of local conditions and for the alien and the national, thus:
legislative choice of methods for controlling an apprehended evil. The case of State vs. Carrol, 124 N. E.
129 (Ohio, 1919) is a parallel case to the one at bar. In Asakura vs. City of Seattle, 210 P. 30 (Washington,
. . . . It may be judicially known, however, that alien coming into this country are without the
1922), the business of pawn brooking was considered as having tendencies injuring public interest, and
intimate knowledge of our laws, customs, and usages that our own people have. So it is
limiting it to citizens is within the scope of police power. A similar statute denying aliens the right to
likewise known that certain classes of aliens are of different psychology from our fellow
engage in auctioneering was also sustained in Wright vs. May, L.R.A., 1915 P. 151 (Minnesota, 1914). So
countrymen. Furthermore, it is natural and reasonable to suppose that the foreign born,
also in Anton vs. Van Winkle, 297 F. 340 (Oregon, 1924), the court said that aliens are judicially known to
whose allegiance is first to their own country, and whose ideals of governmental environment
have different interests, knowledge, attitude, psychology and loyalty, hence the prohibitions of issuance
and control have been engendered and formed under entirely different regimes and political
of licenses to them for the business of pawnbroker, pool, billiard, card room, dance hall, is not an
systems, have not the same inspiration for the public weal, nor are they as well disposed
infringement of constitutional rights. In Templar vs. Michigan State Board of Examiners, 90 N.W. 1058
toward the United States, as those who by citizenship, are a part of the government itself.
(Michigan, 1902), a law prohibiting the licensing of aliens as barbers was held void, but the reason for
Further enlargement, is unnecessary. I have said enough so that obviously it cannot be
the decision was the court's findings that the exercise of the business by the aliens does not in any way
affirmed with absolute confidence that the Legislature was without plausible reason for
affect the morals, the health, or even the convenience of the community. In Takahashi vs. Fish and Game
making the classification, and therefore appropriate discriminations against aliens as it relates
Commission, 92 L. ed. 1479 (1947), a California statute banning the issuance of commercial fishing
to the subject of legislation. . . . .
licenses to person ineligible to citizenship was held void, because the law conflicts with Federal power
over immigration, and because there is no public interest in the mere claim of ownership of the waters
and the fish in them, so there was no adequate justification for the discrimination. It further added that VII. The Due Process of Law Limitation.
the law was the outgrowth of antagonism toward the persons of Japanese ancestry. However, two
Justices dissented on the theory that fishing rights have been treated traditionally as natural resources. a. Reasonability, the test of the limitation; determination by legislature decisive.
We now come to due process as a limitation on the exercise of the police power. It has been stated by Petitioner's main argument is that retail is a common, ordinary occupation, one of those privileges long
the highest authority in the United States that: ago recognized as essential to the orderly pursuant of happiness by free men; that it is a gainful and
honest occupation and therefore beyond the power of the legislature to prohibit and penalized. This
arguments overlooks fact and reality and rests on an incorrect assumption and premise, i.e., that in this
. . . . And the guaranty of due process, as has often been held, demands only that the law shall
country where the occupation is engaged in by petitioner, it has been so engaged by him, by the alien in
not be unreasonable, arbitrary or capricious, and that the means selected shall have a real
an honest creditable and unimpeachable manner, without harm or injury to the citizens and without
and substantial relation to the subject sought to be attained. . . . .
ultimate danger to their economic peace, tranquility and welfare. But the Legislature has found, as we
have also found and indicated, that the privilege has been so grossly abused by the alien, thru the
xxx xxx xxx illegitimate use of pernicious designs and practices, that he now enjoys a monopolistic control of the
occupation and threatens a deadly stranglehold on the nation's economy endangering the national
So far as the requirement of due process is concerned and in the absence of other security in times of crisis and emergency.
constitutional restriction a state is free to adopt whatever economic policy may reasonably be
deemed to promote public welfare, and to enforce that policy by legislation adapted to its The real question at issue, therefore, is not that posed by petitioner, which overlooks and ignores the
purpose. The courts are without authority either to declare such policy, or, when it is declared facts and circumstances, but this, Is the exclusion in the future of aliens from the retail trade
by the legislature, to override it. If the laws passed are seen to have a reasonable relation to a unreasonable. Arbitrary capricious, taking into account the illegitimate and pernicious form and manner
proper legislative purpose, and are neither arbitrary nor discriminatory, the requirements of in which the aliens have heretofore engaged therein? As thus correctly stated the answer is clear. The
due process are satisfied, and judicial determination to that effect renders a court functus law in question is deemed absolutely necessary to bring about the desired legislative objective, i.e., to
officio. . . . (Nebbia vs. New York, 78 L. ed. 940, 950, 957.) free national economy from alien control and dominance. It is not necessarily unreasonable because it
affects private rights and privileges (11 Am. Jur. pp. 1080-1081.) The test of reasonableness of a law is
Another authority states the principle thus: the appropriateness or adequacy under all circumstances of the means adopted to carry out its purpose
into effect (Id.) Judged by this test, disputed legislation, which is not merely reasonable but actually
necessary, must be considered not to have infringed the constitutional limitation of reasonableness.
. . . . Too much significance cannot be given to the word "reasonable" in considering the scope
of the police power in a constitutional sense, for the test used to determine the
constitutionality of the means employed by the legislature is to inquire whether the The necessity of the law in question is explained in the explanatory note that accompanied the bill,
restriction it imposes on rights secured to individuals by the Bill of Rights are unreasonable, which later was enacted into law:
and not whether it imposes any restrictions on such rights. . . .
This bill proposes to regulate the retail business. Its purpose is to prevent persons who are
xxx xxx xxx not citizens of the Philippines from having a strangle hold upon our economic life. If the
persons who control this vital artery of our economic life are the ones who owe no allegiance
to this Republic, who have no profound devotion to our free institutions, and who have no
. . . . A statute to be within this power must also be reasonable in its operation upon the permanent stake in our people's welfare, we are not really the masters of our destiny. All
persons whom it affects, must not be for the annoyance of a particular class, and must not be aspects of our life, even our national security, will be at the mercy of other people.
unduly oppressive. (11 Am. Jur. Sec. 302., 1:1)- 1074-1075.)

In seeking to accomplish the foregoing purpose, we do not propose to deprive persons who
In the case of Lawton vs. Steele, 38 L. ed. 385, 388. it was also held: are not citizens of the Philippines of their means of livelihood. While this bill seeks to take
away from the hands of persons who are not citizens of the Philippines a power that can be
. . . . To justify the state in thus interposing its authority in behalf of the public, it must appear, wielded to paralyze all aspects of our national life and endanger our national security it
first, that the interests of the public generally, as distinguished from those of a particular respects existing rights.
class, require such interference; and second, that the means are reasonably necessary for the
accomplishment of the purpose, and not unduly oppressive upon individuals. . . . The approval of this bill is necessary for our national survival.

Prata Undertaking Co. vs. State Board of Embalming, 104 ALR, 389, 395, fixes this test of If political independence is a legitimate aspiration of a people, then economic independence is none the
constitutionality: less legitimate. Freedom and liberty are not real and positive if the people are subject to the economic
control and domination of others, especially if not of their own race or country. The removal and
In determining whether a given act of the Legislature, passed in the exercise of the police eradication of the shackles of foreign economic control and domination, is one of the noblest motives
power to regulate the operation of a business, is or is not constitutional, one of the first that a national legislature may pursue. It is impossible to conceive that legislation that seeks to bring it
questions to be considered by the court is whether the power as exercised has a sufficient about can infringe the constitutional limitation of due process. The attainment of a legitimate aspiration
foundation in reason in connection with the matter involved, or is an arbitrary, oppressive, of a people can never be beyond the limits of legislative authority.
and capricious use of that power, without substantial relation to the health, safety, morals,
comfort, and general welfare of the public. c. Law expressly held by Constitutional Convention to be within the sphere of legislative action.

b. Petitioner's argument considered.


The framers of the Constitution could not have intended to impose the constitutional restrictions of due from foreigners; etc. Many of these arguments are directed against the supposed wisdom of the law
process on the attainment of such a noble motive as freedom from economic control and domination, which lies solely within the legislative prerogative; they do not import invalidity.
thru the exercise of the police power. The fathers of the Constitution must have given to the legislature
full authority and power to enact legislation that would promote the supreme happiness of the people,
VIII. Alleged defect in the title of the law
their freedom and liberty. On the precise issue now before us, they expressly made their voice clear;
they adopted a resolution expressing their belief that the legislation in question is within the scope of
the legislative power. Thus they declared the their Resolution: A subordinate ground or reason for the alleged invalidity of the law is the claim that the title thereof is
misleading or deceptive, as it conceals the real purpose of the bill which is to nationalize the retail
business and prohibit aliens from engaging therein. The constitutional provision which is claimed to be
That it is the sense of the Convention that the public interest requires the nationalization of
violated in Section 21 (1) of Article VI, which reads:
retail trade; but it abstain from approving the amendment introduced by the Delegate for
Manila, Mr. Araneta, and others on this matter because it is convinced that the National
Assembly is authorized to promulgate a law which limits to Filipino and American citizens the No bill which may be enacted in the law shall embrace more than one subject which shall be
privilege to engage in the retail trade. (11 Aruego, The Framing of the Philippine Constitution, expressed in the title of the bill.
quoted on pages 66 and 67 of the Memorandum for the Petitioner.)
What the above provision prohibits is duplicity, that is, if its title completely fails to appraise the
It would do well to refer to the nationalistic tendency manifested in various provisions of the legislators or the public of the nature, scope and consequences of the law or its operation (I Sutherland,
Constitution. Thus in the preamble, a principle objective is the conservation of the patrimony of the Statutory Construction, Sec. 1707, p. 297.) A cursory consideration of the title and the provisions of the
nation and as corollary the provision limiting to citizens of the Philippines the exploitation, development bill fails to show the presence of duplicity. It is true that the term "regulate" does not and may not
and utilization of its natural resources. And in Section 8 of Article XIV, it is provided that "no franchise, readily and at first glance convey the idea of "nationalization" and "prohibition", which terms express the
certificate, or any other form of authorization for the operation of the public utility shall be granted two main purposes and objectives of the law. But "regulate" is a broader term than either prohibition or
except to citizens of the Philippines." The nationalization of the retail trade is only a continuance of the nationalization. Both of these have always been included within the term regulation.
nationalistic protective policy laid down as a primary objective of the Constitution. Can it be said that a
law imbued with the same purpose and spirit underlying many of the provisions of the Constitution is Under the title of an act to "regulate", the sale of intoxicating liquors, the Legislature may
unreasonable, invalid and unconstitutional? prohibit the sale of intoxicating liquors. (Sweet vs. City of Wabash, 41 Ind., 7; quoted in page
41 of Answer.)
The seriousness of the Legislature's concern for the plight of the nationals as manifested in the approval
of the radical measures is, therefore, fully justified. It would have been recreant to its duties towards the Within the meaning of the Constitution requiring that the subject of every act of the
country and its people would it view the sorry plight of the nationals with the complacency and refuse or Legislature shall be stated in the tale, the title to regulate the sale of intoxicating liquors, etc."
neglect to adopt a remedy commensurate with the demands of public interest and national survival. As sufficiently expresses the subject of an act prohibiting the sale of such liquors to minors and
the repository of the sovereign power of legislation, the Legislature was in duty bound to face the to persons in the habit of getting intoxicated; such matters being properly included within the
problem and meet, through adequate measures, the danger and threat that alien domination of retail subject of regulating the sale. (Williams vs. State, 48 Ind. 306, 308, quoted in p. 42 of Answer.)
trade poses to national economy.

The word "regulate" is of broad import, and necessarily implies some degree of restraint and
d. Provisions of law not unreasonable. prohibition of acts usually done in connection with the thing to be regulated. While word
regulate does not ordinarily convey meaning of prohibit, there is no absolute reason why it
A cursory study of the provisions of the law immediately reveals how tolerant, how reasonable the should not have such meaning when used in delegating police power in connection with a
Legislature has been. The law is made prospective and recognizes the right and privilege of those already thing the best or only efficacious regulation of which involves suppression. (State vs. Morton,
engaged in the occupation to continue therein during the rest of their lives; and similar recognition of 162 So. 718, 182 La. 887, quoted in p. 42 of Answer.)
the right to continue is accorded associations of aliens. The right or privilege is denied to those only upon
conviction of certain offenses. In the deliberations of the Court on this case, attention was called to the The general rule is for the use of general terms in the title of a bill; it has also been said that the title
fact that the privilege should not have been denied to children and heirs of aliens now engaged in the need not be an index to the entire contents of the law (I Sutherland, Statutory Construction, See. 4803,
retail trade. Such provision would defeat the law itself, its aims and purposes. Beside, the exercise of p. 345.) The above rule was followed the title of the Act in question adopted the more general term
legislative discretion is not subject to judicial review. It is well settled that the Court will not inquire into "regulate" instead of "nationalize" or "prohibit". Furthermore, the law also contains other rules for the
the motives of the Legislature, nor pass upon general matters of legislative judgment. The Legislature is regulation of the retail trade which may not be included in the terms "nationalization" or "prohibition";
primarily the judge of the necessity of an enactment or of any of its provisions, and every presumption is so were the title changed from "regulate" to "nationalize" or "prohibit", there would have been many
in favor of its validity, and though the Court may hold views inconsistent with the wisdom of the law, it provisions not falling within the scope of the title which would have made the Act invalid. The use of the
may not annul the legislation if not palpably in excess of the legislative power. Furthermore, the test of term "regulate", therefore, is in accord with the principle governing the drafting of statutes, under which
the validity of a law attacked as a violation of due process, is not its reasonableness, but its a simple or general term should be adopted in the title, which would include all other provisions found in
unreasonableness, and we find the provisions are not unreasonable. These principles also answer the body of the Act.
various other arguments raised against the law, some of which are: that the law does not promote
general welfare; that thousands of aliens would be thrown out of employment; that prices will increase
because of the elimination of competition; that there is no need for the legislation; that adequate One purpose of the constitutional directive that the subject of a bill should be embraced in its title is to
replacement is problematical; that there may be general breakdown; that there would be repercussions apprise the legislators of the purposes, the nature and scope of its provisions, and prevent the
enactment into law of matters which have received the notice, action and study of the legislators or of however, is merely to determine if the law falls within the scope of legislative authority and does not
the public. In the case at bar it cannot be claimed that the legislators have been appraised of the nature transcend the limitations of due process and equal protection guaranteed in the Constitution. Remedies
of the law, especially the nationalization and the prohibition provisions. The legislators took active against the harshness of the law should be addressed to the Legislature; they are beyond our power and
interest in the discussion of the law, and a great many of the persons affected by the prohibitions in the jurisdiction.
law conducted a campaign against its approval. It cannot be claimed, therefore, that the reasons for
declaring the law invalid ever existed. The objection must therefore, be overruled.
The petition is hereby denied, with costs against petitioner.

IX. Alleged violation of international treaties and obligations

Another subordinate argument against the validity of the law is the supposed violation thereby of the
Charter of the United Nations and of the Declaration of the Human Rights adopted by the United Nations
General Assembly. We find no merit in the Nations Charter imposes no strict or legal obligations
regarding the rights and freedom of their subjects (Hans Kelsen, The Law of the United Nations, 1951 ed.
pp. 29-32), and the Declaration of Human Rights contains nothing more than a mere recommendation or
a common standard of achievement for all peoples and all nations (Id. p. 39.) That such is the import of
the United Nations Charter aid of the Declaration of Human Rights can be inferred the fact that members
of the United Nations Organizations, such as Norway and Denmark, prohibit foreigners from engaging in
retail trade, and in most nations of the world laws against foreigners engaged in domestic trade are
adopted.

The Treaty of Amity between the Republic of the Philippines and the Republic of China of April 18, 1947
is also claimed to be violated by the law in question. All that the treaty guarantees is equality of
treatment to the Chinese nationals "upon the same terms as the nationals of any other country." But the
nationals of China are not discriminating against because nationals of all other countries, except those of
the United States, who are granted special rights by the Constitution, are all prohibited from engaging in
the retail trade. But even supposing that the law infringes upon the said treaty, the treaty is always
subject to qualification or amendment by a subsequent law (U. S. vs. Thompson, 258, Fed. 257, 260), and
the same may never curtail or restrict the scope of the police power of the State (plaston vs.
Pennsylvania, 58 L. ed. 539.)

X. Conclusion

Resuming what we have set forth above we hold that the disputed law was enacted to remedy a real
actual threat and danger to national economy posed by alien dominance and control of the retail
business and free citizens and country from dominance and control; that the enactment clearly falls
within the scope of the police power of the State, thru which and by which it protects its own personality
and insures its security and future; that the law does not violate the equal protection clause of the
Constitution because sufficient grounds exist for the distinction between alien and citizen in the exercise
of the occupation regulated, nor the due process of law clause, because the law is prospective in
operation and recognizes the privilege of aliens already engaged in the occupation and reasonably
protects their privilege; that the wisdom and efficacy of the law to carry out its objectives appear to us to
be plainly evident as a matter of fact it seems not only appropriate but actually necessary and that
in any case such matter falls within the prerogative of the Legislature, with whose power and discretion
the Judicial department of the Government may not interfere; that the provisions of the law are clearly
embraced in the title, and this suffers from no duplicity and has not misled the legislators or the segment
of the population affected; and that it cannot be said to be void for supposed conflict with treaty
obligations because no treaty has actually been entered into on the subject and the police power may
not be curtailed or surrendered by any treaty or any other conventional agreement.

Some members of the Court are of the opinion that the radical effects of the law could have been made
less harsh in its impact on the aliens. Thus it is stated that the more time should have been given in the
law for the liquidation of existing businesses when the time comes for them to close. Our legal duty,
Republic of the Philippines Respondents maintain that the status of petitioner as a rice planter does not give him sufficient interest
SUPREME COURT to file the petition herein and secure the relief therein prayed for. We find no merit in this pretense.
Manila Apart from prohibiting the importation of rice and corn "by the Rice and Corn Administration or any
other government agency". Republic Act No. 3452 declares, in Section 1 thereof, that "the policy of the
Government" is to "engage in the purchase of these basic foods directly from those tenants, farmers,
EN BANC
growers, producers and landowners in the Philippines who wish to dispose of their products at a price
that will afford them a fair and just return for their labor and capital investment. ... ." Pursuant to this
G.R. No. L-21897 October 22, 1963 provision, petitioner, as a planter with a rice land of substantial proportion,2 is entitled to a chance to sell
to the Government the rice it now seeks to buy abroad. Moreover, since the purchase of said commodity
RAMON A. GONZALES, petitioner, will have to be effected with public funds mainly raised by taxation, and as a rice producer and
vs. landowner petitioner must necessarily be a taxpayer, it follows that he has sufficient personality and
RUFINO G. HECHANOVA, as Executive Secretary, MACARIO PERALTA, JR., as Secretary of Defense, interest to seek judicial assistance with a view to restraining what he believes to be an attempt to
PEDRO GIMENEZ, as Auditor General, CORNELIO BALMACEDA, as Secretary of Commerce and Industry, unlawfully disburse said funds.
and SALVADOR MARINO, Secretary of Justice, respondents.
II. Exhaustion of administrative remedies.
Ramon A. Gonzales in his own behalf as petitioner.
Office of the Solicitor General and Estanislao Fernandez for respondents. Respondents assail petitioner's right to the reliefs prayed for because he "has not exhausted all
administrative remedies available to him before coming to court". We have already held, however, that
CONCEPCION, J.: the principle requiring the previous exhaustion of administrative remedies is not applicable where the
question in dispute is purely a legal one",3 or where the controverted act is "patently illegal" or was
performed without jurisdiction or in excess of jurisdiction,4 or where the respondent is a department
This is an original action for prohibition with preliminary injunction. secretary, whose acts as an alter-ego of the President bear the implied or assumed approval of the
latter,5 unless actually disapproved by him,6 or where there are circumstances indicating the urgency of
It is not disputed that on September 22, 1963, respondent Executive Secretary authorized the judicial intervention.7 The case at bar fails under each one of the foregoing exceptions to the general
importation of 67,000 tons of foreign rice to be purchased from private sources, and created a rice rule. Respondents' contention is, therefore, untenable.
procurement committee composed of the other respondents herein1 for the implementation of said
proposed importation. Thereupon, or September 25, 1963, herein petitioner, Ramon A. Gonzales a III. Merits of petitioner's cause of action.
rice planter, and president of the Iloilo Palay and Corn Planters Association, whose members are,
likewise, engaged in the production of rice and corn filed the petition herein, averring that, in making
or attempting to make said importation of foreign rice, the aforementioned respondents "are acting Respondents question the sufficiency of petitioner's cause of action upon the theory that the proposed
without jurisdiction or in excess of jurisdiction", because Republic Act No. 3452 which allegedly repeals importation in question is not governed by Republic Acts Nos. 2207 and 3452, but was authorized by the
or amends Republic Act No. 220 explicitly prohibits the importation of rice and corn "the Rice and President as Commander-in-Chief "for military stock pile purposes" in the exercise of his alleged
Corn Administration or any other government agency;" that petitioner has no other plain, speedy and authority under Section 2 of Commonwealth Act No. 1;8 that in cases of necessity, the President "or his
adequate remedy in the ordinary course of law; and that a preliminary injunction is necessary for the subordinates may take such preventive measure for the restoration of good order and maintenance of
preservation of the rights of the parties during the pendency this case and to prevent the judgment peace"; and that, as Commander-in-Chief of our armed forces, "the President ... is duty-bound to
therein from coming ineffectual. Petitioner prayed, therefore, that said petition be given due course; prepare for the challenge of threats of war or emergency withoutwaiting for any special authority".
that a writ of preliminary injunction be forthwith issued restraining respondent their agents or
representatives from implementing the decision of the Executive Secretary to import the Regardless of whether Republic Act No. 3452 repeals Republic Act No. 2207, as contended by petitioner
aforementioned foreign rice; and that, after due hearing, judgment be rendered making said injunction herein - on which our view need not be expressed we are unanimously of the opinion - assuming that
permanent. said Republic Act No. 2207 is still in force that the two Acts are applicable to the proposed
importation in question because the language of said laws is such as to include within the purview
Forthwith, respondents were required to file their answer to the petition which they did, and petitioner's thereof all importations of rice and corn into the Philippines". Pursuant to Republic Act No. 2207, "it shall
pray for a writ of preliminary injunction was set for hearing at which both parties appeared and argued be unlawful for any person, association, corporation orgovernment agency to import rice and corn into
orally. Moreover, a memorandum was filed, shortly thereafter, by the respondents. Considering, later any point in the Philippines", although, by way of exception, it adds, that "the President of the
on, that the resolution said incident may require some pronouncements that would be more appropriate Philippines may authorize the importation of these commodities through any government agency that he
in a decision on the merits of the case, the same was set for hearing on the merits thereafter. The may designate", is the conditions prescribed in Section 2 of said Act are present. Similarly, Republic Act
parties, however, waived the right to argue orally, although counsel for respondents filed their No. 3452 explicitly enjoins "the Rice and Corn Administration or any government agency" from importing
memoranda. rice and corn.

I. Sufficiency of petitioner's interest. Respondents allege, however, that said provisions of Republic Act Nos. 2207 and 3452, prohibiting the
importation of rice and corn by any "government agency", do not apply to importations "made by the
Government itself", because the latter is not a "government agency". This theory is devoid of merit. The
Department of National Defense and the Armed Forces of the Philippines, as well as respondents herein,
and each and every officer and employee of our Government, our government agencies and/or agents. Besides, the stockpiling of rice and corn for purpose of national security and/or national emergency is
The applicability of said laws even to importations by the Government as such, becomes more apparent within the purview of Republic Act No. 3452. Section 3 thereof expressly authorizes the Rice and Corn
when we consider that: Administration "to accumulate stocks as a national reserve in such quantities as it may deem proper and
necessary to meet any contingencies". Moreover, it ordains that "the buffer stocks held as a national
reserve ... be deposited by the administration throughout the country under the proper dispersal plans ...
1. The importation permitted in Republic Act No. 2207 is to be authorized by the "President of the
and may be released only upon the occurrence of calamities or emergencies ...". (Emphasis applied.)
Philippines" and, hence, by or on behalf of the Government of the Philippines;

Again, the provisions of Section 2 of Commonwealth Act No. 1, upon which respondents rely so much,
2. Immediately after enjoining the Rice and Corn administration and any other government agency from
are not self-executory. They merely outline the general objectives of said legislation. The means for the
importing rice and corn, Section 10 of Republic Act No. 3452 adds "that the importation of rice and corn
attainment of those objectives are subject to congressional legislation. Thus, the conditions under which
is left to private parties upon payment of the corresponding taxes", thus indicating that only "private
the services of citizens, as indicated in said Section 2, may be availed of, are provided for in Sections 3, 4
parties" may import rice under its provisions; and
and 51 to 88 of said Commonwealth Act No. 1. Similarly, Section 5 thereof specifies the manner in which
resources necessary for our national defense may be secured by the Government of the Philippines, but
3. Aside from prescribing a fine not exceeding P10,000.00 and imprisonment of not more than five (5) only "during a national mobilization",9which does not exist. Inferentially, therefore, in the absence of a
years for those who shall violate any provision of Republic Act No. 3452 or any rule and regulation national mobilization, said resources shall be produced in such manner as Congress may by other laws
promulgated pursuant thereto, Section 15 of said Act provides that "if the offender is a public provide from time to time. Insofar as rice and corn are concerned, Republic Acts Nos. 2207 and 3452,
official and/or employees", he shall be subject to the additional penalty specified therein. A public and Commonwealth Act No. 138 are such laws.
official is an officer of the Government itself, as distinguished from officers or employees of
instrumentalities of the Government. Hence, the duly authorized acts of the former are those of the
Respondents cite Corwin in support of their pretense, but in vain. An examination of the work
Government, unlike those of a government instrumentality which may have a personality of its own,
cited10 shows that Corwin referred to the powers of the President during "war time"11 or when he has
distinct and separate from that of the Government, as such. The provisions of Republic Act No. 2207 are,
placed the country or a part thereof under "martial law".12 Since neither condition obtains in the case at
in this respect, even more explicit. Section 3 thereof provides a similar additional penalty for any "officer
bar, said work merely proves that respondents' theory, if accepted, would, in effect, place the Philippines
or employee of the Government" who "violates, abets or tolerates the violation of any provision" of said
under martial law, without a declaration of the Executive to that effect. What is worse, it would keep
Act. Hence, the intent to apply the same to transactions made by the very government is patent.
us perpetually under martial law.

Indeed, the restrictions imposed in said Republic Acts are merely additional to those prescribed in
It has been suggested that even if the proposed importation violated Republic Acts Nos. 2207 and 3452,
Commonwealth Act No. 138, entitled "An Act to give native products and domestic entities the
it should, nevertheless, be permitted because "it redounds to the benefit of the people". Salus populi est
preference in the purchase of articles for the Government." Pursuant to Section 1 thereof:
suprema lex, it is said.

The Purchase and Equipment Division of the Government of the Philippines and other officers
If there were a local shortage of rice, the argument might have some value. But the respondents, as
and employees of the municipal and provincial governments and the Government of the
officials of this Government, have expressly affirmed again and again that there is no rice shortage. And
Philippines and of chartered cities, boards, commissions, bureaus, departments, offices,
the importation is avowedly for stockpile of the Army not the civilian population.
agencies, branches, and bodies of any description, including government-owned companies,
authorized to requisition, purchase, or contract or make disbursements for articles, materials,
and supplies for public use, public buildings, or public works shall give preference to But let us follow the respondents' trend of thought. It has a more serious implication that appears on the
materials ... produced ... in the Philippines or in the United States, and to domestic entities, surface. It implies that if an executive officer believes that compliance with a certain statute will not
subject to the conditions hereinbelow specified. (Emphasis supplied.) benefit the people, he is at liberty to disregard it. That idea must be rejected - we still live under a rule of
law.
Under this provision, in all purchases by the Government, including those made by and/or for the armed
forces,preference shall be given to materials produced in the Philippines. The importation involved in the And then, "the people" are either producers or consumers. Now as respondents explicitly admit
case at bar violates this general policy of our Government, aside from the provisions of Republic Acts Republic Acts Nos. 2207 and 3452 were approved by the Legislature for the benefit of producers and
Nos. 2207 and 3452. consumers, i.e., the people, it must follow that the welfare of the people lies precisely in
the compliance with said Acts.
The attempt to justify the proposed importation by invoking reasons of national security predicated
upon the "worsening situation in Laos and Vietnam", and "the recent tension created by the Malaysia It is not for respondent executive officers now to set their own opinions against that of the Legislature,
problem" - and the alleged powers of the President as Commander-in-Chief of all armed forces in the and adopt means or ways to set those Acts at naught. Anyway, those laws permit importation but
Philippines, under Section 2 of the National Defense Act (Commonwealth Act No. 1), overlooks the fact under certain conditions, which have not been, and should be complied with.
that the protection of local planters of rice and corn in a manner that would foster and accelerate self-
sufficiency in the local production of said commodities constitutes a factor that is vital to our ability to
IV. The contracts with Vietnam and Burma
meet possible national emergency. Even if the intent in importing goods in anticipation of such
emergency were to bolster up that ability, the latter would, instead, be impaired if the importation were
so made as to discourage our farmers from engaging in the production of rice. It is lastly contended that the Government of the Philippines has already entered into two (2) contracts
for the Purchase of rice, one with the Republic of Vietnam, and another with the Government of Burma;
that these contracts constitute valid executive agreements under international law; that such question, because, aside from the fact that said obligations may be complied with without importing the
agreements became binding effective upon the signing thereof by representatives the parties thereto; commodity into the Philippines, the proposed importation may still be legalized by complying with the
that in case of conflict between Republic Acts Nos. 2207 and 3452 on the one hand, and aforementioned provisions of the aforementioned laws.
contracts, on the other, the latter should prevail, because, if a treaty and a statute are inconsistent with
each other, the conflict must be resolved under the American jurisprudence in favor of the one
V. The writ of preliminary injunction.
which is latest in point of time; that petitioner herein assails the validity of acts of the Executive relative
to foreign relations in the conduct of which the Supreme Court cannot interfere; and the
aforementioned contracts have already been consummated, the Government of the Philippines having The members of the Court have divergent opinions on the question whether or not respondents herein
already paid the price of the rice involved therein through irrevocable letters of credit in favor of the sell should be enjoined from implementing the aforementioned proposed importation. However, the
of the said commodity. We find no merit in this pretense. majority favors the negative view, for which reason the injunction prayed for cannot be granted.

The Court is not satisfied that the status of said tracts as alleged executive agreements has been WHEREFORE, judgment is hereby rendered declaring that respondent Executive Secretary had and has
sufficiently established. The parties to said contracts do not pear to have regarded the same as executive no power to authorize the importation in question; that he exceeded his jurisdiction in granting said
agreements. But, even assuming that said contracts may properly considered as executive agreements, authority; said importation is not sanctioned by law and is contrary to its provisions; and that, for lack of
the same are unlawful, as well as null and void, from a constitutional viewpoint, said agreements being the requisite majority, the injunction prayed for must be and is, accordingly denied. It is so ordered.
inconsistent with the provisions of Republic Acts Nos. 2207 and 3452. Although the President may, under
the American constitutional system enter into executive agreements without previous legislative
authority, he may not, by executive agreement, enter into a transaction which is prohibited by statutes
enacted prior thereto. Under the Constitution, the main function of the Executive is to enforce laws
enacted by Congress. The former may not interfere in the performance of the legislative powers of the
latter, except in the exercise of his veto power. He may not defeat legislative enactments that have
acquired the status of law, by indirectly repealing the same through an executive agreement providing
for the performance of the very act prohibited by said laws.

The American theory to the effect that, in the event of conflict between a treaty and a statute, the one
which is latest in point of time shall prevail, is not applicable to the case at bar, for respondents not only
admit, but, alsoinsist that the contracts adverted to are not treaties. Said theory may be justified upon
the ground that treaties to which the United States is signatory require the advice and consent of its
Senate, and, hence, of a branch of the legislative department. No such justification can be given as
regards executive agreements not authorized by previous legislation, without completely upsetting the
principle of separation of powers and the system of checks and balances which are fundamental in our
constitutional set up and that of the United States.

As regards the question whether an international agreement may be invalidated by our courts, suffice it
to say that the Constitution of the Philippines has clearly settled it in the affirmative, by providing, in
Section 2 of Article VIII thereof, that the Supreme Court may not be deprived "of its jurisdiction to
review, revise, reverse, modify, or affirm on appeal, certiorari, or writ of error as the law or the rules of
court may provide, final judgments and decrees of inferior courts in (1) All cases in which
the constitutionality or validity of any treaty, law, ordinance, or executive order or regulation is in
question". In other words, our Constitution authorizes the nullification of a treaty, not only when it
conflicts with the fundamental law, but, also, when it runs counter to an act of Congress.

The alleged consummation of the aforementioned contracts with Vietnam and Burma does not render
this case academic, Republic Act No. 2207 enjoins our Government not from entering into contracts for
the purchase of rice, but from importing rice, except under the conditions Prescribed in said Act. Upon
the other hand, Republic Act No. 3452 has two (2) main features, namely: (a) it requires the Government
to purchase rice and corn directly from our local planters, growers or landowners; and (b) it
prohibits importations of rice by the Government, and leaves such importations to private parties. The
pivotal issue in this case is whether the proposed importation which has not been consummated as
yet is legally feasible.

Lastly, a judicial declaration of illegality of the proposed importation would not compel our Government
to default in the performance of such obligations as it may have contracted with the sellers of the rice in
Republic of the Philippines sides. 2. Whenever any motor vehicle is stalled or disabled or is parked for thirty (30) minutes or more
SUPREME COURT on any street or highway, including expressways or limited access roads, the owner, user or driver
Manila thereof shall cause the warning device mentioned herein to be installed at least four meters away to the
front and rear of the motor vehicle staged, disabled or parked. 3. The Land Transportation Commissioner
shall cause Reflectorized Triangular Early Warning Devices, as herein described, to be prepared and
EN BANC
issued to registered owners of motor vehicles, except motorcycles and trailers, charging for each piece
not more than 15 % of the acquisition cost. He shall also promulgate such rules and regulations as are
G.R. No. L-49112 February 2, 1979 appropriate to effectively implement this order. 4. All hereby concerned shall closely coordinate and take
such measures as are necessary or appropriate to carry into effect then instruction. 3 Thereafter, on
LEOVILLO C. AGUSTIN, petitioner, November 15, 1976, it was amended by Letter of Instruction No. 479 in this wise. "Paragraph 3 of Letter
vs. of Instruction No. 229 is hereby amended to read as follows: 3. The Land transportation Commissioner
HON. ROMEO F. EDU, in his capacity as Land Transportation Commissioner; HON. JUAN PONCE ENRILE, shall require every motor vehicle owner to procure from any and present at the registration of his
in his capacity as Minister of National Defense; HON. ALFREDO L. JUINIO, in his capacity as Minister Of vehicle, one pair of a reflectorized early warning device, as d bed of any brand or make chosen by mid
Public Works, Transportation and Communications; and HON: BALTAZAR AQUINO, in his capacity as motor vehicle . The Land Transportation Commissioner shall also promulgate such rule and regulations
Minister of Public Highways, respondents. as are appropriate to effectively implement this order.'" 4There was issued accordingly, by respondent
Edu, the implementing rules and regulations on December 10, 1976. 5 They were not enforced as
President Marcos on January 25, 1977, ordered a six-month period of suspension insofar as the
Leovillo C. Agustin Law Office for petitioner. installation of early warning device as a pre-registration requirement for motor vehicle was
concerned. 6 Then on June 30, 1978, another Letter of Instruction 7 the lifting of such suspension and
Solicitor General Estelito P. Mendoza, Assistant Solicitor General Ruben E. Agpalo and Solicitor Amado D. directed the immediate implementation of Letter of Instruction No. 229 as amended. 8 It was not until
Aquino for respondents. August 29, 1978 that respondent Edu issued Memorandum Circular No. 32, worded thus: "In pursuance
of Letter of Instruction No. 716, dated June 30, 1978, the implementation of Letter of Instruction No.
229, as amended by Letter of Instructions No. 479, requiring the use of Early Warning Devices (EWD) on
motor vehicle, the following rules and regulations are hereby issued: 1. LTC Administrative Order No. 1,
dated December 10, 1976; shall now be implemented provided that the device may come from whatever
FERNANDO, J.: source and that it shall have substantially complied with the EWD specifications contained in Section 2 of
said administrative order; 2. In order to insure that every motor vehicle , except motorcycles, is equipped
with the device, a pair of serially numbered stickers, to be issued free of charge by this Commission, shall
The validity of a letter of Instruction 1 providing for an early seaming device for motor vehicles is assailed
be attached to each EWD. The EWD. serial number shall be indicated on the registration certificate and
in this prohibition proceeding as being violative of the constitutional guarantee of due process and,
official receipt of payment of current registration fees of the motor vehicle concerned. All Orders,
insofar as the rules and regulations for its implementation are concerned, for transgressing the
Circulars, and Memoranda in conflict herewith are hereby superseded, This Order shall take effect
fundamental principle of non- delegation of legislative power. The Letter of Instruction is stigmatized by
immediately. 9 It was for immediate implementation by respondent Alfredo L. Juinio, as Minister of
petitioner who is possessed of the requisite standing, as being arbitrary and oppressive. A temporary
Public Works, transportation, and Communications. 10
restraining order as issued and respondents Romeo F. Edu, Land Transportation Commissioner Juan
Ponce Enrile, Minister of National Defense; Alfredo L. Juinio, Minister of Public Works, Transportation
and Communications; and Baltazar Aquino, Minister of Public Highways; were to answer. That they did in Petitioner, after setting forth that he "is the owner of a Volkswagen Beetle Car, Model 13035, already
a pleading submitted by Solicitor General Estelito P. Mendoza. 2 Impressed with a highly persuasive properly equipped when it came out from the assembly lines with blinking lights fore and aft, which
quality, it makes devoid clear that the imputation of a constitutional infirmity is devoid of justification could very well serve as an early warning device in case of the emergencies mentioned in Letter of
The Letter of Instruction on is a valid police power measure. Nor could the implementing rules and Instructions No. 229, as amended, as well as the implementing rules and regulations in Administrative
regulations issued by respondent Edu be considered as amounting to an exercise of legislative power. Order No. 1 issued by the land transportation Commission," 11 alleged that said Letter of Instruction No.
Accordingly, the petition must be dismissed. 229, as amended, "clearly violates the provisions and delegation of police power, [sic] * * *: " For him
they are "oppressive, unreasonable, arbitrary, confiscatory, nay unconstitutional and contrary to the
precepts of our compassionate New Society." 12 He contended that they are "infected with arbitrariness
The facts are undisputed. The assailed Letter of Instruction No. 229 of President Marcos, issued on
because it is harsh, cruel and unconscionable to the motoring public;" 13 are "one-sided, onerous and
December 2, 1974, reads in full: "[Whereas], statistics show that one of the major causes of fatal or
patently illegal and immoral because [they] will make manufacturers and dealers instant millionaires at
serious accidents in land transportation is the presence of disabled, stalled or parked motor vehicles
the expense of car owners who are compelled to buy a set of the so-called early warning device at the
along streets or highways without any appropriate early warning device to signal approaching motorists
rate of P 56.00 to P72.00 per set." 14 are unlawful and unconstitutional and contrary to the precepts of a
of their presence; [Whereas], the hazards posed by such obstructions to traffic have been recognized by
compassionate New Society [as being] compulsory and confiscatory on the part of the motorists who
international bodies concerned with traffic safety, the 1968 Vienna Convention on Road Signs and
could very well provide a practical alternative road safety device, or a better substitute to the specified
Signals and the United Nations Organization (U.N.); [Whereas], the said Vienna Convention which was
set of EWD's." 15 He therefore prayed for a judgment both the assailed Letters of Instructions and
ratified by the Philippine Government under P.D. No. 207, recommended the enactment of local
Memorandum Circular void and unconstitutional and for a restraining order in the meanwhile.
legislation for the installation of road safety signs and devices; [Now, therefore, I, Ferdinand E. Marcos],
President of the Philippines, in the interest of safety on all streets and highways, including expressways
or limited access roads, do hereby direct: 1. That all owners, users or drivers of motor vehicles shall have A resolution to this effect was handed down by this Court on October 19, 1978: "L-49112 (Leovillo C.
at all times in their motor vehicles at least one (1) pair of early warning device consisting of triangular, Agustin v. Hon. Romeo F. Edu, etc., et al.) Considering the allegations contained, the issues raised and
collapsible reflectorized plates in red and yellow colors at least 15 cms. at the base and 40 cms. at the the arguments adduced in the petition for prohibition with writ of p prohibitory and/or mandatory
injunction, the Court Resolved to (require) the respondents to file an answer thereto within ton (10) days flexible response to conditions and circumstances thus assuring the greatest benefits. In the language of
from notice and not to move to dismiss the petition. The Court further Resolved to [issue] a [temporary Justice Cardozo: 'Needs that were narrow or parochial in the past may be interwoven in the present with
restraining order] effective as of this date and continuing until otherwise ordered by this Court. 16 the well-being of the nation. What is critical or urgent changes with the time.' The police power is thus a
dynamic agency, suitably vague and far from precisely defined, rooted in the conception that men in
organizing the state and imposing upon its government limitations to safeguard constitutional rights did
Two motions for extension were filed by the Office of the Solicitor General and granted. Then on
not intend thereby to enable an individual citizen or a group of citizens to obstruct unreasonably the
November 15, 1978, he Answer for respondents was submitted. After admitting the factual allegations
enactment of such salutary measures calculated to communal peace, safety, good order, and welfare." 24
and stating that they lacked knowledge or information sufficient to form a belief as to petitioner owning
a Volkswagen Beetle car," they "specifically deny the allegations and stating they lacked knowledge or
information sufficient to form a belief as to petitioner owning a Volkswagen Beetle Car, 17 they 2. It was thus a heavy burden to be shouldered by petitioner, compounded by the fact that the particular
specifically deny the allegations in paragraphs X and XI (including its subparagraphs 1, 2, 3, 4) of Petition police power measure challenged was clearly intended to promote public safety. It would be a rare
to the effect that Letter of Instruction No. 229 as amended by Letters of Instructions Nos. 479 and 716 as occurrence indeed for this Court to invalidate a legislative or executive act of that character. None has
well as Land transportation Commission Administrative Order No. 1 and its Memorandum Circular No. 32 been called to our attention, an indication of its being non-existent. The latest decision in point, Edu v.
violates the constitutional provisions on due process of law, equal protection of law and undue Ericta, sustained the validity of the Reflector Law, 25 an enactment conceived with the same end in
delegation of police power, and that the same are likewise oppressive, arbitrary, confiscatory, one-sided, view. Calalang v. Williams found nothing objectionable in a statute, the purpose of which was: "To
onerous, immoral unreasonable and illegal the truth being that said allegations are without legal and promote safe transit upon, and. avoid obstruction on roads and streets designated as national roads * *
factual basis and for the reasons alleged in the Special and Affirmative Defenses of this Answer." 18 Unlike *. 26 As a matter of fact, the first law sought to be nullified after the effectivity of the 1935 Constitution,
petitioner who contented himself with a rhetorical recital of his litany of grievances and merely invoked the National Defense Act, 27 with petitioner failing in his quest, was likewise prompted by the imperative
the sacramental phrases of constitutional litigation, the Answer, in demonstrating that the assailed demands of public safety.
Letter of Instruction was a valid exercise of the police power and implementing rules and regulations of
respondent Edu not susceptible to the charge that there was unlawful delegation of legislative power,
3. The futility of petitioner's effort to nullify both the Letter of Instruction and the implementing rules
there was in the portion captioned Special and Affirmative Defenses, a citation of what respondents
and regulations becomes even more apparent considering his failure to lay the necessary factual
believed to be the authoritative decisions of this Tribunal calling for application. They are Calalang v.
foundation to rebut the presumption of validity. So it was held in Ermita-Malate Hotel and Motel
Williams, 19 Morfe v. Mutuc, 20 and Edu v. Ericta. 21 Reference was likewise made to the 1968 Vienna
Operators Association, Inc. v. City Mayor of Manila. 28 The rationale was clearly set forth in an excerpt
Conventions of the United Nations on road traffic, road signs, and signals, of which the Philippines was a
from a decision of Justice Branders of the American Supreme Court, quoted in the opinion: "The statute
signatory and which was duly ratified. 22 Solicitor General Mendoza took pains to refute in detail, in
here questioned deals with a subject clearly within the scope of the police power. We are asked to
language calm and dispassionate, the vigorous, at times intemperate, accusation of petitioner that the
declare it void on the ground that the specific method of regulation prescribed is unreasonable and
assailed Letter of Instruction and the implementing rules and regulations cannot survive the test of
hence deprives the plaintiff of due process of law. As underlying questions of fact may condition the
rigorous scrutiny. To repeat, its highly-persuasive quality cannot be denied.
constitutionality of legislation of this character, the presumption of constitutionality must prevail in the
absence of some factual foundation of record in overthrowing the statute. 29
This Court thus considered the petition submitted for decision, the issues being clearly joined. As noted
at the outset, it is far from meritorious and must be dismissed.
4. Nor did the Solicitor General as he very well could, rely solely on such rebutted presumption of
validity. As was pointed out in his Answer "The President certainly had in his possession the necessary
1. The Letter of Instruction in question was issued in the exercise of the police power. That is conceded statistical information and data at the time he issued said letter of instructions, and such factual
by petitioner and is the main reliance of respondents. It is the submission of the former, however, that foundation cannot be defeated by petitioner's naked assertion that early warning devices 'are not too
while embraced in such a category, it has offended against the due process and equal protection vital to the prevention of nighttime vehicular accidents' because allegedly only 390 or 1.5 per cent of the
safeguards of the Constitution, although the latter point was mentioned only in passing. The broad and supposed 26,000 motor vehicle accidents that in 1976 involved rear-end collisions (p. 12 of petition).
expansive scope of the police power which was originally Identified by Chief Justice Taney of the Petitioner's statistics is not backed up by demonstrable data on record. As aptly stated by this Honorable
American Supreme Court in an 1847 decision as "nothing more or less than the powers of government Court: Further: "It admits of no doubt therefore that there being a presumption of validity, the necessity
inherent in every sovereignty" 23 was stressed in the aforementioned case of Edu v. Ericta thus: "Justice for evidence to rebut it is unavoidable, unless the statute or ordinance is void on its face, which is not the
Laurel, in the first leading decision after the Constitution came into force, Calalang v. Williams, Identified case here"' * * *. But even as g the verity of petitioner's statistics, is that not reason enough to require
police power with state authority to enact legislation that may interfere with personal liberty or property the installation of early warning devices to prevent another 390 rear-end collisions that could mean the
in order to promote the general welfare. Persons and property could thus 'be subjected to all kinds of death of 390 or more Filipinos and the deaths that could likewise result from head-on or frontal
restraints and burdens in order to we the general comfort, health and prosperity of the state.' Shortly collisions with stalled vehicles?" 30 It is quite manifest then that the issuance of such Letter of Instruction
after independence in 1948, Primicias v. Fugosoreiterated the doctrine, such a competence being is encased in the armor of prior, careful study by the Executive Department. To set it aside for alleged
referred to as 'the power to prescribe regulations to promote the health, morals, peace, education, good repugnancy to the due process clause is to give sanction to conjectural claims that exceeded even the
order or safety, and general welfare of the people. The concept was set forth in negative terms by Justice broadest permissible limits of a pleader's well known penchant for exaggeration.
Malcolm in a pre-Commonwealth decision as 'that inherent and plenary power in the State which
enables it to prohibit all things hurtful to the comfort, safety and welfare of society. In that sense it could
5. The rather wild and fantastic nature of the charge of oppressiveness of this Letter of Instruction was
be hardly distinguishable as noted by this Court in Morfe v. Mutuc with the totality of legislative power.
exposed in the Answer of the Solicitor General thus: "Such early warning device requirement is not
It is in the above sense the greatest and most powerful at. tribute of government. It is, to quote Justice
an expensive redundancy, nor oppressive, for car owners whose cars are already equipped with 1)
Malcolm anew, 'the most essential, insistent, and at least table powers, I extending as Justice Holmes
blinking lights in the fore and aft of said motor vehicles,' 2) "battery-powered blinking lights inside motor
aptly pointed out 'to all the great public needs.' Its scope, ever-expanding to meet the exigencies of the
vehicles," 3) "built-in reflectorized tapes on front and rear bumpers of motor vehicles," or 4) "well-
times, even to anticipate the future where it could be done, provides enough room for an efficient and
lighted two (2) petroleum lamps (the Kinke) * * * because: Being universal among the signatory
countries to the said 1968 Vienna Conventions, and visible even under adverse conditions at a distance the taint of unlawful delegation, there must be a standard, which implies at the very least that the
of at least 400 meters, any motorist from this country or from any part of the world, who sees a legislature itself determines matters of principle and lays down fundamental policy. Otherwise, the
reflectorized rectangular early seaming device installed on the roads, highways or expressways, will charge of complete abdication may be hard to repel A standard thus defines legislative policy, marks its
conclude, without thinking, that somewhere along the travelled portion of that road, highway, or maps out its boundaries and specifies the public agency to apply it. It indicates the circumstances under
expressway, there is a motor vehicle which is stationary, stalled or disabled which obstructs or which the legislative command is to be effected. It is the criterion by which legislative purpose may be
endangers passing traffic. On the other hand, a motorist who sees any of the aforementioned other built carried out. Thereafter, the executive or administrative office designated may in pursuance of the above
in warning devices or the petroleum lamps will not immediately get adequate advance warning because guidelines promulgate supplemental rules and regulations. The standard may be either express or
he will still think what that blinking light is all about. Is it an emergency vehicle? Is it a law enforcement implied. If the former, the non-delegation objection is easily met. The standard though does not have to
car? Is it an ambulance? Such confusion or uncertainty in the mind of the motorist will thus increase, be spelled out specifically. It could be implied from the policy and purpose of the act considered as a
rather than decrease, the danger of collision. 31 whole. In the Reflector Law clearly, the legislative objective is public safety. What is sought to be
attained as in Calalang v. Williams is "safe transit upon the roads.' This is to adhere to the recognition
given expression by Justice Laurel in a decision announced not too long after the Constitution came into
6. Nor did the other extravagant assertions of constitutional deficiency go unrefuted in the Answer of the
force and effect that the principle of non-delegation "has been made to adapt itself to the complexities
Solicitor General "There is nothing in the questioned Letter of Instruction No. 229, as amended, or in
of modern governments, giving rise to the adoption, within certain limits, of the principle of
Administrative Order No. 1, which requires or compels motor vehicle owners to purchase the early
"subordinate legislation" not only in the United States and England but in practically all modern
warning device prescribed thereby. All that is required is for motor vehicle owners concerned like
governments.' He continued: 'Accordingly, with the growing complexity of modern life, the multiplication
petitioner, to equip their motor vehicles with a pair of this early warning device in question, procuring or
of the subjects of governmental regulation, and the increased difficulty of administering the laws, there
obtaining the same from whatever source. In fact, with a little of industry and practical ingenuity, motor
is a constantly growing tendency toward the delegation of greater powers by the legislature and toward
vehicle owners can even personally make or produce this early warning device so long as the same
the approval of the practice by the courts.' Consistency with the conceptual approach requires the
substantially conforms with the specifications laid down in said letter of instruction and administrative
reminder that what is delegated is authority non-legislative in character, the completeness of the statute
order. Accordingly the early warning device requirement can neither be oppressive, onerous, immoral,
when it leaves the hands of Congress being assumed." 34
nor confiscatory, much less does it make manufacturers and dealers of said devices 'instant millionaires
at the expense of car owners' as petitioner so sweepingly concludes * * *. Petitioner's fear that with the
early warning device requirement 'a more subtle racket may be committed by those called upon to 9. The conclusion reached by this Court that this petition must be dismissed is reinforced by this
enforce it * * * is an unfounded speculation. Besides, that unscrupulous officials may try to enforce said consideration. The petition itself quoted these two whereas clauses of the assailed Letter of Instruction:
requirement in an unreasonable manner or to an unreasonable degree, does not render the same illegal "[Whereas], the hazards posed by such obstructions to traffic have been recognized by international
or immoral where, as in the instant case, the challenged Letter of Instruction No. 229 and implementing bodies concerned with traffic safety, the 1968 Vienna Convention on Road Signs and Signals and the
order disclose none of the constitutional defects alleged against it. 32 United Nations Organization (U.N.); [Whereas], the said Vionna Convention, which was ratified by the
Philippine Government under P.D. No. 207, recommended the enactment of local legislation for the
installation of road safety signs and devices; * * * " 35 It cannot be disputed then that this Declaration of
7 It does appear clearly that petitioner's objection to this Letter of Instruction is not premised on lack of
Principle found in the Constitution possesses relevance: "The Philippines * * * adopts the generally
power, the justification for a finding of unconstitutionality, but on the pessimistic, not to say negative,
accepted principles of international law as part of the law of the land * * *." 36 The 1968 Vienna
view he entertains as to its wisdom. That approach, it put it at its mildest, is distinguished, if that is the
Convention on Road Signs and Signals is impressed with such a character. It is not for this country to
appropriate word, by its unorthodoxy. It bears repeating "that this Court, in the language of Justice
repudiate a commitment to which it had pledged its word. The concept of Pacta sunt servanda stands in
Laurel, 'does not pass upon questions of wisdom justice or expediency of legislation.' As expressed by
the way of such an attitude, which is, moreover, at war with the principle of international morality.
Justice Tuason: 'It is not the province of the courts to supervise legislation and keep it within the bounds
of propriety and common sense. That is primarily and exclusively a legislative concern.' There can be no
possible objection then to the observation of Justice Montemayor. 'As long as laws do not violate any 10. That is about all that needs be said. The rather court reference to equal protection did not even elicit
Constitutional provision, the Courts merely interpret and apply them regardless of whether or not they any attempt on the Part of Petitioner to substantiate in a manner clear, positive, and categorical why
are wise or salutary. For they, according to Justice Labrador, 'are not supposed to override legitimate such a casual observation should be taken seriously. In no case is there a more appropriate occasion for
policy and * * * never inquire into the wisdom of the law.' It is thus settled, to paraphrase Chief Justice insistence on what was referred to as "the general rule" in Santiago v. Far Eastern Broadcasting
Concepcion in Gonzales v. Commission on Elections, that only congressional power or competence, not Co., 37 namely, "that the constitutionality of a law wig not be considered unless the point is specially
the wisdom of the action taken, may be the basis for declaring a statute invalid. This is as it ought to be. pleaded, insisted upon, and adequately argued." 38 "Equal protection" is not a talismanic formula at the
The principle of separation of powers has in the main wisely allocated the respective authority of each mere invocation of which a party to a lawsuit can rightfully expect that success will crown his efforts. The
department and confined its jurisdiction to such a sphere. There would then be intrusion not allowable law is anything but that.
under the Constitution if on a matter left to the discretion of a coordinate branch, the judiciary would
substitute its own. If there be adherence to the rule of law, as there ought to be, the last offender should
WHEREFORE, this petition is dismissed. The restraining order is lifted. This decision is immediately
be courts of justice, to which rightly litigants submit their controversy precisely to maintain unimpaired
executory. No costs.
the supremacy of legal norms and prescriptions. The attack on the validity of the challenged provision
likewise insofar as there may be objections, even if valid and cogent on is wisdom cannot be sustained. 33

8. The alleged infringement of the fundamental principle of non-delegation of legislative power is equally
without any support well-settled legal doctrines. Had petitioner taken the trouble to acquaint himself
with authoritative pronouncements from this Tribunal, he would not have the temerity to make such an
assertion. An exempt from the aforecited decision of Edu v. Ericta sheds light on the matter: "To avoid
EN BANC 3. SITUATION:

Criminal incidents in Metro Manila have been perpetrated not only by ordinary criminals but also by
organized syndicates whose members include active and former police/military personnel whose training,
[G.R. No. 141284. August 15, 2000] skill, discipline and firepower prove well-above the present capability of the local police alone to
handle. The deployment of a joint PNP NCRPO-Philippine Marines in the conduct of police visibility patrol
in urban areas will reduce the incidence of crimes specially those perpetrated by active or former
police/military personnel.

INTEGRATED BAR OF THE PHILIPPINES, petitioner, vs. HON. RONALDO B. ZAMORA, GEN. PANFILO M.
4. MISSION:
LACSON, GEN. EDGAR B. AGLIPAY, and GEN. ANGELO REYES, respondents.

The PNP NCRPO will organize a provisional Task Force to conduct joint NCRPO-PM visibility patrols to
DECISION
keep Metro Manila streets crime-free, through a sustained street patrolling to minimize or eradicate all
KAPUNAN, J.: forms of high-profile crimes especially those perpetrated by organized crime syndicates whose members
include those that are well-trained, disciplined and well-armed active or former PNP/Military personnel.

At bar is a special civil action for certiorari and prohibition with prayer for issuance of a temporary
restraining order seeking to nullify on constitutional grounds the order of President Joseph Ejercito 5. CONCEPT IN JOINT VISIBILITY PATROL OPERATIONS:
Estrada commanding the deployment of the Philippine Marines (the Marines) to join the Philippine
National Police (the PNP) in visibility patrols around the metropolis. a. The visibility patrols shall be conducted jointly by the NCRPO [National Capital Regional Police Office]
and the Philippine Marines to curb criminality in Metro Manila and to preserve the internal security of the
In view of the alarming increase in violent crimes in Metro Manila, like robberies, kidnappings and state against insurgents and other serious threat to national security, although the primary responsibility
carnappings, the President, in a verbal directive, ordered the PNP and the Marines to conduct joint over Internal Security Operations still rests upon the AFP.
visibility patrols for the purpose of crime prevention and suppression. The Secretary of National Defense,
the Chief of Staff of the Armed Forces of the Philippines (the AFP), the Chief of the PNP and the Secretary
of the Interior and Local Government were tasked to execute and implement the said order. In b. The principle of integration of efforts shall be applied to eradicate all forms of high-profile crimes
compliance with the presidential mandate, the PNP Chief, through Police Chief Superintendent Edgar B. perpetrated by organized crime syndicates operating in Metro Manila. This concept requires the military
Aglipay, formulated Letter of Instruction 02/2000[1] (the LOI) which detailed the manner by which the and police to work cohesively and unify efforts to ensure a focused, effective and holistic approach in
joint visibility patrols, called Task Force Tulungan, would be conducted.[2] Task Force Tulungan was addressing crime prevention. Along this line, the role of the military and police aside from neutralizing
placed under the leadership of the Police Chief of Metro Manila. crime syndicates is to bring a wholesome atmosphere wherein delivery of basic services to the people and
development is achieved. Hand-in-hand with this joint NCRPO-Philippine Marines visibility patrols, local
Subsequently, the President confirmed his previous directive on the deployment of the Marines in Police Units are responsible for the maintenance of peace and order in their locality.
a Memorandum, dated 24 January 2000, addressed to the Chief of Staff of the AFP and the PNP
Chief.[3] In the Memorandum, the President expressed his desire to improve the peace and order
c. To ensure the effective implementation of this project, a provisional Task Force TULUNGAN shall be
situation in Metro Manila through a more effective crime prevention program including increased police
organized to provide the mechanism, structure, and procedures for the integrated planning, coordinating,
patrols.[4] The President further stated that to heighten police visibility in the metropolis, augmentation
monitoring and assessing the security situation.
from the AFP is necessary.[5]Invoking his powers as Commander-in-Chief under Section 18, Article VII of
the Constitution, the President directed the AFP Chief of Staff and PNP Chief to coordinate with each
other for the proper deployment and utilization of the Marines to assist the PNP in preventing or xxx.[8]
suppressing criminal or lawless violence.[6] Finally, the President declared that the services of the
Marines in the anti-crime campaign are merely temporary in nature and for a reasonable period only, The selected areas of deployment under the LOI are: Monumento Circle, North Edsa (SM City),
until such time when the situation shall have improved.[7] Araneta Shopping Center, Greenhills, SM Megamall, Makati Commercial Center, LRT/MRT Stations and
the NAIA and Domestic Airport.[9]
The LOI explains the concept of the PNP-Philippine Marines joint visibility patrols as follows:
On 17 January 2000, the Integrated Bar of the Philippines (the IBP) filed the instant petition to
xxx annul LOI 02/2000 and to declare the deployment of the Philippine Marines, null and void and
unconstitutional, arguing that:
2. PURPOSE: I

The Joint Implementing Police Visibility Patrols between the PNP NCRPO and the Philippine Marines THE DEPLOYMENT OF THE PHILIPPINE MARINES IN METRO MANILA IS VIOLATIVE OF THE CONSTITUTION,
partnership in the conduct of visibility patrols in Metro Manila for the suppression of crime prevention IN THAT:
and other serious threats to national security.
A) NO EMERGENCY SITUATION OBTAINS IN METRO MANILA AS WOULD JUSTIFY, EVEN ONLY REMOTELY, abuse of discretion amounting to lack or excess of jurisdiction on the part of any branch or
THE DEPLOYMENT OF SOLDIERS FOR LAW ENFORCEMENT WORK; HENCE, SAID DEPLOYMENT IS IN instrumentality of the Government.
DEROGATION OF ARTICLE II, SECTION 3 OF THE CONSTITUTION;
When questions of constitutional significance are raised, the Court can exercise its power of
B) SAID DEPLOYMENT CONSTITUTES AN INSIDIOUS INCURSION BY THE MILITARY IN A CIVILIAN judicial review only if the following requisites are complied with, namely: (1) the existence of an actual
FUNCTION OF GOVERNMENT (LAW ENFORCEMENT) IN DEROGATION OF ARTICLE XVI, SECTION 5 (4), OF and appropriate case; (2) a personal and substantial interest of the party raising the constitutional
THE CONSTITUTION; question; (3) the exercise of judicial review is pleaded at the earliest opportunity; and (4) the
constitutional question is the lis mota of the case.[12]
C) SAID DEPLOYMENT CREATES A DANGEROUS TENDENCY TO RELY ON THE MILITARY TO PERFORM THE
CIVILIAN FUNCTIONS OF THE GOVERNMENT.
The IBP has not sufficiently complied with the requisites of standing in this case.
II

IN MILITARIZING LAW ENFORCEMENT IN METRO MANILA, THE ADMINISTRATION IS UNWITTINGLY Legal standing or locus standi has been defined as a personal and substantial interest in the case
MAKING THE MILITARY MORE POWERFUL THAN WHAT IT SHOULD REALLY BE UNDER THE such that the party has sustained or will sustain direct injury as a result of the governmental act that is
CONSTITUTION.[10] being challenged.[13] The term interest means a material interest, an interest in issue affected by the
decree, as distinguished from mere interest in the question involved, or a mere incidental
interest.[14] The gist of the question of standing is whether a party alleges such personal stake in the
Asserting itself as the official organization of Filipino lawyers tasked with the bounden duty to
outcome of the controversy as to assure that concrete adverseness which sharpens the presentation of
uphold the rule of law and the Constitution, the IBP questions the validity of the deployment and
issues upon which the court depends for illumination of difficult constitutional questions.[15]
utilization of the Marines to assist the PNP in law enforcement.
In the case at bar, the IBP primarily anchors its standing on its alleged responsibility to uphold the
Without granting due course to the petition, the Court in a Resolution, [11] dated 25 January 2000,
rule of law and the Constitution. Apart from this declaration, however, the IBP asserts no other basis in
required the Solicitor General to file his Comment on the petition. On 8 February 2000, the Solicitor
support of its locus standi. The mere invocation by the IBP of its duty to preserve the rule of law and
General submitted his Comment.
nothing more, while undoubtedly true, is not sufficient to clothe it with standing in this case. This is too
The Solicitor General vigorously defends the constitutionality of the act of the President in general an interest which is shared by other groups and the whole citizenry. Based on the standards
deploying the Marines, contending, among others, that petitioner has no legal standing;that the above-stated, the IBP has failed to present a specific and substantial interest in the resolution of the
question of deployment of the Marines is not proper for judicial scrutiny since the same involves a case. Its fundamental purpose which, under Section 2, Rule 139-A of the Rules of Court, is to elevate the
political question; that the organization and conduct of police visibility patrols, which feature the team- standards of the law profession and to improve the administration of justice is alien to, and cannot be
up of one police officer and one Philippine Marine soldier, does not violate the civilian supremacy clause affected by the deployment of the Marines. It should also be noted that the interest of the National
in the Constitution. President of the IBP who signed the petition, is his alone, absent a formal board resolution authorizing
him to file the present action. To be sure, members of the BAR, those in the judiciary included, have
The issues raised in the present petition are: (1) Whether or not petitioner has legal standing; varying opinions on the issue. Moreover, the IBP, assuming that it has duly authorized the National
(2) Whether or not the Presidents factual determination of the necessity of calling the armed forces is President to file the petition, has not shown any specific injury which it has suffered or may suffer by
subject to judicial review; and, (3) Whether or not the calling of the armed forces to assist the PNP in virtue of the questioned governmental act. Indeed, none of its members, whom the IBP purportedly
joint visibility patrols violates the constitutional provisions on civilian supremacy over the military and represents, has sustained any form of injury as a result of the operation of the joint visibility
the civilian character of the PNP. patrols. Neither is it alleged that any of its members has been arrested or that their civil liberties have
been violated by the deployment of the Marines. What the IBP projects as injurious is the supposed
The petition has no merit. militarization of law enforcement which might threaten Philippine democratic institutions and may cause
more harm than good in the long run. Not only is the presumed injury not personal in character, it is
First, petitioner failed to sufficiently show that it is in possession of the requisites of standing to
likewise too vague, highly speculative and uncertain to satisfy the requirement of standing. Since
raise the issues in the petition. Second, the President did not commit grave abuse of discretion
petitioner has not successfully established a direct and personal injury as a consequence of the
amounting to lack or excess of jurisdiction nor did he commit a violation of the civilian supremacy clause
questioned act, it does not possess the personality to assail the validity of the deployment of the
of the Constitution.
Marines. This Court, however, does not categorically rule that the IBP has absolutely no standing to raise
The power of judicial review is set forth in Section 1, Article VIII of the Constitution, to wit: constitutional issues now or in the future. The IBP must, by way of allegations and proof, satisfy this
Court that it has sufficient stake to obtain judicial resolution of the controversy.

Section 1. The judicial power shall be vested in one Supreme Court and in such lower courts as may be Having stated the foregoing, it must be emphasized that this Court has the discretion to take
established by law. cognizance of a suit which does not satisfy the requirement of legal standing when paramount interest is
involved.[16] In not a few cases, the Court has adopted a liberal attitude on the locus standi of a petitioner
Judicial power includes the duty of the courts of justice to settle actual controversies involving rights where the petitioner is able to craft an issue of transcendental significance to the people.[17] Thus, when
which are legally demandable and enforceable, and to determine whether or not there has been grave the issues raised are of paramount importance to the public, the Court may brush aside technicalities of
procedure.[18] In this case, a reading of the petition shows that the IBP has advanced constitutional issues
which deserve the attention of this Court in view of their seriousness, novelty and weight as
precedents. Moreover, because peace and order are under constant threat and lawless violence occurs We now address the Solicitor Generals argument that the issue involved is not susceptible to
in increasing tempo, undoubtedly aggravated by the Mindanao insurgency problem, the legal review by the judiciary because it involves a political question, and thus, notjusticiable.
controversy raised in the petition almost certainly will not go away. It will stare us in the face again. It,
therefore, behooves the Court to relax the rules on standing and to resolve the issue now, rather than As a general proposition, a controversy is justiciable if it refers to a matter which is appropriate for
later. court review.[22] It pertains to issues which are inherently susceptible of being decided on grounds
recognized by law. Nevertheless, the Court does not automatically assume jurisdiction over actual
constitutional cases brought before it even in instances that are ripe for resolution. One class of cases
wherein the Court hesitates to rule on are political questions. The reason is that political questions are
The President did not commit grave abuse of discretion in calling out the Marines. concerned with issues dependent upon the wisdom, not the legality, of a particular act or measure being
assailed. Moreover, the political question being a function of the separation of powers, the courts will
not normally interfere with the workings of another co-equal branch unless the case shows a clear need
for the courts to step in to uphold the law and the Constitution.
In the case at bar, the bone of contention concerns the factual determination of the President of
the necessity of calling the armed forces, particularly the Marines, to aid the PNP in visibility patrols. In As Taada v. Cuenco[23] puts it, political questions refer to those questions which, under the
this regard, the IBP admits that the deployment of the military personnel falls under the Commander-in- Constitution, are to be decided by the people in their sovereign capacity, or in regard to which full
Chief powers of the President as stated in Section 18, Article VII of the Constitution, specifically, the discretionary authority has been delegated to the legislative or executive branch of government. Thus, if
power to call out the armed forces to prevent or suppress lawless violence, invasion or rebellion. What an issue is clearly identified by the text of the Constitution as matters for discretionary action by a
the IBP questions, however, is the basis for the calling of the Marines under the aforestated provision. particular branch of government or to the people themselves then it is held to be a political question. In
According to the IBP, no emergency exists that would justify the need for the calling of the military to the classic formulation of Justice Brennan in Baker v. Carr,[24] [p]rominent on the surface of any case held
assist the police force. It contends that no lawless violence, invasion or rebellion exist to warrant the to involve a political question is found a textually demonstrable constitutional commitment of the issue
calling of the Marines. Thus, the IBP prays that this Court review the sufficiency of the factual basis for to a coordinate political department; or a lack of judicially discoverable and manageable standards for
said troop [Marine] deployment.[19] resolving it; or the impossibility of deciding without an initial policy determination of a kind clearly for
nonjudicial discretion; or the impossibility of a courts undertaking independent resolution without
The Solicitor General, on the other hand, contends that the issue pertaining to the necessity of
expressing lack of the respect due coordinate branches of government; or an unusual need for
calling the armed forces is not proper for judicial scrutiny since it involves a political question and the
unquestioning adherence to a political decision already made; or the potentiality of embarassment from
resolution of factual issues which are beyond the review powers of this Court.
multifarious pronouncements by various departments on the one question.
As framed by the parties, the underlying issues are the scope of presidential powers and limits,
The 1987 Constitution expands the concept of judicial review by providing that (T)he Judicial
and the extent of judicial review. But, while this Court gives considerable weight to the parties
power shall be vested in one Supreme Court and in such lower courts as may be established by
formulation of the issues, the resolution of the controversy may warrant a creative approach that goes
law. Judicial power includes the duty of the courts of justice to settle actual controversies involving rights
beyond the narrow confines of the issues raised. Thus, while the parties are in agreement that the power
which are legally demandable and enforceable, and to determine whether or not there has been a grave
exercised by the President is the power to call out the armed forces, the Court is of the view that the
abuse of discretion amounting to lack or excess of jurisdiction on the part of any branch or
power involved may be no more than the maintenance of peace and order and promotion of the general
instrumentality of the Government.[25]Under this definition, the Court cannot agree with the Solicitor
welfare.[20] For one, the realities on the ground do not show that there exist a state of warfare,
General that the issue involved is a political question beyond the jurisdiction of this Court to
widespread civil unrest or anarchy. Secondly, the full brunt of the military is not brought upon the
review. When the grant of power is qualified, conditional or subject to limitations, the issue of whether
citizenry, a point discussed in the latter part of this decision. In the words of the late Justice Irene Cortes
the prescribed qualifications or conditions have been met or the limitations respected, is justiciable - the
inMarcos v. Manglapus:
problem being one of legality or validity, not its wisdom.[26] Moreover, the jurisdiction to delimit
constitutional boundaries has been given to this Court.[27] When political questions are involved, the
More particularly, this case calls for the exercise of the Presidents powers as protector of the Constitution limits the determination as to whether or not there has been a grave abuse of discretion
peace. [Rossiter, The American Presidency]. The power of the President to keep the peace is not limited amounting to lack or excess of jurisdiction on the part of the official whose action is being questioned.[28]
merely to exercising the commander-in-chief powers in times of emergency or to leading the State
against external and internal threats to its existence. The President is not only clothed with extraordinary By grave abuse of discretion is meant simply capricious or whimsical exercise of judgment that is
powers in times of emergency, but is also tasked with attending to the day-to-day problems of patent and gross as to amount to an evasion of positive duty or a virtual refusal to perform a duty
maintaining peace and order and ensuring domestic tranquility in times when no foreign foe appears on enjoined by law, or to act at all in contemplation of law, as where the power is exercised in an arbitrary
the horizon. Wide discretion, within the bounds of law, in fulfilling presidential duties in times of peace is and despotic manner by reason of passion or hostility.[29]Under this definition, a court is without power
not in any way diminished by the relative want of an emergency specified in the commander-in-chief to directly decide matters over which full discretionary authority has been delegated. But while this
provision. For in making the President commander-in-chief the enumeration of powers that follow Court has no power to substitute its judgment for that of Congress or of the President, it may look into
cannot be said to exclude the Presidents exercising as Commander-in-Chief powers short of the calling of the question of whether such exercise has been made in grave abuse of discretion.[30] A showing that
the armed forces, or suspending the privilege of the writ ofhabeas corpus or declaring martial law, in plenary power is granted either department of government, may not be an obstacle to judicial inquiry,
order to keep the peace, and maintain public order and security. for the improvident exercise or abuse thereof may give rise to justiciable controversy.[31]

When the President calls the armed forces to prevent or suppress lawless violence, invasion or
xxx[21] rebellion, he necessarily exercises a discretionary power solely vested in his wisdom. This is clear from
the intent of the framers and from the text of the Constitution itself. The Court, thus, cannot be called
Nonetheless, even if it is conceded that the power involved is the Presidents power to call out the
upon to overrule the Presidents wisdom or substitute its own. However, this does not prevent an
armed forces to prevent or suppress lawless violence, invasion or rebellion, the resolution of the
examination of whether such power was exercised within permissible constitutional limits or whether it
controversy will reach a similar result.
was exercised in a manner constituting grave abuse of discretion. In view of the constitutional intent to During the suspension of the privilege of the writ, any person thus arrested or detained shall be judicially
give the President full discretionary power to determine the necessity of calling out the armed forces, it charged within three days, otherwise he shall be released.
is incumbent upon the petitioner to show that the Presidents decision is totally bereft of factual
basis. The present petition fails to discharge such heavy burden as there is no evidence to support the
Under the foregoing provisions, Congress may revoke such proclamation or suspension and the
assertion that there exist no justification for calling out the armed forces. There is, likewise, no evidence
Court may review the sufficiency of the factual basis thereof. However, there is no such equivalent
to support the proposition that grave abuse was committed because the power to call was exercised in
provision dealing with the revocation or review of the Presidents action to call out the armed forces. The
such a manner as to violate the constitutional provision on civilian supremacy over the military. In the
distinction places the calling out power in a different category from the power to declare martial law and
performance of this Courts duty of purposeful hesitation[32] before declaring an act of another branch as
the power to suspend the privilege of the writ of habeas corpus, otherwise, the framers of the
unconstitutional, only where such grave abuse of discretion is clearly shown shall the Court interfere
Constitution would have simply lumped together the three powers and provided for their revocation and
with the Presidents judgment. To doubt is to sustain.
review without any qualification. Expressio unius est exclusio alterius. Where the terms are expressly
There is a clear textual commitment under the Constitution to bestow on the President full limited to certain matters, it may not, by interpretation or construction, be extended to other
discretionary power to call out the armed forces and to determine the necessity for the exercise of such matters.[33] That the intent of the Constitution is exactly what its letter says, i.e., that the power to call is
power. Section 18, Article VII of the Constitution, which embodies the powers of the President as fully discretionary to the President, is extant in the deliberation of the Constitutional Commission, to wit:
Commander-in-Chief, provides in part:
FR. BERNAS. It will not make any difference. I may add that there is a graduated power of the President
The President shall be the Commander-in-Chief of all armed forces of the Philippines and whenever it as Commander-in-Chief. First, he can call out such Armed Forces as may be necessary to suppress lawless
becomes necessary, he may call out such armed forces to prevent or suppress lawless violence, invasion violence; then he can suspend the privilege of the writ of habeas corpus, then he can impose martial
or rebellion. In case of invasion or rebellion, when the public safety requires it, he may, for a period not law. This is a graduated sequence.
exceeding sixty days, suspend the privilege of the writ of habeas corpus, or place the Philippines or any
part thereof under martial law. When he judges that it is necessary to impose martial law or suspend the privilege of the writ of habeas
corpus, his judgment is subject to review. We are making it subject to review by the Supreme Court and
xxx subject to concurrence by the National Assembly. But when he exercises this lesser power of calling on
the Armed Forces, when he says it is necessary, it is my opinion that his judgment cannot be reviewed by
The full discretionary power of the President to determine the factual basis for the exercise of the anybody.
calling out power is also implied and further reinforced in the rest of Section 18, Article VII which reads,
thus: xxx
xxx
FR. BERNAS. Let me just add that when we only have imminent danger, the matter can be handled by
Within forty-eight hours from the proclamation of martial law or the suspension of the privilege of the the first sentence: The President may call out such armed forces to prevent or suppress lawless violence,
writ of habeas corpus, the President shall submit a report in person or in writing to the Congress. The invasion or rebellion. So we feel that that is sufficient for handling imminent danger.
Congress, voting jointly, by a vote of at least a majority of all its Members in regular or special session,
may revoke such proclamation or suspension, which revocation shall not be set aside by the MR. DE LOS REYES. So actually, if a President feels that there is imminent danger, the matter can be
President. Upon the initiative of the President, the Congress may, in the same manner, extend such handled by the First Sentence: The President....may call out such Armed Forces to prevent or suppress
proclamation or suspension for a period to be determined by the Congress, if the invasion or rebellion lawless violence, invasion or rebellion. So we feel that that is sufficient for handling imminent danger, of
shall persist and public safety requires it. invasion or rebellion, instead of imposing martial law or suspending the writ of habeas corpus, he must
necessarily have to call the Armed Forces of the Philippines as their Commander-in-Chief. Is that the
The Congress, if not in session, shall within twenty-four hours following such proclamation or idea?
suspension, convene in accordance with its rules without need of a call.
MR. REGALADO. That does not require any concurrence by the legislature nor is it subject to judicial
The Supreme Court may review, in an appropriate proceeding filed by any citizen, the sufficiency of the review.[34]
factual basis of the proclamation of martial law or the suspension of the privilege of the writ or the
extension thereof, and must promulgate its decision thereon within thirty days from its filing. The reason for the difference in the treatment of the aforementioned powers highlights the intent
to grant the President the widest leeway and broadest discretion in using the power to call out because
A state of martial law does not suspend the operation of the Constitution, nor supplant the functioning it is considered as the lesser and more benign power compared to the power to suspend the privilege of
of the civil courts or legislative assemblies, nor authorize the conferment of jurisdiction on military the writ of habeas corpus and the power to impose martial law, both of which involve the curtailment
courts and agencies over civilians where civil courts are able to function, nor automatically suspend the and suppression of certain basic civil rights and individual freedoms, and thus necessitating safeguards
privilege of the writ. by Congress and review by this Court.

Moreover, under Section 18, Article VII of the Constitution, in the exercise of the power to
The suspension of the privilege of the writ shall apply only to persons judicially charged for rebellion or suspend the privilege of the writ of habeas corpus or to impose martial law, two conditions must concur:
offenses inherent in or directly connected with invasion. (1) there must be an actual invasion or rebellion and, (2) public safety must require it. These conditions
are not required in the case of the power to call out the armed forces. The only criterion is that
whenever it becomes necessary, the President may call the armed forces to prevent or suppress lawless belonging to the PNP. In fact, the Metro Manila Police Chief is the overall leader of the PNP-Philippine
violence, invasion or rebellion." The implication is that the President is given full discretion and wide Marines joint visibility patrols.[37] Under the LOI, the police forces are tasked to brief or orient the
latitude in the exercise of the power to call as compared to the two other powers. soldiers on police patrol procedures.[38] It is their responsibility to direct and manage the deployment of
the Marines.[39] It is, likewise, their duty to provide the necessary equipment to the Marines and render
If the petitioner fails, by way of proof, to support the assertion that the President acted without logistical support to these soldiers.[40] In view of the foregoing, it cannot be properly argued that military
factual basis, then this Court cannot undertake an independent investigation beyond the pleadings. The authority is supreme over civilian authority. Moreover, the deployment of the Marines to assist the PNP
factual necessity of calling out the armed forces is not easily quantifiable and cannot be objectively does not unmake the civilian character of the police force. Neither does it amount to an insidious
established since matters considered for satisfying the same is a combination of several factors which incursion of the military in the task of law enforcement in violation of Section 5(4), Article XVI of the
are not always accessible to the courts. Besides the absence of textual standards that the court may use Constitution.[41]
to judge necessity, information necessary to arrive at such judgment might also prove unmanageable for
the courts. Certain pertinent information might be difficult to verify, or wholly unavailable to the In this regard, it is not correct to say that General Angelo Reyes, Chief of Staff of the AFP, by his
courts. In many instances, the evidence upon which the President might decide that there is a need to alleged involvement in civilian law enforcement, has been virtually appointed to a civilian post in
call out the armed forces may be of a nature not constituting technical proof. derogation of the aforecited provision. The real authority in these operations, as stated in the LOI, is
lodged with the head of a civilian institution, the PNP, and not with the military. Such being the case, it
On the other hand, the President as Commander-in-Chief has a vast intelligence network to gather does not matter whether the AFP Chief actually participates in the Task Force Tulungan since he does
information, some of which may be classified as highly confidential or affecting the security of the not exercise any authority or control over the same. Since none of the Marines was incorporated or
state. In the exercise of the power to call, on-the-spot decisions may be imperatively necessary in enlisted as members of the PNP, there can be no appointment to civilian position to speak of. Hence, the
emergency situations to avert great loss of human lives and mass destruction of property. Indeed, the deployment of the Marines in the joint visibility patrols does not destroy the civilian character of the
decision to call out the military to prevent or suppress lawless violence must be done swiftly and PNP.
decisively if it were to have any effect at all. Such a scenario is not farfetched when we consider the
present situation in Mindanao, where the insurgency problem could spill over the other parts of the Considering the above circumstances, the Marines render nothing more than assistance required
country. The determination of the necessity for the calling out power if subjected to unfettered judicial in conducting the patrols. As such, there can be no insidious incursion of the military in civilian affairs nor
scrutiny could be a veritable prescription for disaster, as such power may be unduly straitjacketed by an can there be a violation of the civilian supremacy clause in the Constitution.
injunction or a temporary restraining order every time it is exercised.
It is worth mentioning that military assistance to civilian authorities in various forms persists in
Thus, it is the unclouded intent of the Constitution to vest upon the President, as Commander-in- Philippine jurisdiction. The Philippine experience reveals that it is not averse to requesting the assistance
Chief of the Armed Forces, full discretion to call forth the military when in his judgment it is necessary to of the military in the implementation and execution of certain traditionally civil functions. As correctly
do so in order to prevent or suppress lawless violence, invasion or rebellion. Unless the petitioner can pointed out by the Solicitor General, some of the multifarious activities wherein military aid has been
show that the exercise of such discretion was gravely abused, the Presidents exercise of judgment rendered, exemplifying the activities that bring both the civilian and the military together in a
deserves to be accorded respect from this Court. relationship of cooperation, are:

The President has already determined the necessity and factual basis for calling the armed 1. Elections;[42]
forces. In his Memorandum, he categorically asserted that, [V]iolent crimes like bank/store robberies,
holdups, kidnappings and carnappings continue to occur in Metro Manila...[35] We do not doubt the 2. Administration of the Philippine National Red Cross;[43]
veracity of the Presidents assessment of the situation, especially in the light of present
3. Relief and rescue operations during calamities and disasters;[44]
developments. The Court takes judicial notice of the recent bombings perpetrated by lawless elements in
the shopping malls, public utilities, and other public places. These are among the areas of deployment 4. Amateur sports promotion and development;[45]
described in the LOI 2000. Considering all these facts, we hold that the President has sufficient factual
basis to call for military aid in law enforcement and in the exercise of this constitutional power. 5. Development of the culture and the arts;[46]

6. Conservation of natural resources;[47]

7. Implementation of the agrarian reform program;[48]


The deployment of the Marines does not violate the civilian supremacy clause nor does it infringe the
civilian character of the police force. 8. Enforcement of customs laws;[49]

9. Composite civilian-military law enforcement activities;[50]


Prescinding from its argument that no emergency situation exists to justify the calling of the
10. Conduct of licensure examinations;[51]
Marines, the IBP asserts that by the deployment of the Marines, the civilian task of law enforcement is
militarized in violation of Section 3, Article II[36] of the Constitution. 11. Conduct of nationwide tests for elementary and high school students;[52]
We disagree. The deployment of the Marines does not constitute a breach of the civilian 12. Anti-drug enforcement activities;[53]
supremacy clause. The calling of the Marines in this case constitutes permissible use of military assets for
civilian law enforcement. The participation of the Marines in the conduct of joint visibility patrols is 13. Sanitary inspections;[54]
appropriately circumscribed. The limited participation of the Marines is evident in the provisions of the
LOI itself, which sufficiently provides the metes and bounds of the Marines authority. It is noteworthy 14. Conduct of census work;[55]
that the local police forces are the ones in charge of the visibility patrols at all times, the real authority
15. Administration of the Civil Aeronautics Board;[56]
16. Assistance in installation of weather forecasting devices;[57] regulatory, proscriptive or compulsory military power, the deployment of a handful of Philippine
Marines constitutes no impermissible use of military power for civilian law enforcement.[71]
17. Peace and order policy formulation in local government units.[58]

This unquestionably constitutes a gloss on executive power resulting from a systematic, unbroken, It appears that the present petition is anchored on fear that once the armed forces are deployed,
executive practice, long pursued to the knowledge of Congress and, yet, never before the military will gain ascendancy, and thus place in peril our cherished liberties.Such apprehensions,
questioned.[59] What we have here is mutual support and cooperation between the military and civilian however, are unfounded. The power to call the armed forces is just that - calling out the armed
authorities, not derogation of civilian supremacy. forces. Unless, petitioner IBP can show, which it has not, that in the deployment of the Marines, the
President has violated the fundamental law, exceeded his authority or jeopardized the civil liberties of
In the United States, where a long tradition of suspicion and hostility towards the use of military the people, this Court is not inclined to overrule the Presidents determination of the factual basis for the
force for domestic purposes has persisted,[60] and whose Constitution, unlike ours, does not expressly calling of the Marines to prevent or suppress lawless violence.
provide for the power to call, the use of military personnel by civilian law enforcement officers is allowed
under circumstances similar to those surrounding the present deployment of the Philippine One last point. Since the institution of the joint visibility patrol in January, 2000, not a single
Marines. Under the Posse Comitatus Act[61] of the US, the use of the military in civilian law enforcement citizen has complained that his political or civil rights have been violated as a result of the deployment of
is generally prohibited, except in certain allowable circumstances. A provision of the Act states: the Marines. It was precisely to safeguard peace, tranquility and the civil liberties of the people that the
joint visibility patrol was conceived. Freedom and democracy will be in full bloom only when people feel
secure in their homes and in the streets, not when the shadows of violence and anarchy constantly lurk
1385. Use of Army and Air Force as posse comitatus
in their midst.

Whoever, except in cases and under circumstances expressly authorized by the Constitution or Act of WHEREFORE, premises considered, the petition is hereby DISMISSED.
Congress, willfully uses any part of the Army or the Air Force as posse comitatus or otherwise to execute
SO ORDERED.
the laws shall be fined not more than $10,000 or imprisoned not more than two years, or both.[62]

To determine whether there is a violation of the Posse Comitatus Act in the use of military
personnel, the US courts[63] apply the following standards, to wit:

Were Army or Air Force personnel used by the civilian law enforcement officers at Wounded Knee in
such a manner that the military personnel subjected the citizens to the exercise of military power which
was regulatory, proscriptive, or compulsory[64] George Washington Law Review, pp. 404-433 (1986),
which discusses the four divergent standards for assessing acceptable involvement of military personnel
in civil law enforcement. See likewise HONORED IN THE BREECH: PRESIDENTIAL AUTHORITY TO EXECUTE
THE LAWS WITH MILITARY FORCE, 83 Yale Law Journal, pp. 130-152, 1973. 64 in nature, either presently
or prospectively?

xxx

When this concept is transplanted into the present legal context, we take it to mean that military
involvement, even when not expressly authorized by the Constitution or a statute, does not violate the
Posse Comitatus Act unless it actually regulates, forbids or compels some conduct on the part of those
claiming relief. A mere threat of some future injury would be insufficient. (emphasis supplied)

Even if the Court were to apply the above rigid standards to the present case to determine
whether there is permissible use of the military in civilian law enforcement, the conclusion is inevitable
that no violation of the civilian supremacy clause in the Constitution is committed. On this point, the
Court agrees with the observation of the Solicitor General:

3. The designation of tasks in Annex A[65] does not constitute the exercise of regulatory,
proscriptive, or compulsory military power. First, the soldiers do not control or direct the
operation. This is evident from Nos. 6,[66] 8(k)[67] and 9(a)[68] of Annex A. These soldiers, second,
also have no power to prohibit or condemn. In No. 9(d)[69] of Annex A, all arrested persons are
brought to the nearest police stations for proper disposition. And last, these soldiers apply no
coercive force. The materials or equipment issued to them, as shown in No. 8(c)[70] of Annex A, are
all low impact and defensive in character. The conclusion is that there being no exercise of
Republic of the Philippines The petitioners are high-ranking officers of the Armed Forces of the Philippines (AFP). Both petitioners,
SUPREME COURT Brigadier General Francisco Gudani (Gen. Gudani) and Lieutenant Colonel Alexander Balutan (Col.
Manila Balutan), belonged to the Philippine Marines. At the time of the subject incidents, both Gen. Gudani and
Col. Balutan were assigned to the Philippine Military Academy (PMA) in Baguio City, the former as the
PMA Assistant Superintendent, and the latter as the Assistant Commandant of Cadets.2
EN BANC

On 22 September 2005, Senator Rodolfo Biazon (Sen. Biazon) invited several senior officers of the AFP to
G.R. No. 170165 August 15, 2006
appear at a public hearing before the Senate Committee on National Defense and Security (Senate
Committee) scheduled on 28 September 2005. The hearing was scheduled after topics concerning the
B/GEN. (RET.) FRANCISCO V. GUDANI AND LT. COL. ALEXANDER F. BALUTAN Petitioners, conduct of the 2004 elections emerged in the public eye, particularly allegations of massive cheating and
vs. the surfacing of copies of an audio excerpt purportedly of a phone conversation between President
LT./GEN. GENEROSO S. SENGA CORONA, AS CHIEF OF STAFF OF THE CARPIO-MORALES, ARMED Gloria Macapagal Arroyo and an official of the Commission on Elections (COMELEC) widely reputed as
FORCES OF THE CALLEJO, SR., PHILIPPINES, COL. GILBERTO AZCUNA, JOSE C. ROA AS THE PRE-TRIAL then COMELEC Commissioner Virgilio Garcillano. At the time of the 2004 elections, Gen. Gudani had
TINGA, INVESTIGATING OFFICER, THE CHICO-NAZARIO, PROVOST MARSHALL GENERAL GARCIA, and OF been designated as commander, and Col. Balutan a member, of "Joint Task Force Ranao" by the AFP
THE ARMED FORCES OF THE PHILIPPINES AND THE GENERAL COURT-MARTIAL, Respondents. Southern Command. "Joint Task Force Ranao" was tasked with the maintenance of peace and order
during the 2004 elections in the provinces of Lanao del Norte and Lanao del Sur.3 `
DECISION
Gen. Gudani, Col. Balutan, and AFP Chief of Staff Lieutenant General Generoso Senga (Gen. Senga) were
TINGA, J.: among the several AFP officers who received a letter invitation from Sen. Biazon to attend the 28
September 2005 hearing. On 23 September 2005, Gen. Senga replied through a letter to Sen. Biazon that
he would be unable to attend the hearing due to a previous commitment in Brunei, but he nonetheless
A most dangerous general proposition is foisted on the Court that soldiers who defy orders of their "directed other officers from the AFP who were invited to attend the hearing."4
superior officers are exempt

On 26 September 2005, the Office of the Chief of Staff of the AFP issued a Memorandum addressed to
from the strictures of military law and discipline if such defiance is predicated on an act otherwise valid the Superintendent of the PMA Gen. Cristolito P. Baloing (Gen. Baloing). It was signed by Lt. Col.
under civilian law. Obedience and deference to the military chain of command and the President as Hernando DCA Iriberri in behalf of Gen. Senga.5 Noting that Gen. Gudani and Col. Balutan had been
commander-in-chief are the cornerstones of a professional military in the firm cusp of civilian control. invited to attend the Senate Committee hearing on 28 September 2005, the Memorandum directed the
These values of obedience and deference expected of military officers are content-neutral, beyond the two officers to attend the hearing.6Conformably, Gen. Gudani and Col. Balutan filed their respective
sway of the officers own sense of what is prudent or rash, or more elementally, of right or wrong. A self- requests for travel authority addressed to the PMA Superintendent.
righteous military invites itself as the scoundrels activist solution to the "ills" of participatory democracy.

On 27 September 2005, Gen. Senga wrote a letter to Sen. Biazon, requesting the postponement of the
Petitioners seek the annulment of a directive from President Gloria Macapagal-Arroyo1 enjoining them hearing scheduled for the following day, since the AFP Chief of Staff was himself unable to attend said
and other military officers from testifying before Congress without the Presidents consent. Petitioners hearing, and that some of the invited officers also could not attend as they were "attending to other
also pray for injunctive relief against a pending preliminary investigation against them, in preparation for urgent operational matters." By this time, both Gen. Gudani and Col. Balutan had already departed
possible court-martial proceedings, initiated within the military justice system in connection with Baguio for Manila to attend the hearing.
petitioners violation of the aforementioned directive.

Then on the evening of 27 September 2005, at around 10:10 p.m., a message was transmitted to the
The Court is cognizant that petitioners, in their defense, invoke weighty constitutional principles that PMA Superintendent from the office of Gen. Senga, stating as follows:
center on fundamental freedoms enshrined in the Bill of Rights. Although these concerns will not be

PER INSTRUCTION OF HER EXCELLENCY PGMA, NO AFP PERSONNEL SHALL APPEAR BEFORE
addressed to the satisfaction of petitioners, the Court recognizes these values as of paramount ANY CONGRESSIONAL OR SENATE HEARING WITHOUT HER APPROVAL. INFORM BGEN
importance to our civil society, even if not determinative of the resolution of this petition. Had the FRANCISCO F GUDANI AFP AND LTC ALEXANDER BALUTAN PA (GSC) ACCORDINGLY.7
relevant issue before us been the right of the Senate to compel the testimony of petitioners, the
constitutional questions raised by them would have come to fore. Such a scenario could have very well
been presented to the Court in such manner, without the petitioners having had to violate a direct order The following day, Gen. Senga sent another letter to Sen. Biazon, this time informing the senator that
from their commanding officer. Instead, the Court has to resolve whether petitioners may be subjected "no approval has been granted by the President to any AFP officer to appear" before the hearing
to military discipline on account of their defiance of a direct order of the AFP Chief of Staff. scheduled on that day. Nonetheless, both Gen. Gudani and Col. Balutan were present as the hearing
started, and they both testified as to the conduct of the 2004 elections.

The solicited writs of certiorari and prohibition do not avail; the petition must be denied.
The Office of the Solicitor General (OSG), representing the respondents before this Court, has offered
additional information surrounding the testimony of Gen. Gudani and Col. Balutan. The OSG manifests
I. that the couriers of the AFP Command Center had attempted to deliver the radio message to Gen.
Gudanis residence in a subdivision in Paraaque City late in the night of 27 September 2005, but they legislation. They also equate the "gag order" with culpable violation of the Constitution, particularly in
were not permitted entry by the subdivision guards. The next day, 28 September 2005, shortly before relation to the publics constitutional right to information and transparency in matters of public concern.
the start of the hearing, a copy of Gen. Sengas letter to Sen. Biazon sent earlier that day was handed at Plaintively, petitioners claim that "the Filipino people have every right to hear the [petitioners]
the Senate by Commodore Amable B. Tolentino of the AFP Office for Legislative Affairs to Gen. Gudani, testimonies," and even if the "gag order" were unconstitutional, it still was tantamount to "the crime of
who replied that he already had a copy. Further, Gen. Senga called Commodore Tolentino on the latters obstruction of justice." Petitioners further argue that there was no law prohibiting them from testifying
cell phone and asked to talk to Gen. Gudani, but Gen. Gudani refused. In response, Gen. Senga before the Senate, and in fact, they were appearing in obeisance to the authority of Congress to conduct
instructed Commodore Tolentino to inform Gen. Gudani that "it was an order," yet Gen. Gudani still inquiries in aid of legislation.
refused to take Gen. Sengas call.8
Finally, it is stressed in the petition that Gen. Gudani was no longer subject to military jurisdiction on
A few hours after Gen. Gudani and Col. Balutan had concluded their testimony, the office of Gen. Senga account of his compulsory retirement on 4 October 2005. It is pointed out that Article 2, Title I of the
issued a statement which noted that the two had appeared before the Senate Committee "in spite of the Articles of War defines persons subject to military law as "all officers and soldiers in the active service" of
fact that a guidance has been given that a Presidential approval should be sought prior to such an the AFP.
appearance;" that such directive was "in keeping with the time[-]honored principle of the Chain of
Command;" and that the two officers "disobeyed a legal order, in violation of A[rticles of] W[ar] 65
II.
(Willfully Disobeying Superior Officer), hence they will be subjected to General Court Martial proceedings
x x x" Both Gen. Gudani and Col. Balutan were likewise relieved of their assignments then.9
We first proceed to define the proper litigable issues. Notably, the guilt or innocence of petitioners in
violating Articles 65 and 97 of the Articles of War is not an issue before this Court, especially considering
On the very day of the hearing, 28 September 2005, President Gloria-Macapagal-Arroyo issued Executive
that per records, petitioners have not yet been subjected to court martial proceedings. Owing to the
Order No. 464 (E.O. 464). The OSG notes that the E.O. "enjoined officials of the executive department
absence of such proceedings, the correct inquiry should be limited to whether respondents could
including the military establishment from appearing in any legislative inquiry without her
properly initiate such proceedings preparatory to a formal court-martial, such as the aforementioned
approval."10 This Court subsequently ruled on the constitutionality of the said executive order in Senate
preliminary investigation, on the basis of petitioners acts surrounding their testimony before the Senate
v. Ermita.11 The relevance of E.O. 464 andSenate to the present petition shall be discussed forthwith.
on 28 September 2005. Yet this Court, consistent with the principle that it is not a trier of facts at first
instance,21 is averse to making any authoritative findings of fact, for that function is first for the court-
In the meantime, on 30 September 2005, petitioners were directed by General Senga, through Col. martial court to fulfill.
Henry A. Galarpe of the AFP Provost Marshal General, to appear before the Office of the Provost Marshal
General (OPMG) on 3 October 2005 for investigation. During their appearance before Col. Galarpe, both
Thus, we limit ourselves to those facts that are not controverted before the Court, having been
petitioners invoked their right to remain silent.12 The following day, Gen. Gudani was compulsorily
commonly alleged by petitioners and the OSG (for respondents). Petitioners were called by the Senate
retired from military service, having reached the age of 56.13
Committee to testify in its 28 September 2005 hearing. Petitioners attended such hearing and testified
before the Committee, despite the fact that the day before, there was an order from Gen. Senga (which
In an Investigation Report dated 6 October 2005, the OPMG recommended that petitioners be charged in turn was sourced "per instruction" from President Arroyo) prohibiting them from testifying without
with violation of Article of War 65, on willfully disobeying a superior officer, in relation to Article of War the prior approval of the President. Petitioners do not precisely admit before this Court that they had
97, on conduct prejudicial to the good order and military discipline.14 As recommended, the case was learned of such order prior to their testimony, although the OSG asserts that at the very least, Gen.
referred to a Pre-Trial Investigation Officer (PTIO) preparatory to trial by the General Court Martial Gudani already knew of such order before he testified.22 Yet while this fact may be ultimately material in
(GCM).15 Consequently, on 24 October 2005, petitioners were separately served with Orders respectively the court-martial proceedings, it is not determinative of this petition, which as stated earlier, does not
addressed to them and signed by respondent Col. Gilbert Jose C. Roa, the Pre-Trial Investigating Officer proffer as an issue whether petitioners are guilty of violating the Articles of War.
of the PTIO. The Orders directed petitioners to appear in person before Col. Roa at the Pre-Trial
Investigation of the Charges for violation of Articles 6516 and 9717 of Commonwealth Act No. 408,18 and
What the Court has to consider though is whether the violation of the aforementioned order of Gen.
to submit their counter-affidavits and affidavits of witnesses at the Office of the Judge Advocate
Senga, which emanated from the President, could lead to any investigation for court-martial of
General.19 The Orders were accompanied by respective charge sheets against petitioners, accusing them
petitioners. It has to be acknowledged as a general principle23 that AFP personnel of whatever rank are
of violating Articles of War 65 and 97.
liable under military law for violating a direct order of an officer superior in rank. Whether petitioners did
violate such an order is not for the Court to decide, but it will be necessary to assume, for the purposes
It was from these premises that the present petition for certiorari and prohibition was filed, particularly of this petition, that petitioners did so.
seeking that (1) the order of President Arroyo coursed through Gen. Senga preventing petitioners from
testifying before Congress without her prior approval be declared unconstitutional; (2) the charges
III.
stated in the charge sheets against petitioners be quashed; and (3) Gen. Senga, Col. Galarpe, Col. Roa,
and their successors-in-interest or persons acting for and on their behalf or orders, be permanently
enjoined from proceeding against petitioners, as a consequence of their having testified before the Preliminarily, we must discuss the effect of E.O. 464 and the Courts ruling in Senate on the present
Senate on 28 September 2005.20 petition.Notably, it is not alleged that petitioners were in any way called to task for violating E.O. 464,
but instead, they were charged for violating the direct order of Gen. Senga not to appear before the
Senate Committee, an order that stands independent of the executive order. Distinctions are called for,
Petitioners characterize the directive from President Arroyo requiring her prior approval before any AFP
since Section 2(b) of E.O. 464 listed "generals and flag officers of the Armed Forces of the Philippines and
personnel appear before Congress as a "gag order," which violates the principle of separation of powers
such other officers who in the judgment of the Chief of Staff are covered by the executive privilege," as
in government as it interferes with the investigation of the Senate Committee conducted in aid of
among those public officials required in Section 3 of E.O. 464 "to secure prior consent of the President
prior to appearing before either House of Congress." The Court in Senate declared both Section 2(b) and interests of discipline clearly forbid that the offender should go unpunished. It is held therefore that if
Section 3 void,24 and the impression may have been left following Senate that it settled as doctrine, that before the day on which his service legally terminates and his right to a discharge is complete,
the President is prohibited from requiring military personnel from attending congressional hearings proceedings with a view to trial are commenced against him as by arrest or the service of charges,
without having first secured prior presidential consent. That impression is wrong. the military jurisdiction will fully attach and once attached may be continued by a trial by court-
martial ordered and held after the end of the term of the enlistment of the accused x x x 29
Senate turned on the nature of executive privilege, a presidential prerogative which is encumbered by
significant limitations. Insofar as E.O. 464 compelled officials of the executive branch to seek prior Thus, military jurisdiction has fully attached to Gen. Gudani inasmuch as both the acts complained of and
presidential approval before appearing before Congress, the notion of executive control also comes into the initiation of the proceedings against him occurred before he compulsorily retired on 4 October 2005.
consideration.25 However, the ability of the President to require a military official to secure prior consent We see no reason to unsettle the Abadilla doctrine. The OSG also points out that under Section 28 of
before appearing before Congress pertains to a wholly different and independent specie of presidential Presidential Decree No. 1638, as amended, "[a]n officer or enlisted man carried in the retired list [of the
authoritythe commander-in-chief powers of the President. By tradition and jurisprudence, the Armed Forces of the Philippines] shall be subject to the Articles of War x x x" 30 To this citation,
commander-in-chief powers of the President are not encumbered by the same degree of restriction as petitioners do not offer any response, and in fact have excluded the matter of Gen. Gudanis retirement
that which may attach to executive privilege or executive control. as an issue in their subsequent memorandum.

During the deliberations in Senate, the Court was very well aware of the pendency of this petition as well IV.
as the issues raised herein. The decision in Senate was rendered with the comfort that the nullification of
portions of E.O. 464 would bear no impact on the present petition since petitioners herein were not
We now turn to the central issues.
called to task for violating the executive order. Moreover, the Court was then cognizant that Senate and
this case would ultimately hinge on disparate legal issues. Relevantly, Senate purposely did not touch
upon or rule on the faculty of the President, under the aegis of the commander-in-chief powers26 to Petitioners wish to see annulled the "gag order" that required them to secure presidential consent prior
require military officials from securing prior consent before appearing before Congress. The pertinent to their appearance before the Senate, claiming that it violates the constitutional right to information
factors in considering that question are markedly outside of those which did become relevant in and transparency in matters of public concern; or if not, is tantamount at least to the criminal acts of
adjudicating the issues raised in Senate. It is in this petition that those factors come into play. obstruction of justice and grave coercion. However, the proper perspective from which to consider this
issue entails the examination of the basis and authority of the President to issue such an order in the first
place to members of the AFP and the determination of whether such an order is subject to any
At this point, we wish to dispose of another peripheral issue before we strike at the heart of the matter.
limitations.
General Gudani argues that he can no longer fall within the jurisdiction of the court-martial, considering
his retirement last 4 October 2005. He cites Article 2, Title I of Commonwealth Act No. 408, which
defines persons subject to military law as, among others, "all officers and soldiers in the active service of The vitality of the tenet that the President is the commander-in-chief of the Armed Forces is most crucial
the [AFP]," and points out that he is no longer in the active service. to the democratic way of life, to civilian supremacy over the military, and to the general stability of our
representative system of government. The Constitution reposes final authority, control and supervision
of the AFP to the President, a civilian who is not a member of the armed forces, and whose duties as
This point was settled against Gen. Gudanis position in Abadilla v. Ramos,27 where the Court declared
commander-in-chief represent only a part of the organic duties imposed upon the office, the other
that an officer whose name was dropped from the roll of officers cannot be considered to be outside the
functions being clearly civil in nature.31 Civilian supremacy over the military also countermands the
jurisdiction of military authorities when military justice proceedings were initiated against him before
notion that the military may bypass civilian authorities, such as civil courts, on matters such as
the termination of his service. Once jurisdiction has been acquired over the officer, it continues until his
conducting warrantless searches and seizures.32
case is terminated. Thus, the Court held:

Pursuant to the maintenance of civilian supremacy over the military, the Constitution has allocated
The military authorities had jurisdiction over the person of Colonel Abadilla at the time of the alleged
specific roles to the legislative and executive branches of government in relation to military affairs.
offenses. This jurisdiction having been vested in the military authorities, it is retained up to the end of
Military appropriations, as with all other appropriations, are determined by Congress, as is the power to
the proceedings against Colonel Abadilla. Well-settled is the rule that jurisdiction once acquired is not
declare the existence of a state of war.33Congress is also empowered to revoke a proclamation of martial
lost upon the instance of the parties but continues until the case is terminated.28
law or the suspension of the writ of habeas corpus.34 The approval of the Commission on Appointments
is also required before the President can promote military officers from the rank of colonel or naval
Citing Colonel Winthrops treatise on Military Law, the Court further stated: captain.35 Otherwise, on the particulars of civilian dominance and administration over the military, the
Constitution is silent, except for the commander-in-chief clause which is fertile in meaning and
We have gone through the treatise of Colonel Winthrop and We find the following passage which goes
against the contention of the petitioners, viz implication as to whatever inherent martial authority the President may possess.36

3. Offenders in general Attaching of jurisdiction. It has further been held, and is now settled law, in The commander-in-chief provision in the Constitution is denominated as Section 18, Article VII, which
regard to military offenders in general, that if the military jurisdiction has once duly attached to them begins with the simple declaration that "[t]he President shall be the Commander-in-Chief of all armed
previous to the date of the termination of their legal period of service, they may be brought to trial by forces of the Philippines x x x"37 Outside explicit constitutional limitations, such as those found in Section
court-martial after that date, their discharge being meanwhile withheld. This principle has mostly been 5, Article XVI, the commander-in-chief clause vests on the President, as commander-in-chief, absolute
applied to cases where the offense was committed just prior to the end of the term. In such cases the authority over the persons and actions of the members of the armed forces. Such authority includes the
ability of the President to restrict the travel, movement and speech of military officers, activities which Further traditional restrictions on members of the armed forces are those imposed on free speech and
may otherwise be sanctioned under civilian law. mobility.1wphi1Kapunan is ample precedent in justifying that a soldier may be restrained by a superior
officer from speaking out on certain matters. As a general rule, the discretion of a military officer to
restrain the speech of a soldier under his/her command will be accorded deference, with minimal regard
Reference to Kapunan, Jr. v. De Villa38 is useful in this regard. Lt. Col. Kapunan was ordered confined
if at all to the reason for such restraint. It is integral to military discipline that the soldiers speech be
under "house arrest" by then Chief of Staff (later President) Gen. Fidel Ramos. Kapunan was also
with the consent and approval of the military commander.
ordered, as a condition for his house arrest, that he may not issue any press statements or give any press
conference during his period of detention. The Court unanimously upheld such restrictions, noting:
The necessity of upholding the ability to restrain speech becomes even more imperative if the soldier
desires to speak freely on political matters. The Constitution requires that "[t]he armed forces shall be
[T]he Court is of the view that such is justified by the requirements of military discipline. It cannot be
insulated from partisan politics," and that [n]o member of the military shall engage directly or indirectly
gainsaid that certain liberties of persons in the military service, including the freedom of speech, may
in any partisan political activity, except to vote."47 Certainly, no constitutional provision or military
be circumscribed by rules of military discipline. Thus, to a certain degree, individual rights may be
indoctrination will eliminate a soldiers ability to form a personal political opinion, yet it is vital that such
curtailed, because the effectiveness of the military in fulfilling its duties under the law depends to a
opinions be kept out of the public eye. For one, political belief is a potential source of discord among
large extent on the maintenance of discipline within its ranks. Hence, lawful orders must be followed
people, and a military torn by political strife is incapable of fulfilling its constitutional function as
without question and rules must be faithfully complied with, irrespective of a soldier's personal views
protectors of the people and of the State. For another, it is ruinous to military discipline to foment an
on the matter. It is from this viewpoint that the restrictions imposed on petitioner Kapunan, an officer in
atmosphere that promotes an active dislike of or dissent against the President, the commander-in-chief
the AFP, have to be considered.39
of the armed forces. Soldiers are constitutionally obliged to obey a President they may dislike or distrust.
This fundamental principle averts the country from going the way of banana republics.
Any good soldier, or indeed any ROTC cadet, can attest to the fact that the military way of life
circumscribes several of the cherished freedoms of civilian life. It is part and parcel of the military
Parenthetically, it must be said that the Court is well aware that our countrys recent past is marked by
package. Those who cannot abide by these limitations normally do not pursue a military career and
regime changes wherein active military dissent from the chain of command formed a key, though not
instead find satisfaction in other fields; and in fact many of those discharged from the service are
exclusive, element. The Court is not blind to history, yet it is a judge not of history but of the
inspired in their later careers precisely by their rebellion against the regimentation of military life.
Constitution. The Constitution, and indeed our modern democratic order, frown in no uncertain terms
Inability or unwillingness to cope with military discipline is not a stain on character, for the military mode
on a politicized military, informed as they are on the trauma of absolute martial rule. Our history might
is a highly idiosyncratic path which persons are not generally conscripted into, but volunteer themselves
imply that a political military is part of the natural order, but this view cannot be affirmed by the legal
to be part of. But for those who do make the choice to be a soldier, significant concessions to personal
order. The evolutionary path of our young democracy necessitates a reorientation from this view, reliant
freedoms are expected. After all, if need be, the men and women of the armed forces may be
as our socio-political culture has become on it. At the same time, evolution mandates a similar demand
commanded upon to die for country, even against their personal inclinations.
that our system of governance be more responsive to the needs and aspirations of the citizenry, so as to
avoid an environment vulnerable to a military apparatus able at will to exert an undue influence in our
It may be so that military culture is a remnant of a less democratic era, yet it has been fully integrated polity.
into the democratic system of governance. The constitutional role of the armed forces is as protector of
the people and of the State.40 Towards this end, the military must insist upon a respect for duty and a
Of possibly less gravitas, but of equal importance, is the principle that mobility of travel is another
discipline without counterpart in civilian life.41 The laws and traditions governing that discipline have a
necessary restriction on members of the military. A soldier cannot leave his/her post without the
long history; but they are founded on unique military exigencies as powerful now as in the past.42 In the
consent of the commanding officer. The reasons are self-evident. The commanding officer has to be
end, it must be borne in mind that the armed forces has a distinct subculture with unique needs, a
aware at all times of the location of the troops under command, so as to be able to appropriately
specialized society separate from civilian society. 43 In the elegant prose of the eminent British military
respond to any exigencies. For the same reason, commanding officers have to be able to restrict the
historian, John Keegan:
movement or travel of their soldiers, if in their judgment, their presence at place of call of duty is
necessary. At times, this may lead to unsentimental, painful consequences, such as a soldier being
[Warriors who fight wars have] values and skills [which] are not those of politicians and diplomats. They denied permission to witness the birth of his first-born, or to attend the funeral of a parent. Yet again,
are those of a world apart, a very ancient world, which exists in parallel with the everyday world but military life calls for considerable personal sacrifices during the period of conscription, wherein the
does not belong to it. Both worlds change over time, and the warrior world adopts in step to the civilian. higher duty is not to self but to country.
It follows it, however, at a distance. The distance can never be closed, for the culture of the warrior can
never be that of civilization itself.44
Indeed, the military practice is to require a soldier to obtain permission from the commanding officer
before he/she may leave his destination. A soldier who goes from the properly appointed place of duty
Critical to military discipline is obeisance to the military chain of command. Willful disobedience of a or absents from his/her command, guard, quarters, station, or camp without proper leave is subject to
superior officer is punishable by court-martial under Article 65 of the Articles of War.45 "An individual punishment by court-martial.48 It is even clear from the record that petitioners had actually requested
soldier is not free to ignore the lawful orders or duties assigned by his immediate superiors. For there for travel authority from the PMA in Baguio City to Manila, to attend the Senate Hearing. 49 Even
would be an end of all discipline if the seaman and marines on board a ship of war [or soldiers deployed petitioners are well aware that it was necessary for them to obtain permission from their superiors
in the field], on a distant service, were permitted to act upon their own opinion of their rights [or their before they could travel to Manila to attend the Senate Hearing.
opinion of the
It is clear that the basic position of petitioners impinges on these fundamental principles we have
Presidents intent], and to throw off the authority of the commander whenever they supposed it to be discussed. They seek to be exempted from military justice for having traveled to the Senate to testify
unlawfully exercised."46 before the Senate Committee against the express orders of Gen. Senga, the AFP Chief of Staff. If
petitioners position is affirmed, a considerable exception would be carved from the unimpeachable right pass in this petition, since petitioners testified anyway despite the presidential prohibition. Yet the Court
of military officers to restrict the speech and movement of their juniors. The ruinous consequences to is aware that with its pronouncement today that the President has the right to require prior consent
the chain of command and military discipline simply cannot warrant the Courts imprimatur on from members of the armed forces, the clash may soon loom or actualize.
petitioners position.
We believe and hold that our constitutional and legal order sanctions a modality by which members of
V. the military may be compelled to attend legislative inquiries even if the President desires otherwise, a
modality which does not offend the Chief Executives prerogatives as commander-in-chief. The remedy
lies with the courts.
Still, it would be highly myopic on our part to resolve the issue solely on generalities surrounding military
discipline. After all, petitioners seek to impress on us that their acts are justified as they were responding
to an invitation from the Philippine Senate, a component of the legislative branch of government. At the The fact that the executive branch is an equal, coordinate branch of government to the legislative
same time, the order for them not to testify ultimately came from the President, the head of the creates a wrinkle to any basic rule that persons summoned to testify before Congress must do so. There
executive branch of government and the commander-in-chief of the armed forces. is considerable interplay between the legislative and executive branches, informed by due deference and
respect as to their various constitutional functions. Reciprocal courtesy idealizes this relationship; hence,
it is only as a last resort that one branch seeks to compel the other to a particular mode of behavior. The
Thus, we have to consider the question: may the President prevent a member of the armed forces from
judiciary, the third coordinate branch of government, does not enjoy a similar dynamic with either the
testifying before a legislative inquiry? We hold that the President has constitutional authority to do so,
legislative or executive branches. Whatever weakness inheres on judicial power due to its inability to
by virtue of her power as commander-in-chief, and that as a consequence a military officer who defies
originate national policies and legislation, such is balanced by the fact that it is the branch empowered
such injunction is liable under military justice. At the same time, we also hold that any chamber of
by the Constitution to compel obeisance to its rulings by the other branches of government.
Congress which seeks the appearance before it of a military officer against the consent of the President
has adequate remedies under law to compel such attendance. Any military official whom Congress
summons to testify before it may be compelled to do so by the President. If the President is not so As evidenced by Arnault v. Nazareno54 and Bengzon v. Senate Blue Ribbon Committee,55 among others,
inclined, the President may be commanded by judicial order to compel the attendance of the military the Court has not shirked from reviewing the exercise by Congress of its power of legislative
officer. Final judicial orders have the force of the law of the land which the President has the duty to inquiry.56 Arnaultrecognized that the legislative power of inquiry and the process to enforce it, "is an
faithfully execute.50 essential and appropriate auxiliary to the legislative function."57 On the other
hand, Bengzon acknowledged that the power of both houses of Congress to conduct inquiries in aid of
legislation is not "absolute or unlimited", and its exercise is circumscribed by Section 21, Article VI of the
Explication of these principles is in order.
Constitution.58 From these premises, the Court enjoined the Senate Blue Ribbon Committee from
requiring the petitioners in Bengzon from testifying and producing evidence before the committee,
As earlier noted, we ruled in Senate that the President may not issue a blanket requirement of prior holding that the inquiry in question did not involve any intended legislation.
consent on executive officials summoned by the legislature to attend a congressional hearing. In doing
so, the Court recognized the considerable limitations on executive privilege, and affirmed that the
Senate affirmed both the Arnault and Bengzon rulings. It elucidated on the constitutional scope and
privilege must be formally invoked on specified grounds. However, the ability of the President to
limitations on the constitutional power of congressional inquiry. Thus:
prevent military officers from testifying before Congress does not turn on executive privilege, but on
the Chief Executives power as commander-in-chief to control the actions and speech of members of
the armed forces. The Presidents prerogatives as commander-in-chief are not hampered by the same As discussed in Arnault, the power of inquiry, "with process to enforce it," is grounded on the necessity
limitations as in executive privilege. of information in the legislative process. If the information possessed by executive officials on the
operation of their offices is necessary for wise legislation on that subject, by parity of reasoning,
Congress has the right to that information and the power to compel the disclosure thereof.
Our ruling that the President could, as a general rule, require military officers to seek presidential
approval before appearing before Congress is based foremost on the notion that a contrary rule unduly
diminishes the prerogatives of the President as commander-in-chief. Congress holds significant control As evidenced by the American experience during the so-called "McCarthy era", however, the right of
over the armed forces in matters such as budget appropriations and the approval of higher-rank Congress to conduct inquirites in aid of legislation is, in theory, no less susceptible to abuse than
promotions,51 yet it is on the President that the Constitution vests the title as commander-in-chief and all executive or judicial power. It may thus be subjected to judicial review pursuant to the Courts certiorari
the prerogatives and functions appertaining to the position. Again, the exigencies of military discipline powers under Section 1, Article VIII of the Constitution.
and the chain of command mandate that the Presidents ability to control the individual members of the
armed forces be accorded the utmost respect. Where a military officer is torn between obeying the
For one, as noted in Bengzon v. Senate Blue Ribbon Committee, the inquiry itself might not properly be in
President and obeying the Senate, the Court will without hesitation affirm that the officer has to choose
aid of legislation, and thus beyond the constitutional power of Congress. Such inquiry could not usurp
the President. After all, the Constitution prescribes that it is the President, and not the Senate, who is
judicial functions. Parenthetically, one possible way for Congress to avoid such result as occurred
the commander-in-chief of the armed forces.52
in Bengzon is to indicate in its invitations to the public officials concerned, or to any person for that
matter, the possible needed statute which prompted the need for the inquiry. Given such statement in
At the same time, the refusal of the President to allow members of the military to appear before its invitations, along with the usual indication of the subject of inquiry and the questions relative to and
Congress is still subject to judicial relief. The Constitution itself recognizes as one of the legislatures in furtherance thereof, there would be less room for speculation on the part of the person invited on
functions is the conduct of inquiries in aid of legislation.53 Inasmuch as it is ill-advised for Congress to whether the inquiry is in aid of legislation.
interfere with the Presidents power as commander-in-chief, it is similarly detrimental for the President
to unduly interfere with Congresss right to conduct legislative inquiries. The impasse did not come to
Section 21, Article VI likewise establishes critical safeguards that proscribe the legislative power of issues ultimately detract from the main point that they testified before the Senate despite an order
inquiry. The provision requires that the inquiry be done in accordance with the Senate or Houses duly from their commanding officer and their commander-in-chief for them not to do so,61 in contravention of
published rules of procedure, necessarily implying the constitutional infirmity of an inquiry conducted the traditions of military discipline which we affirm today.1wphi1 The issues raised by petitioners could
without duly published rules of procedure. Section 21 also mandates that the rights of persons appearing have very well been raised and properly adjudicated if the proper procedure was observed. Petitioners
in or affected by such inquiries be respected, an imposition that obligates Congress to adhere to the could have been appropriately allowed to testify before the Senate without having to countermand their
guarantees in the Bill of Rights. Commander-in-chief and superior officer under the setup we have prescribed.

These abuses are, of course, remediable before the courts, upon the proper suit filed by the persons We consider the other issues raised by petitioners unnecessary to the resolution of this petition.
affected, even if they belong to the executive branch. Nonetheless, there may be exceptional
circumstances wherein a clear pattern of abuse of the legislative power of inquiry might be established,
Petitioners may have been of the honest belief that they were defying a direct order of their
resulting in palpable violations of the rights guaranteed to members of the executive department under
Commander-in-Chief and Commanding General in obeisance to a paramount idea formed within their
the Bill of Rights. In such instances, depending on the particulars of each case, attempts by the Executive
consciences, which could not be lightly ignored. Still, the Court, in turn, is guided by the superlative
Branch to forestall these abuses may be accorded judicial sanction59 .
principle that is the Constitution, the embodiment of the national conscience. The Constitution simply
does not permit the infraction which petitioners have allegedly committed, and moreover, provides for
In Senate, the Court ruled that the President could not impose a blanket prohibition barring executive an orderly manner by which the same result could have been achieved without offending constitutional
officials from testifying before Congress without the Presidents consent notwithstanding the invocation principles.
of executive privilege to justify such prohibition. The Court did not rule that the power to conduct
legislative inquiry ipso facto superseded the claim of executive privilege, acknowledging instead that the
WHEREFORE, the petition is DENIED. No pronouncement as to costs.
viability of executive privilege stood on a case to case basis. Should neither branch yield to the other
branchs assertion, the constitutional recourse is to the courts, as the final arbiter if the dispute. It is only
the courts that can compel, with conclusiveness, attendance or non-attendance in legislative inquiries. SO ORDERED.

Following these principles, it is clear that if the President or the Chief of Staff refuses to allow a member
of the AFP to appear before Congress, the legislative body seeking such testimony may seek judicial relief
to compel the attendance. Such judicial action should be directed at the heads of the executive branch
or the armed forces, the persons who wield authority and control over the actions of the officers
concerned. The legislative purpose of such testimony, as well as any defenses against the same
whether grounded on executive privilege, national security or similar concerns would be accorded
due judicial evaluation. All the constitutional considerations pertinent to either branch of government
may be raised, assessed, and ultimately weighed against each other. And once the courts speak with
finality, both branches of government have no option but to comply with the decision of the courts,
whether the effect of the decision is to their liking or disfavor.

Courts are empowered, under the constitutional principle of judicial review, to arbitrate disputes
between the legislative and executive branches of government on the proper constitutional parameters
of power.60 This is the fair and workable solution implicit in the constitutional allocation of powers
among the three branches of government. The judicial filter helps assure that the particularities of each
case would ultimately govern, rather than any overarching principle unduly inclined towards one branch
of government at the expense of the other. The procedure may not move as expeditiously as some may
desire, yet it ensures thorough deliberation of all relevant and cognizable issues before one branch is
compelled to yield to the other. Moreover, judicial review does not preclude the legislative and
executive branches from negotiating a mutually acceptable solution to the impasse. After all, the two
branches, exercising as they do functions and responsibilities that are political in nature, are free to
smooth over the thorns in their relationship with a salve of their own choosing.

And if emphasis be needed, if the courts so rule, the duty falls on the shoulders of the President, as
commander-in-chief, to authorize the appearance of the military officers before Congress. Even if the
President has earlier disagreed with the notion of officers appearing before the legislature to testify,
the Chief Executive is nonetheless obliged to comply with the final orders of the courts.

Petitioners have presented several issues relating to the tenability or wisdom of the Presidents order on
them and other military officers not to testify before Congress without the Presidents consent. Yet these
Republic of the Philippines In order to avoid a bloody confrontation, the government sent negotiators to dialogue with the soldiers.
SUPREME COURT The aim was to persuade them to peacefully return to the fold of the law. After several hours of
Manila negotiation, the government panel succeeded in convincing them to lay down their arms and defuse the
explosives placed around the premises of the Oakwood Apartments. Eventually, they returned to their
barracks.
EN BANC

A total of 321 soldiers, including petitioners herein, surrendered to the authorities.


G.R. No. 164007 August 10, 2006

The National Bureau of Investigation (NBI) investigated the incident and recommended that the military
LT. (SG) EUGENE GONZALES, LT. (SG) ANDY TORRATO, LT. (SG) ANTONIO TRILLANES IV, CPT. GARY
personnel involved be charged with coup detat defined and penalized under Article 134-A of the Revised
ALEJANO, LT. (SG) JAMES LAYUG, CPT. GERARDO GAMBALA, CPT. NICANOR FAELDON, LT. (SG)
Penal Code, as amended. On July 31, 2003, the Chief State Prosecutor of the Department of Justice (DOJ)
MANUEL CABOCHAN, ENS. ARMAND PONTEJOS, LT. (JG) ARTURO PASCUA, and 1LT. JONNEL
recommended the filing of the corresponding Information against them.
SANGGALANG,Petitioners,
vs.
GEN. NARCISO ABAYA, in his capacity as Chief of Staff of the Armed Forces of the Philippines, and B. Meanwhile, on August 2, 2003, pursuant to Article 70 of the Articles of War, respondent General Narciso
GEN. MARIANO M. SARMIENTO, JR., in his capacity as the Judge Advocate General of the Judge Abaya, then AFP Chief of Staff, ordered the arrest and detention of the soldiers involved in the Oakwood
Advocate Generals Office (JAGO), Respondents. incident and directed the AFP to conduct its own separate investigation.

DECISION On August 5, 2003, the DOJ filed with the Regional Trial Court (RTC), Makati City an Information for coup
detat 2against those soldiers, docketed as Criminal Case No. 03-2784 and eventually raffled off to Branch
61, presided by Judge Romeo F. Barza. 3 Subsequently, this case was consolidated with Criminal Case No.
SANDOVAL-GUTIERREZ, J.:
03-2678, involving the other accused, pending before Branch 148 of the RTC, Makati City, presided by
Judge Oscar B. Pimentel.
For our resolution is the Petition for Prohibition (with prayer for a temporary restraining order) filed by
the above-named members of the Armed Forces of the Philippines (AFP), herein petitioners, against the
On August 13, 2003, the RTC directed the DOJ to conduct a reinvestigation of Criminal Case No. 03-2784.
AFP Chief of Staff and the Judge Advocate General, respondents.

On the same date, respondent Chief of Staff issued Letter Order No. 625 creating a Pre-Trial Investigation
The facts are:
Panel tasked to determine the propriety of filing with the military tribunal charges for violations of the
Articles of War under Commonwealth Act No. 408, 4 as amended, against the same military personnel.
On July 26, 2003, President Gloria Macapagal Arroyo received intelligence reports that some members of Specifically, the charges are: (a) violation of Article 63 for disrespect toward the President, the Secretary
the AFP, with high-powered weapons, had abandoned their designated places of assignment. Their aim of National Defense, etc., (b) violation of Article 64 for disrespect toward a superior officer, (c) violation
was to destabilize the government. The President then directed the AFP and the Philippine National of Article 67 for mutiny or sedition, (d) violation of Article 96 for conduct unbecoming an officer and a
Police (PNP) to track and arrest them. gentleman, and (e) violation of Article 97 for conduct prejudicial to good order and military discipline.

On July 27, 2003 at around 1:00 a.m., more than 300 heavily armed junior officers and enlisted men of Of the original 321 accused in Criminal Case No. 03-2784, only 243 (including petitioners herein) filed
the AFP mostly from the elite units of the Armys Scout Rangers and the Navys Special Warfare Group with the RTC, Branch 148 an Omnibus Motion praying that the said trial court assume jurisdiction over all
entered the premises of the Oakwood Premier Luxury Apartments on Ayala Avenue, Makati City. They the charges filed with the military tribunal. They invoked Republic Act (R.A.) No. 7055. 5
disarmed the security guards and planted explosive devices around the building.
On September 15, 2003, petitioners filed with the Judge Advocate Generals Office (JAGO) a motion
Led by Navy Lt. (SG) Antonio Trillanes IV, the troops sported red armbands emblazoned with the emblem praying for the suspension of its proceedings until after the RTC shall have resolved their motion to
of the"Magdalo" faction of the Katipunan. 1 The troops then, through broadcast media, announced their assume jurisdiction.
grievances against the administration of President Gloria Macapagal Arroyo, such as the graft and
corruption in the military, the illegal sale of arms and ammunition to the "enemies" of the State, and the
On October 29, 2003, the Pre-Trial Investigation Panel submitted its Initial Report to the AFP Chief of
bombings in Davao City intended to acquire more military assistance from the US government. They
Staff recommending that the military personnel involved in the Oakwood incident be charged before a
declared their withdrawal of support from their Commander-in-Chief and demanded that she resign as
general court martial with violations of Articles 63, 64, 67, 96, and 97 of the Articles of War.
President of the Republic. They also called for the resignation of her cabinet members and the top brass
of the AFP and PNP.
Meanwhile, on November 11, 2003, the DOJ, after conducting a reinvestigation, found probable cause
against only 31 (petitioners included) of the 321 accused in Criminal Case No. 03-2784. Accordingly, the
About noontime of the same day, President Arroyo issued Proclamation No. 427 declaring a state of
prosecution filed with the RTC an Amended Information. 6
rebellion, followed by General Order No. 4 directing the AFP and PNP to take all necessary measures to
suppress the rebellion then taking place in Makati City. She then called the soldiers to surrender their
weapons at five oclock in the afternoon of that same day.
In an Order dated November 14, 2003, the RTC admitted the Amended Information and dropped the In his Comment, the Solicitor General prays that the Supplemental Petition be denied for lack of merit.
charge ofcoup detat against the 290 accused. He alleges that "contrary to petitioners pretensions, all the accused were duly arraigned on July 13 and
18, 2005." 16 The "(r)ecords show that in the hearing on July 13, 2005, all the 29 accused were present"
and, "(o)n that day, Military Prosecutor Captain Karen Ong Jags read the Charges and Specifications from
Subsequently, or on December 12, 2003, the Pre-Trial Investigation Panel submitted its Final Pre-Trial
the Charge Sheet in open court (pp. 64, TSN, July 13, 2005)." 17
Investigation Report 7 to the JAGO, recommending that, following the "doctrine of absorption," those
charged withcoup detat before the RTCshould not be charged before the military tribunal for violation
of the Articles of War. The sole question for our resolution is whether the petitioners are entitled to the writ of prohibition.

For its part, the RTC, on February 11, 2004, issued an Order 8 stating that "all charges before the court There is no dispute that petitioners, being officers of the AFP, are subject to military law. Pursuant to
martial against the accusedare hereby declared not service-connected, but rather absorbed and in Article 1 (a) of Commonwealth Act No. 408, as amended, otherwise known as the Articles of War, the
furtherance of the alleged crime of coup detat." The trial court then proceeded to hear petitioners term "officer" is "construed to refer to a commissioned officer." Article 2 provides:
applications for bail.
Art. 2. Persons Subject to Military Law. The following persons are subject to these articles and shall be
In the meantime, Colonel Julius A. Magno, in his capacity as officer-in-charge of the JAGO, reviewed the understood as included in the term "any person subject to military law" or "persons subject to military
findings of the Pre-Trial Investigation Panel. He recommended that 29 of the officers involved in the law," whenever used in these articles:
Oakwood incident, including petitioners, be prosecuted before a general court martial for violation of
Article 96 (conduct unbecoming an officer and a gentleman) of the Articles of War.
(a) All officers and soldiers in the active service of the Armed Forces of the Philippines or of the
Philippine Constabulary, all members of the reserve force, from the dates of their call to active duty and
On June 17, 2004, Colonel Magnos recommendation was approved by the AFP top brass. The AFP Judge while on such active duty; all trainees undergoing military instructions; and all other persons lawfully
Advocate General then directed petitioners to submit their answer to the charge. Instead of complying, called, drafted, or ordered into, or to duty or for training in the said service, from the dates they are
they filed with this Court the instant Petition for Prohibition praying that respondents be ordered to required by the terms of the call, draft, or order to obey the same.
desist from charging them with violation of Article 96 of the Articles of War in relation to the Oakwood
incident. 9
Upon the other hand, Section 1 of R.A. No. 7055 reads:

Petitioners maintain that since the RTC has made a determination in its Order of February 11, 2004 that
SEC. 1. Members of the Armed Forces of the Philippines and other persons subject to military law,
the offense for violation of Article 96 (conduct unbecoming an officer and a gentleman) of the Articles of
including members of the Citizens Armed Forces Geographical Units, who commit crimes or offenses
War is not service-connected, but is absorbed in the crime of coup detat, the military tribunal cannot
penalized under the Revised Penal Code, other special penal laws, or local government ordinances,
compel them to submit to its jurisdiction.
regardless of whether or not civilians are co-accused, victims, or offended parties, which may be natural
or juridical persons, shall be tried by the proper civil court, except when the offense, as determined
The Solicitor General, representing the respondents, counters that R.A. No. 7055 specifies which before arraignment by the civil court, is service-connected, in which case, the offense shall be tried by
offenses covered by the Articles of War areservice-connected. These are violations of Articles 54 to 70, court-martial, Provided, That the President of the Philippines may, in the interest of justice, order or
72 to 92, and 95 to 97. The law provides that violations of these Articles are properly cognizable by the direct at any time before arraignment that any such crimes or offenses be tried by the proper civil courts.
court martial. As the charge against petitioners is violation of Article 96 which, under R.A. No. 7055 is a
service-connected offense, then it falls under the jurisdiction of the court martial.
As used in this Section, service-connected crimes or offenses shall be limited to those defined in Articles
54 to 70, Articles 72 to 92, and Articles 95 to 97 of Commonwealth Act No. 408, as amended.
Subsequently, petitioners filed with this Court a Supplemental Petition raising the additional issue that
the offense charged before the General Court Martial has prescribed. Petitioners alleged therein that
In imposing the penalty for such crimes or offenses, the court-martial may take into consideration the
during the pendency of their original petition, respondents proceeded with the Pre-Trial Investigation for
penalty prescribed therefor in the Revised Penal Code, other special laws, or local government
purposes of charging them with violation of Article 96 (conduct unbecoming an officer and a gentleman)
ordinances.
of the Articles of War; that the Pre-Trial Investigation Panel then referred the case to the General Court
Martial; that "almost two years since the Oakwood incident on July 27, 2003, only petitioner Lt. (SG)
Antonio Trillanes was arraigned, and this was done under questionable circumstances;" 10 that in the Section 1 of R.A. No. 7055, quoted above, is clear and unambiguous. First, it lays down the general rule
hearing of July 26, 2005, herein petitioners moved for the dismissal of the case on the ground that they that members of the AFP and other persons subject to military law, including members of the Citizens
were not arraigned within the prescribed period of two (2) years from the date of the commission of the Armed Forces Geographical Units, who commit crimes or offenses penalized under the Revised Penal
alleged offense, in violation of Article 38 of the Articles of War; 11 that "the offense charged prescribed Code (like coup detat), other special penal laws, or local ordinances shall be tried by the proper civil
on July 25, 2005;" 12 that the General Court Martial ruled, however, that "the prescriptive period shall court. Next, it provides the exception to the general rule, i.e., where the civil court, before arraignment,
end only at 12:00 midnight of July 26, 2005;" 13 that "(a)s midnight of July 26, 2005 was approaching and has determined the offense to be service-connected, then the offending soldier shall be tried by a court
it was becoming apparent that the accused could not be arraigned, the prosecution suddenly changed its martial. Lastly, the law states an exception to the exception, i.e., where the President of the Philippines,
position and asserted that 23 of the accused have already been arraigned;" 14 and that petitioners moved in the interest of justice, directs before arraignment that any such crimes or offenses be tried by the
for a reconsideration but it was denied by the general court martial in its Order dated September 14, proper civil court.
2005. 15
The second paragraph of the same provision further identifies the "service-connected crimes or the alleged crime of coup detat," hence, triable by said court (RTC). The RTC, in making such declaration,
offenses" as "limited to those defined in Articles 54 to 70, Articles 72 to 92, and Articles 95 to 97" of the practically amended the law which expressly vests in the court martial the jurisdiction over "service-
Articles of War. Violations of these specified Articles are triable by court martial. This delineates the connected crimes or offenses." What the law has conferred the court should not take away. It is only the
jurisdiction between the civil courts and the court martial over crimes or offenses committed by military Constitution or the law that bestows jurisdiction on the court, tribunal, body or officer over the subject
personnel. matter or nature of an action which can do so. 22 And it is only through a constitutional amendment or
legislative enactment that such act can be done. The first and fundamental duty of the courts is merely
to apply the law "as they find it, not as they like it to be." 23 Evidently, such declaration by the RTC
Such delineation of jurisdiction by R.A. No. 7055 is necessary to preserve the peculiar nature of military
constitutes grave abuse of discretion tantamount to lack or excess of jurisdiction and is, therefore, void.
justice system over military personnel charged with service-connected offenses. The military justice
system is disciplinary in nature, aimed at achieving the highest form of discipline in order to ensure the
highest degree of military efficiency. 18 Military law is established not merely to enforce discipline in In Navales v. Abaya., 24 this Court, through Mr. Justice Romeo J. Callejo, Sr., held:
times of war, but also to preserve the tranquility and security of the State in time of peace; for there is
nothing more dangerous to the public peace and safety than a licentious and undisciplined military
We agree with the respondents that the sweeping declaration made by the RTC (Branch 148) in the
body. 19 The administration of military justice has been universally practiced. Since time immemorial, all
dispositive portion of its Order dated February 11, 2004 that all charges before the court-martial against
the armies in almost all countries of the world look upon the power of military law and its administration
the accused were not service-connected, but absorbed and in furtherance of the crime of coup detat,
as the most effective means of enforcing discipline. For this reason, the court martial has become
cannot be given effect. x x x, such declaration was made without or in excess of jurisdiction; hence, a
invariably an indispensable part of any organized armed forces, it being the most potent agency in
nullity.
enforcing discipline both in peace and in war. 20

The second paragraph of the above provision (referring to Section 1 of R.A. No. 7055) explicitly specifies
Here, petitioners are charged for violation of Article 96 (conduct unbecoming an officer and a
what are considered "service-connected crimes or offenses" under Commonwealth Act No. 408, as
gentleman) of the Articles of War before the court martial, thus:
amended, also known as the Articles of War, to wit:

All persons subject to military law, did on or about 27 July 2003 at Oakwood Hotel, Makati City, Metro
Articles 54 to 70:
Manila, willfully, unlawfully and feloniously violate their solemn oath as officers to defend the
Constitution, the law and the duly-constituted authorities and abused their constitutional duty to
protect the people and the State by, among others, attempting to oust the incumbent duly-elected and Art. 54. Fraudulent Enlistment.
legitimate President by force and violence, seriously disturbing the peace and tranquility of the people
and the nation they are sworn to protect,thereby causing dishonor and disrespect to the military Art. 55. Officer Making Unlawful Enlistment.
profession, conduct unbecoming an officer and a gentleman, in violation of AW 96 of the Articles of
War.
Art. 56. False Muster.

CONTRARY TO LAW. (Underscoring ours)


Art. 57. False Returns.

Article 96 of the Articles of War 21 provides:


Art. 58. Certain Acts to Constitute Desertion.

ART. 96. Conduct Unbecoming an Officer and Gentleman. Any officer, member of the Nurse Corps,
cadet, flying cadet, or probationary second lieutenant, who is convicted of conduct unbecoming an Art. 59. Desertion.
officer and a gentleman shall be dismissed from the service. (Underscoring ours)
Art. 60. Advising or Aiding Another to Desert.
We hold that the offense for violation of Article 96 of the Articles of War is service-connected. This is
expressly provided in Section 1 (second paragraph) of R.A. No. 7055. It bears stressing that the charge Art. 61. Entertaining a Deserter.
against the petitioners concerns the alleged violation of their solemn oath as officers to defend the
Constitution and the duly-constituted authorities.Such violation allegedly caused dishonor and
disrespect to the military profession. In short, the charge has a bearing on Art. 62. Absence Without Leave.
their professional conduct or behavior as military officers. Equally indicative of the "service-connected"
nature of the offense is the penalty prescribed for the same dismissal from the service imposable Art. 63. Disrespect Toward the President, Vice-President,
only by the military court.Such penalty is purely disciplinary in character, evidently intended to cleanse
the military profession of misfits and to preserve the stringent standard of military discipline.
Congress of the Philippines, or Secretary of National

Obviously, there is no merit in petitioners argument that they can no longer be charged before the court
Defense.
martial for violation of Article 96 of the Articles of War because the same has been declared by the RTC
in its Order of February 11, 2004 as "not service-connected, but rather absorbed and in furtherance of
Art. 64. Disrespect Toward Superior Officer. Issued to Soldiers.

Art. 65. Assaulting or Willfully Disobeying Superior Officer. Art. 86. Drunk on Duty.

Art. 66. Insubordinate Conduct Toward Non-Commissioned Officer. Art. 87. Misbehavior of Sentinel.

Art. 67. Mutiny or Sedition. Art. 88. Personal Interest in Sale of Provisions.

Art. 68. Failure to Suppress Mutiny or Sedition. Art. 88-A. Unlawful Influencing Action of Court.

Art. 69. Quarrels; Frays; Disorders. Art. 89. Intimidation of Persons Bringing Provisions.

Art. 70. Arrest or Confinement. Art. 90. Good Order to be Maintained and Wrongs Redressed.

Articles 72 to 92: Art. 91. Provoking Speeches or Gestures.

Art. 72. Refusal to Receive and Keep Prisoners. Art. 92. Dueling.

Art. 73. Report of Prisoners Received. Articles 95 to 97:

Art. 74. Releasing Prisoner Without Authority. Art. 95. Frauds Against the Government.

Art. 75. Delivery of Offenders to Civil Authorities. Art. 96. Conduct Unbecoming an Officer and Gentleman.

Art. 76. Misbehavior Before the Enemy. Art. 97. General Article.

Art. 77. Subordinates Compelling Commander to Surrender. Further, Section 1 of Rep. Act No. 7055 vests on the military courts the jurisdiction over the foregoing
offenses. x x x.
Art. 78. Improper Use of Countersign.
It is clear from the foregoing that Rep. Act No. 7055 did not divest the military courts of jurisdiction to try
cases involving violations of Articles 54 to 70, Articles 72 to 92, and Articles 95 to 97 of the Articles of
Art. 79. Forcing a Safeguard.
War as these are considered "service-connected crimes or offenses." In fact, it mandates that these shall
be tried by the court-martial.
Art. 80. Captured Property to be Secured for Public Service.
Moreover, the observation made by Mr. Justice Antonio T. Carpio during the deliberation of this case is
Art. 81. Dealing in Captured or Abandoned Property. worth quoting, thus:

Art. 82. Relieving, Corresponding With, or Aiding the Enemy. The trial court aggravated its error when it justified its ruling by holding that the charge of Conduct
Unbecoming an Officer and a Gentleman is absorbed and in furtherance to the alleged crime of coup
Art. 83. Spies. detat. Firstly, the doctrine of absorption of crimes is peculiar to criminal law and generally applies to
crimes punished by the same statute,25 unlike here where different statutes are involved. Secondly, the
doctrine applies only if the trial court has jurisdiction over both offenses. Here, Section 1 of R.A. 7055
Art. 84. Military Property.Willful or Negligent Loss, Damage deprives civil courts of jurisdiction over service-connected offenses, including Article 96 of the Articles of
War. Thus, the doctrine of absorption of crimes is not applicable to this case.
or wrongful Disposition.
Military law is sui generis (Calley v. Callaway, 519 F.2d 184 [1975]), applicable only to military personnel
Art. 85. Waste or Unlawful Disposition of Military Property because the military constitutes an armed organization requiring a system of discipline separate from
that of civilians (see Orloff v. Willoughby, 345 U.S. 83 [1953]). Military personnel carry high-powered
arms and other lethal weapons not allowed to civilians. History, experience, and the nature of a military
organization dictate that military personnel must be subjected to a separate disciplinary system not
applicable to unarmed civilians or unarmed government personnel.

A civilian government employee reassigned to another place by his superior may question his
reassignment by asking a temporary restraining order or injunction from a civil court. However, a soldier
cannot go to a civil court and ask for a restraining or injunction if his military commander reassigns him
to another area of military operations. If this is allowed, military discipline will collapse.

xxx

This Court has recognized that courts-martial are instrumentalities of the Executive to enable the
President, as Commander-in-Chief, to effectively command, control, and discipline the armed forces (see
Ruffy v. Chief of Staff, 75 Phil. 875 [1946], citing Winthrops Military Law and Precedents, 2nd edition, p.
49). In short, courts-martial form part of the disciplinary system that ensures the Presidents control, and
thus civilian supremacy, over the military. At the apex of this disciplinary system is the President who
exercises review powers over decisions of courts-martial (citing Article 50 of the Articles of War; quoted
provisions omitted).

xxx

While the Court had intervened before in courts-martial or similar proceedings, it did so sparingly and
only to release a military personnel illegally detained (Ognir v. Director of Prisons, 80 Phil. 401 [1948] or
to correct objectionable procedures (Yamashita v. Styer, 75 Phil. 563 [1945]). The Court has never
suppressed court-martial proceedings on the ground that the offense charged is absorbed and in
furtherance of another criminal charge pending with the civil courts. The Court may now do so only if
the offense charged is not one of the service-connected offenses specified in Section 1 of RA 7055. Such
is not the situation in the present case.

With respect to the issue of prescription raised by petitioners in their Supplemental Petition, suffice it to
say that we cannot entertain the same. The contending parties are at loggerheads as to (a) who among
the petitioners were actually arraigned, and (b) the dates of their arraignment. These are matters
involving questions of fact, not within our power of review, as we are not a trier of facts. In a petition for
prohibition, such as the one at bar, only legal issues affecting the jurisdiction of the tribunal, board or
officer involved may be resolved on the basis of the undisputed facts. 26

Clearly, the instant petition for prohibition must fail. The office of prohibition is to prevent the unlawful
and oppressive exercise of authority and is directed against proceedings that are done without or in
excess of jurisdiction, or with grave abuse of discretion, there being no appeal or other plain, speedy,
and adequate remedy in the ordinary course of law. 27 Stated differently, prohibition is the remedy to
prevent inferior courts, corporations, boards, or persons from usurping or exercising a jurisdiction or
power with which they have not been vested by law. 28

In fine, this Court holds that herein respondents have the authority in convening a court martial and in
charging petitioners with violation of Article 96 of the Articles of War.

WHEREFORE, the instant petition for prohibition is DISMISSED.

SO ORDERED.
EN BANC Petitioner contends that the subject EO violates the constitutional provision on the separation of
Church and State.[7] It is unconstitutional for the government to formulate policies and guidelines on the
halal certification scheme because said scheme is a function only religious organizations, entity or
scholars can lawfully and validly perform for the Muslims.According to petitioner, a food product
becomes halal only after the performance of Islamic religious ritual and prayer. Thus, only practicing
[G.R. No. 153888. July 9, 2003] Muslims are qualified to slaughter animals for food. A government agency like herein respondent OMA
cannot therefore perform a religious function like certifying qualified food products as halal.

Petitioner also maintains that the respondents violated Section 10, Article III of the 1987
Constitution which provides that (n)o law impairing the obligation of contracts, shall be passed. After the
ISLAMIC DAWAH COUNCIL OF THE PHILIPPINES, INC., herein represented by PROF. ABDULRAFIH H. subject EO was implemented, food manufacturers with existing contracts with petitioner ceased to
SAYEDY, petitioner, vs. OFFICE OF THE EXECUTIVE SECRETARY of the Office of the President obtain certifications from the latter.
of the Philippines, herein represented by HON. ALBERTO G. ROMULO, Executive Secretary,
and the OFFICE ON MUSLIM AFFAIRS, herein represented by its Executive Director, HABIB Moreover, petitioner argues that the subject EO violates Sections 15 and 16 of Article XIII of the
MUJAHAB HASHIM, respondents. 1987 Constitution which respectively provide:

DECISION ROLE AND RIGHTS OF PEOPLES ORGANIZATIONS

CORONA, J.:
Sec. 15. The State shall respect the role of independent peoples organizations to enable the people to
pursue and protect, within the democratic framework, their legitimate and collective interests and
Before us is a petition for prohibition filed by petitioner Islamic Dawah Council of the Philippines, aspirations through peaceful and lawful means.
Inc. (IDCP) praying for the declaration of nullity of Executive Order (EO) 46, s. 2001 and the prohibition of
herein respondents Office of the Executive Secretary and Office of Muslim Affairs (OMA) from
implementing the subject EO. Peoples organizations are bona fide associations of citizens with demonstrated capacity to promote the
public interest and with identifiable leadership, membership, and structure.
Petitioner IDCP, a corporation that operates under Department of Social Welfare and
Development License No. SB-01-085, is a non-governmental organization that extends voluntary services
Sec. 16. The rights of the people and their organizations to effective and reasonable participation at all
to the Filipino people, especially to Muslim communities. It claims to be a federation of national Islamic levels of social, political, and economic decision-making shall not be abridged. The State shall, by law,
organizations and an active member of international organizations such as the Regional Islamic Dawah
facilitate, the establishment of adequate consultation mechanisms.
Council of Southeast Asia and the Pacific (RISEAP)[1] and The World Assembly of Muslim Youth. The
RISEAP accredited petitioner to issue halal[2] certifications in the Philippines. Thus, among the functions
petitioner carries out is to conduct seminars, orient manufacturers on halal food and issue halal According to petitioner, the subject EO was issued with utter haste and without even consulting
certifications to qualified products and manufacturers. Muslim peoples organizations like petitioner before it became effective.

Petitioner alleges that, on account of the actual need to certify food products as halal and also We grant the petition.
due to halal food producers request, petitioner formulated in 1995 internal rules and procedures based
OMA was created in 1981 through Executive Order No. 697 (EO 697) to ensure the integration of
on the Quran[3] and the Sunnah[4] for the analysis of food, inspection thereof and issuance of halal
Muslim Filipinos into the mainstream of Filipino society with due regard to their beliefs, customs,
certifications. In that same year, petitioner began to issue, for a fee, certifications to qualified products
traditions, and institutions.[8] OMA deals with the societal, legal, political and economic concerns of the
and food manufacturers. Petitioner even adopted for use on its halal certificates a distinct sign or logo
Muslim community as a national cultural community and not as a religious group. Thus, bearing in mind
registered in the Philippine Patent Office under Patent No. 4-2000-03664.
the constitutional barrier between the Church and State, the latter must make sure that OMA does not
On October 26, 2001, respondent Office of the Executive Secretary issued EO 46[5] creating the intrude into purely religious matters lest it violate the non-establishment clause and the free exercise of
Philippine Halal Certification Scheme and designating respondent OMA to oversee its religion provision found in Article III, Section 5 of the 1987 Constitution.[9]
implementation. Under the EO, respondent OMA has the exclusive authority to issue halal certificates
Freedom of religion was accorded preferred status by the framers of our fundamental law. And
and perform other related regulatory activities.
this Court has consistently affirmed this preferred status, well aware that it is "designed to protect the
On May 8, 2002, a news article entitled OMA Warns NGOs Issuing Illegal Halal Certification was broadest possible liberty of conscience, to allow each man to believe as his conscience directs, to profess
published in the Manila Bulletin, a newspaper of general circulation. In said article, OMA warned Muslim his beliefs, and to live as he believes he ought to live, consistent with the liberty of others and with the
consumers to buy only products with its official halal certification since those without said certification common good.[10]
had not been subjected to careful analysis and therefore could contain pork or its derivatives.
Without doubt, classifying a food product as halal is a religious function because the standards
Respondent OMA also sent letters to food manufacturers asking them to secure the halal certification
used are drawn from the Quran and Islamic beliefs. By giving OMA the exclusive power to classify food
only from OMA lest they violate EO 46 and RA 4109.[6] As a result, petitioner lost revenues after food
products as halal, EO 46 encroached on the religious freedom of Muslim organizations like herein
manufacturers stopped securing certifications from it.
petitioner to interpret for Filipino Muslims what food products are fit for Muslim consumption. Also, by
Hence, this petition for prohibition. arrogating to itself the task of issuing halal certifications, the State has in effect forced Muslims to accept
its own interpretation of the Quran and Sunnah on halal food.
To justify EO 46s intrusion into the subject religious activity, the Solicitor General argues that the right to health is protected. The halal certifications issued by petitioner and similar organizations come
freedom of religion is subservient to the police power of the State. By delegating to OMA the authority forward as the official religious approval of a food product fit for Muslim consumption.
to issue halal certifications, the government allegedly seeks to protect and promote the muslim Filipinos
right to health, and to instill health consciousness in them. We do not share respondents apprehension that the absence of a central administrative body to
regulate halal certifications might give rise to schemers who, for profit, will issue certifications for
We disagree. products that are not actually halal. Aside from the fact that Muslim consumers can actually verify
through the labels whether a product contains non-food substances, we believe that they are discerning
Only the prevention of an immediate and grave danger to the security and welfare of the enough to know who the reliable and competent certifying organizations in their community are. Before
community can justify the infringement of religious freedom.[11] If the government fails to show the purchasing a product, they can easily avert this perceived evil by a diligent inquiry on the reliability of the
seriousness and immediacy of the threat, State intrusion is constitutionally unacceptable. In a society concerned certifying organization.
with a democratic framework like ours, the State must minimize its interference with the affairs of its
citizens and instead allow them to exercise reasonable freedom of personal and religious activity. WHEREFORE, the petition is GRANTED. Executive Order 46, s. 2001, is hereby declared NULL AND
VOID. Consequently, respondents are prohibited from enforcing the same.
In the case at bar, we find no compelling justification for the government to deprive Muslim
organizations, like herein petitioner, of their religious right to classify a product as halal, even on the SO ORDERED.
premise that the health of Muslim Filipinos can be effectively protected by assigning to OMA the
exclusive power to issue halal certifications. The protection and promotion of the Muslim Filipinos right
to health are already provided for in existing laws and ministered to by government agencies charged
with ensuring that food products released in the market are fit for human consumption, properly labeled
and safe. Unlike EO 46, these laws do not encroach on the religious freedom of Muslims.

Section 48(4) of the Administrative Code of 1987 gives to the National Meat Inspection
Commission (NMIC) of the Department of Agriculture (DOA) the power to inspect slaughtered animals
intended for human consumption to ensure the safety of the meat released in the market. Another law,
RA 7394, otherwise known as The Consumer Act of 1992, gives to certain government departments the
duty to protect the interests of the consumer, promote his general welfare and to establish standards of
conduct for business and industry.[12] To this end, a food product, before its distribution to the market, is
required to secure the Philippine Standard Certification Mark after the concerned department inspects
and certifies its compliance with quality and safety standards.[13]

One such government agency designated by RA 7394 is the Bureau of Food and Drugs (BFD) of the
Department of Health (DOH). Under Article 22 of said law, BFD has the duty to promulgate and enforce
rules and regulations fixing and establishing a reasonable definition and standard of identity, a standard
of quality and a standard of fill of containers for food. The BFD also ensures that food products released
in the market are not adulterated.[14]

Furthermore, under Article 48 of RA 7394, the Department of Trade and Industry (DTI) is tasked to
protect the consumer against deceptive, unfair and unconscionable sales acts or practices as defined in
Article 50.[15] DTI also enforces compulsory labeling and fair packaging to enable the consumer to obtain
accurate information as to the nature, quality and quantity of the contents of consumer products and to
facilitate his comparison of the value of such products.[16]

With these regulatory bodies given detailed functions on how to screen and check the quality and
safety of food products, the perceived danger against the health of Muslim and non-Muslim Filipinos
alike is totally avoided. Of great help are the provisions on labeling of food products (Articles 74 to
85)[17] of RA 7394. In fact, through these labeling provisions, the State ably informs the consuming public
of the contents of food products released in the market. Stiff sanctions are imposed on violators of said
labeling requirements.

Through the laws on food safety and quality, therefore, the State indirectly aids Muslim
consumers in differentiating food from non-food products. The NMIC guarantees that the meat sold in
the market has been thoroughly inspected and fit for consumption. Meanwhile, BFD ensures that food
products are properly categorized and have passed safety and quality standards. Then, through the
labeling provisions enforced by the DTI, Muslim consumers are adequately apprised of the products that
contain substances or ingredients that, according to their Islamic beliefs, are not fit for human
intake. These are the non-secular steps put in place by the State to ensure that the Muslim consumers
Republic of the Philippines the unabated hemorrhage of the country's vital life support systems and continued rape of Mother
SUPREME COURT Earth."
Manila
The controversy has its genesis in Civil Case No. 90-77 which was filed before Branch 66 (Makati, Metro
EN BANC Manila) of the Regional Trial Court (RTC), National Capital Judicial Region. The principal plaintiffs therein,
now the principal petitioners, are all minors duly represented and joined by their respective parents.
Impleaded as an additional plaintiff is the Philippine Ecological Network, Inc. (PENI), a domestic, non-
stock and non-profit corporation organized for the purpose of, inter alia, engaging in concerted action
geared for the protection of our environment and natural resources. The original defendant was the
G.R. No. 101083 July 30, 1993 Honorable Fulgencio S. Factoran, Jr., then Secretary of the Department of Environment and Natural
Resources (DENR). His substitution in this petition by the new Secretary, the Honorable Angel C. Alcala,
JUAN ANTONIO, ANNA ROSARIO and JOSE ALFONSO, all surnamed OPOSA, minors, and represented was subsequently ordered upon proper motion by the petitioners. 1 The complaint 2was instituted as a
by their parents ANTONIO and RIZALINA OPOSA, ROBERTA NICOLE SADIUA, minor, represented by her taxpayers' class suit 3 and alleges that the plaintiffs "are all citizens of the Republic of the Philippines,
parents CALVIN and ROBERTA SADIUA, CARLO, AMANDA SALUD and PATRISHA, all surnamed FLORES, taxpayers, and entitled to the full benefit, use and enjoyment of the natural resource treasure that is the
minors and represented by their parents ENRICO and NIDA FLORES, GIANINA DITA R. FORTUN, minor, country's virgin tropical forests." The same was filed for themselves and others who are equally
represented by her parents SIGRID and DOLORES FORTUN, GEORGE II and MA. CONCEPCION, all concerned about the preservation of said resource but are "so numerous that it is impracticable to bring
surnamed MISA, minors and represented by their parents GEORGE and MYRA MISA, BENJAMIN ALAN them all before the Court." The minors further asseverate that they "represent their generation as well
V. PESIGAN, minor, represented by his parents ANTONIO and ALICE PESIGAN, JOVIE MARIE ALFARO, as generations yet unborn." 4 Consequently, it is prayed for that judgment be rendered:
minor, represented by her parents JOSE and MARIA VIOLETA ALFARO, MARIA CONCEPCION T.
CASTRO, minor, represented by her parents FREDENIL and JANE CASTRO, JOHANNA DESAMPARADO, . . . ordering defendant, his agents, representatives and other persons acting in his
minor, represented by her parents JOSE and ANGELA DESAMPRADO, CARLO JOAQUIN T. NARVASA, behalf to
minor, represented by his parents GREGORIO II and CRISTINE CHARITY NARVASA, MA. MARGARITA,
JESUS IGNACIO, MA. ANGELA and MARIE GABRIELLE, all surnamed SAENZ, minors, represented by
(1) Cancel all existing timber license agreements in the country;
their parents ROBERTO and AURORA SAENZ, KRISTINE, MARY ELLEN, MAY, GOLDA MARTHE and
DAVID IAN, all surnamed KING, minors, represented by their parents MARIO and HAYDEE KING,
DAVID, FRANCISCO and THERESE VICTORIA, all surnamed ENDRIGA, minors, represented by their (2) Cease and desist from receiving, accepting, processing, renewing or approving
parents BALTAZAR and TERESITA ENDRIGA, JOSE MA. and REGINA MA., all surnamed ABAYA, minors, new timber license agreements.
represented by their parents ANTONIO and MARICA ABAYA, MARILIN, MARIO, JR. and MARIETTE, all
surnamed CARDAMA, minors, represented by their parents MARIO and LINA CARDAMA, CLARISSA,
and granting the plaintiffs ". . . such other reliefs just and equitable under the premises." 5
ANN MARIE, NAGEL, and IMEE LYN, all surnamed OPOSA, minors and represented by their parents
RICARDO and MARISSA OPOSA, PHILIP JOSEPH, STEPHEN JOHN and ISAIAH JAMES, all surnamed
QUIPIT, minors, represented by their parents JOSE MAX and VILMI QUIPIT, BUGHAW CIELO, The complaint starts off with the general averments that the Philippine archipelago of 7,100 islands has a
CRISANTO, ANNA, DANIEL and FRANCISCO, all surnamed BIBAL, minors, represented by their parents land area of thirty million (30,000,000) hectares and is endowed with rich, lush and verdant rainforests in
FRANCISCO, JR. and MILAGROS BIBAL, and THE PHILIPPINE ECOLOGICAL NETWORK, INC., petitioners, which varied, rare and unique species of flora and fauna may be found; these rainforests contain a
vs. genetic, biological and chemical pool which is irreplaceable; they are also the habitat of indigenous
THE HONORABLE FULGENCIO S. FACTORAN, JR., in his capacity as the Secretary of the Department of Philippine cultures which have existed, endured and flourished since time immemorial; scientific
Environment and Natural Resources, and THE HONORABLE ERIBERTO U. ROSARIO, Presiding Judge of evidence reveals that in order to maintain a balanced and healthful ecology, the country's land area
the RTC, Makati, Branch 66, respondents. should be utilized on the basis of a ratio of fifty-four per cent (54%) for forest cover and forty-six per cent
(46%) for agricultural, residential, industrial, commercial and other uses; the distortion and disturbance
of this balance as a consequence of deforestation have resulted in a host of environmental tragedies,
Oposa Law Office for petitioners.
such as (a) water shortages resulting from drying up of the water table, otherwise known as the
"aquifer," as well as of rivers, brooks and streams, (b) salinization of the water table as a result of the
The Solicitor General for respondents. intrusion therein of salt water, incontrovertible examples of which may be found in the island of Cebu
and the Municipality of Bacoor, Cavite, (c) massive erosion and the consequential loss of soil fertility and
agricultural productivity, with the volume of soil eroded estimated at one billion (1,000,000,000) cubic
meters per annum approximately the size of the entire island of Catanduanes, (d) the endangering
and extinction of the country's unique, rare and varied flora and fauna, (e) the disturbance and
DAVIDE, JR., J.: dislocation of cultural communities, including the disappearance of the Filipino's indigenous cultures, (f)
the siltation of rivers and seabeds and consequential destruction of corals and other aquatic life leading
In a broader sense, this petition bears upon the right of Filipinos to a balanced and healthful ecology to a critical reduction in marine resource productivity, (g) recurrent spells of drought as is presently
which the petitioners dramatically associate with the twin concepts of "inter-generational responsibility" experienced by the entire country, (h) increasing velocity of typhoon winds which result from the
and "inter-generational justice." Specifically, it touches on the issue of whether the said petitioners have absence of windbreakers, (i) the floodings of lowlands and agricultural plains arising from the absence of
a cause of action to "prevent the misappropriation or impairment" of Philippine rainforests and "arrest the absorbent mechanism of forests, (j) the siltation and shortening of the lifespan of multi-billion peso
dams constructed and operated for the purpose of supplying water for domestic uses, irrigation and the
generation of electric power, and (k) the reduction of the earth's capacity to process carbon dioxide This act of defendant constitutes a misappropriation and/or impairment of the
gases which has led to perplexing and catastrophic climatic changes such as the phenomenon of global natural resource property he holds in trust for the benefit of plaintiff minors and
warming, otherwise known as the "greenhouse effect." succeeding generations.

Plaintiffs further assert that the adverse and detrimental consequences of continued and deforestation 15. Plaintiffs have a clear and constitutional right to a balanced and healthful
are so capable of unquestionable demonstration that the same may be submitted as a matter of judicial ecology and are entitled to protection by the State in its capacity as the parens
notice. This notwithstanding, they expressed their intention to present expert witnesses as well as patriae.
documentary, photographic and film evidence in the course of the trial.
16. Plaintiff have exhausted all administrative remedies with the defendant's
As their cause of action, they specifically allege that: office. On March 2, 1990, plaintiffs served upon defendant a final demand to
cancel all logging permits in the country.
CAUSE OF ACTION
A copy of the plaintiffs' letter dated March 1, 1990 is hereto attached as Annex
"B".
7. Plaintiffs replead by reference the foregoing allegations.

17. Defendant, however, fails and refuses to cancel the existing TLA's to the
8. Twenty-five (25) years ago, the Philippines had some sixteen (16) million
continuing serious damage and extreme prejudice of plaintiffs.
hectares of rainforests constituting roughly 53% of the country's land mass.

18. The continued failure and refusal by defendant to cancel the TLA's is an act
9. Satellite images taken in 1987 reveal that there remained no more than 1.2
violative of the rights of plaintiffs, especially plaintiff minors who may be left with
million hectares of said rainforests or four per cent (4.0%) of the country's land
a country that is desertified (sic), bare, barren and devoid of the wonderful flora,
area.
fauna and indigenous cultures which the Philippines had been abundantly blessed
with.
10. More recent surveys reveal that a mere 850,000 hectares of virgin old-growth
rainforests are left, barely 2.8% of the entire land mass of the Philippine
19. Defendant's refusal to cancel the aforementioned TLA's is manifestly contrary
archipelago and about 3.0 million hectares of immature and uneconomical
to the public policy enunciated in the Philippine Environmental Policy which, in
secondary growth forests.
pertinent part, states that it is the policy of the State

11. Public records reveal that the defendant's, predecessors have granted timber
(a) to create, develop, maintain and improve conditions under which man and
license agreements ('TLA's') to various corporations to cut the aggregate area of
nature can thrive in productive and enjoyable harmony with each other;
3.89 million hectares for commercial logging purposes.

(b) to fulfill the social, economic and other requirements of present and future
A copy of the TLA holders and the corresponding areas covered is hereto attached
generations of Filipinos and;
as Annex "A".

(c) to ensure the attainment of an environmental quality that is conductive to a life


12. At the present rate of deforestation, i.e. about 200,000 hectares per annum or
of dignity and well-being. (P.D. 1151, 6 June 1977)
25 hectares per hour nighttime, Saturdays, Sundays and holidays included
the Philippines will be bereft of forest resources after the end of this ensuing
decade, if not earlier. 20. Furthermore, defendant's continued refusal to cancel the aforementioned
TLA's is contradictory to the Constitutional policy of the State to
13. The adverse effects, disastrous consequences, serious injury and irreparable
damage of this continued trend of deforestation to the plaintiff minor's generation a. effect "a more equitable distribution of opportunities, income and wealth" and
and to generations yet unborn are evident and incontrovertible. As a matter of "make full and efficient use of natural resources (sic)." (Section 1, Article XII of the
fact, the environmental damages enumerated in paragraph 6 hereof are already Constitution);
being felt, experienced and suffered by the generation of plaintiff adults.
b. "protect the nation's marine wealth." (Section 2, ibid);
14. The continued allowance by defendant of TLA holders to cut and deforest the
remaining forest stands will work great damage and irreparable injury to plaintiffs
c. "conserve and promote the nation's cultural heritage and resources (sic)"
especially plaintiff minors and their successors who may never see, use,
(Section 14, Article XIV,id.);
benefit from and enjoy this rare and unique natural resource treasure.
d. "protect and advance the right of the people to a balanced and healthful On the other hand, the respondents aver that the petitioners failed to allege in their complaint a specific
ecology in accord with the rhythm and harmony of nature." (Section 16, Article legal right violated by the respondent Secretary for which any relief is provided by law. They see nothing
II, id.) in the complaint but vague and nebulous allegations concerning an "environmental right" which
supposedly entitles the petitioners to the "protection by the state in its capacity as parens patriae." Such
allegations, according to them, do not reveal a valid cause of action. They then reiterate the theory that
21. Finally, defendant's act is contrary to the highest law of humankind the
the question of whether logging should be permitted in the country is a political question which should
natural law and violative of plaintiffs' right to self-preservation and
be properly addressed to the executive or legislative branches of Government. They therefore assert
perpetuation.
that the petitioners' resources is not to file an action to court, but to lobby before Congress for the
passage of a bill that would ban logging totally.
22. There is no other plain, speedy and adequate remedy in law other than the
instant action to arrest the unabated hemorrhage of the country's vital life support
As to the matter of the cancellation of the TLAs, respondents submit that the same cannot be done by
systems and continued rape of Mother Earth. 6
the State without due process of law. Once issued, a TLA remains effective for a certain period of time
usually for twenty-five (25) years. During its effectivity, the same can neither be revised nor cancelled
On 22 June 1990, the original defendant, Secretary Factoran, Jr., filed a Motion to Dismiss the complaint unless the holder has been found, after due notice and hearing, to have violated the terms of the
based on two (2) grounds, namely: (1) the plaintiffs have no cause of action against him and (2) the issue agreement or other forestry laws and regulations. Petitioners' proposition to have all the TLAs
raised by the plaintiffs is a political question which properly pertains to the legislative or executive indiscriminately cancelled without the requisite hearing would be violative of the requirements of due
branches of Government. In their 12 July 1990 Opposition to the Motion, the petitioners maintain that process.
(1) the complaint shows a clear and unmistakable cause of action, (2) the motion is dilatory and (3) the
action presents a justiciable question as it involves the defendant's abuse of discretion.
Before going any further, We must first focus on some procedural matters. Petitioners instituted Civil
Case No. 90-777 as a class suit. The original defendant and the present respondents did not take issue
On 18 July 1991, respondent Judge issued an order granting the aforementioned motion to dismiss. 7 In with this matter. Nevertheless, We hereby rule that the said civil case is indeed a class suit. The subject
the said order, not only was the defendant's claim that the complaint states no cause of action against matter of the complaint is of common and general interest not just to several, but to all citizens of the
him and that it raises a political question sustained, the respondent Judge further ruled that the Philippines. Consequently, since the parties are so numerous, it, becomes impracticable, if not totally
granting of the relief prayed for would result in the impairment of contracts which is prohibited by the impossible, to bring all of them before the court. We likewise declare that the plaintiffs therein are
fundamental law of the land. numerous and representative enough to ensure the full protection of all concerned interests. Hence, all
the requisites for the filing of a valid class suit under Section 12, Rule 3 of the Revised Rules of Court are
Plaintiffs thus filed the instant special civil action for certiorari under Rule 65 of the Revised Rules of present both in the said civil case and in the instant petition, the latter being but an incident to the
Court and ask this Court to rescind and set aside the dismissal order on the ground that the respondent former.
Judge gravely abused his discretion in dismissing the action. Again, the parents of the plaintiffs-minors
not only represent their children, but have also joined the latter in this case. 8 This case, however, has a special and novel element. Petitioners minors assert that they represent their
generation as well as generations yet unborn. We find no difficulty in ruling that they can, for
On 14 May 1992, We resolved to give due course to the petition and required the parties to submit their themselves, for others of their generation and for the succeeding generations, file a class suit. Their
respective Memoranda after the Office of the Solicitor General (OSG) filed a Comment in behalf of the personality to sue in behalf of the succeeding generations can only be based on the concept of
respondents and the petitioners filed a reply thereto. intergenerational responsibility insofar as the right to a balanced and healthful ecology is concerned.
Such a right, as hereinafter expounded, considers
the "rhythm and harmony of nature." Nature means the created world in its entirety. 9 Such rhythm and
Petitioners contend that the complaint clearly and unmistakably states a cause of action as it contains harmony indispensably include, inter alia, the judicious disposition, utilization, management, renewal
sufficient allegations concerning their right to a sound environment based on Articles 19, 20 and 21 of and conservation of the country's forest, mineral, land, waters, fisheries, wildlife, off-shore areas and
the Civil Code (Human Relations), Section 4 of Executive Order (E.O.) No. 192 creating the DENR, Section other natural resources to the end that their exploration, development and utilization be equitably
3 of Presidential Decree (P.D.) No. 1151 (Philippine Environmental Policy), Section 16, Article II of the accessible to the present as well as future generations. 10 Needless to say, every generation has a
1987 Constitution recognizing the right of the people to a balanced and healthful ecology, the concept of responsibility to the next to preserve that rhythm and harmony for the full enjoyment of a balanced and
generational genocide in Criminal Law and the concept of man's inalienable right to self-preservation healthful ecology. Put a little differently, the minors' assertion of their right to a sound environment
and self-perpetuation embodied in natural law. Petitioners likewise rely on the respondent's correlative constitutes, at the same time, the performance of their obligation to ensure the protection of that right
obligation per Section 4 of E.O. No. 192, to safeguard the people's right to a healthful environment. for the generations to come.

It is further claimed that the issue of the respondent Secretary's alleged grave abuse of discretion in The locus standi of the petitioners having thus been addressed, We shall now proceed to the merits of
granting Timber License Agreements (TLAs) to cover more areas for logging than what is available the petition.
involves a judicial question.

After a careful perusal of the complaint in question and a meticulous consideration and evaluation of the
Anent the invocation by the respondent Judge of the Constitution's non-impairment clause, petitioners issues raised and arguments adduced by the parties, We do not hesitate to find for the petitioners and
maintain that the same does not apply in this case because TLAs are not contracts. They likewise submit rule against the respondent Judge's challenged order for having been issued with grave abuse of
that even if TLAs may be considered protected by the said clause, it is well settled that they may still be discretion amounting to lack of jurisdiction. The pertinent portions of the said order reads as follows:
revoked by the State when the public interest so requires.
xxx xxx xxx preserve the first and protect and advance the second, the day would not be too far when all else would
be lost not only for the present generation, but also for those to come generations which stand to
inherit nothing but parched earth incapable of sustaining life.
After a careful and circumspect evaluation of the Complaint, the Court cannot help
but agree with the defendant. For although we believe that plaintiffs have but the
noblest of all intentions, it (sic) fell short of alleging, with sufficient definiteness, a The right to a balanced and healthful ecology carries with it the correlative duty to refrain from impairing
specific legal right they are seeking to enforce and protect, or a specific legal the environment. During the debates on this right in one of the plenary sessions of the 1986
wrong they are seeking to prevent and redress (Sec. 1, Rule 2, RRC). Furthermore, Constitutional Commission, the following exchange transpired between Commissioner Wilfrido Villacorta
the Court notes that the Complaint is replete with vague assumptions and vague and Commissioner Adolfo Azcuna who sponsored the section in question:
conclusions based on unverified data. In fine, plaintiffs fail to state a cause of
action in its Complaint against the herein defendant.
MR. VILLACORTA:

Furthermore, the Court firmly believes that the matter before it, being impressed
Does this section mandate the State to provide sanctions
with political color and involving a matter of public policy, may not be taken
against all forms of pollution air, water and noise
cognizance of by this Court without doing violence to the sacred principle of
pollution?
"Separation of Powers" of the three (3) co-equal branches of the Government.

MR. AZCUNA:
The Court is likewise of the impression that it cannot, no matter how we stretch
our jurisdiction, grant the reliefs prayed for by the plaintiffs, i.e., to cancel all
existing timber license agreements in the country and to cease and desist from Yes, Madam President. The right to healthful (sic)
receiving, accepting, processing, renewing or approving new timber license environment necessarily carries with it the correlative duty
agreements. For to do otherwise would amount to "impairment of contracts" of not impairing the same and, therefore, sanctions may be
abhored (sic) by the fundamental law. 11 provided for impairment of environmental balance. 12

We do not agree with the trial court's conclusions that the plaintiffs failed to allege with sufficient The said right implies, among many other things, the judicious management and conservation of the
definiteness a specific legal right involved or a specific legal wrong committed, and that the complaint is country's forests.
replete with vague assumptions and conclusions based on unverified data. A reading of the complaint
itself belies these conclusions. Without such forests, the ecological or environmental balance would be irreversiby disrupted.

The complaint focuses on one specific fundamental legal right the right to a balanced and healthful Conformably with the enunciated right to a balanced and healthful ecology and the right to health, as
ecology which, for the first time in our nation's constitutional history, is solemnly incorporated in the well as the other related provisions of the Constitution concerning the conservation, development and
fundamental law. Section 16, Article II of the 1987 Constitution explicitly provides: utilization of the country's natural resources, 13 then President Corazon C. Aquino promulgated on 10
June 1987 E.O. No. 192, 14 Section 4 of which expressly mandates that the Department of Environment
Sec. 16. The State shall protect and advance the right of the people to a balanced and Natural Resources "shall be the primary government agency responsible for the conservation,
and healthful ecology in accord with the rhythm and harmony of nature. management, development and proper use of the country's environment and natural resources,
specifically forest and grazing lands, mineral, resources, including those in reservation and watershed
areas, and lands of the public domain, as well as the licensing and regulation of all natural resources as
This right unites with the right to health which is provided for in the preceding
may be provided for by law in order to ensure equitable sharing of the benefits derived therefrom for
section of the same article:
the welfare of the present and future generations of Filipinos." Section 3 thereof makes the following
statement of policy:
Sec. 15. The State shall protect and promote the right to health of the people and
instill health consciousness among them.
Sec. 3. Declaration of Policy. It is hereby declared the policy of the State to
ensure the sustainable use, development, management, renewal, and
While the right to a balanced and healthful ecology is to be found under the Declaration of Principles and conservation of the country's forest, mineral, land, off-shore areas and other
State Policies and not under the Bill of Rights, it does not follow that it is less important than any of the natural resources, including the protection and enhancement of the quality of the
civil and political rights enumerated in the latter. Such a right belongs to a different category of rights environment, and equitable access of the different segments of the population to
altogether for it concerns nothing less than self-preservation and self-perpetuation aptly and fittingly the development and the use of the country's natural resources, not only for the
stressed by the petitioners the advancement of which may even be said to predate all governments present generation but for future generations as well. It is also the policy of the
and constitutions. As a matter of fact, these basic rights need not even be written in the Constitution for state to recognize and apply a true value system including social and
they are assumed to exist from the inception of humankind. If they are now explicitly mentioned in the environmental cost implications relative to their utilization, development and
fundamental charter, it is because of the well-founded fear of its framers that unless the rights to a conservation of our natural resources.
balanced and healthful ecology and to health are mandated as state policies by the Constitution itself,
thereby highlighting their continuing importance and imposing upon the state a solemn obligation to
This policy declaration is substantially re-stated it Title XIV, Book IV of the Administrative Code of A cause of action is defined as:
1987, 15specifically in Section 1 thereof which reads:
. . . an act or omission of one party in violation of the legal right or rights of the
Sec. 1. Declaration of Policy. (1) The State shall ensure, for the benefit of the other; and its essential elements are legal right of the plaintiff, correlative
Filipino people, the full exploration and development as well as the judicious obligation of the defendant, and act or omission of the defendant in violation of
disposition, utilization, management, renewal and conservation of the country's said legal right. 18
forest, mineral, land, waters, fisheries, wildlife, off-shore areas and other natural
resources, consistent with the necessity of maintaining a sound ecological balance
It is settled in this jurisdiction that in a motion to dismiss based on the ground that the complaint fails to
and protecting and enhancing the quality of the environment and the objective of
state a cause of action, 19 the question submitted to the court for resolution involves the sufficiency of
making the exploration, development and utilization of such natural resources
the facts alleged in the complaint itself. No other matter should be considered; furthermore, the truth of
equitably accessible to the different segments of the present as well as future
falsity of the said allegations is beside the point for the truth thereof is deemed hypothetically admitted.
generations.
The only issue to be resolved in such a case is: admitting such alleged facts to be true, may the court
render a valid judgment in accordance with the prayer in the complaint? 20 In Militante vs.
(2) The State shall likewise recognize and apply a true value system that takes into Edrosolano, 21 this Court laid down the rule that the judiciary should "exercise the utmost care and
account social and environmental cost implications relative to the utilization, circumspection in passing upon a motion to dismiss on the ground of the absence thereof [cause of
development and conservation of our natural resources. action] lest, by its failure to manifest a correct appreciation of the facts alleged and deemed
hypothetically admitted, what the law grants or recognizes is effectively nullified. If that happens, there
is a blot on the legal order. The law itself stands in disrepute."
The above provision stresses "the necessity of maintaining a sound ecological balance and protecting
and enhancing the quality of the environment." Section 2 of the same Title, on the other hand,
specifically speaks of the mandate of the DENR; however, it makes particular reference to the fact of the After careful examination of the petitioners' complaint, We find the statements under the introductory
agency's being subject to law and higher authority. Said section provides: affirmative allegations, as well as the specific averments under the sub-heading CAUSE OF ACTION, to be
adequate enough to show, prima facie, the claimed violation of their rights. On the basis thereof, they
may thus be granted, wholly or partly, the reliefs prayed for. It bears stressing, however, that insofar as
Sec. 2. Mandate. (1) The Department of Environment and Natural Resources
the cancellation of the TLAs is concerned, there is the need to implead, as party defendants, the grantees
shall be primarily responsible for the implementation of the foregoing policy.
thereof for they are indispensable parties.

(2) It shall, subject to law and higher authority, be in charge of carrying out the
The foregoing considered, Civil Case No. 90-777 be said to raise a political question. Policy formulation or
State's constitutional mandate to control and supervise the exploration,
determination by the executive or legislative branches of Government is not squarely put in issue. What
development, utilization, and conservation of the country's natural resources.
is principally involved is the enforcement of a right vis-a-vis policies already formulated and expressed in
legislation. It must, nonetheless, be emphasized that the political question doctrine is no longer, the
Both E.O. NO. 192 and the Administrative Code of 1987 have set the objectives which will serve as the insurmountable obstacle to the exercise of judicial power or the impenetrable shield that protects
bases for policy formulation, and have defined the powers and functions of the DENR. executive and legislative actions from judicial inquiry or review. The second paragraph of section 1,
Article VIII of the Constitution states that:
It may, however, be recalled that even before the ratification of the 1987 Constitution, specific statutes
already paid special attention to the "environmental right" of the present and future generations. On 6 Judicial power includes the duty of the courts of justice to settle actual
June 1977, P.D. No. 1151 (Philippine Environmental Policy) and P.D. No. 1152 (Philippine Environment controversies involving rights which are legally demandable and enforceable, and
Code) were issued. The former "declared a continuing policy of the State (a) to create, develop, maintain to determine whether or not there has been a grave abuse of discretion
and improve conditions under which man and nature can thrive in productive and enjoyable harmony amounting to lack or excess of jurisdiction on the part of any branch or
with each other, (b) to fulfill the social, economic and other requirements of present and future instrumentality of the Government.
generations of Filipinos, and (c) to insure the attainment of an environmental quality that is conducive to
a life of dignity and well-being." 16 As its goal, it speaks of the "responsibilities of each generation as
Commenting on this provision in his book, Philippine Political Law, 22 Mr. Justice Isagani A. Cruz, a
trustee and guardian of the environment for succeeding generations." 17 The latter statute, on the other
distinguished member of this Court, says:
hand, gave flesh to the said policy.

The first part of the authority represents the traditional concept of judicial power,
Thus, the right of the petitioners (and all those they represent) to a balanced and healthful ecology is as
involving the settlement of conflicting rights as conferred as law. The second part
clear as the DENR's duty under its mandate and by virtue of its powers and functions under E.O. No.
of the authority represents a broadening of judicial power to enable the courts of
192 and the Administrative Code of 1987 to protect and advance the said right.
justice to review what was before forbidden territory, to wit, the discretion of the
political departments of the government.
A denial or violation of that right by the other who has the corelative duty or obligation to respect or
protect the same gives rise to a cause of action. Petitioners maintain that the granting of the TLAs, which
As worded, the new provision vests in the judiciary, and particularly the Supreme
they claim was done with grave abuse of discretion, violated their right to a balanced and healthful
Court, the power to rule upon even the wisdom of the decisions of the executive
ecology; hence, the full protection thereof requires that no further TLAs should be renewed or granted.
and the legislature and to declare their acts invalid for lack or excess of jurisdiction held that the granting of license does not create irrevocable rights, neither is it
because tainted with grave abuse of discretion. The catch, of course, is the property or property rights (People vs. Ong Tin, 54 O.G. 7576).
meaning of "grave abuse of discretion," which is a very elastic phrase that can
expand or contract according to the disposition of the judiciary.
We reiterated this pronouncement in Felipe Ysmael, Jr. & Co., Inc. vs. Deputy Executive Secretary: 26

In Daza vs. Singson, 23 Mr. Justice Cruz, now speaking for this Court, noted:
. . . Timber licenses, permits and license agreements are the principal instruments
by which the State regulates the utilization and disposition of forest resources to
In the case now before us, the jurisdictional objection becomes even less tenable the end that public welfare is promoted. And it can hardly be gainsaid that they
and decisive. The reason is that, even if we were to assume that the issue merely evidence a privilege granted by the State to qualified entities, and do not
presented before us was political in nature, we would still not be precluded from vest in the latter a permanent or irrevocable right to the particular concession area
revolving it under the expanded jurisdiction conferred upon us that now covers, in and the forest products therein. They may be validly amended, modified, replaced
proper cases, even the political question. Article VII, Section 1, of the Constitution or rescinded by the Chief Executive when national interests so require. Thus, they
clearly provides: . . . are not deemed contracts within the purview of the due process of law clause
[See Sections 3(ee) and 20 of Pres. Decree No. 705, as amended. Also, Tan v.
Director of Forestry, G.R. No. L-24548, October 27, 1983, 125 SCRA 302].
The last ground invoked by the trial court in dismissing the complaint is the non-impairment of contracts
clause found in the Constitution. The court a quo declared that:
Since timber licenses are not contracts, the non-impairment clause, which reads:
The Court is likewise of the impression that it cannot, no matter how we stretch
our jurisdiction, grant the reliefs prayed for by the plaintiffs, i.e., to cancel all Sec. 10. No law impairing, the obligation of contracts shall be passed. 27
existing timber license agreements in the country and to cease and desist from
receiving, accepting, processing, renewing or approving new timber license
cannot be invoked.
agreements. For to do otherwise would amount to "impairment of contracts"
abhored (sic) by the fundamental law. 24
In the second place, even if it is to be assumed that the same are contracts, the instant case does not
involve a law or even an executive issuance declaring the cancellation or modification of existing timber
We are not persuaded at all; on the contrary, We are amazed, if not shocked, by such a sweeping
licenses. Hence, the non-impairment clause cannot as yet be invoked. Nevertheless, granting further that
pronouncement. In the first place, the respondent Secretary did not, for obvious reasons, even invoke in
a law has actually been passed mandating cancellations or modifications, the same cannot still be
his motion to dismiss the non-impairment clause. If he had done so, he would have acted with utmost
stigmatized as a violation of the non-impairment clause. This is because by its very nature and purpose,
infidelity to the Government by providing undue and unwarranted benefits and advantages to the
such as law could have only been passed in the exercise of the police power of the state for the purpose
timber license holders because he would have forever bound the Government to strictly respect the said
of advancing the right of the people to a balanced and healthful ecology, promoting their health and
licenses according to their terms and conditions regardless of changes in policy and the demands of
enhancing the general welfare. In Abe vs. Foster Wheeler
public interest and welfare. He was aware that as correctly pointed out by the petitioners, into every
Corp. 28 this Court stated:
timber license must be read Section 20 of the Forestry Reform Code (P.D. No. 705) which provides:

The freedom of contract, under our system of government, is not meant to be


. . . Provided, That when the national interest so requires, the President may
absolute. The same is understood to be subject to reasonable legislative regulation
amend, modify, replace or rescind any contract, concession, permit, licenses or
aimed at the promotion of public health, moral, safety and welfare. In other
any other form of privilege granted herein . . .
words, the constitutional guaranty of non-impairment of obligations of contract is
limited by the exercise of the police power of the State, in the interest of public
Needless to say, all licenses may thus be revoked or rescinded by executive action. It is not a health, safety, moral and general welfare.
contract, property or a property right protested by the due process clause of the Constitution.
In Tan vs. Director of Forestry, 25 this Court held:
The reason for this is emphatically set forth in Nebia vs. New York, 29 quoted in Philippine American Life
Insurance Co. vs. Auditor General, 30 to wit:
. . . A timber license is an instrument by which the State regulates the utilization
and disposition of forest resources to the end that public welfare is promoted. A
Under our form of government the use of property and the making of contracts
timber license is not a contract within the purview of the due process clause; it is
are normally matters of private and not of public concern. The general rule is that
only a license or privilege, which can be validly withdrawn whenever dictated by
both shall be free of governmental interference. But neither property rights nor
public interest or public welfare as in this case.
contract rights are absolute; for government cannot exist if the citizen may at will
use his property to the detriment of his fellows, or exercise his freedom of
A license is merely a permit or privilege to do what otherwise would be unlawful, contract to work them harm. Equally fundamental with the private right is that of
and is not a contract between the authority, federal, state, or municipal, granting the public to regulate it in the common interest.
it and the person to whom it is granted; neither is it property or a property right,
nor does it create a vested right; nor is it taxation (37 C.J. 168). Thus, this Court
In short, the non-impairment clause must yield to the police power of the state. 31

Finally, it is difficult to imagine, as the trial court did, how the non-impairment clause could apply with
respect to the prayer to enjoin the respondent Secretary from receiving, accepting, processing, renewing
or approving new timber licenses for, save in cases of renewal, no contract would have as of yet existed
in the other instances. Moreover, with respect to renewal, the holder is not entitled to it as a matter of
right.

WHEREFORE, being impressed with merit, the instant Petition is hereby GRANTED, and the challenged
Order of respondent Judge of 18 July 1991 dismissing Civil Case No. 90-777 is hereby set aside. The
petitioners may therefore amend their complaint to implead as defendants the holders or grantees of
the questioned timber license agreements.

No pronouncement as to costs.

SO ORDERED.
Republic of the Philippines In their Second Amended Petition, petitioners also claim that PD 1869 is contrary to the declared
SUPREME COURT national policy of the "new restored democracy" and the people's will as expressed in the 1987
Manila Constitution. The decree is said to have a "gambling objective" and therefore is contrary to Sections 11,
12 and 13 of Article II, Sec. 1 of Article VIII and Section 3 (2) of Article XIV, of the present Constitution (p.
3, Second Amended Petition; p. 21, Rollo).
EN BANC

The procedural issue is whether petitioners, as taxpayers and practicing lawyers (petitioner Basco being
also the Chairman of the Committee on Laws of the City Council of Manila), can question and seek the
annulment of PD 1869 on the alleged grounds mentioned above.
G.R. No. 91649 May 14, 1991
The Philippine Amusements and Gaming Corporation (PAGCOR) was created by virtue of P.D. 1067-A
ATTORNEYS HUMBERTO BASCO, EDILBERTO BALCE, SOCRATES MARANAN AND LORENZO dated January 1, 1977 and was granted a franchise under P.D. 1067-B also dated January 1, 1977 "to
SANCHEZ,petitioners, establish, operate and maintain gambling casinos on land or water within the territorial jurisdiction of
vs. the Philippines." Its operation was originally conducted in the well known floating casino "Philippine
PHILIPPINE AMUSEMENTS AND GAMING CORPORATION (PAGCOR), respondent. Tourist." The operation was considered a success for it proved to be a potential source of revenue to
fund infrastructure and socio-economic projects, thus, P.D. 1399 was passed on June 2, 1978 for PAGCOR
H.B. Basco & Associates for petitioners. to fully attain this objective.
Valmonte Law Offices collaborating counsel for petitioners.
Aguirre, Laborte and Capule for respondent PAGCOR. Subsequently, on July 11, 1983, PAGCOR was created under P.D. 1869 to enable the Government to
regulate and centralize all games of chance authorized by existing franchise or permitted by law, under
the following declared policy

Sec. 1. Declaration of Policy. It is hereby declared to be the policy of the State to centralize
PARAS, J.: and integrate all games of chance not heretofore authorized by existing franchises or
permitted by law in order to attain the following objectives:

A TV ad proudly announces:
(a) To centralize and integrate the right and authority to operate and conduct games of
chance into one corporate entity to be controlled, administered and supervised by the
"The new PAGCOR responding through responsible gaming." Government.

But the petitioners think otherwise, that is why, they filed the instant petition seeking to annul the (b) To establish and operate clubs and casinos, for amusement and recreation, including
Philippine Amusement and Gaming Corporation (PAGCOR) Charter PD 1869, because it is allegedly sports gaming pools, (basketball, football, lotteries, etc.) and such other forms of amusement
contrary to morals, public policy and order, and because and recreation including games of chance, which may be allowed by law within the territorial
jurisdiction of the Philippines and which will: (1) generate sources of additional revenue to
A. It constitutes a waiver of a right prejudicial to a third person with a right recognized by law. fund infrastructure and socio-civic projects, such as flood control programs, beautification,
It waived the Manila City government's right to impose taxes and license fees, which is sewerage and sewage projects, Tulungan ng Bayan Centers, Nutritional Programs, Population
recognized by law; Control and such other essential public services; (2) create recreation and integrated facilities
which will expand and improve the country's existing tourist attractions; and (3) minimize, if
not totally eradicate, all the evils, malpractices and corruptions that are normally prevalent
B. For the same reason stated in the immediately preceding paragraph, the law has intruded on the conduct and operation of gambling clubs and casinos without direct government
into the local government's right to impose local taxes and license fees. This, in contravention involvement. (Section 1, P.D. 1869)
of the constitutionally enshrined principle of local autonomy;

To attain these objectives PAGCOR is given territorial jurisdiction all over the Philippines. Under its
C. It violates the equal protection clause of the constitution in that it legalizes PAGCOR Charter's repealing clause, all laws, decrees, executive orders, rules and regulations, inconsistent
conducted gambling, while most other forms of gambling are outlawed, together with therewith, are accordingly repealed, amended or modified.
prostitution, drug trafficking and other vices;

It is reported that PAGCOR is the third largest source of government revenue, next to the Bureau of
D. It violates the avowed trend of the Cory government away from monopolistic and crony Internal Revenue and the Bureau of Customs. In 1989 alone, PAGCOR earned P3.43 Billion, and directly
economy, and toward free enterprise and privatization. (p. 2, Amended Petition; p. 7, Rollo) remitted to the National Government a total of P2.5 Billion in form of franchise tax, government's
income share, the President's Social Fund and Host Cities' share. In addition, PAGCOR sponsored other
socio-cultural and charitable projects on its own or in cooperation with various governmental agencies,
and other private associations and organizations. In its 3 1/2 years of operation under the present With particular regard to the requirement of proper party as applied in the cases before us,
administration, PAGCOR remitted to the government a total of P6.2 Billion. As of December 31, 1989, We hold that the same is satisfied by the petitioners and intervenors because each of them
PAGCOR was employing 4,494 employees in its nine (9) casinos nationwide, directly supporting the has sustained or is in danger of sustaining an immediate injury as a result of the acts or
livelihood of Four Thousand Four Hundred Ninety-Four (4,494) families. measures complained of. And even if, strictly speaking they are not covered by the definition,
it is still within the wide discretion of the Court to waive the requirement and so remove the
impediment to its addressing and resolving the serious constitutional questions raised.
But the petitioners, are questioning the validity of P.D. No. 1869. They allege that the same is "null and
void" for being "contrary to morals, public policy and public order," monopolistic and tends toward
"crony economy", and is violative of the equal protection clause and local autonomy as well as for In the first Emergency Powers Cases, ordinary citizens and taxpayers were allowed to
running counter to the state policies enunciated in Sections 11 (Personal Dignity and Human Rights), 12 question the constitutionality of several executive orders issued by President Quirino
(Family) and 13 (Role of Youth) of Article II, Section 1 (Social Justice) of Article XIII and Section 2 although they were involving only an indirect and general interest shared in common with the
(Educational Values) of Article XIV of the 1987 Constitution. public. The Court dismissed the objection that they were not proper parties and ruled that
"the transcendental importance to the public of these cases demands that they be settled
promptly and definitely, brushing aside, if we must technicalities of procedure." We have
This challenge to P.D. No. 1869 deserves a searching and thorough scrutiny and the most deliberate
since then applied the exception in many other cases. (Association of Small Landowners in the
consideration by the Court, involving as it does the exercise of what has been described as "the highest
Philippines, Inc. v. Sec. of Agrarian Reform, 175 SCRA 343).
and most delicate function which belongs to the judicial department of the government." (State v.
Manuel, 20 N.C. 144; Lozano v. Martinez, 146 SCRA 323).
Having disposed of the procedural issue, We will now discuss the substantive issues raised.
As We enter upon the task of passing on the validity of an act of a co-equal and coordinate branch of the
government We need not be reminded of the time-honored principle, deeply ingrained in our Gambling in all its forms, unless allowed by law, is generally prohibited. But the prohibition of gambling
jurisprudence, that a statute is presumed to be valid. Every presumption must be indulged in favor of its does not mean that the Government cannot regulate it in the exercise of its police power.
constitutionality. This is not to say that We approach Our task with diffidence or timidity. Where it is
clear that the legislature or the executive for that matter, has over-stepped the limits of its authority
The concept of police power is well-established in this jurisdiction. It has been defined as the "state
under the constitution, We should not hesitate to wield the axe and let it fall heavily, as fall it must, on
authority to enact legislation that may interfere with personal liberty or property in order to promote
the offending statute (Lozano v. Martinez, supra).
the general welfare." (Edu v. Ericta, 35 SCRA 481, 487) As defined, it consists of (1) an imposition or
restraint upon liberty or property, (2) in order to foster the common good. It is not capable of an exact
In Victoriano v. Elizalde Rope Workers' Union, et al, 59 SCRA 54, the Court thru Mr. Justice Zaldivar definition but has been, purposely, veiled in general terms to underscore its all-comprehensive embrace.
underscored the (Philippine Association of Service Exporters, Inc. v. Drilon, 163 SCRA 386).

. . . thoroughly established principle which must be followed in all cases where questions of Its scope, ever-expanding to meet the exigencies of the times, even to anticipate the future where it
constitutionality as obtain in the instant cases are involved. All presumptions are indulged in could be done, provides enough room for an efficient and flexible response to conditions and
favor of constitutionality; one who attacks a statute alleging unconstitutionality must prove circumstances thus assuming the greatest benefits. (Edu v. Ericta, supra)
its invalidity beyond a reasonable doubt; that a law may work hardship does not render it
unconstitutional; that if any reasonable basis may be conceived which supports the statute, it
It finds no specific Constitutional grant for the plain reason that it does not owe its origin to the charter.
will be upheld and the challenger must negate all possible basis; that the courts are not
Along with the taxing power and eminent domain, it is inborn in the very fact of statehood and
concerned with the wisdom, justice, policy or expediency of a statute and that a liberal
sovereignty. It is a fundamental attribute of government that has enabled it to perform the most vital
interpretation of the constitution in favor of the constitutionality of legislation should be
functions of governance. Marshall, to whom the expression has been credited, refers to it succinctly as
adopted. (Danner v. Hass, 194 N.W. 2nd 534, 539; Spurbeck v. Statton, 106 N.W. 2nd 660,
the plenary power of the state "to govern its citizens". (Tribe, American Constitutional Law, 323, 1978).
663; 59 SCRA 66; see also e.g. Salas v. Jarencio, 46 SCRA 734, 739 [1970]; Peralta v.
The police power of the State is a power co-extensive with self-protection and is most aptly termed the
Commission on Elections, 82 SCRA 30, 55 [1978]; and Heirs of Ordona v. Reyes, 125 SCRA 220,
"law of overwhelming necessity." (Rubi v. Provincial Board of Mindoro, 39 Phil. 660, 708) It is "the most
241-242 [1983] cited in Citizens Alliance for Consumer Protection v. Energy Regulatory Board,
essential, insistent, and illimitable of powers." (Smith Bell & Co. v. National, 40 Phil. 136) It is a dynamic
162 SCRA 521, 540)
force that enables the state to meet the agencies of the winds of change.

Of course, there is first, the procedural issue. The respondents are questioning the legal personality of
What was the reason behind the enactment of P.D. 1869?
petitioners to file the instant petition.

P.D. 1869 was enacted pursuant to the policy of the government to "regulate and centralize thru an
Considering however the importance to the public of the case at bar, and in keeping with the Court's
appropriate institution all games of chance authorized by existing franchise or permitted by law" (1st
duty, under the 1987 Constitution, to determine whether or not the other branches of government have
whereas clause, PD 1869). As was subsequently proved, regulating and centralizing gambling operations
kept themselves within the limits of the Constitution and the laws and that they have not abused the
in one corporate entity the PAGCOR, was beneficial not just to the Government but to society in
discretion given to them, the Court has brushed aside technicalities of procedure and has taken
general. It is a reliable source of much needed revenue for the cash strapped Government. It provided
cognizance of this petition. (Kapatiran ng mga Naglilingkod sa Pamahalaan ng Pilipinas Inc. v. Tan, 163
funds for social impact projects and subjected gambling to "close scrutiny, regulation, supervision and
SCRA 371)
control of the Government" (4th Whereas Clause, PD 1869). With the creation of PAGCOR and the direct
intervention of the Government, the evil practices and corruptions that go with gambling will be Therefore, only the National Government has the power to issue "licenses or permits" for the operation
minimized if not totally eradicated. Public welfare, then, lies at the bottom of the enactment of PD 1896. of gambling. Necessarily, the power to demand or collect license fees which is a consequence of the
issuance of "licenses or permits" is no longer vested in the City of Manila.
Petitioners contend that P.D. 1869 constitutes a waiver of the right of the City of Manila to impose taxes
and legal fees; that the exemption clause in P.D. 1869 is violative of the principle of local autonomy. They (d) Local governments have no power to tax instrumentalities of the National Government. PAGCOR is a
must be referring to Section 13 par. (2) of P.D. 1869 which exempts PAGCOR, as the franchise holder government owned or controlled corporation with an original charter, PD 1869. All of its shares of stocks
from paying any "tax of any kind or form, income or otherwise, as well as fees, charges or levies of are owned by the National Government. In addition to its corporate powers (Sec. 3, Title II, PD 1869) it
whatever nature, whether National or Local." also exercises regulatory powers thus:

(2) Income and other taxes. a) Franchise Holder: No tax of any kind or form, income or Sec. 9. Regulatory Power. The Corporation shall maintain a Registry of the affiliated
otherwise as well as fees, charges or levies of whatever nature, whether National or Local, entities, and shall exercise all the powers, authority and the responsibilities vested in the
shall be assessed and collected under this franchise from the Corporation; nor shall any form Securities and Exchange Commission over such affiliating entities mentioned under the
or tax or charge attach in any way to the earnings of the Corporation, except a franchise tax preceding section, including, but not limited to amendments of Articles of Incorporation and
of five (5%) percent of the gross revenues or earnings derived by the Corporation from its By-Laws, changes in corporate term, structure, capitalization and other matters concerning
operations under this franchise. Such tax shall be due and payable quarterly to the National the operation of the affiliated entities, the provisions of the Corporation Code of the
Government and shall be in lieu of all kinds of taxes, levies, fees or assessments of any kind, Philippines to the contrary notwithstanding, except only with respect to original
nature or description, levied, established or collected by any municipal, provincial or national incorporation.
government authority (Section 13 [2]).
PAGCOR has a dual role, to operate and to regulate gambling casinos. The latter role is governmental,
Their contention stated hereinabove is without merit for the following reasons: which places it in the category of an agency or instrumentality of the Government. Being an
instrumentality of the Government, PAGCOR should be and actually is exempt from local taxes.
Otherwise, its operation might be burdened, impeded or subjected to control by a mere Local
(a) The City of Manila, being a mere Municipal corporation has no inherent right to impose taxes (Icard v.
government.
City of Baguio, 83 Phil. 870; City of Iloilo v. Villanueva, 105 Phil. 337; Santos v. Municipality of Caloocan, 7
SCRA 643). Thus, "the Charter or statute must plainly show an intent to confer that power or the
municipality cannot assume it" (Medina v. City of Baguio, 12 SCRA 62). Its "power to tax" therefore must The states have no power by taxation or otherwise, to retard, impede, burden or in any
always yield to a legislative act which is superior having been passed upon by the state itself which has manner control the operation of constitutional laws enacted by Congress to carry into
the "inherent power to tax" (Bernas, the Revised [1973] Philippine Constitution, Vol. 1, 1983 ed. p. 445). execution the powers vested in the federal government. (MC Culloch v. Marland, 4 Wheat
316, 4 L Ed. 579)
(b) The Charter of the City of Manila is subject to control by Congress. It should be stressed that
"municipal corporations are mere creatures of Congress" (Unson v. Lacson, G.R. No. 7909, January 18, This doctrine emanates from the "supremacy" of the National Government over local governments.
1957) which has the power to "create and abolish municipal corporations" due to its "general legislative
powers" (Asuncion v. Yriantes, 28 Phil. 67; Merdanillo v. Orandia, 5 SCRA 541). Congress, therefore, has
Justice Holmes, speaking for the Supreme Court, made reference to the entire absence of
the power of control over Local governments (Hebron v. Reyes, G.R. No. 9124, July 2, 1950). And if
power on the part of the States to touch, in that way (taxation) at least, the instrumentalities
Congress can grant the City of Manila the power to tax certain matters, it can also provide for
of the United States (Johnson v. Maryland, 254 US 51) and it can be agreed that no state or
exemptions or even take back the power.
political subdivision can regulate a federal instrumentality in such a way as to prevent it from
consummating its federal responsibilities, or even to seriously burden it in the accomplishment
(c) The City of Manila's power to impose license fees on gambling, has long been revoked. As early as of them. (Antieau, Modern Constitutional Law, Vol. 2, p. 140, emphasis supplied)
1975, the power of local governments to regulate gambling thru the grant of "franchise, licenses or
permits" was withdrawn by P.D. No. 771 and was vested exclusively on the National Government, thus:
Otherwise, mere creatures of the State can defeat National policies thru extermination of what local
authorities may perceive to be undesirable activities or enterprise using the power to tax as "a tool for
Sec. 1. Any provision of law to the contrary notwithstanding, the authority of chartered cities regulation" (U.S. v. Sanchez, 340 US 42).
and other local governments to issue license, permit or other form of franchise to operate,
maintain and establish horse and dog race tracks, jai-alai and other forms of gambling is
The power to tax which was called by Justice Marshall as the "power to destroy" (Mc Culloch v.
hereby revoked.
Maryland, supra) cannot be allowed to defeat an instrumentality or creation of the very entity which has
the inherent power to wield it.
Sec. 2. Hereafter, all permits or franchises to operate, maintain and establish, horse and dog
race tracks, jai-alai and other forms of gambling shall be issued by the national government
(e) Petitioners also argue that the Local Autonomy Clause of the Constitution will be violated by P.D.
upon proper application and verification of the qualification of the applicant . . .
1869. This is a pointless argument. Article X of the 1987 Constitution (on Local Autonomy) provides:
Sec. 5. Each local government unit shall have the power to create its own source of revenue require situations which are different in fact or opinion to be treated in law as though they were the
and to levy taxes, fees, and other charges subject to such guidelines and limitation as the same (Gomez v. Palomar, 25 SCRA 827).
congress may provide, consistent with the basic policy on local autonomy. Such taxes, fees
and charges shall accrue exclusively to the local government. (emphasis supplied)
Just how P.D. 1869 in legalizing gambling conducted by PAGCOR is violative of the equal protection is not
clearly explained in the petition. The mere fact that some gambling activities like cockfighting (P.D 449)
The power of local government to "impose taxes and fees" is always subject to "limitations" which horse racing (R.A. 306 as amended by RA 983), sweepstakes, lotteries and races (RA 1169 as amended by
Congress may provide by law. Since PD 1869 remains an "operative" law until "amended, repealed or B.P. 42) are legalized under certain conditions, while others are prohibited, does not render the
revoked" (Sec. 3, Art. XVIII, 1987 Constitution), its "exemption clause" remains as an exception to the applicable laws, P.D. 1869 for one, unconstitutional.
exercise of the power of local governments to impose taxes and fees. It cannot therefore be violative but
rather is consistent with the principle of local autonomy.
If the law presumably hits the evil where it is most felt, it is not to be overthrown because
there are other instances to which it might have been applied. (Gomez v. Palomar, 25 SCRA
Besides, the principle of local autonomy under the 1987 Constitution simply means "decentralization" (III 827)
Records of the 1987 Constitutional Commission, pp. 435-436, as cited in Bernas, The Constitution of the
Republic of the Philippines, Vol. II, First Ed., 1988, p. 374). It does not make local governments sovereign
The equal protection clause of the 14th Amendment does not mean that all occupations
within the state or an "imperium in imperio."
called by the same name must be treated the same way; the state may do what it can to
prevent which is deemed as evil and stop short of those cases in which harm to the few
Local Government has been described as a political subdivision of a nation or state which is concerned is not less than the harm to the public that would insure if the rule laid down were
constituted by law and has substantial control of local affairs. In a unitary system of made mathematically exact. (Dominican Hotel v. Arizona, 249 US 2651).
government, such as the government under the Philippine Constitution, local governments
can only be an intra sovereign subdivision of one sovereign nation, it cannot be
Anent petitioners' claim that PD 1869 is contrary to the "avowed trend of the Cory Government away
an imperium in imperio. Local government in such a system can only mean a measure of
from monopolies and crony economy and toward free enterprise and privatization" suffice it to state
decentralization of the function of government. (emphasis supplied)
that this is not a ground for this Court to nullify P.D. 1869. If, indeed, PD 1869 runs counter to the
government's policies then it is for the Executive Department to recommend to Congress its repeal or
As to what state powers should be "decentralized" and what may be delegated to local government units amendment.
remains a matter of policy, which concerns wisdom. It is therefore a political question. (Citizens Alliance
for Consumer Protection v. Energy Regulatory Board, 162 SCRA 539).
The judiciary does not settle policy issues. The Court can only declare what the law is and not
what the law should be.1wphi1 Under our system of government, policy issues are within
What is settled is that the matter of regulating, taxing or otherwise dealing with gambling is a State the domain of the political branches of government and of the people themselves as the
concern and hence, it is the sole prerogative of the State to retain it or delegate it to local governments. repository of all state power. (Valmonte v. Belmonte, Jr., 170 SCRA 256).

As gambling is usually an offense against the State, legislative grant or express charter power On the issue of "monopoly," however, the Constitution provides that:
is generally necessary to empower the local corporation to deal with the subject. . . . In the
absence of express grant of power to enact, ordinance provisions on this subject which are
Sec. 19. The State shall regulate or prohibit monopolies when public interest so requires. No
inconsistent with the state laws are void. (Ligan v. Gadsden, Ala App. 107 So. 733 Ex-Parte
combinations in restraint of trade or unfair competition shall be allowed. (Art. XII, National
Solomon, 9, Cals. 440, 27 PAC 757 following in re Ah You, 88 Cal. 99, 25 PAC 974, 22 Am St.
Economy and Patrimony)
Rep. 280, 11 LRA 480, as cited in Mc Quinllan Vol. 3 Ibid, p. 548, emphasis supplied)

It should be noted that, as the provision is worded, monopolies are not necessarily prohibited by the
Petitioners next contend that P.D. 1869 violates the equal protection clause of the Constitution, because
Constitution. The state must still decide whether public interest demands that monopolies be regulated
"it legalized PAGCOR conducted gambling, while most gambling are outlawed together with
or prohibited. Again, this is a matter of policy for the Legislature to decide.
prostitution, drug trafficking and other vices" (p. 82, Rollo).

On petitioners' allegation that P.D. 1869 violates Sections 11 (Personality Dignity) 12 (Family) and 13
We, likewise, find no valid ground to sustain this contention. The petitioners' posture ignores the well-
(Role of Youth) of Article II; Section 13 (Social Justice) of Article XIII and Section 2 (Educational Values) of
accepted meaning of the clause "equal protection of the laws." The clause does not preclude
Article XIV of the 1987 Constitution, suffice it to state also that these are merely statements of principles
classification of individuals who may be accorded different treatment under the law as long as the
and, policies. As such, they are basically not self-executing, meaning a law should be passed by Congress
classification is not unreasonable or arbitrary (Itchong v. Hernandez, 101 Phil. 1155). A law does not have
to clearly define and effectuate such principles.
to operate in equal force on all persons or things to be conformable to Article III, Section 1 of the
Constitution (DECS v. San Diego, G.R. No. 89572, December 21, 1989).
In general, therefore, the 1935 provisions were not intended to be self-executing principles
ready for enforcement through the courts. They were rather directives addressed to the
The "equal protection clause" does not prohibit the Legislature from establishing classes of individuals or
executive and the legislature. If the executive and the legislature failed to heed the directives
objects upon which different rules shall operate (Laurel v. Misa, 43 O.G. 2847). The Constitution does not
of the articles the available remedy was not judicial or political. The electorate could express
their displeasure with the failure of the executive and the legislature through the language of
the ballot. (Bernas, Vol. II, p. 2)

Every law has in its favor the presumption of constitutionality (Yu Cong Eng v. Trinidad, 47 Phil. 387;
Salas v. Jarencio, 48 SCRA 734; Peralta v. Comelec, 82 SCRA 30; Abbas v. Comelec, 179 SCRA 287).
Therefore, for PD 1869 to be nullified, it must be shown that there is a clear and unequivocal breach of
the Constitution, not merely a doubtful and equivocal one. In other words, the grounds for nullity must
be clear and beyond reasonable doubt. (Peralta v. Comelec, supra) Those who petition this Court to
declare a law, or parts thereof, unconstitutional must clearly establish the basis for such a declaration.
Otherwise, their petition must fail. Based on the grounds raised by petitioners to challenge the
constitutionality of P.D. 1869, the Court finds that petitioners have failed to overcome the presumption.
The dismissal of this petition is therefore, inevitable. But as to whether P.D. 1869 remains a wise
legislation considering the issues of "morality, monopoly, trend to free enterprise, privatization as well as
the state principles on social justice, role of youth and educational values" being raised, is up for
Congress to determine.

As this Court held in Citizens' Alliance for Consumer Protection v. Energy Regulatory Board, 162 SCRA 521

Presidential Decree No. 1956, as amended by Executive Order No. 137 has, in any case, in its
favor the presumption of validity and constitutionality which petitioners Valmonte and the
KMU have not overturned. Petitioners have not undertaken to identify the provisions in the
Constitution which they claim to have been violated by that statute. This Court, however, is
not compelled to speculate and to imagine how the assailed legislation may possibly offend
some provision of the Constitution. The Court notes, further, in this respect that petitioners
have in the main put in question the wisdom, justice and expediency of the establishment of
the OPSF, issues which are not properly addressed to this Court and which this Court may not
constitutionally pass upon. Those issues should be addressed rather to the political
departments of government: the President and the Congress.

Parenthetically, We wish to state that gambling is generally immoral, and this is precisely so when the
gambling resorted to is excessive. This excessiveness necessarily depends not only on the financial
resources of the gambler and his family but also on his mental, social, and spiritual outlook on life.
However, the mere fact that some persons may have lost their material fortunes, mental control,
physical health, or even their lives does not necessarily mean that the same are directly attributable to
gambling. Gambling may have been the antecedent, but certainly not necessarily the cause. For the same
consequences could have been preceded by an overdose of food, drink, exercise, work, and even sex.

WHEREFORE, the petition is DISMISSED for lack of merit.

SO ORDERED.

S-ar putea să vă placă și